You are on page 1of 292

BIOLOGY

CBSE (PROBLEMS SOLUTION)

S.No. Content Page


SYLLABUS I & II

01. SEXUAL REPRODUCTION IN FLOWERING PLANTS 01-22

02. HUMAN REPRODUCTION 23-46

03. REPRODUCTIVE HEALTH 47-62

04. PRINCIPLES OF INHERITANCE AND VARIATION 63-88

05. MOLECULAR BASIS OF INHERITANCE 89-114

06. EVOLUTION 115-132

07. HUMAN HEALTH AND DISEASE 133-152

08. MICROBES IN HUMAN WELFARE 153-172

09. BIOTECHNOLOGY PRINCIPLES AND PROCESSES 173-192

10. BIOTECHNOLOGY AND ITS APPLICATIONS 193-210

11. ORGANISMS AND POPULATIONS 211-228

12. ECOSYSTEM 229-248

13. BIODIVERSITY AND CONSERVATION 249-268

14. SAMPLE QUESTION PAPER 269–278

15. SAMPLE QUESTION PAPER (SOLUTION) 279–288


All rights including trademark and copyrights and rights of translation etc. reserved and vested exclusively with
Allen Career Institute Private Limited. (Allen)
No part of this work may be copied, reproduced, adapted, abridged or translated, transcribed, transmitted,
stored or distributed in any form retrieval system, computer system, photographic or other system or
transmitted in any form or by any means whether electronic, magnetic, chemical or manual, mechanical, digital,
optical, photocopying, recording or otherwise, or stood in any retrieval system of any nature without the written
permission of the Allen Career Institute Private Limited. Any breach will entail legal action and prosecution
without further notice.
This work is sold/distributed by Allen Career Institute Private Limited subject to the condition and undertaking
given by the student that all proprietary rights (under the Trademark Act, 1999 and Copyright Act, 1957) of the
work shall be exclusively belong to Allen Career Institute Private Limited. Neither the Study Materials and/or
Test Series and/or the contents nor any part thereof i.e. work shall be reproduced, modify, re-publish,
sub-license, upload on website, broadcast, post, transmit, disseminate, distribute, sell in market, stored in a
retrieval system or transmitted in any form or by any means for reproducing or making multiple copies of it.
Any person who does any unauthorised act in relation to this work may be liable to criminal prosecution and
civil claims for damages. Any violation or infringement of the propriety rights of Allen shall be punishable
under Section- 29 & 52 of the Trademark Act, 1999 and under Section- 51, 58 & 63 of the Copyright Act, 1957 and
any other Act applicable in India. All disputes are subjected to the exclusive jurisdiction of courts, tribunals and
forums at Kota, Rajasthan only.
Note:- This publication is meant for educational and learning purposes. All reasonable care and diligence have
been taken while editing and printing this publication. Allen Career Institute Private Limited shall not hold any
responsibility for any error that may have inadvertently crept in. Allen Career Institute Private Limited is not
responsible for the consequences of any action taken on the basis of this publication.
BIOLOGY (Code No. 044)
COURSE STRUCTURE
CLASS XII (2023 - 24)

EVALUATION SCHEME
THEORY
Units Title Marks
VI Reproduction 16
VII Genetics and Evolution 20
VIII Biology and Human Welfare 12
IX Biotechnology and its Applications 12
X Ecology and Environment 10
Total 70
UNIT-VI REPRODUCTION
Chapter-2: Sexual Reproduction in Flowering Plants
Flower structure; development of male and female gametophytes; pollination - types, agencies
and examples; out breeding devices; pollen-pistil interaction; double fertilization; post
fertilization events - development of endosperm and embryo, development of seed and formation
of fruit; special modes- apomixis, parthenocarpy, polyembryony; Significance of seed dispersal
and fruit formation.
Chapter-3: Human Reproduction
Male and female reproductive systems; microscopic anatomy of testis and ovary; gametogenesis
- spermatogenesis and oogenesis; menstrual cycle; fertilisation, embryo development upto
blastocyst formation, implantation; pregnancy and placenta formation (elementary idea);
parturition (elementary idea); lactation (elementary idea).
Chapter-4: Reproductive Health
Need for reproductive health and prevention of Sexually Transmitted Diseases (STDs); birth
control - need and methods, contraception and medical termination of pregnancy (MTP);
amniocentesis; infertility and assisted reproductive technologies - IVF, ZIFT, GIFT (elementary
idea for general awareness).
UNIT-VII GENETICS AND EVOLUTION
Chapter-5: Principles of Inheritance and Variation
Heredity and variation: Mendelian inheritance; deviations from Mendelism – incomplete
dominance, co-dominance, multiple alleles and inheritance of blood groups, pleiotropy;
elementary idea of polygenic inheritance; chromosome theory of inheritance; chromosomes and
genes; Sex determination - in humans, birds and honey bee; linkage and crossing over; sex linked
inheritance - haemophilia, colour blindness; Mendelian disorders in humans - thalassemia;
chromosomal disorders in humans; Down's syndrome, Turner's and Klinefelter's syndromes.

E I
Chapter-6: Molecular Basis of Inheritance
Search for genetic material and DNA as genetic material; Structure of DNA and RNA; DNA
packaging; DNA replication; Central Dogma; transcription, genetic code, translation; gene
expression and regulation - lac operon; Genome, Human and rice genome projects; DNA
fingerprinting.
Chapter-7: Evolution
Origin of life; biological evolution and evidences for biological evolution (paleontology,
comparative anatomy, embryology and molecular evidences); Darwin's contribution, modern
synthetic theory of evolution; mechanism of evolution - variation (mutation and recombination)
and natural selection with examples, types of natural selection; Gene flow and genetic drift;
Hardy - Weinberg's principle; adaptive radiation; human evolution.
UNIT-VIII BIOLOGY AND HUMAN WELFARE
Chapter-8: Human Health and Diseases
Pathogens; parasites causing human diseases (malaria, dengue, chikungunya, filariasis, ascariasis,
typhoid, pneumonia, common cold, amoebiasis, ring worm) and their control; Basic concepts of
immunology - vaccines; cancer, HIV and AIDS; Adolescence - drug and alcohol abuse.
Chapter-10: Microbes in Human Welfare
Microbes in food processing, industrial production, sewage treatment, energy generation and
microbes as bio-control agents and bio-fertilizers. Antibiotics; production and judicious use.
UNIT-IX BIOTECHNOLOGY AND ITS APPLICATIONS
Chapter-11: Biotechnology - Principles and Processes
Genetic Engineering (Recombinant DNA Technology).
Chapter-12: Biotechnology and its Applications
Application of biotechnology in health and agriculture: Human insulin and vaccine production,
stem cell technology, gene therapy; genetically modified organisms - Bt crops; transgenic
animals; biosafety issues, biopiracy and patents.
UNIT-X ECOLOGY AND ENVIRONMENT
Chapter-13: Organisms and Populations
Population interactions - mutualism, competition, predation, parasitism; population attributes -
growth, birth rate and death rate, age distribution. (Topics excluded: Organism and its
Environment, Major Abiotic Factors, Responses to Abiotic Factors, Adaptations)
Chapter-14: Ecosystem
Ecosystems: Patterns, components; productivity and decomposition; energy flow; pyramids of
number, biomass, energy (Topics excluded: Ecological Succession and Nutrient Cycles)
Chapter-15: Biodiversity and its Conservation
Biodiversity-Concept, patterns, importance; loss of biodiversity; biodiversity conservation;
hotspots, endangered organisms, extinction, Red Data Book, Sacred Groves, biosphere reserves,
national parks, wildlife, sanctuaries and Ramsar sites.
II E
CHAPTER 1
(A) NCERT QUESTIONS & SOLUTIONS
1. Name the parts of an angiosperm flower in which development of male and female
gametophyte take place.
Ans. Development of male and female gametophytes takes place in anther and ovary Respectively.
2. Differentiate between microsporogenesis and megasporogenesis. Which type of cell division
occurs during these events? Name the structures formed at the end of these two events.
Ans. S.No Microsporogenesis Megasporogenesis
1 In this process, haploid microspore are In this process, haploid megaspore are
formed from diploid mother cell or pollen formed from diploid mother cell
mother cells (MMC or PMC). (MMC).
2 It occurs inside the microsporangium or It occurs inside the nucellus of ovule or
pollen sac of an anther. megasporangium.
3 There are many microspore mother cells in a There are generally a single megaspore
microsporangium. mother cell in a megasporangium.
4 The four microspore formed from a single The four megaspore formed from a
microspore mother cell are generally megaspore mother cells are arranged in
arranged in a tetrahedral structure. the form of a linear tetrad.
5 All the four mircrospore arranged in a Only one megaspore remains functional
tetrahedral tetrad are functional. while the other three degenerates.
6 The microspores give rise to male The functional megaspore gives rise to
gametophytes. female gametophytes.
Meiosis occurs during micro and megasporogenesis. Microspores (pollen grains) are formed at
the end of microsporogenesis and female gametophyte (embryo sac) are formed at the end of
megasporogenesis.
3. Arrange the following terms in the correct developmental sequence: pollen grain,
sporogenous tissue, microspore tetrad, pollen mother cell, male gametes.
Ans. Sporogenous tissue - Pollen mother cell - Microspore tetrad - Pollen grain - Male gametes
4. With a neat, Labelled diagram, describe the parts of a typical angiosperm ovule.
Ans. Structure of megasporangium.

Raphe Chalaza
Nucellus
Outer integument
Inner integument
Antipodals
Polar nuclei
Egg apparatus
Nucellus
Micropyle
Funicle
Fig:- Structure of megasporangium
(Ovule)
E 1
5. What is meant by monosporic development of a female gametophyte?
Ans. Out of the four megaspores, three degenerate and only one remains functional which develops
into a female gametophyte or embryo sac. This is called monosporic development i.e. when
embryo sac develops from one single megaspore it is called monosporic embryo sac.
6. With a neat diagram and explain the 7-celled, 8-nucleate nature of the female
gametophyte. [IMP.]
Ans. Female gametophyte or embryo sac is a small oval structure that contains a 3-celled egg
apparatus, 3 antipodal cells and one binucleate central cell hence it is 7 - celled and 8 - nucleate
structure.
(a) Egg apparatus: It consists of two synergids and an egg cell towards the micropylar end. The
cells of egg apparatus are uninucleate. Synergids towards its micropylar end have cellular
thickenings called filiform apparatus that helps in guiding the pollen tubes into the synergids
(b) Antipodal cells: Chalazal end of embryo sac contain three cells of various shapes and sizes
called antipodal cells.
(c) Central cell: It is the single and largest cell which is bounded by a membrane of embryo sac.
It contains two polar nuclei which later fuse to form diploid secondary nucleus. After
fertilisation the central cell gets converted into triploid primary endosperm cell (PEC) which
forms endosperm.

Chalazal end
Antipodals

Polar nuclei
Central cell
Egg
nuclei

Synergids

Filiform
apparatus
Micropylar end
A Mature Embryo-Sac

7. What are chasmogamous flowers? Can cross-pollination occur in cleistogamous flowers?


Give reasons in your answer.
Ans. Chasmogamous flowers are open flowers with exposed stamens and stigma which facilitate
cross pollination.
No cross-pollination occurs in cleistogamous flowers as these flowers are closed and never
open and thus no transfer of pollen from outside to stigma of the flower is possible.
8. Mention two strategies evolved to prevent self-pollination in flowers.
Ans. (i) Maturation of anthers and stigma at different time periods in a bisexual flower prevents self
pollination (dichogamy).
(ii) Production of unisexual flowers.
9. What is self-incompatibility? Why does self-pollination not lead to seed formation in
self-incompatible species?
Ans. Self-incompatibility or self sterility is the inability of an intersexual or bisexual plant to produce
viable seeds on self-pollination in spite of producing functional male and female gametes. Since
fertilization does not take place, no seeds are produced. It is a genetic mechanism that prevents
self pollen from fertilizing ovules by inhibiting pollen tube growth in pistil.
10. What is bagging technique? How is it useful in a plant breeding Programme.
Ans. It is the covering of female plant with butter paper germ to avoid their contamination from
foreign pollens during breeding programme.
11. What is triple fusion? Where and how does it take place? Name the nuclei involved in triple
fusion.
Ans. Triple fusion is fusion of one male gamete and two polar nuclei (or secondary nucleus if the two
have already fused) in the central cell of embryo sac to form primary endosperm nucleus. It takes
place in the central cell of an embryo sac. Three nuclei are involved in triple fusion i.e. nuclei in
the central cell one male nucleus and two polar nuclei in the central cell.
12. Why do you think the zygote is dormant for sometime in a fertilised ovule?
Ans. The zygote is dormant for sometime in a fertilised ovule because the embryo which is developed
from zygote may obtain from endosperm.
13. Differentiate between: [IMP.]
(a) hypocotyl and epicotyl; (b) coleoptile and (Coleorhiza)
(c) integument and testa; (d) perisperm and pericarp
Ans. (a) hypocotyl and epicotyl
S.No. Hypocotyl Epicotyle
1. The region of the embryonal axis that The region of the embryonal axis that lies
lies between the radical and the point of between the plumule and cotyledons is
attachment of cotyledons is called as called as epicotyle.
hypocotyle.
2. Hypocotyle pushes the seed above the Epicotyle pushes the plumule above the
soil in epigeal germination. soil in hypogeal germination.
3. It is an important component of It is an important component of embryonic
embryonic root system. shoot system.
(b) coleoptile and coleorhiza
S.No. Coleoptile Coleorhiza
1. The shoot apex and few leaf Primordia The radical and root cap are sitiated at the
are enclosed in a hollow foliar structure lower end of embryonal axis are enclosed
in epicotyl region in monocots and is by undifferentiated sheath called
called coleoptiles. coleorhiza.
2. It emerges from the soil turns green It remains in the soil and is non green in
and carries out photosynthesis. colour.
E 3
(c) integument and testa
S.No. Integument Testa
1. It is part of pre-fertilization events. It is part of post-fertilization events.
2. It is the protective covering of the ovule It is the protective outer covering of the
may differentiate into inner and outer seed formed from outer integument.
integument.
3. It is thin and living in nature. It is thick and dead in nature.
(d) perisperm and pericarp
S.No. Perisperm Pericarp
1. It represent persistent remains of nucellus in It represent the wall of fruit formed
the seed. by the ovarian wall.
2. It is a part that belongs to seed. It is a part that belongs to fruit.
3. It is usually dry. It can be dry or fleshy.
14. Why is apple called a false fruit? Which part (s) of the flower forms the fruit?
Ans. False fruits are those fruits in which accessory floral parts also contribute to fruit formation. In
apple, the thalamus also contributes to fruit formation. Therefore, it is called a false fruit. The
fruit develops from the ovary of the flower.
15. What is meant by emasculation? When and why does a plant breeder employ this
technique? [IMP.]
Ans. Emasculation is a practice of removal of stamens/anthers before the anther dehiscence from
bisexual flowers in female parent. A plant breeder employs this technique in the immature bud
condition before the anthers begin to differentiate. It is required to prevent self-pollination.
Uses in plant breeding :
(i) Prevention of contamination and pollination of stigma of female flowers with foreign
undesirable pollens.
(ii) Prevention of damage by animals.
16. If one can induce parthenocarpy through the application of growth substances, which fruits
would you select to induce parthenocarpy and why?
Ans. Only fleshy fruits like orange watermelon, lemon, etc. should be selected as parthenocarpic fruit.
Here seeds of fruits are irritant during consumption of seeds the fruits even more valuable. It is
easy to make fruit juices, jams, etc. with seedless fruits.
17. Explain the role of tapetum in the formation of pollen-grain wall.
Ans. Tapetum is the innermost wall layer of a microsporangium. It nourishes the developing pollen
grains and also help in the formation of wall of pollen grains. The cells of tapetum secrete Ubisch
granules that provide sporopollenin and other materials for exine formation.
18. What is apomixis and what is its importance?
Ans. Apomixis is a type of asexual reproduction that mimics sexual reproduction to form seeds
without fertilisation. In apomictic seeds parental characters are maintained in the
progeny/offspring as there is no meiosis or segregation of characters. If desired hybrid seeds are
made apomictics the farmers can keep on using the hybrid seeds to rise new crops year after year.
(B) PREVIOUS YEAR QUESTIONS
1. Select the option that shows the correctly identified 'U', 'X', 'Y' and 'Z' in a developing dicot
embryo. [CBSE 2023]

X
Z

Y
U

(1) X-Plumule (2n), Y-Suspensor (n), Z-Cotyledon (2n), U-Radicle (2n).


(2) X-Plumule (2n), Y-Suspensor (2n), Z-Radicle (2n), U-Cotyledon (2n).
(3) X-Suspensor (2n), Y-Cotyledon (2n), Z-Radicle (2n), U-Plumule (2n).
(4) X-Cotyledon (2n), Y-Radicle (n), Z-Plumule (n), U-Suspensor (n).
Ans. (3) X-Suspensor (2n), Y-Cotyledon (2n), Z-Radicle (2n), U-Plumule (2n).
2. (i) Explain the monosporic development of embryo sac in the ovule of an angiosperm.
(ii) Draw a diagram of the mature embryo sac of an angiospermic ovule and label any
four parts in it. [CBSE 2023]
Ans. (i) The process of formation of megaspores from megaspore mother cell (MMC) is called
megasporogenesis.
 The four haploid megaspores are generally arranged in linear tetrad.
 In a majority of flowering plants the lower most or chalazal megaspore remains functional
out of four megaspores and the other three which lie towards the micropyle degenerate.
 The functional megaspore produces female gametophyte (embryo sac).
 This method of embryo sac formation from a single megaspore is termed monosporic
development.
(ii)
Chalazal end
Antipodals

Polar nuclei
Central cell
Egg

Synergids

Filiform
apparatus
Micropylar end
A Mature Embryo-Sac

E 5
3. One of the major approaches of crop improvement programme is Artificial Hybridisation.
Explain the steps involved in making sure that only the desired pollen grain pollinate the
stigma of a bisexual flower by a plant breeder. [CBSE 2023]
Ans. Artificial hybridisation is one of the major approaches of crop improvement programme. In such
crossing experiments it is important to make sure that only the desired pollen grains are used for
pollination and the stigma is protected from contamination (from unwanted pollen).
This is achieved by emasculation and bagging techniques.
 If the female parent bears bisexual flowers, removal of anthers from the flower bud before
the anther dehisces using a pair of forceps is necessary. This step is referred to as
emasculation.
 Emasculated flowers have to be covered with a bag of suitable size, generally made up of
butter paper, to prevent contamination of its stigma with unwanted pollen. This process is
called bagging.
 When the stigma of bagged flower attains receptivity, mature pollen grains collected from
anthers of the male parent are dusted on the stigma, and the flowers are rebagged, and the
fruits allowed to develop.
4. The hilum in a typical angiospermic ovule represents the junction between:-
[CBSE Term-I 2022]
(1) Integuments and the embryo sac. (2) Embryo sac and the nucellus
(3) Body of the ovule and the funicle (4) Nucellus and the fumicle
Ans. (3) Body of the ovule and the funicle
5. In the given diagram of a transverse section of a young anther. Choose the labellings showing the
correct placement of the wall layers from the table given below. [CBSE Term-I 2022]
(iii)
(ii)

(iv)

(i)

(i) (ii) (ii) (iii)


(1) Epidermis Middle layers Tapetum Endothecium
(2) Tapetum Endothecium Epidermis Middle layers
(3) Endothecium Tapetum Middle layers Epidermis
(4) Middle layers Epidermis Endothecium Tapetum
Ans. (2) (i) Tapetum (ii)Endothecium (iii)Epidermis (iv) Middle layers
6. The term used for the embryo entering into the state of inactivity as the seed mature is:-
[CBSE Term-I 2022]
(1) Quiescent (2) Parthenogenesis (3) Parthenocarpy (4) Dormancy
Ans. (4) Dormancy
7. The ploidy of the apomictic embryo developed from the integument cells and megaspore mother
cell without reduction division respectively will be:– [CBSE Term-I 2022]
(1) 2 n and 2n (2) n and n (3) 2n and n (4) 3n and 2n
Ans. (1) 2 n and 2n
6 E
8. Given below is a diagrammatic representation of a mature embryo sac of a typical angiosperm
plant. [CBSE Term-I 2022]
Choose the option showing the correct labellings for the parts W, X, Y and Z from the table
given below.
(X)

(Z)

(Y)

(W)

W X Y Z
(1) Micropylar end Antipodals Synergids Central cell
(2) Chalazal end Antipodals Central cell Synergids
(3) Micropylar end Synergids Central cell Antipodals
(4) Chalazal end Synergids Central cell Antipodals
Ans. (1) (W) Micropylar end (X) Antipodals (Y) Synergids (Z) Central cell
9. Given below is a figure of an angiosperm plant showing two different types of flowers 'X' and 'Y'
and the possible type of pollination in them: [CBSE Term-I 2022]

(X)

(Y)

Select the correct option for the flower (X) and flower (Y) and the possible type of pollination
from the given table:
Flower X Flower Y
(1) Chasmogamous, assured seed set Cleistogamous, cross pollination
(2) Cleistogamous self/cross pollination Chasmogamous, assured seed set
(3) Chasmogamous self/cross pollination Cleistogamous, self-pollination
(4) Cleistogamous self-pollination only Chasmogamous, cross pollination only
Ans. (3) Flower X - Chasmogamous self/cross pollination
Flower Y - Cleistogamous, self-pollination
10. An undifferentiated sheath covering the root cap of a monocotyledonous embryo is:
[CBSE Term-I 2022]
(1) Scutellum (2) Coleorhiza (3) Coleoptile (4) Epiblast
Ans. (2) Coleorhiza

E 7
11. Why does endosperm development precede embryo development? [CBSE IMP-Question]
Ans. The cells of endosperm are filled with reserve food materials and are used for the nutrition of the
developing embryo.
12. How many meiotic divisions are required to produce 76 seeds in a Guava fruit?
[CBSE IMP-Question]
Ans. 95
13. How does pollination take place in water hyacinth and water lily? [CBSE IMP-Question]
Ans. In water hyacinth and water lily, the flowers emerge above the level of water and are pollinated
by insects or wind as in most of the land plants.
14. Self-pollination is fully ensured if [CBSE 2020]
(1) The flower is bisexual. (2) The style is longer that the filament.
(3) The flower is cleistogamous. (4) The time of pistil and anther maturity is different.
Ans. (3)
15. Draw a schematic transverse section of a mature anther of an angiosperm. Label its
epidermis, middle layers, tapetum, endothecium, sporogenous tissue and the connective.
[CBSE 2018,20]
Ans.
Epidermis

Endothecium
Connective Middle layers
Epidermis
Endothecium Microspore
Sporogenous mother cells
tissue
Tapetum
Middle layers Tapetum

(A) T.S of young anther (B) Enlarged view of one microsporangium

16. Differentiate between wind pollinated and insect pollinated flowers. [CBSE 2020]
Ans.
Wind Pollinated Insect Pollinated
Wind Pollinated flowers do not Insect Pollinated flowers are rich in nectar
have nectar. to attract insects.
The petals are dull and does not The petals of these flowers are very
attract the insects. colourful and attractive.
They do not have any scent. The scent of these flowers are another factor
that attracts insects.
The pollen grains are non-sticky, The pollen grains are sticky.
light.
Stigma is feathery, to catch the Stigma is non-feathery and sticky.
pollen grain.

8 E
17. Some flowers, selected for artificial hybridization, do not require emasculation but bagging is
essential for them. Give a reason. [CBSE 2019]
Ans. As some flowers are unisexual, to prevent contamination of its stigma with unwanted pollen
grains.
18. Write any two ways by which apomictic seeds may be developed in angiosperms.
[CBSE 2019]
Ans. Develops from a diploid egg cell (formed without reduction division) which grows into an
embryo without fertilization.
Develops from nuclear cell which divides and protrudes into the embryo sac and develops
into an embryo
19. Draw a labeled diagram of a mature male gametophyte of an angiosperm. [CBSE 2019]
Ans.

Exine
Intine Nucleus
Nucleus
Vegetative
cell Germ Pore

Generative
cell
Mature male gametophyte

20. Draw a diagram of L.S. of an embryo of grass and label any six parts. [CBSE 2019]
Ans.

Scutellum

Coleoptile

Shoot apex

Epiblast

Radicle
Root cap
Coleorhiza

L.S. of an embryo of grass

21. (a) Draw a diagram of Pistil showing pollen tube growth in angiosperm and label
(i) Stigma (ii) male gametes (iii) micropyle and (iv) Ovule.
(b) Write the function of micropyle. [CBSE 2018]
Ans. (a)
E 9
STIGMA

OVULE

MICROPYLE
MALE GAMETES

(b) The pollen tube enters the ovule through micropyle, it facilitates the entry of oxygen and
water for seed germination.

22. State one difference and one similarity between geitonogamy and xenogamy. [CBSE 2018]

Ans. Difference - In geitonogamy pollen grains from one flower are transferred to the stigma of anther
flower on the same plant whereas in xenogamy the pollen grains are transferred to the stigma of a
flower on another plant (of the same species) genetically similar, genetically different.

Similarity - In both types of pollination pollen grains from the anther are transferred to the
stigma of another flower of the same species.

23. Explain any three devices developed in flowering plants to discourage self pollination and
encourage cross pollination. [CBSE 2018]

Ans. Pollen release & stigma receptivity not synchronised / hence the maturity of stigma and pollen
are different /Protandry / Protogyny- Anther and Stigma are placed at different positions so that
pollen cannot come in contact with stigma of the same flower.

Self incompatibility/ Self sterility.

Production of unisexual flowers.

24. Write one advantage and one disadvantage of cleistogamy to flowering plants. [CBSE 2018]

Ans. Advantage - Assured seed set / maintain purelines.

Disadvantage - No variation / only parental characters are preserved / it can lead to inbreeding
depression

25. If the meiocyte of a maize plant contains 20 chromosomes, write the number of
chromosomes in the endosperm and embryo of the maize grain and give reasons in support
of your answer. [CBSE 2018]
Ans. Endosperm = 30, Embryo = 20

Diploid meiocyte (20 chromosomes) form haploid gametes (10 chromosomes) Two haploid
gametes fuse to form diploid (20) zygote which develops into a (diploid = 20) embryo / syngamy
of two haploid gametes to form a diploid zygote.

One haploid gamete (chromosome 10) fuses with two polar nuclei (chromosome 10 + 10) to form
(triploid - 30) endosperm nuclei (which divides to form endosperm) / Triple fusion of three
haploid nuclei (1 gamete + 2 polar nuclei) to form a triploid endosperm.

26. (a) Describe the process of microsporogenesis upto the formation of a microspore.

(b) Write the function of 'germ pore' in a pollen grain of an angiosperm. [CBSE 2018]

Ans. (a) The process of formation of microspore from a pollen mother cell by meiosis is called
micro sporogenesis.

Each cell of the sprogenous tissue in microsporangium acts as potential PMC (Pollen mother
cell/ or micro sporemother cell.)

PMC under goes meiotic divisions to form cluster of four cells called microspore tetrad.

On maturity, the anther dehydrates and the microspores separate from each other to form
pollen grains.

Sporogenous tissue Microspore mother cell microspore tetrad

(b) Germ pores allow the germinating / growing pollen tube with contents of the pollen grain/
male gametes + vegetative cell to come out of the pollen grains

E 11
(C) MULTIPLE CHOICE QUESTIONS
1. A typical angiosperm anther is _________ and _________.
(1) Bilobed, tetrasporangiate (2) Bilobed, monosporangiate
(3) Bilobed, bisporangiate (4) Tetralobed, monosporangiate
Ans. (1) Bilobed, Tetrasporangiate
2. The innermost wall layer of anther
(1) Is nutritive in function (2) Helps in dehiscence of anther
(3) Is haploid and protective in function (4) Forms microspores
Ans. (1) Is nutritive in function
3. The process of formation of microspores from a pollen mother cell is called
(1) Megasporogenesis (2) Microsporogenesis
(3) Megagametogenesis (4) Microgametogenesis
Ans. (2) Microsporogenesis
4. The pollen grain represents
(1) Male gamete (2) Male gametophyte
(3) Microsporophyll (4) Microsporangium
Ans. (2) Male gametophyte
5. The most resistant organic material known which makes up the outermost layer of pollen wall is
(1) Pectin (2) Cellulose (3) Sporopollenin (4) Lignin
Ans. (3) Sporopollenin
6. The ploidy level of nucellus and female gametophyte respectively is
(1) n, n (2) n, 2n (3) 2n, n (4) 2n, 2n
Ans. (3) 2n,n
7. The number of nuclei in a mature embryo sac are
(1) Eight (2) Seven (3) Six (4) Four
Ans. (1) Eight
8. The largest cell of the mature embryo sac is
(1) Antipodal cells (2) Synergids (3) Central cell (4) Egg cell
Ans. (3) Central cell
9. The structures which guide the pollen tube into synergid is
(1) Antipodals (2) Germ pore (3) Aril (4) Filiform apparatus
Ans. (4) Filiform apparatus
10. Geitonogamy is
(1) Genetically autogamous (2) Ecologically autogamous
(3) Genetically allogamous (4) Functionally autogamous
Ans. (1) Genetically autogamous
12
11. Which of the following plant provides safe place to insect for laying eggs?
(1) Sage plant (2) Amorphophallus (3) Ophrys (4) Mango
Ans. (2) Amorphophallus
12. Production of seed without fertilization is called
(1) Parthenocarpy (2) Parthenogenesis (3) Apomixis (4) Apogamy
Ans. (3) Apomixis
13. Examples of water pollinated flowers are
(1) Zostera, Lotus, Water lily (2) Lotus, Vallisneria, Hydrilla
(3) Potamogeton, Vallisneria, Lotus (4) Vallisneria, Hydrilla, Zostera
Ans. (4) Vallisneria, Hydrilla, Zostera
14. The central cell after triple fusion becomes the
(1) PEC (2) PEN (3) Endosperm (4) Embryo
Ans. (1) PEC
15. Choose the correct option w.r.t. the function of the germ pore.
(1) It allows growth of pollen tube (2) It allows water absorption in seed
(3) It helps dehiscence of pollen grain (4) More than one option is correct
Ans. (1) It allows growth of pollen tube
16. The thin and continuous wall layer of pollen is
(1) Exine (2) Intine (3) Germ pore (4) Endothecium
Ans. (2) Intine
17. The two-celled stage of mature pollen grain consists of
(1) Vegetative cell, generative cell (2) Vegetative cell, one male gamete
(3) Two male gametes (4) Generative cell, one male gamete
Ans. (1) Vegetative cell, generative cell
18. In 40% angiosperms, the pollen grains are shed at
(1) Four-celled stage (2) Three-celled stage (3) Two-celled stage (4) Five-celled stage
Ans. (2) Three-celled stage
19. Pollen allergy is caused by pollens of
(1) Rose (2) Clematis (3) Parthenium (4) Sunflower
Ans. (3) Parthenium
20. The pollen viability period of rice and pea respectively, is
(1) 30 minutes and several months (2) Several months and 30 minutes
(3) Few days and few months (4) Few days in both the cases
Ans. (1) 30 minutes and several months
13
(D) ASSERTION – REASON QUESITONS
 Directions: In the following questions, a statement of assertion is followed by a statement of
reason. Mark the correct choice as:
(1) If both Assertion and Reason are true and Reason is the correct explanation of Assertion.
(2) If both Assertion and Reason are true but Reason is not the correct explanation of Assertion.
(3) If Assertion is true but Reason is false.
(4) If both Assertion and Reason are false.
1. Assertion : Pollen mother cells (PMCs) are the first male gametophytic cells.
Reason : Each PMC gives rise to two pollens.
Ans (4)
2. Assertion: Gynoecium consists of pistil.
Reason: It represents the male reproductive part in flowering plants.
Ans (3)
3. Assertion: Flowers are the structures related to sexual reproduction in flowering plants.
Reason: Various embryological processes of plants occur in a flower.
Ans (1)
4. Assertion: A typical microsporangium of angiosperms is generally surrounded by four wall
layers.
Reason: The outer three wall layers perform the function of protection and help in dehiscence of
anther to release the pollen.
Ans (2)
5. Assertion: Exine of a pollen grain is made up of sporopollenin which are resistant to high
temperatures, strong acids or alkali as well as enzymatic degradation.
Reason: Sporopollenins are absent in the region of germ pores.
Ans (2)
6. Assertion: An angiospermous flower represents the modified condensed shoot which performs
the functions of sexual reproduction.
Reason: The fertile leaves of the shoot become modified into microsporophylls and
magasporophylls which bear ovules and anthers respectively.
Ans (3)
7. Assertion: In angiosperms, the male gametophyte is the pollen grain.
Reason: Pollen grain contains stigma, style and ovary.
Ans (3)
8. Assertion: Tapetum helps in the dehiscence of microspores from tetrad.
Reason: It shows callose activity.
Ans (1)
9. Assertion: Megaspore mother cell undergoes meiosis to produce four haploid gametes.
Reason: Megaspore mother cell is 2n, meiosis gives haploid structure.
Ans (1)
10. Assertion: Megaspore mother cell undergoes meiotic division.
Reason: All four megaspores form female gametophyte.
Ans (3)
(E) VERY SHORT ANSWER QUESTIONS

1. The diploid number of chromosomes in an angiospermic plant is 16. What will be the
number of chromosomes in its endosperm and antipodal cells ?

Ans. Endosperm - 24 chromosomes, Antipodals - 8 chromosomes.

2. State the reason why pollen grains lose their viability when the tapetum in the anther
malfunctioning.

Ans. Lack of nourishment for the developing pollen grain due to malfunctional tapetum.

3. Can a plant flowering in Mumbai be pollinated by pollen grains of the same species
growing in New Delhi ? Provide explanations to your answer.

Ans. Yes, By artificial means (any relevant explanation)

4. How many microsporangia are present in a typical anther of an angiosperm?

Ans. Four

5. Name the part of the flower which the tassels of the corn-cob represent.

Ans. Style and stigma.

6. Write the function of coleoptile.

Ans. It protects the plumule of the monocot embryo.

7. Name the part of gynoecium that determines the compatible nature of pollen grain.

Ans. Stigma

8. What is shield-shaped single cotyledon of monocots called?

Ans. Scutellum

9. Who discovered double fertilisation in agiosperms?

Ans. S.G. Nawaschin (1897) discovered double fertilisation in angiosperms.

10. Name a plant is which dichogamy is found.

Ans. Magnolia

E 15
(F) SHORT ANSWER QUESTIONS
1. How do plants produce seeds through apomixis?
Ans.
Megaspore mother Free nuclear division
Nucellus (2n) Mitosis cell (2n) Embryo sac (2n)

Seed (2n) Embryo (2n) Without fertilization Egg cell (2n)

2. Why do moss plants produce very large numbers of male gametes? Provide one reason.
What are these gametes called?
Ans. To ensure the fertilization of egg cell, because it depend on water. High numbers of male
gametes confirm the continuity of life. These gametes are called as antherozoid.
3. Explain the role of tapetum in the formation of pollen –grain wall.
Ans. The tapetum forms ubsich bodies sporollenin, which participate in the formation of outer hard
layer of pollen grain, called exine. Tapetum also deposits compatibility proteins in exine as well
as pollen –kitt in insect pollinated pollen grains.
4. Geitonogamous flowering plants are genetically autogamous but functionally pollinated.
Justify.
Ans. In these plants male and female flowers are present on same plant but different branches.
Pollination occur in between these male and female flowers by medium. So, functionally cross
pollination. Genetically pollen grain belongs to the same plant so it is genetically autogamous.
5. Differentiate between Parthenocarpy and Parthenogenesis. Give one example of each.
Ans. S.No. Parthenocarpy Parthenogenesis
1 Formation of fruit without fertilization New organism develops without fertilization
2 -e.g. banana / grapes / any other e.g. Drones /rotifers/male honey bee / turkey
6. A single pea plant in your kitchen garden produces pods with viable seeds, but the
individual papaya plant does not. Explain.
Ans. Pea- flowers of pea plants are bisexual, monoecious / self pollinated (to produce pods with
viable seeds)
Papaya- Dioecious plant / unisexual plant bearing male and female flowers on separate plants,
unable to produce viable seeds as there is no cross pollination / it could be a male plant which is
unable to produce fruit and seeds

16 E
(G) LONG ANSWER QUESTIONS
1. As a senior biology student you have been asked to demonstrate to the students of
secondary level in you school, the procedure(s) that shall ensure cross-pollination in a
hermaphrodite flower. List the different steps that you would suggest and provide reasons
for each one of them.

Ans. The following steps would be followed :


(i) Emasculation or removal of anthers from the flower bud, before the anther dehisce, to avoid
self pollination.
(ii) Bagging, to prevent contamination of its stigma with unwanted pollen grains.
(iii)Rebagging, the stigma of the mature ovary are dustred with desired pollen grains and
rebagged to allow the fruit to develop.
2. (a) Why both wind water pollinated flowers are not very colourful and do not produce
nector.
(b) How do flowers reward their insect pollinators? Explain.
(c) Write the characteristics feature of anther, pollen and stigma of wind pollinated flowers.
Ans. (a) Because there pollinator are abiotic (water and wind) so that do not required any substrate
for attraction of water and wind.
(b) Insect pollinators are rewarded in following ways:
• The flowers offer floral reward like nector and pollen grain.
• In some species floral reward provided safe place to laying egg.
(c) The characteristics of wind pollinated flowers are:
• Pollen grains are light in weight, non sticky, dry and winged, so that they can be easily
transported.
• Well –exposed stamens for easy dispersal of pollen grains in the wind.
• The stigma is sticky, large, feathery to trap the pollen grains in air.
• Numerous flower are packed together to form inflorescence.
• The flowers are small and inconspicuous.
3. A flower of brinjal plant following the process of sexual reproduction procedure 360 viable
seeds. Answer the following questions giving a reasons :
(a) How many ovule minimally involved?
(b) How many megaspore mother cells are involved?
(c) What is the minimum number of pollen grains that must land on stigma for
pollination?
(d) How many male gametes are involves in the above case?
(e) How many microspore mother cells must have undergone reduction division prior to
dehiscence of anther in the above case?

E 17
Ans. (a) 360 ovules are involved. One ovule after fertilization forms one seed.

(b) 360 MMC. Each MMC form four megaspore out of which only one remains functional.
(c) 360 pollen grains .One pollen grains participate in fertilization of one ovule.
(d) 720 male gametes are involved. Each pollen grains carries two male gametes (which
participate in double fertilization) (360 × 2 = 720)
(e) 90 MMC undergoes reduction division. Each microspore mother cells meiotically divided to
form four pollen grains. (360 / 4 = 90)
4. Describe the process of megasporogenesis upto fully developed embryo sac formation in an
angiosperm.
Ans. A single large cell of nucellus with dense cytoplasm and prominent nucleus differentiated as
megaspore mother cell (MMC), in the micropylar region, the megaspore mother cell undergoes
meiosis to form 4 megaspores, 3 cells degenerates and one is functional. Functional megaspore
undergoes three successive mitotic divisions, to develop 8 nucleated (7 celled) embryo sac,
(3 antipodals, one egg cell (female gamete), 2 synergids, two polar nuclei.)

For the above explanation the following diagram can be considered

Micropylar end Micropylar end Micropylar end


Nucellus Nucellus

Megaspore
Megaspore Megasporedyad tetrad
mother cell

(a)
Micropylar end Chalazal end
Antipodals
Synergids
Eggs
Central
cell Polar nuclei
2 polar Central cell
nuclei Egg
Antipodals
Synergids
(a)
Filiform
Micropylar end apparatus
(c)

(a) - Parts of the ovule showing a large megaspore mother cell to tetrad

(b) - 2, 4 and 8 nucleate of embryo sac and mature embryo sac

(c) A diagrammatic representation of the mature embryo sac

18 E
(H) CASE STUDY BASED QUESTION
1. Study the following and answer the questions given below :
Study the given case carefully related to the gametogenesis in angiosperm plants and given the
answer of asked questions.

"Y" "Z"
v v
"X" Cell

(i) Out the right labelling for X, Y and Z into given process of gametogenesis.

Ans. (i) X - megaspore mother cell

Y - Megaspore tetrad

Z - Polar nuclei

(ii) How many meiotic and mitotic division are required for complete given gametogenesis

process?

Ans. Meiotic and 3 Mitotic division

(iii) How many nucleus and cells are present in a typical mature angiosperm endosperm?

Ans. 8 nucleus, 7 cells

(iv) State the function of filiform apparatus found in mature embryo sac of an angiosperm.

Ans. Guiding the entry of pollen tubes up to the synergids.

2. Study the following and answer the questions given below:


The given below are types of pollination, study it and given the answer of asked question.
Subject-1 Subject-2
Method-"X" Method-"Y"

Plant-A Plant-B
Plant-A

E 19
(i) What are appropriate terms for method X and Y of subject-1 and subject-2 respectively.
Ans. (i) X  Geitonogamy, Y Xenogamy,
(ii) Differentiate between geitonogamy and xenogamy.
Ans.
Geitonogamy Xenogamy
Transfer of pollen grains Transfer of pollen grains from
from anther of one flower anther of one flower to stigma
to the stigma of another of another flower of
flower on "Same Plant". a. different plant.
(iii) Out of many Papaya plants growing in your garden, any a few bear fruits. Give reason.
Ans. The individual papaya plant is prevented from both autogamy and geitonogamy. In this plants
male and female flowers are present on different plants, eg.,  each plants is either male or
female.
(iv) If the plant - A and plant - B are belong from the grass family, than which of the following
will be work as pollination agent?
Ans. Wind
3. Read the following and answer the questions given below:
The gynoecium represents the female reproductive part of the flower. The gynoecium may
consist of a single or more than one pistil. They may be fused or may be free. The placenta is
located inside the ovarian cavity. Megasporangium (ovule) consists of a small structure attached
to the placenta by a stalk called a funicle. The body of the ovules fuses with a funicle in the
region called hilum. The nucleus is located in the embryo sac. The process of formation of,
megaspore from the megaspore mother cell is called Megasporogenesis. Meiosis result in the
formation of four megaspore.
(i) Define the term monocarpellary and Multicarpellary.
Ans. Monocarpellary - Occurrence of single pistil.
Multicarpellary - Occurrence of more than one pistil
(ii) Write the appropriate term for fused and free conditions of pistil.
Ans. Fused Pistil- Syncarpous, Free Pistil- Apocarpous
(iii) Write the name of parts of pistil.
Ans. Stigma, Style and Ovary
(iv) How many microspore mother cells must have undergone reductional division to produce
400 microspore.
Ans. 100 Microspore Mother Cells
(v) What do you means by monosporic development of female gametophytes?
Ans. Out of megaspore tetrad, one of the megaspore is functional while the other three degenerated.
The functional megaspore develops into female gametophyte this is called as monosporic
development.
20 E
4. Read the following and answer the questions given below:-

In angiosperm, the seed is the final product of sexual reproduction. It is described as a fertilized
ovule. The seeds are formed inside the fruit. The seed consists of a seed coat, cotyledon, and the
embryo axis. A mature seed is usually non - albuminous or albuminous. Integument of ovules
harder as tough protective seed coat. Sometimes due to reduced water content, the general
metabolic activity of the seed slows down and the seed enters a state of inactivity. In the mature
plant, the fruit develops from the ovary they are called true fruit. The fruit is the result of
fertilization. There are a few species in which fruit develop without fertilization banana is such
an example.

(i) What are non-albuminous and albuminous seeds?


Ans. Non- albuminous- Do not have residual endosperm as it is completely consumed during embryo
development. Albuminous- The seeds retains a part of endosperm as it is not completely used up
during embryo development.
(ii) What is perisperm ?
Ans. In some seeds such as black pepper and beet, remnants of nucellus are persistent called as
perisperm.
(iii) What is the fate of integument of ovule ?
Ans. After fertilization the integuments of ovule converted into seeds coat testa and tagmen.
(iv) What are false and parthenocarpic fruits?
Ans. False fruit- The fruits which develops rather than ovary.e.g.; Apple
Parthenocarpic fruit- The fruits which develops without fertilization.e.g.; Banana
(v) Write the appropriate term for condition of seeds where water content reduced, metabolic
activity slow down and seeds enters a state of inactivity.
Ans. Dormancy

21
IMPORTANT NOTE

_______________________________________________________________________
_______________________________________________________________________
_______________________________________________________________________
_______________________________________________________________________
_______________________________________________________________________
_______________________________________________________________________
_______________________________________________________________________
_______________________________________________________________________
_______________________________________________________________________
_______________________________________________________________________
_______________________________________________________________________
_______________________________________________________________________
_______________________________________________________________________
_______________________________________________________________________
_______________________________________________________________________
_______________________________________________________________________
_______________________________________________________________________
_______________________________________________________________________
_______________________________________________________________________
_______________________________________________________________________
_______________________________________________________________________
_______________________________________________________________________
_______________________________________________________________________

_______________________________________________________________________

_______________________________________________________________________
_______________________________________________________________________
CHAPTER 2
(A) NCERT QUESTIONS & SOLUTIONS
1. Fill in the blanks :
(a) Humans reproduce ----------------. (asexually/sexually)
Ans. Sexually
(b) Human are -------------------. (oviparous, viviparous, ovoviviparous)
Ans. Viviparous
(c) Fertilization is------------------in human. (external/internal)
Ans. Internal
(d) Male and female gametes are--------.(diploid/haploid)
Ans. Haploid
(e) Zygote is--------------------------. (diploid/haploid).
Ans. Diploid
(f) The process of release of ovum from a mature follicle is called --------------------------.
Ans. Ovulation
(g) Ovulation is induced by a hormone called --------------------------.
Ans. Luteinising hormone (LH)
(h) The fusion of male and female gametes is called --------------------------.
Ans. Fertilisation
(i) Fertilisation takes place in --------------------------.
Ans. Ampulla of the oviduct
(j) Zygote divides to form -------------------------- which is implanted in uterus.
Ans. Blastocyst
(k) The structure which provides vascular connection between foetus and uterus is called
--------------------------.
Ans. Placenta
2. Draw a labelled diagram of male reproductive system.
Ans.
Ureter Urinary
bladder
Vas deferens Seminal
vesicle
Prostate
Bulbourethral
gland
Epididymis
Vasa efferentia Urethra
Rete testis

Testicular lobules Testis

Foreskin
Glans penis

Diagrammatic view of male reproductive system

23
3. Draw a labelled diagram of female reproductive system.
Ans.
Uterine fundus
Uterine cavity
Isthmus
Fallopian
Ampulla tube
Infundibulum

Endometrium Ovary
Myometrium
Body of Fimbriae
Perimetrium uterus
Cervix

Cervical canal

Vagina

Diagrammatic view of female reproductive system

4. Write two major functions each of testis and ovary.


Ans. (1) Functions of the Testis :
They produce male gametes (sperm) by the process of called spermatogenesis.
The leydig cells of the seminiferous tubules secrete the male sex hormone called
testosterone.
(2) unctions of the ovary :
They produce female gametes (ovum) by the process of oogenesis.
The growing Graafian follicles secrete the female sex hormone called estrogen.
5. Describe the structure of a seminiferous tubule. [IMP.]
Ans. A seminiferous tubule is made up of layer of male germ cells and large Sertoli cells
(nurse cell).
The male germ cells undergo spermatogenesis to produce spermatocytes, spermatids and
sperms.
The regions outside the seminiferous tubules called interstitial spaces have connective tissue
which included blood vessels and leydig cell.
Leydig cells synthesise and secrete the male sex hormones called androgens, of which
testosterone is the principle one.
Spermatozoa

Spermatid

Secondary
spermatocyte
Primary
Spermatocyte

Sertoli cell

Spermatogonium

Diagrammatic sectional view of a seminiferous tubule

24 E
6. What is spermatogenesis? Briefly describe the process of spermatogenesis.
Ans. It is the process of formation of haploid sperms in seminiferous tubules of testes.
Primordial germ cells with
2n diploid chromosome

Multiplication phase
2n 2n Spermatogonia

Spermatocytogenesis
2n 2n 2n 2n

Growth
Primary
phase
spermatocytes
2n Meiosis-I
Secondary
n n spermatocytes
Maturation

Meiosis-II
phase

n n n n
Spermiogenesis

Spermatid

Sperm

Process of spermatogenesis

7. Name the hormones involves in regulation of spermatogenesis.


Ans. GnRH (Gonadotropin releasing hormone), LH (Luteinising hormone), FSH (Follicle stimulating
hormone), androgens.
8. Define spermiogenesis and spermiation.
Ans. (1) Spermiogenesis : The process of transformation of spermatids into spermatozoa in
seminiferous tubules, is called spermiogenesis.
(2) Spermiation : Sperm are finally released from the seminiferous tubules by the process
called spermiation.
9. Draw a labelled diagram of sperm. [IMP.]
Ans.
Plasma
membrane
Acrosome
Head Nucleus containing
chromosomal material
Centriole
Neck
Manchette
Middle
piece Mitochondria (Nebenkern Sheath)
energy source for swimming)

Tail (Longest part)


Axonema
(Axial filament)

Structure of a sperm
25
10. What are the major components of seminal plasma?
Ans. The major components of seminal plasma are :
Secretions of the accessory sex glands of males – prostate gland, seminal vesicles, bulbourethral
glands.
Mainly composed of – calcium, fructose, and other enzymes
11. What are the major functions of male accessory ducts and glands?
Ans. The major functions of the male accessory ducts and glands are as follows:
Male accessory ducts
(a) Vasa efferentia : Conducts sperms from the rete testis to the epididymis.
(b) Rete testis : Stores sperms that are produced by seminiferous tubules.
(c) Epididymis : Physiological maturation of sperms, storage and nourishment.
(d) Vas deferens : Conducts sperms from epididymis to the urethra.
Male accessory glands
(a) Seminal vesicles : Activates and provides energy to facilitate sperm motility after
ejaculation.
(b) Prostate gland : Nourishes and activates sperms, enhances sperm motility.
(c) Cowper’s gland : It enhances mobility & survival potentiality of sperms in the genital tract
of female reproductive system and neutralizes the activity of acidic female vaginal
secretions.
12. What is oogenesis? give a brief account of oogenesis. [IMP.]
Ans. Oogenesis is the phenomenon of formation of haploid female gametes known as ova from diploid
oogonia in the ovary is called as oogenesis. Oogenesis can be explain through following flow
chart.
13. Draw a labelled diagram of a section through ovary.
CHROMOSOME
NUMBER
PER CELL
Oogonia
46
Mitosis
differentiation
Fetal life
Primary oocyte
Ist meiotic
division
Birth (completed
Childhood prior to
Puberty
23 ovulation)
First Secondary
Adult polar body oocyte
Reproductive
life Ovum
Second
23 polar body

Schematic representation of Oogenesis

26 E
Ans. The diagram of a section of an ovary is as follows -
Primary Tertiary follicle
Blood follicle Showing antrum
Vessels Graafian
follicle

Ovum

Corpus
luteum

Diagrammatic section view of ovary

14. Draw a labelled diagram of a Graafian follicle.


Ans. The diagram of a Graafian follicle is as follows :
Granulosa cells
Cumulus oophorus

Theca externa
Theca interna

Antrum
(filled with liquor folliculi) Secondary oocyte

Blood vessels

Discus proligerous

Mature Graafian follicle

15. Name the functions of the following :


(a) Corpus luteum (b) Endometrium (c) Acrosome
(d) Sperm tail (e) Fimbriae
Ans. The functions of each are as follows :
(a) Corpus luteum – The corpus luteum secretes the hormone progesterone during the luteal
phase of the menstrual cycle.
(b) Endometrium –The endometrium is the innermost layer of the uterus comprising of glands
that undergoes cyclic changes during different stages of the menstrual cycle in order to
prepare itself for the embryo-implantation process.
(c) Acrosome - It consists of the sperm lysin enzyme that helps in penetration of the outer
membrane of the egg by sperm which facilitates the sperm to perforate through the egg
during fertilization.

E 27
(d) Sperm tail – The sperm tail makes up for the longest part of the sperm, enabling the

movement of the sperm, once it has entered the female reproductive tract.

(e) Fimbriae – Towards the ovarian end of the fallopian tube, finger-like projections emerge,

these are the fimbriae which assist in collection of the ovum after the ovulation.

16. Identify True/False statements. Correct each false statement to make it true.

(a) Androgens are produced by Sertoli cells. (True/False)

Ans. False, Correct statement : Androgens are produced by the Leydig cells.

(b) Spermatozoa get nutrition from Sertoli cells. (True/False)

Ans. True

(c) Leydig cells are found in ovary. (True/False)

Ans. False, Correct statement : Leydig cells are found in testes.

(d) Leydig cells synthesise androgens. (True/False)

Ans. True

(e) Oogenesis takes place in corpus luteum. (True/False)

Ans. False, Correct statement : Oogenesis takes place in ovary.

(f) Menstrual cycle ceases during pregnancy. (True/False)

Ans. True

(g) Presence or absence of hymen is not a reliable indicator of virginity or sexual experience.

(True/False)

Ans. True

17. What is menstrual cycle ? Which hormones regulate menstrual cycle?

Ans. It is a reproductive cycle in the female primates (e.g. monkeys, apes and human beings), in

females menstruation is repeated at an average interval of about 28/29 days, the cycle of events

starting from one menstruation till the next one is called the menstrual cycle.

The different hormones that regulate the menstrual cycle are LH – Luteinising hormone,

FSH – Follicle stimulating hormone, progesterone, estrogen.

28 E
18. What is parturition? Which hormones are involved in induction of parturition?

Ans. Parturition is the process wherein a fully developed foetus from the mother’s womb is expelled

out after the completion of the gestation period. The two critical hormones are involved in the

induction of parturition are :

Oxytocin – It directs the full-term foetus towards the birth canal, as it causes the contraction of

the smooth muscles of the myometrium of the uterus leading the baby to be expelled out.

Relaxin – It relaxes the pelvic ligaments, widening the pelvis to assist in an easier child birth.

19. In our society the women are often blamed for giving birth to daughters. Can you explain

why this is not correct? [IMP.]

Ans. This is not correct to blame women for giving birth to daughter.

The male sperm contain either X or Y chromosome whereas the female egg contain only

X chromosome.

At the time of fertilization, sperm with Y chromosome combine with egg containing

X chromosome formed which would be male.

Thus scientifically sex of the baby is determined by the father and not by the mother as

blamed in our society.

20. How many eggs are released by a human ovary in a month? How many eggs do you think

would have been released if the mother gave birth to identical twins? Would your answer

change if the twins born were fraternal?

Ans. Typically in a month, human ovaries release only one egg, rarely two. In case of identical twins

or monozygotic twins one egg is released by the ovary which splits into two post fertilization.

This is why identical twins exhibit the same genetic features.

On the other hand in fraternal twins or dizygotic twins, two eggs are released which are fertilized

by two different sperms causing the fraternal twins to exhibit different genetic characters.

21. How many eggs do you think were released by the ovary of a female dog which gave birth

to 6 puppies?

Ans. In order to have given birth to six puppies, the ovary of the female dog released six eggs. Hence,

six zygotes were formed for each to develop into a puppy.

29
(B) PREVIOUS YEAR QUESTIONS
1. Given below are structural details of a human mammary gland : [CBSE 2023]
(i) The glandular tissue in the breast has 15-20 clusters of cells called alveoli.
(ii) The milk is stored in the lumen of alveoli.
(iii) The alveoli join to form the mammary ducts.
(iv) Mammary ampulla is connected to lactiferous ducts.
Choose the option that gives the correct detail of human mammary gland.
(1) (i) and (ii) (2) (ii) and (iii) (3) (ii) and (iv) (4) (i) and (iii)
Ans. (3) (ii) and (iv)
2. The graph given below shows the number of primordial follicles per ovary in women at different
ages. Study. the graph and answer the questions that follow. [CBSE 2023]
Primordial follicles/ovary

100,000 Premenopausal Women (regular menses)


10,000
Perimenopausal Women
1000
(irregular menses for atleast one year)
100
Post menopausal Women
10 (No menses for atleast one year)
1

0
10 20 30 40 50 60

Age (y)
(a) What is the average age of the women at the onset of menopause?
(b) At what age are maximum primordial follicles present in the ovary, according to the given
graph?
Ans. (a) 45-50 year (b) 0-10 year
3. (i) Explain the formation of placenta after the implantation in a human female.

(ii) Draw a diagram showing human foetus within the uterus and label any four parts in
its. [CBSE 2023]
Ans. (i) After implantation, finger-like projections appear on the trophoblast called chorionic villi
which are surrounded by the uterine tissue and maternal blood.
 The chorionic villi and uterine tissue become interdigitated with each other and jointly
form a structural and functional unit between developing embryo (foetus) and maternal
body called placenta
(ii) The human foetus within the uterus.
The human foetus within the uterus
4. Breast-feeding the baby acts as a natural contraceptive for the mother because it prevents:
[CBSE Term-I 2022]
(i) Ovulation (ii) Menstruation (iii) Insemination (iv) Fertilisation
(1) (ii) and (iv) (2) (i) and (iii) (3) (i) and (iv) (4) (i) and (ii)
Ans. (4) (i) and (ii)
5. The given figure shows the different stages of human embryo [CBSE Term-I 2022]

(Y) (Z) (X) (W)

Identify the correct labelings for W, X, Y and Z and choose the correct option from the table
below.
W X Y Z
(1) Cleavage Blastocyst Morula Fertilisation
(2) Blastocyst Morula Cleavage Fertilisation
(3) Morula Cleavage Blastocyst Fertilisation
(4) Morula Blastocyst Cleavage Fertilisation
Ans. (3)
W-Morula X-Cleavage Y-Blastocyst Z-Fertilisation
6. During human embryonic development the external genital organs are well developed in the
foetus by the end of – [CBSE Term-I 2022]
(1) 6 weeks of pregnancy (2) 12 weeks of pregnancy
(3) 18 weeks of pregnancy (4) 24 weeks of pregnancy
Ans. (2) 12 weeks of pregnancy
7. The accessory ducts in the human male reproductive system consists of [CBSE Term – I 2022]
(1) Epididymis, Prostate, Rete testis (2) Rete testis, Vas efferentia, Seminal vesicles
(3) Vas efferentia, Bulbourethral, Epididymis (4) Rete testis, epididymis, Vas deferens
Ans. (4) Rete testis, epididymis, Vas deferens
31
8. The source of gonadotropin LH and its corresponding function is: [CBSE Term-I 2022]
(1) Anterior pituitary, ovulation (2) Anterior pituitary, Graafian follicle formation
(3) Hypothalamus, Ovulation (4) Hypothalamus, Graafian follicle formation
Ans. (1) Anterior pituitary, ovulation

CASE BASED QUESTIONS


9. A women of 35 years age with a married life of eight years and having normal reproductive
cycles visits a doctor along with her husband for consultation for infertility. They were not using
any contraceptive methods. They have no child. The doctor advises them after a detailed physical
examination of both of them to undergo following investigations:
– Seminal analysis of the husband.
– Follicular study of the wife.
– Blood test for follicle Stimulating Hormone (FSH) estimation for both
With your basic knowledge of human embryology and the case given above, answer the
following questions:
(i) Seminal analysis of the husband was done for determining [CBSE Term-I 2022]
(i) Sperm morphology
(ii) Quantity and pH of semen
(iii) Rate of sperm release into the Vagina
(1) (i) only (2) (i) and (ii) (3) (ii) and (iii) (4) (ii) only
Ans. (2) (i) and (ii)
(ii) An ultrasound – guided follicular study was done for the wife for determining the size and
physical appearance of the [CBSE Term-I 2022]
(1) Ovary (2) Oogonia (3) Antral follicles (4) Corpus Luteum
Ans. (1) Ovary
(iii) The blood test report of the wife showed low FSH value, which is indicative of –
[CBSE Term-I 2022]
(1) low rate of formation of ovarian follicles (2) high rate of formation of ovarian follicles
(3) low rate maturation of ovarian follicles (4) high rate of maturation of ovarian follicles
Ans. (1) low rate of formation of ovarian follicles
(iv) In the above case if the husband is found to have sperm count of less than 20 million/mL and the
wife is diagnosed with blockage in the oviduct, the couple would be advised for:
[CBSE Term-I 2022]
(i) ZIFT (ii) AI (iii) IVF (iv) ICSI
(1) (i) and (iii) (2) (ii) and (iii) (3) (iii) and (iv) (4) (i) and (iv)
Ans. (1) (i) and (iii)
(v) The high level of which gonadotropin/ovarian hormone in the blood sample of the wife taken on
day 20 of her reproductive (menstrual) cycle would indicate the letual phase of the ovarian cycle?
[CBSE Term-I 2022]
(1) FSH (2) LH (3) Estrogens (4) Progesterone
Ans. (4) Progesterone
32 E
(vi) In which phase of the menstrual cycle is the blood sample of a women taken if, on analysis, it
shows high levels of L.H. and estrogen? [CBSE Term-I 2022]
(1) Ovulatory phase (2) Menstrual phase (3) Secretory phase (4) Follicular phase
Ans. (1) Ovulatory phase
10. Name the glands that contribute to human seminal plasma. [CBSE IMP-Question]
Ans. Prostate, Seminal vesicle and Bulbourethral gland. (any two)
11. A fully developed foetus initiates its delivery from the mother’s womb. Justify the
statement. [CBSE IMP-Question]
Ans.  The signals for parturition originate from the fully developed foetus and the placenta which
induce mild uterine contractions called foetal ejection reflex.
 This triggers the release of oxytocin from the maternal pituitary.
 Oxytocin acts on the uterine muscle and causes stronger uterine contractions, which in turn
stimulates further secretion of oxytocin.
 The stimulatory reflex between the uterine contraction and oxytocin secretion continues
resulting in stronger and stronger contractions.
 Parturition is induced by a complex neuroendocrine mechanism involving cortisol,
oestrogens and oxytocin.
 This leads to expulsion of the baby out of the uterus through the birth canal – parturition.
12. Study the graph given below related with menstrual cycle in females:[CBSE IMP-Question]
a. Identify ovarian hormones X and Y mentioned in the graph and specify their source.

Ovulation
Hormone levels
Ovarian

X
Y
Day
1 3 5 7 9 11 13 15 17 19 21 23 25 27 29/1

b. Correlate and describe the uterine events that take place according to the ovarian
hormone levels X and Y mentioned in the graph on -
i. 6 – 15 days
ii. 16 – 25 days
iii. 26 – 28 days (when ovum is not fertilized)
33
Ans. a. X- Estrogen secreted by growing follicles;
Y – Progesterone secreted by corpus luteum
b. Uterine events that take place according to the ovarian hormone levels X and Y on -
(i) 6 – 15 days: Endometrium of the uterus regenerates by proliferation under the influence of estrogen.
(ii) 16 – 25 days: Under the influence of Progesterone the endometrium of the uterus is maintained
for implantation of fertilised ovum and other events of pregnancy.
(iii) 26 – 28 days (when ovum is not fertilized): in the absence of fertilisation, corpus luteum
degenerates which causes disintegration of endometrium leading to menstruation, marking a
new cycle.
13. The following figure shows a foetus within the uterus. On the basis of the given figure,
answer the questions that follow: [CBSE IMP-Question]

(a) In the above figure, choose and name the correct part (A, B, C or D) that act as a
temporary endocrine gland and substantiate your answer. Why is it also called the
functional junction?
(b) Mention the role of B in the development of the embryo.
(c) Name the fluid surrounding the developing embryo. How is it misused for sex-
determination?
Ans. (a) Part labeled A -Placenta. It acts as an endocrine tissue as it produces several hormones like
human chorionic gonadotropin (hCG), human placental lactogen (hPL),estrogens,
progestogens, etc. It is also called the functional junction because it facilitates the supply of
oxygen and nutrients to the embryo and removes carbon dioxide and excretory/waste materials
produced by the embryo.
(b)The placenta is connected to the embryo through an umbilical cord which helps in the
transport of substances to the embryo.
(c) Amniotic fluid; a foetal sex determination test is based on the chromosomal pattern of the
cells in the amniotic fluid surrounding the developing embryo.
14. Study the given diagram where A is an embryonic stage that gets transformed into B, which
in turn gets implanted in the endometrium in human females. [CBSE 2020]

Transform into
 

(a) Identify A, B and its parts C and D.


(b) Write the name of cavity present in the stage B.
(c) State the fate of C and D in the course of embryonic development in human.
Ans. (a) A - Morula,
B - Blastocyst,
C - Inner cell mass,
D - Trophoblast
(b) Blastocoel
(c) C-It differentiated as the embryo.
D-It helps in implantation and further develop into placenta.
15. Given below is the diagram of a human ovum surrounded by a few sperms. Study the
diagram and answer the following questions: [CBSE 2019]

B
C
A

D
Ovum

E
(a) Which one of the sperms would reach the ovum earlier?
(b) Identify ‘D’ and ‘E’.
(c) Mention what helps the entry of sperm into the ovum and write the changes occurring
in the ovum during the process.
(d) Name the specific region in the female reproductive system where the event
represented in the diagram takes place.
Ans. (a) Sperm ‘A’
(b) D = Cells of corona radiata, E = zona pellucida,
(c) The secretion of acrosome (enzymes) helps the entry of sperm into the ovum, induces the
completion of meiotic division of secondary oocyte.
(d) Ampulla /ampullary - isthmic junction of the fallopian tube
16. Medically it is advised to all young mothers that breast feeding is the best for their newborn
babies. Do you agree ? Give reasons in support of your answer. [CBSE 2018]
Ans. Yes, The milk produced during the initial few days of lactation is called colostrum.
The colostrum contains several antibodies (IgA) absolutely essential to develop resistance for the
new-born babies. Breast-feeding during the initial period of infant growth is recommended by
doctors for bringing up a healthy baby.
17. State from where do the signals for parturition originate in human females. [CBSE 2017]
Ans Fully developed foetus, and the placenta.

35
TG: @Chalnaayaaar
(C) MULTIPLE CHOICE QUESTIONS
1. Semen is constituted of –
(1) Sperm (2) Spermatogonia (3) Seminal Plasma (4) Both (1) & (3)
Ans. (4) Both (1) & (3)
2. Seminiferous tubules are composed of
(1) Spermatogonia (2) Glandular epithelium
(3) Sensory epithelium (4) Germinal epithelium
Ans. (4) Germinal epithelium
3. Cells of Leydig occur in
(1) Liver (2) Ovary (3) Testis (4) Spleen
Ans. (3) Testis
4. Which piece of a sperm is called power house ?
(1) Head piece (2) Neck piece (3) Middle piece (4) Tail piece
Ans. (3) Middle piece
5. Which is not a secondary sex organ ?
(1) Vagina (2) Penis (3) Prostate (4) Mammary gland
Ans. (4) Mammary gland
6. At puberty woman start producing
(1) Sperms (2) Urine (3) Young ones (4) Ovum
Ans. (4) Ovum
7. Release of sperm from testes is called :-
(1) Spermiation (2) Semination (3) In semination (4) Ejaculation
Ans. (2) Semination
8. Which part of the spermatid forms acrosome of sperm?
(1) Mitochondria (2) Golgi body (3) Nucleus (4) Lysosome
Ans. (2) Golgi body
9. Polar body is produced during the formation of :
(1) Sperm (2) Secondary oocyte (3) Oogonium (4) Spermatocyte
Ans. (2) Secondary oocyte
10. First meiotic division during oogenesis occurs in :
(1) First polar body (2) Second polar body (3) Primary oocytes (4) Secondary polar
Ans. (3) Primary oocyte
36 E
11. Humans are ____ and ____ organisms.
(1) Sexually reproducing, oviparous (2) Asexually reproducing, ovoviviparous
(3) Asexually reproducing, viviparous (4) Sexually reproducing, viviparous
Ans. (4) Sexually reproducing, viviparous
12. The formation of gametes is termed as
(1) Gametogamy (2) Syngamy (3) Gametogenesis (4) Gestation
Ans. (3) Gametogenesis
13. The transfer of sperms into the female genital tract is called
(1) Insemination (2) Gametogenesis (3) Fertilization (4) Gestation
Ans. (1) Insemination
14. The fusion of male and female gametes is known as
(1) Insemination (2) Fertilization (3) Implantation (4) Parturition
Ans. (2) Fertilization
15. The following statements are true except
(1) In an individual, reproductive changes occur after puberty.
(2) Sperm formation occurs even in old men.
(3) Formation of ovum continues in women after fifty years.
(4) Humans are sexually producing and viviparous
Ans. (3) Formation of ovum continues in women after fifty years.
16. The testes are situated ____ the abdominal cavity within a pouch called ____.
(1) inside, testicular lobules (2) outside, scrotum
(3) outside, vas deferens (4) inside, scrotum
Ans. (2) outside, scrotum
17. The scrotum helps in maintaining a temperature ____ lower than the internal body temperature.
(1) 1 to 1.5°C (2) 2 to 2.5°C (3) 3 to 3.5°C (4) 4 to 4.5°C
Ans. (2) 2 to 2.5°C
18. Male germ cells are known as
(1) Sperms (2) Spermatogonia (3) Spermatid (4) Sertoli cells
Ans. (2) Spermatogonia
19. Seminiferous tubules contain ____ for providing nutrition to sperm cells.
(1) Leydig cells (2) Interstitial cells (3) Sertoli cells (4) Germ cells
Ans. (3) Sertoli cells
20. The cells which secrete androgens are
(1) Spermatozoa (2) Interstitial cells (3) Sertoli cells (4) Germ cells
Ans. (2) Interstitial cells
E 37
(D) ASSERTION – REASON QUESTIONS
 Directions: In the following questions, a statement of assertion is followed by a statement of
reason. Mark the correct choice as:
(1) If both Assertion and Reason are true and Reason is the correct explanation of Assertion.
(2) If both Assertion and Reason are true but Reason is not the correct explanation of Assertion.
(3) If Assertion is true but Reason is false.
(4) If both Assertion and Reason are false.
1. Assertion : In human male, testes are extra abdominal and lie in scrotal sacs.
Reason : Scrotum acts as thermo regulator and keeps testicular temperature lower by 2-2.5°C for
normal spermatogenesis.
Ans (1)
2. Assertion : Testicular lobules are the compartments present in testes.
Reason : These lobules are involved in the process of fertilization.
Ans (3)
3. Assertion : Interstitial cell is present in the region outside the seminiferous tubule called
interstitial spaces.
Reason : Interstitial cells provide nutrition to the sertoli cells.
Ans. (3)
4. Assertion: The testes are situated outside the abdominal cavity within the scrotum.
Reason: Muscles in scrotum helps to maintain low temperature of testes, necessary for
spermatogenesis.
Ans (1)
5. Assertion: The bulbourethral gland is a male accessory gland.
Reason: Its secretion helps in the lubrication of the penis, thereby facilitating reproduction.
Ans (1)
6. Assertion: Human male ejaculates about 200 to 300 million sperms during coitus.
Reason: Only few reach the isthmus ampullary junction for process of fertilisation.
Ans (1)
7. Assertion: Epididymis is divided into three parts.
Reason: Epididymis is the organ that stores spermatozoa
Ans (2)
8. Assertion: Graafian follicle ruptures at the mid of menstrual cycle releasing the ovum.
Reason: Both LH and FSH attain a peak level at the middle of cycle.
Ans (1)
9. Assertion: Progesterone is required for maintenance of the endometrium.
Reason: Endometrium is essential for implanation of embryo.
Ans (2)
10. Assertion: The endometrium undergoes cyclical changes during menstrual cycle.
Reason: The myometrium exhibits strong contractions during delivery of the baby.
Ans (2)

38 E
(E) VERY SHORT ANSWER QUESTIONS
1. Write the function of FSH.

Ans FSH acts on sertoli cells and stimulates the secretion of some factors like ABP = androgen

binding protein which helps in spermatogenesis.

2. Write names of enzymes present in the secretion of prostate gland.

Ans Fibrinolysin, spermin, fibrinogenase.

3. How is 'oogenesis' markedly different from 'spermatogenesis' with respect to the growth till

puberty in the humans?

Ans. Oogenesis is initiated at the embryonic stage.

Spermatogenesis begins only at puberty.

4. Where is acrosome present in humans? Write its function?

Ans It is present at the tip or head of the sperm. It helps to penetrate through the zona pellucida in

human ovum.

5. Write the function of each one of the following:

(a) (Oviducal) Fimbriae

(b) Oxytocin

Ans (a) Collection of ovum released by ovary.

(b) Causes uterine contraction for parturition/promotes milk ejection.

6. What is the number of chromosomes in the following cells? Primary oocyte, secondary

oocyte, ootid and follicle.

Ans. The number of chromosome in the cells is as follows:

Primary oocyte: 23 pairs. Secondary oocyte: 23. Ootid: 23. Follicle: 23 pairs.

7. What is corpus luteum. How dose it functions as endocrine gland ?

Ans. After ovulation, the Graafian follicle ruptures & forms corpus luteum. Corpus luteum functions

as endocrine glands as they secrete progesterone & estrogen in large quantities.

E 39
(F) SHORT ANSWER QUESTIONS
1. Give the function of
(a) Corpus luteum (b) Endometrium
Ans. Corpus luteum : It secretes progesterone which prepares endometrium of uterus for implantation
and normal development of foetus.
Endometrium : It undergoes cyclic changes during menstrual cycle and prepares itself for
implantation of blastocyst.
2. Give reason for the following :
(a) The first half of the menstrual cycle is called follicular phase as well as proliferative phase.
(b) The second half of the menstrual cycle is called luteal phase as well as secretory phase.
Ans. (a) During this phase, primary follicles transform into Graafian follicle under FSH stimulation.
Graafian follicles secrete estrogens with stimulate enlargement of Endometrium of uterus.
(b) During this phase, Corpus luteum is fully formed and secretes large quantity of Progesterone.
3. What is meant by L.H. Surge ? Write the role of L.H.
Ans. LH surge refers to maximum level of luteinising hormone middle of menstrual cycle.
LH cause ovulation.
4. Explain significance of the condition in which the testes remain suspended in scrotum
outside the abdomen.
Ans. Human sperm cells cannot develop at body temperature. Spermatogenesis and maintenance of
the seminiferous tubules requires a temperature slightly lower than that of the body. This is
provided by the scrotum, which lies outside the abdominal cavity.
5. Describe the structure of a sperm with a diagram.
Ans. The human sperm is a microscopic structure with a head, middle piece and a tail. The head has
the haploid nucleus and an anterior acrosome that contains the enzymes required for the
fertilization of the egg. The middle piece has numerous mitochondria to produce the energy for
the mobility of the tail of the sperm.
6. Enlist any two functions of a female placenta.
Ans. The structural and the functional unit between the developing embryo and the mother called
placenta facilitates the supply of nutrients, oxygen to the embryo and also the removal of carbon
dioxide and other excretory products produced by the embryo. It also acts as endocrine tissue and
produces several hormones.
40 E
(G) LONG ANSWER QUESTIONS
1. (a) Explain the menstrual phase in a human female. State the levels of ovarian and

pituitary hormones during this phase.

(b) Why is follicular phase in the menstrual cycle also referred as proliferative phase?

Explain.

(c) Explain the events that occur in a Graafian follicle at the time of ovulation and there

after.

Ans. (a) Menstrual phase occurs when released ovum not fertilized, break down of endometrial lining

(of the uterus) and its blood vessel form the liquid that comes out through the vagina, lasts

for 3 to 5 days,

Note - Low level of ovarian and pituitary hormones.

(b) In the ovary follicular development takes place and in the same phase the regeneration of

endometrium through proliferation takes place inside uterus.

(c) Graafian follicle ruptures to release the ovum (secondary oocyte), remaining parts of the

Graafian follicle transform into corpus luteum.

2. Mention the name and role of hormones which are involved in regulation of gamete

formation in human male.

Ans. GnRH : Stimulates adenohypophysis to secrete gonadotropins.

GSH : Stimulates Sertoli cells to secrete factors while help in spermatogenesis.

ICSH : Stimulates interstitial cells to secrete testosterone.

3. Differentiate between spermatogenesis and oogenesis.

Ans. Spermatogenesis :

1. It occurs inside the testes.

2. All the stages are completed inside the testes.

3. Spermatogonia develop from the germinal epithelium lining in the seminiferous tubules.

4. All spermatogonia give rise to spermatocytes.

5. Primary spermatocytes divide by meiosis I to give rise to two secondary spermatocytes.

6. Secondary spermatocyte divides by meiosis

E 41
Oogenesis :
1. It occurs inside the ovary.
2. Majority occurs inside the ovary but last stages occur in the oviduct.
3. Oogonia develop from the germinal epithelium overlying the ovary.
4. Only few oogonia give rise to oocytes.
5. Primary oocyte undergoes meiosis I to give rise to one secondary oocyte and a polar body.
4. Answer the following questions.
(i) State the levels of FSH, LH and Progesterone simply by mentioning high or low around
13th and 14th day and 21st to 23rd day.
(ii) In which of the above mentioned phases does egg travel to fallopian tube ?
(iii) Why there is no mensuration after fertilisation ?
Ans. (i) 13-14th day 21st -23rd day
FSH - High Low
LH - High Low
Progesterone - Low High
(ii) End of follicular or proliferative phase.
(iii) Menstruation does not occur during pregnancy upon fertilization due to high level of
progesterone secreted by persisting corpus luteum and Placenta.
5. (a) Read the graph given below. Correlate the ovarian events that take place in the human
female according to the level of the pituitary hormone during the following day.
(i) 10th - 14th days (ii) 14th -15th days
(iii) 16th - 23th days (iv) 25th - 29th days
If the ovum is not fertilised)
(b) What are the uterine events that follow beyond 29th day if the ovum is not fertilised ?
Ans. (a) (i) Gonadotropins and FSH increases
(ii) LH attains peak level but FSH decreases
(iii) LH and FSH level decreases
(iv) LH remains low and FSH increases.
(b) After 29th day there is a menstrual flow involving discharge of blood and cast off
endometrium lining.
42 E
6. T.S. of mammalian testis revealing seminiferous tubules show different types of cell.

(i) Name the two types of cells of germinal epithelium.

(ii) Name of cells scattered in connective tissue and lying between seminiferous tubules.

Differentiate between them on the basis of their functions.

Ans. (i) Germinal epithelium have two types of cell. (1) Spermatogonium (2) Sertoli cells

(ii) Leydig cells or Interstitial cells.

Functions

Spermatogonium undergoes meiotic division leading to sperm formation.

Sertoli cell : Nourishes germ cells

Leydig cell : Synthesise and Secrete hormone androgen.

7. What are the various male accessory glands ? Give their function.

Ans. The male accessory glands include paired seminal vesicles, a prostrate gland and paired

bulbourethral glands.

These glands secrete seminal plasma rich in fructose, calcium and certain enzymes. Secretions of

bulbourethral glands help in lubrication of the penis.

8. Explain the menstrual cycle with a diagram.

Ans. Menstrual cycle has three phases: menstrual, proliferative and secretory.

(a) Menstrual Phase: The phase lasts for 3-5 days in human females and during this period the

endometrial lining of the uterus is cast off and is slowly passed out from vagina as a mixture

of blood.

(b) Proliferative or Follicular Phase: It lasts for 11 days between 6th to 16th day of the cycle.

During this phase one ovarian follicle is changed into Graafian follicle and the endometrial

layer is rebuilt along with repair of the ruptured blood vessels. Estrogen increases. It ends

with ovulation.

(c) Secretory Phase: It lasts for 12 days between 17-28 days. The Graafian follicle is converted

to Corpus Luteum. The endometrium grows and thickens further. Progesterone increases. It

end with the conversion of corpus luteum to corpus albicans.

43
(H) CASE BASED QUESTIONS
1. Study the following and answer the questions given below:
The following us the illustration of sequence of ovarian events during menstrual cycle in human
female. Observe it and give the answer of question that follow:
Subject-1 Subject-2

Ovarian events
Ovarian events

Ovulation Ovulation

Menstrual Cycle Days Menstrual Cycle Days

(i) Which structure in subject-1 and subject-2 form corpus luteum?


Ans. Graafian Follicle
(ii) Ovulation takes place on which day of menstrual cycle?
Ans. About the 14th day
(iii) In subject-1 structure ‘C’ is?
Ans. Corpus albicans
(iv) Withdrawal of which hormone causes degeneration of corpus luteum in subject-1?
Ans. LH
2. To answer the questions, study the diagram below for Subject 1 and 2 showing different
types of structure.

(i) At the end of first meiotic division in subject-1 the structure A differentiate into?
Ans. Secondary spermatocytes
(ii) Structure-A in subject-2 is produced during the formation of?
Ans. Secondary oocyte
(iii) What is the correct sequence of structure-E formation in subject-1?
Ans. Spermatogonia, spermatocyte, spermatid, spermatozoa
(iv) Polar bodies are formed during in which subject?
Ans. Subject-2
3. Read the following and answer the questions given below:

The female reproductive system consists of a pair of ovaries along with a pair of oviducts, uterus,

cervix, vagina and the external genitalia located in the pelvic region. These parts of the system

along with a pair of the mammary glands are integrated structurally and functionally to support

the processes of ovulation, fertilisation, pregnancy, birth and child care. Each ovary is about 2 to

4 cm in length and is connected to the pelvic wall and uterus by ligaments. The oviducts which

consist of 3 parts, uterus and vagina constitute the female accessory ducts. The uterus is single

and it is also called the womb. The shape of the uterus is like an inverted pear. The wall of the

uterus has three layers of tissue.

(i) Why ovaries consider as primary sex organs of FRS ?

Ans. They produce the female gametes (Ovum) and several steroid hormone.

(ii) Write the name of parts of fallopian tube .

Ans. Infundibulum, Ampulla, Isthmus

(iii) Write the name of external genitilia of FRS.

Ans. Mons pubis, Labia majora, Labia minora, Hymen and Clitoris.

(iv) Write the name of layers of uterus.

Ans. Perimetrium, Myometrium and Endometrium.

(v) “The presence or absence of hymen is not a reliable indicators of virginity or sexual

experience’’. Explain.

Ans. The hymen can also be broken by a sudden fall or jolt, insertion of a vaginal tampon, active

participation in some sports like horseback riding, cycling, etc. In some women the hymen

persists even after coitus.

E 45
4. Read the following and answer the questions given below:

The process of formation of a mature female gamete is called oogenesis which is markedly

different from spermatogenesis. Oogenesis is initiated during the embryonic development stage

when a couple of million gamete mother cells (oogonia) are formed within each fetal ovary; no

more oogonia are formed and added after birth. These cells start division and enter into

prophase-I of the meiotic division and get temporarily arrested at that stage, called primary

oocytes. Each primary oocyte then gets surrounded by a layer of granulosa cells and then called

the primary follicle A large number of these follicles degenerate during the phase from birth to

puberty. Therefore, at puberty only 60,000-80,000 primary follicles are left in each ovary. The

primary follicles get surrounded by more layers of granulosa cells and a new theca and called

secondary follicles.

(i) Differentiate between primary oocytes and primary follicle.

Ans. Primary oocytes: - These cells start division and enter into prophase-I of the meiotic division

and get temporarily arrested at that stage, called primary oocytes.

Primary follicle: - Each primary oocyte then gets surrounded by a layer of granulosa cells and

then called the primary follicle.

(ii) Where and when the oogenesis initiated in female.

Ans. In the fetal ovary during embryonic development stage.

(iii) Name the ovarian hormone that induced the oogenesis process.

Ans. Estrogen

(iv) Write the name of stage in which the gametes mother cell get temporarily arrested?

Ans. Dictyotene of prophase-I of meiosis- I.

(v) How many primary follicles there are left in ovaries at puberty?

Ans. At puberty only 120,000-1600,000 primary follicles are left in ovaries.


46 E
CHAPTER 3
(A) NCERT QUESTIONS & SOLUTIONS
1. What do you think is the significance of reproductive health in a society?
Ans. In a society if the people are aware of birth control methods to avoid sexually transmitted
diseases and the importance of breast-feeding and post-natal care of the mother and baby then the
society can have healthy children who are the future citizens of a nation.
2. Suggest the aspects of reproductive health which need to be given special attention in the
present scenario?
Ans. Important aspects :
 Counselling and creating awareness among people especially the youth about various aspects
of reproductive health such as sexually transmitted diseases, available contraceptive methods,
case of pregnant mothers, adolescence etc.
 Providing support and facilities such as medical assistance to people during pregnancy, STIs,
abortions, contraceptives, infertility, etc. for building a reproductively healthy society.
3. Is sex education necessary in schools? Why? [IMP.]
Ans. Yes, sex education is necessary in schools because
• It will provide proper information about reproductive organs, adolescence, safe, hygienic
sexual practices and sexually transmitted infection (STIs).
• It will provide right information to avoid myths and misconceptions about sex related queries.
4. Do you think that reproductive health in our country has improved in the past 50 years? If
yes, mention some such areas of improvement.
Ans. Yes, the reproductive health has tremendously improved in India in the last 50 years. The areas
of improvement are as follows.
• Batter awareness about sex related matters.
• Increase number of medically assisted deliveries and better post natal care of child and mother
leading to decrease maternal and infant mortality rate.
• Increase numbers of couples with small families.
• Better detection and cure of STIs and overall increased medical facilities for all sex related
problems.

47
5. What are the suggested reasons for population explosion?
Ans. The suggested reasons of population explosion are :
(a) rapid decline in death rate.
(b) decline in maternal mortality rate (MMR)
(c) decline in infant mortality rate (IMR) and
(d) increase in number of people in reproducible Age
6. Is the use of contraceptives justified? Give reasons.
Ans. Yes, the use of contraceptives is absolutely justified. The human population is increasing
tremendously. Therefore to regulate the population growth by regulating reproduction has
become a necessary demand in the present times. Various contraceptive devices have been
devised to reduce unwanted pregnancies which help in bringing down the increased birth rate and
hence, in checking population explosion.
7. Removal of gonads cannot be considered as a contraceptive option. Why?
Ans. The removal of gonads not only stops the production of gametes but will also stop the secretion
of various important hormone which required for body functions. This method is irreversible and
thus, cannot be considered as a contraceptive method.
8. Amniocentesis for sex determination is banned in our country. Is this ban necessary?
Comment. [IMP.]
Ans. Yes, the ban is necessary because amniocentesis is misused for determining the sex of the foetus
and then aborting the child if it is a female.
9. Suggest some methods to assist infertile couples to have children.
Ans. The reasons of infertility could be physical, congenital diseases, drugs, immunological and even
psychological. Specialized health care units called infertility clinics could help in diagnosis and
corrective treatment of some of these disorders. Through special techniques called assisted
reproductive technologies (ART), infertile couple could be assisted to have children.
ART includes:
(i) In vitro fertilisation,
(ii) Gamete intra fallopian transfer (GIFT),
(iii) Intra cytoplasmic sperm injection (ICSI), and
(iv) Artificial insemination (AI).
10. What are the measures one has to take to prevent from contracting STDs? [IMP.]
Ans. Sexually transmitted Infection (STIs) get transferred from one individual to the other through
sexual contact. Adolescents and young adults are at the greatest risk of acquiring these sexually
transmitted diseases. Hence,
• Creating awareness among the adolescents regarding its after-effects can prevent them from
contracting STIs.
• The use of contraceptives such as condoms etc. while intercourse can prevent the transfer of
these diseases.
• Sex with unknown partners or multiple partners should be avoided as they may have such
diseases.
• Specialists should be consulted immediately in case of doubt so as to assure early detection and
cure of the disease.
11. State True / False with explanation.
(a) Abortions could happen spontaneously too. (True / False)
(b) Infertility is defined as the inability to produce a viable offspring and is always due to
abnormalities/ defects in the female partner. (True / False)
(c) Complete lactation could help as a natural method of contraception. (True / False)
(d) Creating awareness about sex related aspects is an effective method to improve reproductive
health of the people. (True / False)
Ans. (a) True - Due to internal factors like incompatibility, abortion could happen spontaneously.
(b) False - Infertility may also be caused sue to male partner when sperm count is low or their
mobility is less.
(c) True - Lactational amenorrhea is a method of contraception as ovulation does not occur
during the period of intense lactation following parturition.
(d) True - Creating awareness about sex related aspects removes the myths and misconceptions
about these problems.
12. Correct the following statements:
(a) Surgical methods of contraception prevent gamete formation.
(b) All sexually transmitted diseases are completely curable.
(c) Oral pills are very popular contraceptives among the rural women.
(d) In E. T. techniques, embryos are always transferred into the uterus.
Ans. (a) Surgical method of contraception prevents gamete motility.
(b) All sexually transmitted diseases are curable if they are detected early and treated properly.
AIDS is still an incurable disease.
(c) Oral pills are very popular contraceptives among urban women.
(d) In embryo transfer technique, 8 celled embryos are transferred into the fallopian tube while
more than 8 celled embryos are transferred into the uterus.
49
(B) PREVIOUS YEAR QUESTIONS
1. Given below are four aspects of Reproductie Health in Column A and their related information in
Column B : [CBSE 2023]
Column - A Column - B
S. Terms used in Reproductive S. Significant information
No. Heath No.
(A) MTP (i) Analysing fetal cells from amniotic
fluid of the foetus
(B) Amniocentesis (ii) Legalised in 1971
(C) Saheli (iii) Programme initiated in 1951
(D) Family Planning (iv) Non - steroidal oral contraceptive
Select the correct match from the following options :
(1) (A) - (iv), B - (ii), (C) - (iii), (D) - (i) (2) (A) - (ii), B - (i), (C) - (iv), (D) - (iii)
(3) (A) - (i), B - (iii), (C) - (ii), (D) - (iv) (4) (A) - (ii), B - (i), (C) - (iii), (D) - (iv)
Ans. (2) (A) - (ii), B - (i), (C) - (iv), (D) - (iii)
2. Given below are Column - A with a list of certain Assisted Reproductive Technologies (ART)
and in Column - B the procedures followed during ART : [CBSE 2023]
Column - A Column - B
S.No. Names of ART S.No. Procedures
(A) GIFT (i) Transfer of ovum from a donor into
the fallopian tube of another
female.
(B) ICSI (ii) Transfer of semen from the donor
into the vagina of the female.
(C) ZIFT (iii) Injecting sperms directly into the
ovum.
(D) IUI (iv) Transfer of early embroys into the
fallopian tube.
Choose the option where ART correctly matches with the procedure.
(1) (A) - (i), (B) - (ii), (C) - (iii), (D) - (iv) (2) (A) - (iv), (B) - (i), (C) - (ii), (D) - (iii)
(3) (A) - (iv), (B) - (iii), (C) - (i), (D) - (ii) (4) (A) - (i), (B) - (iii), (C) - (iv), (D) - (ii)
Ans. (4) (A) - (i), (B) - (iii), (C) - (iv), (D) - (ii)
3. Assertion (A): Through Reproductive and Child Health (RCH) programmes in India ; we could
bring down the population growth rate.
Reason (R): A rapid increase MMR and IMP were the reasons, along other reasons for this.
[CBSE Term-I 2022]
(1) Both Assertion and Reason are true and Reason is the correct explanation of Assertion.
(2) Both Assertion and Reason are true but Reason is not the correct explanation of Assertion.
(3) Assertion is true but Reason is false.
(4) Both Assertion and Reason are false.
Ans. (3) Assertion is true but Reason is false.
4. Assertion (A): Sterilisation methods are generally advised for male/female partner as a terminal
method to prevent any more pregnancies.
Reason (R): These techniques are less effective and have high reversibility. [CBSE Term-I 2022]
(1) Both Assertion and Reason are true and Reason is the correct explanation of Assertion.
(2) Both Assertion and Reason are true but Reason is not the correct explanation of Assertion.
(3) Assertion is true but Reason is false.
(4) Both Assertion and Reason are false.
Ans. (3) Assertion is true but Reason is false.
5. Select the correct option for Human Chorionic Gonadotropin (hCG) released during embryonic
development in humans. [CBSE Term-I 2022]
(i) Helps in maintenance of pregnancy.
(ii) Leads to rupture of Graafian follicle.
(iii) Cause strong uterine contraction during childbirth.
(iv) Brings metabolic changes in the mother.
(1) (i) and (ii) (2) (i) and (iv) (3) (ii) and (iii) (4) (ii) and (iv)
Ans. (2) (i) and (iv)
6. A specialized procedure to form an embryo in the laboratory in which sperm is directly, injected
into the ovum is : [CBSE Term-I 2022]
(1) IUT (2) IUI (3) ICSI (4) ZIFT
Ans. (3) ICSI
7. Listed below are all reproductive tract infections except [CBSE Term-I 2022]
(1) Genital herpes (2) Filariasis (3) Trichomoniasis (4) Syphilis
Ans. (2) Filariasis
8. Assertion (A) : Determining the sex of an unborn child followed by MTP is an illegal practice.
Reason (R) : Amniocentesis is a practice to test the presence of genetic disorders also.
[CBSE 2023]
(1) Both Assertion and Reason are true and Reason is the correct explanation of Assertion.
(2) Both Assertion and Reason are true but Reason is not the correct explanation of Assertion.
(3) Assertion is true but Reason is false.
(4) Both Assertion and Reason are false.
Ans. (2) Both Assertion and Reason are true but Reason is not the correct explanation of
Assertion.
9. Name and explain a surgical contraceptive method that can be adopted by the male partner
of a couple. [CBSE 2023]
Ans. Vasectomy. In vasectomy, a small part of the vas deferens is removed or tied up through a small
incision on the scrotum.
10. State the composition and principle of oral pills as a contraceptive measure taking the
example of Saheli. [CBSE-IMP Question]
Ans. The composition of oral pills comprises: Either progestogens alone or progestogen – estrogen
combination. Saheli is a Non-steroidal preparation. It inhibits ovulation and implantation. It also
alters the quality of cervical mucus to prevent/ retard the entry of sperms.

E 51
11. (a) IUDs are said to be effective contraceptives. Name any two commonly used IUDs and
write the mode of their actions.
(b) When is sterilisation advised to married couples? How is it carried out in a human male
and a female, respectively? [CBSE 2020]
Ans. (a) Hormone releasing IUDs- It makes the uterus unsuitable for implantation and the cervix
hostile to the sperms.
Copper releasing IUDs – Cu ions released suppress sperm motility and the fertilising capacity
of sperms.
(b) When a desired family size is attained, and couple don’t need any more child then only
sterilisation methods are advised.
Sterilisation procedure
(i) Male- in male it is called ‘vasectomy’- In vasectomy, a small part of the vas deferens is
removed or tied up through a small incision on the scrotum.
(ii) Female- in female it is called ‘tubectomy’- In tubectomy, a small part of the fallopian tube
is removed or tied up through a small incision in the abdomen or through vagina.
12. Explain one application of each one of the following :
(a) Amniocentesis
(b) Lactational amenorrhea
(c) ZIFT
(d) Prepare a poster for the school programme depicting the objectives of : “Reproductive
and Child Health Care Programme”. [CBSE 2019]
Ans. (a) To detect chromosomal disorders / sex determination (legally banned) / detect genetic disorder.
(b) To prevent pregnancy / means of natural contraception.
(c) To assist an infertile couple to have children by tranferring the zygote / early embryo / embryo
at eight blastomere stage into fallopian tube.
(d) A poster made on RCH - Any relevant slogan or sketch made should be awarded like
e.g. - Hum Do Hamare Do,
- Gender selection and detection is punishable, or (Any other relevant theme)
(C) MULTIPLE CHOICE QUESTIONS
1. What is the full form of WHO?
(1) Ware House Organization (2) War and Health Organization
(3) World Health Office (4) World Health Organization
Ans. (4) World Health Organization
2. What are the various aspects of reproduction covered by WHO?
(1) Physical, Emotional, Behavioural (2) Physical, Emotional, Behavioural, Social
(3) Physical, Emotional, Gestational, Social (4) Physical, Emotional, Social
Ans. (2) Physical, Emotional, Behavioural, Social.
3. Which was the first country in the world to initiate a nationwide programme for reproductive
health?
(1) China (2) USA (3) India (4) Russia
Ans. (3) India
4. The figure indicates which contraceptive device?

(1) Condom for female (2) Condom for male


(3) Diaphragm (4) Cervical cap
Ans. (2) Condom for male
5. The programme of ‘Family Planning’ was initiated in the year ________.
(1) 1950 (2) 1947 (3) 1949 (4) 1951
Ans. (4) 1951
6. The reproductive program RCH stands for
(1) Reproductive and Community Health Care
(2) Restorative and Communal Health Care
(3) Reproductive and Child Health Care
(4) Reproductive and Congenital Health Care
Ans. (3) Reproductive and Child Health Care
7. In context of reproductive health, STIs stands for
(1) Sexually Terminal Infections (2) Sexually Transmitted Infections
(3) Sexually Transformed Infections (4) Sexually Transducted Infections
Ans. (2) Sexually Transmitted Infections
8. The fluid which envelops the developing foetus is called
(1) Chorionic fluid (2) Placental fluid (3) Amniotic fluid (4) Uterine fluid
Ans. (3) Amniotic fluid
9. Statutory ban has been laid on ________ to check female foeticide.
(1) Choriocentesis (2) Amniocentesis (3) Uterocentesis (4) Embryocentesis
Ans. (2) Amniocentesis
10. The amniocentesis test is based on ________ to determine the sex of developing foetus.
(1) External genitalia (2) Secondary sexual characters
(3) Chromosomal pattern (4) Embryonic enzymes
Ans. (3) Chromosomal pattern
53
11. CDRI, Lucknow developed which new female contraceptive?
(1) ‘Sakhi’ (2) ‘Saheli’ (3) ‘Saloni’ (4) ‘Smiti’
Ans. (2) ‘Saheli’
12. CDRI stands for
(1) Contraceptive Drug Research Institute (2) Central District Research Institute
(3) Central Drug Research Institute (4) Central Dermatologic Research Institute
Ans. (3) Central Drug Research Institute
13. The world population was 2000 million in the year
(1) 1980 (2) 1970 (3) 1960 (4) 1900
Ans. (4) 1900
14. Which IUD is shown in the given figure?

(1) Lippes loop (2) Progestasert (3) Copper T (4) Multiload-375


Ans. (3) Copper T
15. Coitus interrupts/withdrawal method concerns with
(1) Withdrawal of penis from vagina before ejaculation
(2) Withdrawal of penis from vagina after ejaculation
(3) Prevention of coitus
(4) Alternate prevention of coitus
Ans. (1) Withdrawal of penis from vagina before ejaculation
16. After parturition, which natural contraception way can be utilized?
(1) Lactational menorrhea (2) Lactational amenorrhea
(3) Lactational deficiency (4) Lactational prevention
Ans. (2) Lactational amenorrhea
17. In lactational amenorrhea, which event does not occur in menstrual cycle?
(1) Menstrual flow (2) Ovulation (3) Funicular phase (4) Luteal phase
Ans. (2) Ovulation
18. Lactational amenorrhea is effective only up to a maximum of ________ months.
(1) Two (2) Four (3) Six (4) Eight
Ans. (3) Six
19. ________ is a popular brand of condom for males.
(1) ‘Nishodh’ (2) ‘Nirodh’ (3) ‘Nidosh’ (4) ‘Nirdosh’
Ans. (2) ‘Nirodh’
20. Which of the following is not applicable to females for contraception?
(1) Diaphragms (2) Vasectomy (3) Condoms (4) Cervical caps
Ans. (2) Vasectomy
(D) ASSERTION – REASON QUESTIONS

 Directions: In the following questions, a statement of assertion is followed by a statement of


reason. Mark the correct choice as:
(1) If both Assertion and Reason are true and Reason is the correct explanation of Assertion.
(2) If both Assertion and Reason are true but Reason is not the correct explanation of Assertion.
(3) If Assertion is true but Reason is false.
(4) If both Assertion and Reason are false.
1. Assertion: A person should be considered reproductively healthy if they have healthy
reproductive organs but are emotionally imbalanced.
Reason: This statement about reproductive health was given by WHO.
Ans (4)
2. Assertion: Family planning is an action plan to attain reproductive health among people.
Reason: Improved programmes covering reproduction related areas were propagated by RCH to
create awareness among people.
Ans (2)
3. Assertion: Reproductive and Child Healthcare Programmes is for reproduction related areas.
Reason: It deals with creating awareness among various reproduction related aspects.
Ans (1)
4. Assertion : Introduction of sex education in schools should be encouraged.
Reason : This will encourage children to believe in myths about sex related aspects.
Ans (3)
5. Assertion: Natality increases both population density and population size.
Reason: Natality increases the number of individuals in an area by births.
Ans (1)
6. Assertion: Zero population growth should be achieved as early as possible to control human
population.
Reason: This as requires not two children per couple but a lilttle more.
Ans (1)
7. Assertion: Periodic abstinence is a method in which couples avoid from coitus from day 17 to
27 of menstrual cycle.
Reason: It is a very effective method and 100% sure of birth control.
Ans (4)
8. Assertion: Syphilis, gonorrhea and AIDS are STIs.
Reason: These diseases are transmitted through sexual intercourse.
Ans (1)
9. Assertion : HIV infection can be avoided by use of condoms.
Reason : Condoms secrete anti-viral interferon's.
Ans (3)
10. Assertion: STIs are transmitted through sexual intercourse.
Reason: STIs can be prevented by using barrier contraceptives like condoms.
Ans (1)
E 55
(E) VERY SHORT ANSWER QUESTIONS
1. Ideal contraceptives for the females who wants to delay pregnancy.

Ans. IUDs

2. Oral pills for females causes

Ans. Inhibit ovulation and implantation

3. Once a-week oral pill is :

Ans. Saheli

4. In emergency which contraceptive methods may be used within 72 hours of coitus.

Ans. IUD & progestogen or progestogen estrogen combination pills.

5. Which contraceptive method are also known as sterilisation?

Ans. Surgical method

6. Sterilisation in male is known as :

Ans. Vasectomy

7. Government of India legalised MTP in.

Ans. 1971

8. MTP is considered relatively safe during.

Ans. First trimester

9. In ZIFT-early embryo is transferred in (upto 8 blastomeres)

Ans. Fallopian tube

10. Full form of ZIFT

Ans. Zygote Intra Fallopian Transfer

11. In IUT, embryos with more than 8 blastomeres are transferred in

Ans. Uterus

12. Full form of GIFT

Ans. Gamete Intra Fallopian Transfer

13. Procedure to form an embryo in the laboratory in which a sperm is directly injected into

the cytoplasm of ovum

Ans. ICSI-Intracytoplasmic sperm injection.


56 E
(F) SHORT ANSWER QUESTIONS
1. How does 'Saheli' Work like a contraceptive?
Ans. 'Saheli' is a non-steroid oral contraceptive. It contains hormone. It shows following effects -
1. Alter the quality of cervical mucus to prevent or retard entry of sperms into uterus.
2. It prevents implantation.
2. What is copper –T?
Ans. It is an intra uterine device. It is implanted into uterus through vagina by expert doctor or trained
nurse. It is T shaped, made of copper. It releases Cu++ which decrease the mortality and
fertilizing ability of sperms.
3. Name the physical methods of contraceptives.
Ans. They are barriers. They prevent physical association of sperm and ovum. Example are-
1. Condoms
2. Others - Diaphragm, Vaults, Cervical caps.
4. What is infertility? Suggest the methods used for curing the infertility.
Ans. Infertility - When couples are unable to produce children inspite of un protected sexual co-
habitation for at least two or more years duration.
- ART = Assisted Reproductive Technologies are used to cure infertility.
- These technologies are-
1. In-vitro fertilisation = IVF
(a) ZIFT (b) IUT
2. In vivo fertilization 3. GIFT 4. ICSI
5. What are the suggested reasons for population explosion?
Ans. The suggested reasons of population explosion are:
(a) Rapid decline in death rate.
Decline in maternal mortality rate (MMR)
Decline in infant mortality rate (IMR) and
(b) Increase in number of people in reproducible age.
6. Our government has intentionally imposed strict conditions for M.T.P. in our country.
Justify giving a reason.
Ans. (i) To prevent female foeticide
(ii) To maintain sex ratio
(iii) To avoid any danger for (young) mother (and foetus)
E 57
(G) LONG ANSWER QUESTIONS
1. Reproductive and Child Healthcare (RCH) programmes are currently in operation. One of
the major tasks of these programmes is to create awareness amongst people about the wide
range of reproduction related aspects. As this is important and essential for building a
reproductively healthy society.
(a) “Providing sex education in schools is one of the ways to meet this goal.” Give four
points in support of your opinion regarding this statement.
(b) List any two ‘indicators’ that indicate a reproductively healthy society.
Ans. (a) Provide right information to the young so as to discourage children from believing in myths
and misconception about sex related aspects. Proper information about reproductive organs-
Proper information about adolescence and related changes-Safe hygienic practices
STDs/AIDS - Available birth control options – Care of pregnant mothers-Post natal care-
Importance of breast feeding - Equal opportunities for male and female child awareness of
problems due uncontrolled population growth - Sex abuse - Sex related crimes (Any four)
(b) Better awareness about sex related matters / increase number of assisted deliveries/better post
natal care/decrease in IMR (Infant Mortality Rate)/decrease MMR(Maternal Mortality
Rate)/increase number of couples with small families/better detection and cure of STDs/
overall increased medical facilities for sex related problems / total well being in all aspects of
reproduction/physical - behavioural - social / physically and functionally normal reproductive
organs/normal emotional and behavioural interaction among all sex related aspects.
2. (a) Name and explain the mode of action of any two types of IUDs.
(b) List the advantages of using 'Saheli' as a contraceptive.
Ans. (a)
 Non- medicated (e.g. lippes loop), phagocytosis of sperms
 Copper releasing IUDs (CuT, Cu7, Multiload 375), suppress sperm motility/reduces
fertilizing capacity of sperm.
Hormone releasing IUDs (Progestasert, LNG – 20), makes uterus unsuitable for
implantation/cervix hostile to sperms.
(b) Advantages of Saheli-
(i) Non -steroidal (ii) Once a week (iii) High contraceptive value (iv) Less side effects
3. Name two hormones that are constituents of contraceptive pills. Why do they have high and
effective contraceptive value? Name a commonly prescribed non-steroidal oral pill.
Ans. Progestogen-estrogen combination, Progestogen or progesterone are present in contraceptive
pills. They inhibit ovulation, implantation and alter quality of cervical mucus to retard entry of
sperm. Saheli is a commonly prescribed oral pill.
4. (a) Mention the problems that are taken care of by Reproduction and Child Health Care
programme.
(b) What is amniocentesis and why there is a statutory ban on it?
Ans. (a) (i) Uncontrolled population growth (ii) social evil like sex abuse
(iii) sex related crime (iv) STDs.
(b) Foetal sex determination test based on chromosomal pattern in the amniotic fluid/to study
chromosomal abnormalities in the foetus, banned to legally check female foeticide.
58 E
(H) CASE-STUDY BASED QUESTIONS
1. Study the following and answer the questions given below:
The following us the illustration of male reproductive system. Observe it and give the answer of
question that follow:

Subject-1 Subject-2

(i) In subject-1 a small portion of which duct is removed and tied up?
Ans. Vas deferens
(ii) Which subject needs barrier?
Ans. Subject-2
(iii) Subject-1 shows infertility due to?
Ans. Vasectomy
(iv) What are the permanent method of family planning?
Ans. Vasectomy, Tubectomy

2. Study the following and answer the questions given below:


The following us the illustration of female reproductive system. Observe it and give the answer
of question that follow:

Subject-1 Subject-2

E 59
(i) Which part is removed in a subject-1?
Ans. Fallopian tube
(ii) Ovulation occur in which subject?
Ans. Both subject-1 and subject-2
(iii) In which subject does not show menstruation cycle?
Ans. Subject-1
(iv) In which subject fertilization does occurs?
Ans. Subject-2 Only

3. Read the following and answer the questions given below:


Intentional or voluntary termination of pregnancy before full term is called MTP or induced
abortion. Nearly 45 to 50 million MTPs are performed in a year all over the world which
accounts to 1/5th of the total number of conceived pregnancies in a year. Obviously, MTP has a
significant role in decreasing the population though it is not meant for that purpose. Whether to
accept /legalise MTP or not is being debated upon in many countries due to emotional, ethical,
religious and social issues involved in it. Government of India legalised MTP in 1971 with some
strict conditions to avoid its misuse. Such restrictions are all the more important to check
indiscriminate and illegal female foeticides which are reported to be high in India.

(i) Expand MTP.


Ans. Medical Termination of Pregnancy
(ii) What is relatively safe duration for MTPs ?
Ans. First trimester, i.e., upto 12 weeks of pregnancy.
(iii) When Government of India legalised MTP with some strict conditions to avoid its misuse?
Ans. 1971

(iv) What is disturbing trend observed related to MTPs?


Ans. The MTPs are performed illegally by unqualified quacks which are not only unsafe but could
be fatal too.
(v) Amniocentesis for sex determination is banned in our country. Is this ban necessary?
Comment.
Ans. Yes, the ban is necessary because amniocentesis is misused for determining the sex of the
foetus and then aborting the child if it is a female.

60 E
4. Read the following and answer the questions given below:

India was amongst the first countries in the world to initiate action plans and programmes at a

national level to attain total reproductive health as a social goal. These programmes called ‘family

planning’ were initiated in 1951 and were periodically assessed over the past decades. Improved

programmes covering wider reproduction-related areas are currently in operation under the

popular name RCH programmes’. Creating awareness among people about various reproduction

related aspects and providing facilities and support for building up a reproductively healthy

society are the major tasks under these programmes. With the help of audio-visual and the print-

media governmental and non-governmental agencies have taken various steps to create awareness

among the people about reproduction-related aspects.

(i) Expand RCH.

Ans. Reproductive and Child Health Care

(ii) Mention the problems that are taken care of by RCH.

Ans. The uncontrolled population growth, social evil like sex abuse , sex related crime and STDs.

(iii) Prepare a slogan for poster for the school programme depicting the objectives of :

“Reproductive and Child Health Care Programme”.

Ans. A poster made on RCH - Any relevant slogan or sketch made should be awarded marks e.g. Hum

Do Hamare Do, Gender selection and detection is punishable.

(iv) “Providing sex education in schools is one of the ways to meet the goal of RCH ”. Give a

points in support of your opinion regarding this statement.

Ans. Provide right information to the young so as to discourage children from believing in myths

and misconception about sex related aspects.

(v) What do you think is the significance of reproductive health in a society?

Ans. In a society if the people are aware of birth control methods to avoid sexually transmitted

diseases and the importance of breast-feeding and post-natal care of the mother and baby, then

the society can have healthy children who are the future citizens of a nation.

E 61
IMPORTANT NOTE

______________________________________________________________________

_______________________________________________________________________
_______________________________________________________________________
_______________________________________________________________________
_______________________________________________________________________
_______________________________________________________________________
_______________________________________________________________________
_______________________________________________________________________
_______________________________________________________________________
_______________________________________________________________________
_______________________________________________________________________
_______________________________________________________________________
_______________________________________________________________________
_______________________________________________________________________
_______________________________________________________________________
_______________________________________________________________________
_______________________________________________________________________
_______________________________________________________________________
_______________________________________________________________________
_______________________________________________________________________
_______________________________________________________________________
_______________________________________________________________________
_______________________________________________________________________

_______________________________________________________________________

_______________________________________________________________________
_______________________________________________________________________
62 E
CHAPTER 4
(A) NCERT QUESTIONS & SOLUTIONS
1. Mention the advantages of selecting pea plant for experiment by Mendel.
Ans. • Annual plant with short life cycle of 2-3 months.
• It has many contrasting trait.
• Pea seeds are large.
• Easy to cultivate.
• Pea plant is naturally self pollinating.
• Artificial cross pollination can be easily performed.
2. Differentiate between the following – [IMP.]
(A) Dominance and Recessive (B) Homozygous and Heterozygous
(C) Monohybrid and Dihybrid
Ans. (A) Dominance and Recessive
Dominance Recessive

Allele which can express itself both in Allele which can express itself only in
homozygous and heterozygous condition homozygous condition is known as
is known as dominant allele. recessive allele.

(B) Homozygous and Heterozygous


Homozygous Heterozygous

In diploid individual, if similar alleles of In diploid individual, if dissimilar


a gene is present then it is said to be alleles of a gene is present then it is said
homozygous. to be heterozygous.
Also termed as Pure /true breeding. Also termed as Impure/Hybrid.
(C) Monohybrid and Dihybrid.

Monohybrid Dihybrid

Monohybrid involves cross between Dihybrid involves cross between parents


parents which differs in only one pair of differs in two pairs of contrasting
contrasting characters. characters.

For example The cross between tall and For example The cross between pea
dwarf pea plant is a monohybrid cross. plants having yellow round seed with
those having green wrinkled seeds is a
dihybrid cross.

E 63
3. A diploid organism is heterozygous for 4 loci, how many types of gametes can be produced?
Ans. A diploid organism heterozygous for 4 loci, will have four different contrasting characters at four
different loci.
For example if an organism is heterozygous at four loci with four characters say AaBbCcDd then
during meiosis it will segregate to form 16 separate gametes.
Type of gamete = 2n where n is no of heterozygous pair
So n = 4 so 24 so 16 gametes.
4. Explain the Law of Dominance using a monohybrid cross.
Ans. When two different factors (genes) or a pair of contrasting forms of a character are present in an
organism, only one expresses itself in the F1 generation and is termed as dominant while the
other remains unexpressed and called recessive factors (gene).
5. Define a test - cross?
Ans. When an individual is crossed with the homozygous recessive parent. It is called test cross.
6. Using a Punnett Square, workout the distribution of phenotypic features in the first filial
generation after a cross between a homozygous female and a heterozygous male for a single
locus.
Ans.

Parents  Tt tt

T t t t

Gametes  t t

Tt Tt
F1 generation T
Tall Tall
tt tt
t
Dwarf Dwarf
Phenotypic Ratio:- 1:1
Genotypic Ratio:- 1:1
7. When a cross in made between tall plant with yellow seeds (TtYy) and tall plant with green
seed (Ttyy), what proportions of phenotype in the offspring could be expected to be [IMP.]
(a) Tall and green. (b) Dwarf and green.
Ans.
Tall & Yellow Seeds Tall & Green Seeds

TtYy Ttyy

Ty ty

TY TTYy (Tall & Yellow) TtYy (Tall & Yellow)


Ty Ttyy (Tall & Green) Ttyy (Tall & Green)

tY TtYy (Tall & Yellow) ttYy (Dwarf & Yellow)


ty Ttyy (Tall & Green) ttyy (Dwarf & Green)

Phenotype ratio 
(a) Tall and green : 3 (b) Dwarf and green : 1
64 E
8. Two heterozygous parents are crossed. If the two loci are linked what would be the
distribution of phenotypic features in F1 generation for a dihybrid cross?
Ans. The co-existence of two or more genes in the same chromosome is termed as linkage. If the
genes are located close to each other and on the same chromosome, they are inherited together
and are referred to as linked genes. If two heterozygous parents exhibit linkage, then the outcome
is as follows:
BbLl x BbLl
Blue long Blue long
So in F1 generation the parental combination will comparatively be more than the newer
combinations which are less in number.
9. Briefly mention the contribution of T.H. Morgan in genetics.
Ans. Thomas Hunt Morgan is called father of experimental genetics.
Experimental verification of the chromosomal theory of inheritance was given by Thomas Hunt
Morgan and his colleagues, led to discovering the basis for the variation that sexual reproduction
produced.
Morgan worked with the tiny fruit files, Drosophila melanogaster, which were found very
suitable for such studies.
Morgan carried out several dihybrid crosses in Drosophila to study genes that were sex-linked.
He stated and established that genes are located on the chromosome.
He established the principle of linkage, crossing over, sex linked inheritance and discovered the
relationship between gene and chromosome.
He established the technique of chromosome mapping.
He observed and worked on mutation.
10. What is pedigree analysis? Suggest how such an analysis, can be useful.
Ans. A pedigree is a record of inheritance of a specific genetic trait for two or more generations which
is presented in the form of a diagram or family tree. Pedigree analysis is an analysis of several
generations of a family which is used on human beings.
Usefulness of pedigree analysis:
Serves as a powerful tool which can be used to trace the inheritance of a particular trait, disease
or an abnormality
It is helpful for genetic counsellors to suggest couples about the possibility of having children
with genetic abnormalities such as colour blindness, haemophilia, thalassaemia, sickle-cell
anaemia etc
Helpful in reasoning why marriage between close relatives is harmful.
E 65
11. How is sex determined in human beings?
Ans. Sex determination in human is done by XX-XY type method. In humans, females have
XX chromosomes and males have two different type sex chromosomes (XY).
Female is homogametic Male is heterogametic
AA + XX (female) AA + XY (male)

A+X A+X A+X A+Y

A+X A+Y
+ A+X AA + XX AA + XY
A+X AA + XX AA + XY

Male progeny 50% female progeny - 50%


12. A child has blood group O. If the father has blood group A and mother blood group B, work
out the genotypes of the parents and the possible genotypes of the other offsprings. [IMP.]
Ans. IA Io × IBIO

A O
Father (I I ) B O
Mother (I I )

B O
I
A
I
O
+ I I

IB IO
IA I AI B IAIO
IO IBIO IOIO

Possible genotype of other offsprings - IAIB, IAIO, IBIO, IOIO


13. Explain the following terms with example.
(a) Co-dominance (b) Incomplete dominance
Ans. (a) Co-dominance: In this phenomenon both the alleles are able to express themselves
independently when found together in a heterozygote. They are termed as co-dominant
alleles.
E.g., - ABO blood group
(b) Incomplete dominance: Incomplete dominance may be defined as the partial /expression of
both alleles in a heterozygote so that the phenotype is intermediate between those of two
homozygotes.
E.g., - (i) Flower colours of Mirabilis jalapa (4 o'clock plant) (ii) Snapdragon

66
14. What is point mutation? Give one example.
Ans. Mutations arising due to change in single base pair of DNA is called point mutation. Eg: sickle
cell anaemia.
15. Who had proposed the chromosomal theory of the inheritance?
Ans. In 1902 the chromosomal theory of inheritance was proposed by Theodore Boveri and Walter
Sutton.
16. Mention any two autosomal genetic disorders with their symptoms. [IMP.]
Ans. Sickle cell anemia -
Symptoms:-
Shape of RBCs change from biconcave to sickle-shaped(curved) under the influence of low
oxygen tension.
These sickle-shaped RBCs are more rapidly destructed than the normal ones causing anaemia
Phenylketonuria:-
This inborn error of metabolism is also inherited as the autosomal recessive trait. The affected
individual lacks an enzyme that converts the amino acid phenylalanine into tyrosine. As a result
of this phenylalanine is accumulated and converted into phenylpyruvic acid and other
derivatives.
Symptoms:-
Accumulation of these in brain results in mental retardation. These are also excreted through
urine because of its poor absorption by kidney.

E 67
(B) PREVIOUS YEAR QUESTIONS
1. Assertion (A) : In Thalassemia an abnormal myoglobin chain is synthesized due to a gene defect.
Reason (R) : Thalassemia is controlled by genes HBA1 and HBA2 on chromosome 16.
[CBSE 2023]
Ans. (4) Assertion (A) is false but Reason (R) is also true.
2. By using Punnett square depict the genotypes and phenotypes of test crosses (where green
pod colour (G) is dominant over yellow pod colour (g) in Garden pea with unknown
genotype. [CBSE 2023]
Ans.

Homozygous Homozygous
recessive recessive
(gg) (gg)

(g) (g) (g) (g)

Gg Gg (G) (G) Gg Gg
GG Gg
Gg Gg (G) (g) gg gg
Dominant phenotype
[Genotype unknown]
Result Pod colour is green Half of the pods are green
of all pods and half pods are yellow
Interpretation Unknown pod colour is Unknown pods is
homozygous dominant heterozygous

Diagrammatic representation of a test cross

3. It is sometimes observed that the F1 progency has a phenotype that does not resemble either
of the two parents and has intermediate phenotype. Explain by taking a suitable example
and working out the cross upto F2 progeny. [CBSE 2023]
Ans. When in heterozygous condition, dominant allele cannot express completely over recessive allele
this is known as Incomplete dominance.
It is the exception of law of dominance.
In case of incomplete dominance the F1 had a phenotype that did not resemble either of the two
parents and was in between the two.

In heterozygous condition intermediate form appears.


The inheritance of flower colour in the dog flower (snapdragon or Antirrhinum majus) is a good
example to understand incomplete dominance.

68 E
Incomplete dominance was first discovered by Correns for flower colour in Mirabilis jalapa.
Incomplete dominance is found in Snapdragon is similar to Mirabilis.
Intermediate starch grain size in pea plant is also example of incomplete dominance.
4. "It is sometimes observed that the F1 progeny shows a phenotype that resembles both the
parents." Explain this type of inheritance using the example of A,B,O blood groups in
human. [CBSE 2023]
Ans. When both the alleles of a gene express themselves simultaneously in a heterozygote, this
condition is known as Co-dominance.
In case of co-dominance the F1 generation resembles both the parents.
ABO blood group in humans beings comprises of four blood groups i.e. A,B,AB,O
These blood groups are due to the presence of special antigens on the surface of RBC.
Antigens are controlled by gene I.
Gene I has three alleles.
A
I Dominant
Gene I B
Chromosome I Dominant
no 9 O
I or i Recessive
Gene- I

A O
I I
B I or i
A antigen B antigen No antigen
ABO BLOOD GROUP
69
Human is diploid organism so each person possesses any two of the three I gene alleles.

Genotype Blood group

IAIA
A
A O
I I

IBIB
B
B O
I I

IAIB AB (Codominance)

IOIO O

6 Genotypes 4 Phenotypes
5. The chromosome number is fixed for all normal organisms leading to species specification
whereas any abnormality in the chromosome number of an organism results into abnormal
individuals. For example, in humans 46 is the fixed number of chromosomes both in male and
female. In male it is '44 + XY' and in female it is '44 + XX'. Thus the human male is
heterogametic, in other words produce two different types of gametes one with '22 + X'
chromosomes and the other with '22 + Y' chromosomes respectively. Human female, on the other
hand is homogametic i.e. produces only on type of gamete with '22 + X' chromosomes only.
Sometimes an error may occur during meiosis of cell cycle, where the sister chromatids fail to
segregate called nondisjunction, leading to the production of abnormal gametes with altered
chromosome number. On fertilisation such gametes develop into abnormal individuals.
(a) State what is aneuploidy.

(b) If during spermatogenesis, the chromatids of sex chromosomes fail to the segregate during

meiosis, write only the different types of gametes with altered chromosome number that

could possibly be produced.

(c) A normal human sperm (22 + Y) fertilises an ovum with karyotype '22 + XX'. Name the

disorder the offspring thus produced would suffer from and write any two symptoms of the

disorder. [CBSE 2023]

OR

(c) Name a best known and most common autosomal aneuploid abnormality in human and
write any two symptoms.
70
Ans. (a) Failure of segregation of chromatids during cell division cycle results in the gain or loss of a
chromosome(s), called aneuploidy.
(b) 22+XY and 22 + 0
(c) 44+XXYKlinefelter’s Syndrome - This genetic disorder is also caused due to the presence of
an additional copy of X-chromosome resulting into a karyotype of 47, XXY. Such an
individual has overall masculine development, however, the feminine development
(development of breast, i.e., Gynaecomastia) is also expressed such individuals are sterile.
OR
(c) Down’s Syndrome : The cause of this genetic disorder is the presence of an additional copy
of the chromosome number 21 (trisomy of 21). The affected individual is short statured with
small round head, furrowed tongue and partially open mouth. Palm is broad with
characteristic palm crease. Physical, psychomotor and mental development is retarded.
6. The case of Down's syndrome in humans is: [CBSE Term-I 2022]
(1) Extra copy of an autosome (2) Extra copy of a sex chromosome
(3) Absence of an autosome (4) Absence of a sex chromosome
Ans. (1) Extra copy of an autosome
7. Which of the following features show the mechanism of sex determination in honey-bee?
[CBSE Term-I 2022]
(1) An offspring formed from the union of a sperm and egg develops as a male.
(2) Males have half the number of chromosomes than that of female.
(3) The females are diploid having 32 chromosomes.
(4) Males have father and can produce sons.
Ans. (2) Males have half the number of chromosomes than that of female.
8. Select the pair that is incorrect: [CBSE Term-I 2022]
(1) Sickle-cell anaemia : Autosome liked recessive
(2) Haemophilia : Autosome linked recessive trait
(3) Colour blindness : Sex linked recessive trait
(4)Thalassemia : Autosome linked recessive trait
Ans. (2) Haemophilia : Autosome linked recessive trait
9. An example of a human trait where a single gene can exhibit multiple phenotypic expression is :-
[CBSE Term-I 2022]
(1) Phenylketonuria (2) Cystic fibrosis
(3) Thalassemia (4) Haemophilia
Ans. (1) Phenylketonuria
10. Life cycle of Drosophila melanogaster is completed in :- [CBSE Term-I 2022]
(1) 7 days (2) 14 days (3) 21 days (4) 28 days
Ans. (2) 14 days
11. How many types of gametes would develop by an organism with genotype AaBBCcDD?
[CBSE Term-I 2022]
(1) 1 (2) 2 (3) 3 (4) 4
Ans. (4) 4

E 71
12. In Pisum sativum the flower colour may be Violet (V) or White (v). What proportion of the
offsprings in a cross of VV × vv would be expected to be violet? [CBSE Term-I 2022]
(1) 25% (2) 50% (3) 75% (4) 100%
Ans. (4) 100%
13. Which one of the gene pair is expected to give a ratio of 1 : 1 : 1 : 1 in the progeny of a
Mendelian Dihybrid cross? [CBSE Term-I 2022]
(1) AaBb × AbBb (2) AABB × AaBb
(3) AaBb × aabb (4) AABB × aabb
Ans. (3) AaBb × aabb
14. The progeny of a cross between two snapdragon plants heterozygous for flower colour, bearing
different coloured flower would be: [CBSE Term-I 2022]
(1) 50% pink, 50% white (2) 25% red, 50% pink, 25% white
(3) 50% red, 50% white (4) 75% red, 25% white
Ans. (2) 25% red, 50% pink, 25% white
15. Study the given pedigree of a family and select the trait that shows this pattern of inheritance
[CBSE Term-I 2022]
(1) Autosomal recessive, Phenylketonuria
(2) Sex-linked recessive, Colour blindness
(3) Autosomal dominant, Myotonic dystrophy
(4) Sex-linked dominant, Vitamin D resistant rickets
Ans. (1) Autosomal recessive, Phenylketonuria

16. A child with blood group A has father with blood group B and mother with blood group AB.
What would be the possible genotypes of parents and the child? Choose the correct option:

[CBSE Term-I 2022]

Father Mother Child

(1) IAi IBi IAi

(2) IAIB IAi IAIA

(3) IBi IAIB IAi

(4) IBIB IAIB IAIA

Ans. (3) IB i IAIB IAi

72 E
17. In a dihybrid Mendelian cross, garden pea plants heterozygous for violet flowers and round seeds
are crossed with homozygous white flowers and wrinkled seeds. The genotypic and phenotypic
ratio of F1 progeny would be: [CBSE Term-I 2022]
(1) 9 : 3 : 3 : 1 (2) 1 : 2 : 2 : 1 (3) 1 : 1 : 1 : 1 (4) 3 : 1
Ans. (3) 1 : 1 : 1 : 1
18. Colour blindness is a sex linked recessive trait in humans. A man with normal colour vision
marries a women who is colourblind. What would be the possible genotypes of the parents, the
son and the daughter of this couple? [CBSE Term-I 2022]
Mother Father Daughter Son
C C
(1) XX X Y X X XY
(2) XCXC XCY XCXC XCY
(3) XCX XCY XCX XY
C C C C
(4) X X X Y X X XCY
Ans. (4) XCXC XCY XCX XCY
19. How many types of gametes can be produced in a diploid organism which is heterozygous for 4
loci? [CBSE Term-I 2022]
(1) 4 (2) 8 (3) 16 (4) 32
Ans. (3) 16
20. The recombinant Frequency between the four linked genes is as follows:
(i) between X and Y is 40%. (ii) between Y and Z is 30%.
(iii) between Z and W is 10%. (iv) between W and X is 20%.
Select the option that shows the correct order of the position of W, X, Y and Z genes on the
chromosome: [CBSE Term-I 2022]
(1) Y – X – Z – W (2) Y – W – Z – X (3) X – Y – Z – W (4) Z – X – Y – W
Ans. (2) Y– W – Z – X
21. A snapdragon plant with violet flowers was crossed with another such plant with white
flowers. The F1 progeny obtained had pink flowers. Explain, in brief, the inheritance
pattern seen in offsprings of F1 generation? [CBSE IMP Question]
Ans. The inheritance is incomplete dominance. In this, a new intermediate phenotype between the two
original phenotypes is obtained. One allele for a specific trait is not completely expressed over
the other allele for the same trait.
22. Karyotype of a child shows trisomy of chromosome number 21. Identify the disorder and
state the symptoms which are likely to be exhibited in this case. [CBSE IMP Question]
Ans. Disorder - Down’s Syndrome
Symptoms: The affected individual is short statured with small round head, has furrowed tongue,
partially open mouth, Palm is broad with characteristic palm crease, Physical, psychomotor and
mental development is retarded (any three symptoms)
E 73
23. How would you find out the genotype of a pea plant with violet flowers? Explain with the
help of Punnets’ square showing crosses. [CBSE IMP Question]
Ans.

W W
W W

W W

W w

Diagrammatic representation of a test cross


24. What is aneuploidy? Name a chromosomal disorder in humans caused due to (a) gain of an
autosome, and (b) loss of a sex chromosome in females. [CBSE 2020]
Ans. Aneuploidy is Failure of segregation of chromatids (during cell division), resulting in gain or loss
of a chromosome (s)
(a) Gain of an autosome - Down’s syndrome
(b) Loss of a sex chromosome - Turner’s syndrome
25. A normal couple has their first child, who is haemophilic. Work out a cross to show how it
is possible. State the possibility of the normal and the haemophilic children, along with
their sexes, that can be born to them. [CBSE 2020]
Ans.
XXh XY
Carrier Female Normal Male

Gametes X Xh X Y

XX XY XXh XhY
Normal daughter Normal Son Carrier daughter Haemophilic Son
25% 25% 25% 25%

74 E
26. State Mendel’s Law of Independent Assortment. [CBSE 2019]

Ans. When two pairs of traits (characters) are combined in a hybrid, segregation of one pair of

characters is independent of the other pair of characters.

27. Write one example each of organisms exhibiting (i) male heterogamety, and (ii) female

heterogamety. [CBSE 2019]

Ans. (i) Human / Drosophila / Grasshopper (ii) Birds / Chicken

28. Why is the frequency of red-green colour blindness more in human males than in females?

Explain. [CBSE 2019]

Ans. Gene for colour blindness is located on X chromosome in human, it is a recessive gene, since

human males have single X chromosome the recessive gene always expresses when present,

where as in human females as they have two X chromosomes the trait is expressed only if both

the sex chromosomes have this recessive gene.

29. How is polygenic inheritance different from pleiotropy ? Give one example of each.

[CBSE 2019]

Ans.
Polygenic inheritance Pleiotropy

a) A single trait influenced by many a) A single gene can exhibit multiple


genes phenotypic expression

b) e.g human height/ skin colour in b) e.g phenylketonuria , characterised


humans controlled by three or by mental retardation / reduction
more genes. in hairs and / skin pigmentation

30. Write the sex of a human having XXY chromosomes with 22 pairs of autosomes. Name the

disorder this human suffers from. [CBSE 2018]

Ans. Male, Klinefelter’s syndrome

E 75
(C) MULTIPLE CHOICE QUESTIONS
1. An organism’s genetic constitution is called its
(1) Genotype (2) Phenotype (3) Holotype (4) None of these
Ans. (1) Genotype
2. An organism with two identical alleles for a given trait is
(1) Homozygous (2) Segregating (3) Dominant (4) A hermaphrodite
Ans. (1) Homozygous
3. What type of gametes will be formed by genotype RrYy?
(1) RY, Ry, YY, ry (2) RY, Ry, ry, ry (3) RY, Ry, rY, ry (4) Rr, RR, Yy, YY
Ans. (3) RY, Ry, rY, ry
4. Which genotype characterizes an organism that is heterozygous for two genes?
(1) RRYy (2) RrYY (3) RRYY (4) RrYy
Ans. (4) RrYy
5. Which of the following is the dominant character according to Mendel?
(1) Dwarf plant and yellow fruit (2) Terminal fruit and wrinkled seed
(3) White testa and yellow pericarp (4) Green coloured pod and rounded seed
Ans. (4) Green coloured pod and rounded seed
6. Self-pollination between Tt and Tt plants results into the genotype ratio of
(1) 3 : 1 (2) 1 : 2 : 1 (3) 1 : 3 (4) 4 : 0
Ans. (2) 1 : 2 : 1
7. Mendel’s law of heredity can be explained with the help of
(1) Mitosis (2) Meiosis
(3) Both mitosis and meiosis (4) None of these
Ans. (2) Meiosis
8. Genes do not occur in pair in :-
(1) Gametes (2) Embryo (3) Zygote (4) Somatic Cell
Ans. (1) Gametes
9. When heterozygous tall plants are self-pollinated, then tall and dwarf plants are obtained. This
explains
(1) Law of purity of gamete (2) Law of segregation
(3) Division in spores (4) Independent assortment
Ans. (2) Law of segregation

TG: @Chalnaayaaar
76 E

TG: @Chalnaayaaar
10. Mendel’s principle of segregation was based on the separation of alleles in the garden pea during
(1) Pollination (2) Embryonic development
(3) Seed formation (4) Gamete formation
Ans. (4) Gamete formation
11. A cross between a homozygous recessive and a heterozygous plant is called
(1) Monohybrid cross (2) Dihybrid cross (3) Test cross (4) Back cross
Ans. (3) Test cross
12. Cross between F1 plant and recessive female plant is called
(1) Back cross (2) Test cross (3) Out cross (4) Mutation
Ans. (2) Test cross
13. In F2 generation, a phenotypic ratio of 1 : 1 : 1 : 1 exhibits
(1) Back cross (2) Monohybrid test cross
(3) Lethality (4) Dihybrid test cross
Ans. (4) Dihybrid test cross
14. Which of the following depicts the Mendel’s dihybrid ratio?
(1) 3 : 1 (2) 9 : 3 : 3 : 1 (3) 9 : 7 (4) 15 : 1
Ans. (2) 9 : 3 : 3 : 1
15. Inheritance of ABO blood group system is an example of
(1) Multiple allelism (2) Partial dominance (3) Epistasis (4) Dominance
Ans. (1) Multiple allelism
16. Genotype of blood group ‘A’ will be
(1) IAIA (2) IBIB (3) IAIA or IAIO (4) IAIO
Ans. (3) IAIA or IAIO
17. Blood group ‘B’ will have alleles
(1) ii (2) IAIA (3) IBIB (4) IAIB
Ans. (3) IBIB
18. Identify A to E in this figure.

A
B
C
D
E

(1) A: Petal; B: Stigma; C: Anther; D: Stamen; E: Carpel


(2) A: Anther; B: Petal; C: Stigma; D: Carpel; E: Stamen
(3) A: Carpel; B: Stamen; C: Anther; D: Stigma; E: Petal
(4) A: Stigma; B: Petal; C: Stamen; D: Anther; E: Carpel
Ans. (1) A: Petal; B: Stigma; C: Anther; D: Stamen; E: Carpel

E 77
19. The below diagram represents

(1) Back cross (2) Out cross (3) Test cross (4) Dihybrid cross

Ans. (3) Test cross

20. If in a dihybrid cross, Mendel had used two such characters which have linked, he would have

faced difficulty in explaining the results on the basis of

(1) Law of segregation (2) Law of multiple factor hypothesis

(3) Law of independent assortment (4) Law of dominance

Ans. (3) Law of independent assortment

78 E
(D) ASSERTION & REASON QUESTIONS
 Directions: In the following questions, a statement of assertion is followed by a statement of
reason. Mark the correct choice as:
(1) If both Assertion and Reason are true and Reason is the correct explanation of Assertion.
(2) If both Assertion and Reason are true but Reason is not the correct explanation of Assertion.
(3) If Assertion is true but Reason is false.
(4) If both Assertion and Reason are false.
1. Assertion: A good example of multiple alleles is ABO blood group system.
Reason: When IA and IB alleles are present together in ABO blood group system, they both
express their own types of sugar.
Ans. (2)
2. Assertion : In Mirabilis, selfing of F1 pink flower plants produces same phenotypic & genotypic
ratio.
Reason : Flower colour gene shows incomplete dominance.
Ans. (1)
3. Assertion: By means of dihybrid cross, the law of independent assortment can be studied.
Reason: The law of independent assortment is applicable only to linked genes.
Ans. (3)
4. Assertion : In case of incomplete linkage, linked gene show new combination along with
parental combination.
Reason : In case of incomplete linkage, linked genes are separated by crossing over.
Ans. (1)
5. Assertion: When white eyed, yellow bodied Drosophila females were hybridized with red eyed,
brown-bodied males; and F1 progeny was intercrossed, F2 ratio deviated from 9 : 3 : 3 : 1.
Reason: When two genes in a dihybrid are on the same chromosome, the proportion of parental
gene combinations are much higher than the non parental type.
Ans. (1)
6. Assertion: The maximum frequency of recombination, that can result from crossing over
between linked genes, is 50 percent.
Reason: Higher frequency of crossing over is shown in linked genes if distance between them is
longer.
Ans. (2)
7. Assertion : The linked genes tend to get inherited together.
Reason : The link between them fails to break.
Ans. (1)
8. Assertion : The females are diploid having 32 chromosomes in honeybee.
Reason : The male are haploid having 16 chromosomes in honeybee.
Ans. (2)
9. Assertion : Most of experiments regarding sex determination were done on Drosophila.
Reason : It is fruit fly.
Ans. (2)
10. Assertion : In birds, the chromosome composition of the egg determines the sex.
Reason : Female birds are heterogametic.
Ans. (1)
E 79
(E) VERY SHORT ANSWER QUESTIONS
1. How many type of gametes produced by the individual with genotype AABBCCDD and
AaBbCcDd?
Ans. One type of gamete by individual AABBCCDD and sixteen (24 = 16) type of gametes by
individual AaBbCcDd.
2. Mention the type of allele that expresses itself only in homozygous state in an organism.
Ans. Recessive allele.
3. Write the percentage of F2 homozygous and heterozygous populations in a typical
monohybrid cross.
Ans. The ratio of a typical monohybrid cross is 1 : 2 : 1 where 50% are homozygous and 50% are
heterozygous populations. (25% homozygous dominant, 25% homozygous recessive).
4. Name the type of cross that would help to find the genotype of a pea plant bearing violet
flowers.
Ans. Test cross.
5. A cross was carried out between two pea plants showing the contrasting traits of height of
the plant. The result of the cross showed 50% of parental characters. Name the type of
cross.
Ans. Test cross
6. Why, in a test cross, did Mendel cross a tall pea plant with a dwarf pea plant only?
Ans. To determine the genotype of the tall plant, whether it is homozygous dominant or heterozygous,
as dwarfness is a recessive trait which is expressed only in homozygous condition and he was
sure of genotype of dwarf plant.
7. Name the stage of cell division where segregation of an independent pair of chromosomes
occurs.
Ans. Anaphase-I of Meiosis-I.
8. In a dihybrid cross, when would the proportion of parental gene combinations be much
higher than non-parental types, as experimentally shown by Morgan and his group?
Ans. When the genes are linked.
9. If the frequency of a parental form is higher than 25% in dihybrid test cross, what does
that indicate about the two genes involved?
Ans. It shows that the two genes are linked.
10. If two genes are located far apart from each other on a chromosome, how the frequency of
recombination will get affected?
Ans. Frequency of recombination will be higher.
80 E
(F) SHORT ANSWER QUESTIONS
1. Name the base change and the amino acid change, responsible for sickle cell anaemia.
Ans. AG changes as GUG, Glutamic acid is substituted by Valine.
2. Name any one plant that shows the phenomenon of incomplete dominance during the
inheritance of its flower colour.
Ans. Snapdragon or Antirrhinum sp.
3. What is point mutation and frame shift mutation ?
Ans. Point Mutation : Arises due to change in a single base pair of DNA e.g., sickle cell anaemia.
Frame shift mutation : Deletion or insertion/ duplication/addition of one or two bases in DNA.
4. What is pedigree analysis?
Ans. A pedigree is a record of inheritance of a specific genetic trait for two or more generations which
is presented in the form of a diagram or family tree. Pedigree analysis is an analysis of several
generations of a family which is used on human beings.
5. Write the sex of a human having XXY chromosomes with 22 pairs of autosomes. Name the
disorder this human suffers from.
Ans. Male, Klinefelter’s syndrome
6. A haemophilic father can never pass the gene for haemophilia to his son. Explain.
Ans. It is a sex linked recessive disorder in which X-chromosome has the haemophilic gene.
 Son inherits a Y chromosome from father and gene for haemophilia is not present on Y
chromosome
7. A colour blind boy is born to a couple with a normal colour vision. Write the genotype of
the parents.
Ans. Father - XY, Mother -XXC
8. What is a test cross ? How can it decipher the heterozygosity of a plant?
Ans. A cross to analyse whether genotype of dominant individual is homozygous or heterozygous
On crossing with a recessive parent, if 50% of progeny have dominant trait and 50% have
recessive trait then the plant is said to be heterozygous
9. What happens when chromatids fail to segregate during cell division cycle? Explain your
answer with an example.
Ans. Failure of segregation of chromatids during cell division cycle results in the gain or loss of
chromosome called aneuploidy.
E.g. (i) Down' syndrome results in the gain of extra copy of chromosome 21
(ii) Turner's syndrome results due to loss of an X-chromosome in human female.
10. ABO blood groups is a good example of co-dominance. Justify.
Ans. – ABO blood group in humans is contributed by gene 'I' that has 3 alleles 'IA' 'IB' and 'i'.
 Because human beings are diploid each person has two of the three alleles.
 IA and IB produce two different types of sugar while allele i does not produce sugar on the plasma
membrane of RBC.
 When IA and IB are present they both express their own type of sugar- this is codominance
E 81
(G) LONG ANSWER QUESTIONS
1. Given below is the representation of a relevant part of amino acid composition of the
-chain of haemoglobin, related to the shape of human red blood cells.
GTG
CAC

GUG

Hbs peptide
(a) Is this representation of the sequence of amino acids indicating a normal human or a sufferer
from a certain blood related genetic disease? Give reason in Support of your answer.
(b) Why is the disease referred to as a Mendelian disorder? Explain.
Ans. (a) Suffered human, because at 6th position of the chain Valine is present.
(b) Alteration / mutation occurs in a single gene, at 6th position Glutamic acid replaces Valine.
2. (a) How would you find out whether a given tall garden pea plant is homozygous or
heterozygous?
Substantiate your answer with the help of Punnett squares.
(b) Given below are the F2-phenotypic ratios of two independently carried Monohybrid crosses:
(i) 1 : 2 : 1 (ii) 3 : 1
Mention what does each ratio suggest.
Ans. (a) By Test Cross / crossing the organism with dominant phenotype (but unknown genotype) with
the recessive individual.

(b) (i) Incomplete Dominance (ii) Dominance


3. Give the answer of following questions.
(a) Why are thalassemia and haemophilia categorized as Mendelian disorders ? Write the
symptoms of these diseases. Explain their pattern of inheritance in humans.
(b) Write the genotypes of the normal parents producing a haemophilic son.
82 E
Ans. (a) The thalassemia and haemophilia categorized as Mendelian disorder because these are caused
due to alteration or mutation in single gene.
S.No. Haemophilia Thalassemia
1 Single protein involve in the clotting of Defects in the synthesis of globin leading to
blood is affected formation of abnormal haemoglobin
2 Sex linked recessive disorder Autosomal recessive disorder
3 Blood does not clot Results in anaemia
Inheritance pattern:
• The thalassemia is autosomal recessive disorder which inheritate like non criss-cross form
both heterozygous parents to any progeny or both recessive parents to all progeny.
• The haemophilia is Sex linked recessive disorder so that inheritate like criss-cross, from
carrier mother to affected son to carrier daughter. In haemophilia the the female are carrier
while male are patient, female may be patient in case of their parents (both male and
female) carry gene for haemophilia
(b) The genotype of parents of haemophilic son is that father is normal (XY) and mother is
carrier( ).
4. (a) Compare in any three ways the chromosomal theory of inheritance as proposed by
Sutton and Bovery with that of experimental results on pea plant presented by
Mendel.
(b) Mendel published his work on inheritance of characters in 1865 but it remained
unrecognized till 1900. What are the reasons behind this.
Ans. (a)
S.No. Sutton And Bovery Mendel
1 Chromosomes occur in pairs Factors occur in pairs
2 Chromosomes segregate at the time of Factors segregate at gamete formation
gamete formation such that only one of stage and only one of each pair is
each pair is transmitted to a gametes transmitted to a gamete
3 Independent pairs of chromosomes One pair of factors segregate
segregate independently of each other independently of another pairs

(b) There are following reasons behind unrecognization of Mendel work.


(i) The communication was not easy in those days and his work could not be widely published.

(ii) His concept of genes or factor as stable and discrete unit that control the expression of traits and,
of the pair of allele which did not blend each other, was not accepted by his contemporaries as an
explanation for the apparently continuous variations seen in nature.
(iii) Mendel’s approach of using mathematics to explain biological phenomena was totally new and
unacceptable to many of the biologists of his time.
(iv) Finally, though Mendel’s work suggested that factors (genes) were discrete units, he could not
provide any physical proof for the existence of factor.

E 83
5. The given below is the figure of sex determination into three different animal
(A ,B and C). Observe it and give the answer of question that follow.

(a) Which of the animal show male heterogamety?


(b) Is the sperm or egg responsible for the sex of the chicks?
(c) In our society the women are often blamed for giving birth to daughters. Can you explain
why this is not correct?
Ans. (a) The male parent of both A and B show male haterogamety that they formed two type of
gametes.
(b) The egg is responsible for the sex of chicks because the female is heterogametic which
formed two different type of gametes with sex chromosome ( Z and W) while male
formed only one type of gametes (Z).The combination of W from female with Z of male
formed the female progeny.
(c) This is not correct to blame women for giving birth to daughter. The male sperm contain
either X or Y chromosome whereas the female egg contain only X chromosome. At the
time of fertilization, sperm with Y chromosome combine with egg containing X
chromosome formed which would be male. Thus scientifically sex of the baby is
determined by the father and not by the mother as blamed in our society.

84 E
(H) CASE-STUDY BASED QUESTIONS
1. Study the following and answer the questions given below :

During a study of inheritance of two genes, teacher asked students to perform an experiment. The

students crossed white eyed, yellow bodied female Drosophila with a red eyed, brown bodied

male Drosophila (i.e., wild). They observed that progenies in F2 generation had 1.3 percent

recombinants and 98.7 percent parental type combinations. The experimental cross with results is

shown in the given figure.

Cross A Cross B

Parental

F1 generation

Gametes

F2 generation

85
(i) Identify in which of the crosses, the strength of linkage between the gene is higher. Give
reasons in support of your Answer?
Ans. The strength of linkage is higher in the cross A than in cross B because linkage is higher when
two genes are present closely on the same chromosome than those gene which are far a part. In
cross B the chance of crossing over or recombination are higher because the genes are lossely
linked.
(ii) Write the scientific name of fruitfly?
Ans. Drosophila melanogaster
(iii) Why did morgan prefer to work with fruit flies for his experiments? State any two reasons.
Ans. (i) It complete its life cycle in about two weeks. (ii) a single mating could produce a large
number of progeny flies.
(iv) How did morgan show the deviation in inheritance - Pattern in Drosophila with respect to
this law.
Ans. Morgan found that linkage is an exception to the law of Independent assortment.
2. Study the following and answer the questions given below:
The given below is inheritance pattern of plant height (Subject-1) and flower colour in
snapdragon plant (Subject-2) study it give the answer of asked questions.
Subject-1 Subject-2

P generation
Red (RR) White (rr)

Gametes
Tall Dwarf
TT tt
Gametes T t Gametes F1 generation

T t All pink (Rr)


Tt
F1 generation Gametes Gametes
Tt Tt

Tt F2 generation

Tall Selfing Tall


Tt Tt
Gametes T T Gametes

t t
TT
F2 generation
Tt Tt

tt
A punnett square

86
(i) What type of cross represented by subject - 1 and subject - 2 respectively?
Ans. Subject-1  Monohybrid cross
Subject-2 Incomplete Dominance
(ii) What are genotype and phenotype ratio of F2 generation of subject - 1?
Ans. Genotype = 1 : 2 : 1
Phenotype = 3 : 1
(iii) The appearance of pink flowers is not known as blending why? (Subject-2)
Ans. Blending is the mixing of two colours but in this example red and white colours appear
indepently at cellular level. Thus no blending occurs. The red and white colours reappear in F 2
generation.
(iv) Phenotypic and genotypic ratio is similar in case of -
Ans. Incomplete dominance (Subject-2)
3. Read the following and answer the questions given below:
Haemophilia is a sex linked disease which is also known as bleeder's disease as the patient will
continue to bleed even from a minor cut since he or she does not possess the natural phenomenon
of blood clotting due to absence of anti-haemophilic globulin or factor VIII and plasma
thromboplastin factor IX essential for it. As a result of continuous bleeding the patient may die of
blood loss. Colour blindness is another type of sex linked trait in which the eye fails to
distinguish red and green colours. Vision is however, not affected and the colour blind can, lead a
normal life, reading, writing and driving (distinguishing traffic lights by their position).
(i) Which of the blood clotting factors are absent in the patient who suffered from
haemophilia?
Ans. Anti-haemophilic globulin or factor VIII and plasma thromboplastin factor IX essential for it.
(ii) A man whose father was colour blind and mother was normal marries a woman whose
father was haemophilic and mother was normal . Write the ration of possible progenies.
Ans. 25% male progenies and 25% female progenies carry the gene of haemophilia.
(iii) If a haemophilic man marries a woman whose father was haemophilic and mother was
normal then what will be normal, carrier and haemphilic in female progenies.?
Ans. Of the total number of daughters,0% daughter are normal 50% daughters are carriers and 50%
are haemophilic.
(iv) A hemophilic father can never pass the gene for hemophilia to his son. Explain.
Ans. The hemophilia is X-linked recessive disorder where female is carrier while male is patient in
case of present of single X chromosome. The X chromosome of male only passed to daughter and
Y to son. So that the son of hemophilic father is never be a patient.
87
(v) About 8% male the human population are colourblind whereas only 0.4% of females are
colour – blind. Write an explanation to show how it is possible.
Ans. Colour blind is a X linked recessive disorder. Male have higher chances of getting affected in
comparison to females because male have only one X with Y chromosome and female have XX
chromosome Thus for a female to get affected by colour blindness,she has to have the mutate
gene on both the X chromosome while male can be affected, if they carry it on the single X
chromosome.

4. Read the following and answer the questions given below:

Turner's syndrome is an example of monosomy. It is formed by the union of an allosome free egg
and a normal 'X' containing sperm or a normal egg and an allosome free sperm. The individual
has 2n = 45 chromosomes = (44 + X0) instead of 46. Such individuals are sterile females who
have rudimentary ovaries, under developed breasts, small uterus, short stature, webbed neck and
abnormal intelligence. They may not menstruate or ovulate . This disorder can be treated by
giving female sex hormone to the women from the age of puberty to make them develop breasts
and have menstruation. This makes them feel more normal.

(i) Write the karyotype of Turner's syndrome.

Ans. 44+ XO

(ii) Write the number of Bar body present in a female with Turner's syndrome.

Ans. 0 Bar Body.

(iii) What is the reason behind a person to be Turner's syndrome?

Ans. It is due to the union of an allosome free egg and a normal 'X' containing sperm or a normal egg
and an allosome free sperm.

(iv) Write the symptoms of Turner's syndrome.

Ans. Such individuals are sterile females who have rudimentary ovaries, under developed breasts,
small uterus, short stature, webbed neck and abnormal intelligence. They may not menstruate or
ovulate.

(v) Write the treatment for Turner's syndrome.

Ans. This disorder can be treated by giving female sex hormone to the women from the age of puberty
to make them develop breasts and have menstruation. This makes them feel more normal.

88
CHAPTER 5
(A) NCERT QUESTIONS & SOLUTIONS
1. Group the following as nitrogenous bases and nucleosides :
Adenine, Cytidine, Thymine, Guanosine, Uracil and Cytosine.
Ans. Nitrogenous bases-Adenine, Thymine, Uracil and Cytosine
Nucleosides-Cytidine and Guanosine.
2. If a double stranded DNA has 20 per cent of cytosine, calculate the per cent of adenine in
the DNA.
Ans. Cytosine 20%, therefore Guanine = 20%
According to Chargaff's rule,
A+T =100 - (G + C)
A+T =100-40. Since both adenine and thymine are in equal amounts
60
Thymine = Adenine = = 30 %
2
3. If the sequence of one strand of DNA is written as follows: [IMP.]
5'-ATGCATGCATGCATGCATGCATGCATGC-3'
Write down the sequence of complementary strand in 5'3' direction.
Ans. In 3'  5' direction, 3-TACGTACGTACGTACGTACGTACGTACG-5'
In 5'  3' direction, 5'-GCATGCATGGATGCATGGATGCATGCAT-3
4. If the sequence of the coding strand in a transcription unit is written as follows: [IMP.]
5'-ATGCATGCATGCATGCATGCATGCATG-3'
Write down the sequence of mRNA.
Ans. 5'-AUGCAUGCAUGCAUGCAUGCAUGCAUG-C-3'
5. Which property of DNA double helix led Watson and Crick to hypothesis semi-conservative
mode of DNA replication? Explain.
Ans. Watson and crick observed that the two DNA strands are antiparallel, and have opposite polarity.
This means that 5' phosphate of one strand faces that 3' hydroxyl group of other strand and that
the 5' phosphate group of two strands are present in opposite position. The antiparallel
arrangement of two helices allows hydrogen bonding between amino and carbonyl group of
complementary base pairs. 'This led them to the hypothesis of the semi-conservative mode of
DNA replication where in two strands of DNA first separate from each other followed by
copying of each template strands to form DNA molecules each carrying one parental strand and
newly synthesized strands.
89
6. Depending upon the chemical nature of the template (DNA or RNA) and the nature of

nucleic acids synthesized from it (DNA or RNA), list the types of nucleic acid polymerases.

Ans. DNA template

(i) DNA polymerase for DNA replication.

(ii) RNA polymerase for RNA synthesis or transcription.

RNA template

(i) RNA-dependent RNA polymerase for synthesis of RNA in some RNA viruses.

(ii) Reverse transcriptase to synthesise cDNA (complementary DNA) over RNA template.

7. How did Hershey and Chase differentiate between DNA and protein in their experiment

while proving that DNA is the genetic material?

Ans. Hershey and Chase (1952) conducted experiments in bacteriophage to prove that DNA is the

genetic material.

Procedure:

Some bacteriophage virus were grown on a medium that contained radioactive phosphorus

(32P) and some in another medium with radioactive sulphur (35S).

(a) Viruses grown in the presence of radioactive phosphorus (32P) contained radioactive DNA.

(b) Similar viruses grown in presence of radioactive sulphur (35S) contained radioactive problem.

(c) Both the radioactive virus types were allowed to infect E.coli separately.

(d) Soon after infection, the bacterial cells were gently agitated in blender to remove viral coats from

the bacteria.

(e) The culture was also centrifuged to separate the viral particle from the bacteria cell.

Observation and Conclusions:-

(a) Only radioactive (32P) was found to be associated with bacterial cell, whereas radioactive (35S)

was only found in surrounding medium and not in the bacterial cell.

(b) This indicates that only DNA and not the protein coat entered the bacterial cell.

(c) This proves that DNA is the genetic material which is passed from virus to bacteria and not

protein.

90 E
Bacteriophage Radioactive (32P)
Radioactive (35S) labelled labelled DNA
protein capsule

1. Infection

2. Blending

3. Centrifugation
No Radioactive (35S) Radioactive (32P)
detected in cells detected in cells
+ +
Radioactive (35S) No Radioactivity
detected in supernatant detected in supernatant

Fig. The Hershey – Chase Experiment

8. Differentiate between the following : [IMP.]


(a) Repetitive DNA and satellite DNA
(b) mRNA and tRNA
(c) Template strand and coding strand
Ans. (a) Repetitive DNA and satellite DNA :-
S.No. Repetitive DNA satellite DNA
1 DNA in which certain base sequence are DNA in which portion of the gene is
repeated many times are called repetitive tandemly repeated is called satellite
DNA. DNA.
2 Repetitive DNA sequence are transcribed Satellite DNA sequences are not
transcribed
(b) mRNA and tRNA:-
S.No. mRNA tRNA
1 It is linear. It is clover –leaf shaped.
2 It carries coded information. It carries information for association with
an amino acid and an anticodon for its
incorporation in a polypeptide.
3 mRNA undergoes additional processing, It does not required any processing.
capping and tailing splicing .
4 Nitrogen bases are unmodified Nitrogen bases may be modified.-

E 91
(c) Template strand and coding strand :-
S.No. Template strand Coding strand
1 It is the strand of DNA which taken part It is the strand that dose not take part in
in transcription. transcription.
2 The polarity is 3' 5' The polarity is 5'  3'.
3 Nucleotide sequence is complementary. The nucleotide sequence is same as the
one present in mRNA except for presence
of thymine instead of uracil.
9. List two essential roles of ribosome during translation.
Ans. Two essential roles of ribosome during translation are:
(i) One of the rRNA (23S in prokaryotes) acts as a peptidyl transferase ribozymne for formation of
peptide bonds
(ii) Ribosome provides sites for attachment of mRNA and charged (RNAs for polypeptide
synthesis).
10. In the medium where E.coli was growing, lactose was added, which induced the lac operon.
Then, why does lac operon shut down some time after addition of lactose in the medium?
Ans. It is because the repressor protein binds to the operator region of the operon and prevent RNA
polymerase from transcribing the operon.

Figure - The lac Operon


11. Explain (in one or two lines) the function of the following :
(a) Promoter (b) tRNA (c) Exons
Ans. (a) Promoter : It is the segment of DNA which lies adjacent to the operator and functions as the
binding site for RNA polymerase tocarry transcription if allowed by operator.
(b) tRNA : It acts as an adaptor molecule that picks up a particular amino acid from cellular poll
and takes the same over to site A of mRNA for incorporation into polypeptide chain.
(c) Exons : These are the coding segments present in primary transcript which after splicing
joined to form functional mRNA.
92 E
12. Why is the Human Genome Project called a mega project?
Ans. Human Genome Project is called a mega project because of following reasons:
• Sequencing of more than 3 × 109 bp.
• Identification of all the approximately 20,000 – 25,000 genes in human DNA.
• High expenditure of more than 9 billion US dollars.
• Identification of all the alleles of genes and their functions.
• Storage of data for sequencing would require space equal to 3300 books of 1000 pages each if
each page will consist of 1000 letters.
13. What is DNA fingerprinting? Mention its application.
Ans. DNA fingerprinting is the technique to determine the relationship between by studying the
similarity and dissimilarity of VNTR (variable number of tandem repeats). its applications are:
• It is used as a tool in forensic tests to identify criminals.
• To settle paternity disputes.
• To identify racial groups to study biological evolution.
14. Briefly describe the following:
(a) Transcription (b) Polymorphism (c) Translation (d) Bioinformatics
Ans. (a) Transcription: It is the formation of RNA over the template of DNA. It forms
single-stranded RNA which has a coded information similar to the sense or coding strand of
DNA with the exception that thymine is replaced by uracil.
(b) Polymorphism: Genetic polymorphism means occurrence of genetic material in more than
one form. It is of two major types, i.e allelic polymorphism and SNP.
Allelic polymorphism: Allelic polymorphism occurs due to multiple alleles of a gene Allele
possess different mutations which alter the structure and function of a protein formed by as a
result, change phenotype may occur.
SNPs or single nucleotide polymorphism: Over 1.4 million single-base DNA differences
have been observed in human beings. According to SNP every human being is unique. SNP is
very useful for locating alleles, identifying disease –associated sequences and tracing human
history.
(c) Translation: It is the process during which the genetic information which is stored in the
sequence of nucleotides in a mRNA molecules is converted following direction of the genetic
code into the sequence of amino acids in the polypeptide. It takes place in cytoplasm in both
eukaryotes and prokaryotes.
(d) Bioinformatics: The science which deals with handling, storing of huge information of
genomics as databases, analysing, modeling and providing various aspects of biological
information especially the molecules connected with genomics and proteomics is called
bioinformatics.

E 93
(B) PREVIOUS YEAR QUESTIONS
1. Given below is the restriction site of a restriction endonuclease Pst-I and the cleavage sites on a
DNA molecule. [CBSE 2023]

5' C – T – G – C – A – G 3'
3' G – A – C – G – T – C 5'

Choose the option that gives the correct resultant fragments by the action of the enzyme Pst-I.
(1) 5' C – T – G C – A – G 3'
3' G – A – C – G – T C 5'
(2) 5' C – T G – C – A – G 3'
3' G – A – G – C T – C 5'
(3) 5' C – T – G – C A – G 3'
3' G – A – C – G T – C 5'
(4) 5' C – T – G – C – A G 3'
3' G A – C – G – T – C 5'
Ans. (4) 5' C – T – G – C – A____________ G 3'
3' G _______________ A – C – G – T – C 5'
2. Given below is a sequence of bases in mRNA of a bacterial cell. Identify the amino acid that
would be incorporated at codon position 3 and codon position 5 during the process of its
translation. [CBSE 2023]
3' AUCAGGUUUGUGAUGGUACGA 5'
(1) Phenylalanine, Methionine (2) Cysteine, Glycine
(3) Alanine, Proline (4) Serine, Valine
Ans. (1) Phenylalanine, Methionine
3. Human Genome Project (HGP) was a mega project launched in the year 1990 with some
important goals. [CBSE 2023]
(a) Enlist any four prime goals of HGP.
(b) Name any one common non - human animal model organism which has also been
sequenced thereafter.
Ans. (a) (i) Identify all the approximately 20,000-25,000 genes in human DNA;
(ii) Determine the sequences of the 3 billion chemical base pairs that make up human DNA;
(iii) Improve tools for data analysis;
(iv) Transfer related technologies to other sectors, such as industries;
(v) Address the ethical, legal, and social issues (ELSI) that may arise from the project.
(b) Many non-human model organisms, such as bacteria, yeast, Caenorhabditis elegans (a free
living non-pathogenic nematode), Drosophila (the fruit fly), plants (rice and Arabidopsis),
etc.
94 E
4.(a) (i) How and why is charging of tRNA essential in the process of transition?
(ii) State the function of ribosome as a catalyst in bacteria during the process of translation
(iii) Explain the process of binding of ribosomal units to mRNA during protein synthesis.
[CBSE 2023]
OR
(b) (i) Describe the dihybrid cross upto F2 generation as conducted by Gregor Mendel using
pure lines of Garden Pea for characters seed shape and seed colour. [CBSE 2023]
Ans. (a) (i) The amino acids are joined by a bond which is known as a peptide bond. Formation of a
peptide bond requires energy. Therefore, in the first phase itself amino acids are
activated in the presence of ATP and linked to their cognate tRNA–a process
commonly called as charging of tRNA or aminoacylation of tRNA to be more
specific.
Amino acid + ATP  enzyme
 Amino acyl AMP enzyme complex + PP(Pyrophosphate)
Amino acyl AMP enzyme complex + t-RNA   Amino acyl t-RNA complex
(ii) The ribosome consists of structural RNAs and about 80 different proteins. In its inactive
state, it exists as two subunits; a large subunit and a small subunit. When the small
subunit encounters an mRNA, the process of translation of the mRNA to protein begins.
There are two sites in the large subunit, for subsequent amino acids to bind to and thus,
be close enough to each other for the formation of a peptide bond. The ribosome also
acts as a catalyst (23S rRNA in bacteria is the enzyme- ribozyme) for the formation of
peptide bond.
(iii) Translation refers to the process of polymerisation of amino acids to form a polypeptide.
AUG

m-RNA
5’ 3’

Attachment of
smaller subunit of
ribosome(30s)

m-RNA
5’ 3’

30s 16s r-RNA

Attachment of First
AA1 charged t-RNA by
IF2 and GTP

AA1

UAC

Initiator t-RNA by IF2 and GTP

UAC

AUG
m-RNA
5’ 3’
30s 30S Ribosome m-RNA
t-RNA complex

E 95
Attachment of larger
subunit of
ribosomes(50s) by Mg+2
AA1
P A

P A 50s E

50s E AUG GGG


m-RNA
5’ 3’
30s

The process of binding of ribosomal units to mRNA


OR
(b) A cross in which there is study of inheritance of 2 character at a time. From this cross
Mendel wanted to observe the effect of one character over another character during
inheritance.

P generation
Round yellow Wrinkled green

Gametes

Round yellow
F1 generation
Selfing

Gametes Gametes

F2 generation

Phenotypic ratio

96 E
LAW OF INDEPENDENT ASSORTMENT
It is concluded from F2 generation of Dihybrid cross.
The law states that "When two pairs of traits are combined in a hybrid, segregation of one pair of
characters is independent of the other pair of characters."
Assortment occurs when non alleles are located on different homologous chromosome.
5. (a) Name and describe the steps involved in the technique widely used in forensics that
serves as the basis of paternity testing in case of disputes. [CBSE 2023]
Ans. (a) The technique widely used in forensics that serves as the basis of paternity testing in case of
disputes in DNA profiling or DNA fingerprinting. The steps involved in this technique are as
follows :
(i) isolation of DNA,
(ii) digestion of DNA by restriction endonucleases,
(iii) separation of DNA fragments by electrophoresis,
(iv) transferring (blotting) of separated DNA fragments to synthetic membranes, such as
nitrocellulose or nylon,
(v) hybridisation using labelled VNTR probe, and
(vi) detection of hybridised DNA fragments by autoradiography.
6. (i) Explain the process of aminoacylation of tRNA and its role in the process of translation.
(ii) How does initiation of the translation process occur in prokarotes? Explain.
(iii) Where are the untraslated regions located on mRNA and why? [CBSE 2023]
Ans. (i) in the first phase itself amino acids are activated in the presence of ATP and linked to their
cognate tRNA–a process commonly called as charging of tRNA or aminoacylation of tRNA to
be more specific.
1. Activation of amino acid :-

+ PP(Pyrophosphate)

2. Charging of t-RNA

(ii) (1) m-RNA (2) Charged t-RNA,


(3) 30s and 50s sub units of ribosome
(4) GTP
(5) Mg+2
(6) Initiation factors - In prokaryotes 3 factors:- IF1, IF2, IF3

E 97
AUG

m-RNA
5’ 3’

Attachment of
smaller subunit of
ribosome(30s)

m-RNA
5’ 3’

30s 16s r-RNA

Attachment of First
AA1 charged t-RNA by
IF2 and GTP

AA1

UAC

Initiator t-RNA by IF2 and GTP

UAC

AUG
m-RNA
5’ 3’
30s 30S Ribosome m-RNA
t-RNA complex

Attachment of larger
subunit of ribosomes(50s)
by Mg+2
AA1
P A

P A 50s E

50s E AUG GGG


m-RNA
5’ 3’

30s

(iii) mRNA also has some additional sequences that are not translated and are referred as untranslated
regions (UTR).
The UTRs are present at both 5'end (before start codon) and at 3'end (after stop codon).
The UTR (untranslated regions) present on mRNA are required for efficient translation process
(by recognising the smaller subunit of ribosome by mRNA).
7. Given below are the observation drawn in HGP. Select the options that shows the correct
observations. [CBSE Term – I 2022]
(i) The human genome contains 3164.7 billion base pairs.
(ii) The average gene consists of 3000 bases.
(iii) Less than 2% of the genome codes for proteins.
(iv) Chromosome one has most genes (2698)
(1) (i) and (ii) (2) (ii) and (iii) (3) (iii) and (iv) (4) (i) and (iii)
Ans. (2) (ii) and (iii)
98 E
8. The phosphoester linkage in the formation of a nucleotide involves the bonding between
[CBSE Term – I 2022]
(1) Phosphate group and OH of 3C of a nucleoside
(2) Phosphate group and OH of 5C of a nucleoside
(3) Phosphate group and H of 3C of a nucleoside
(4) Phosphate group and H of 5C of a nucleoside
Ans. (2) Phosphate group and OH of 5C of a nucleoside
9. The switching 'on' and 'off' of the lac operon in prokaryotes is regulated by
[CBSE Term – I 2022]
(1) Glucose (2) Galactose (3) Lactose (4) Fructose
Ans. (3) Lactose
10. For 'in-vitro' DNA replication, which one of the following substrates need to be added along with
the necessary enzymes the DNA template and specific conditions? [CBSE Term – I 2022]
(1) Ribonucleotide triphosphate (2) Deoxyribonucleoside triphosphate
(3) Deoxyribonucleotide triphosphate (4) Ribonucleoside triphosphate
Ans. (2) Deoxyribonucleoside triphosphate
11. Which one of the following factor will associate transiently with RNA polymerase to terminate
transcription in prokaryotes? [CBSE Term – I 2022]
(1) sigma factor (2) RHO factor (3) delta factor (4) theta factor
Ans. (2) RHO factor
12. Choose the correct pair of codon with its corresponding amino acid from the following list:
[CBSE Term – I 2022]
(1) UAG : Glycine (2) AUG : Arginine (3) UUU : Phenylalanine (4) UGA : Methionine
Ans. (3) UUU : Phenylalanine
13. During elongation process of translation, the peptide bond formation between amino acids is
catalysed :– [CBSE Term – I 2022]
(1) ribosomal RNA (2) protein in small subunit of ribosome
(3) protein in large subunit of ribosome (4) transfer RNA
Ans. (1) ribosomal RNA
14. A region of coding strand of DNA has the following nucleotide sequence:
5–TGCGCCA – 3
The sequence of bases on mRNA transcribed by this DNA stand would be:
[CBSE Term – I 2022]
(1) 3 – ACGCGGT – 5 (2) 5 – ACGCGGT – 3
(3) 5 – UGCGCCA – 3 (4) 3 – UGCGCCA – 5
Ans. (3) 5 – UGCGCCA – 3
15. A DNA molecule is 160 base pairs long. It has 20% adenine. How many cytosine bases are
present in this DNA molecule? [CBSE Term – I 2022]
(1) 192 (2) 96 (3) 64 (4) 42
Ans. (2) 96
E 99
16. A templated strand in a bacterial DNA has the following base sequence :
5 – TTTAACGAGG – 3 [CBSE Term – I 2022]
(1) 5 – AAATTGCTCC – 3 (2) 3 – AATTGCTCC – 5
(3) 3 – AAAUUGCUCC – 3 (4) 5 – CCUCGUUAAA – 5
Ans. (4) 5 – CCUCGUUAAA – 5
17. tRNA has an ___________ that has bases complementary to the codon. Its actual structure is a
compact molecule which looks like _______. [CBSE Term – I 2022]
Select the option that has correct choices for the two 'blanks'
(1) amino acid acceptor end, clover-leaf (2) anticodon loop, clover-leaf
(3) amino acid acceptor end, inverted L (4) anticodon loop, inverted L
Ans. (4) anticodon loop, inverted L
18. Which type of RNA is correctly paired with its function? [CBSE Term – I 2022]
(1) small nuclear RNA Processes rRNA (2) transfer RNA : attaches to amino acid
(3) ribosomal RNA : involved in transcription (4) micro RNA : involved in translation
Ans. (2) transfer RNA : attaches to amino acid
19. The figure given below has labellings (i), (ii) and (iii), which two labellings in the given figure
are components of a nucleosome? Select the correct option. [CBSE Term – I 2022]
(i)

(ii)
(iii)

(1) (i) – HI histone, (ii) – DNA


(2) (i) – DNA, (ii) – Histone Octamer
(3) (ii) – DNA, (iii) – HI Histone
(4) (ii) – Histone octamer, (ii) – DNA
Ans. (4) (ii) – Histone octamer, (ii) – DNA
20. In molecular biology who proposed that genetic information flows in one direction?
[CBSE Term – I 2022]
(1) Har gobind Khorana (2) Francis Crick
(3) Watson and Crick (4) Marshall Nirenberg
Ans. (2) Francis Crick
21. Predict the effect if, the codon UAU coding for an amino acid at the 25 th position of a
polypeptide of 50 amino acids, is mutated to UAA. [CBSE IMP. Question]
Ans. A polypeptide of 24 amino acids will be formed as UAA is a stop codon which will prevent
further translation.
100 E
22. Assertion (A) : Primary transcripts in eukaryotes are nonfunctional. [CBSE IMP. Question]
Reason (R) : Methyl guanosine triphosphate is attached to 5' end of hnRNA.
(1) Both assertion and reason are true, and reason is the correct explanation of assertion.
(2) Both assertion and reason are true, but reason is not the correct explanation of assertion.
(3) Assertion is true but reason is false.
(4) Both assertion and reason are false.
OR
Assertion (A) : An organism with lethal mutation may not even develop beyond the zygote
stage.
Reason (R) : All types of gene mutations are lethal. [CBSE IMP. Question]
(1)Both assertion and reason are true, and the reason is the correct explanation of the assertion.
(2)Both assertion and reason are true, but the reason is not the correct explanation of the
assertion.
(3)Assertion is true but reason is false.
(4)Both assertion and reason are false
Ans. (2)Both Assertion and Reason are true, but Reason is not the correct explanation of
Assertion.
OR
(2) Both Assertion and Reason are true but Reason, is not the correct explanation of
Assertion.
23. Evaluate the suitability of DNA and RNA as genetic material and justify the suitability of
the one that is preferred as an ideal genetic material. [CBSE IMP. Question]
OR
Explain the mechanism of DNA replication as suggested by Watson and Crick.
[CBSE IMP. Question]
Ans. Evaluation of DNA and RNA on the basis of the properties of the genetic material:
1. It should be able to generate its replica (Replication): As per the rule of base pairing and
complementarity, both the nucleic acids (DNA and RNA) have the ability to direct their
duplications.
2. The genetic material should be chemically and structurally stable enough not to change with
different stages of life cycle, age or with change in physiology of the organism.
Presence of 2'-OH group and uracil make RNA more reactive and structurally less stable than
DNA. Therefore, DNA is a better genetic material than RNA.
3. It should provide the scope for slow changes (mutation) that are required for evolution: Both
DNA and RNA are able to mutate. In fact, RNA being unstable, mutates at a faster rate.
Consequently, viruses having RNA genome and having shorter life span mutate and evolve
faster.
4. It should be able to express itself in the form of 'Mendelian Characters’: RNA can directly
code for the synthesis of proteins, hence can easily express the characters. DNA, however, is
dependent on RNA for synthesis of proteins. The protein synthesising machinery has evolved
around RNA.
E 101
5. The above discussion indicate that both RNA and DNA can function as genetic material, but
DNA being more stable is preferred for storage of genetic information
OR
Mechanism of Replication of DNA suggested by Watson and Crick
 The two strands of DNA would separate and act as a template for the synthesis of new
complementary strands. After the completion of replication, each DNA molecule would have
one parental and one newly synthesised strand. This scheme was termed as semiconservative
DNA replication.
 In living cells, such as E. coli, the process of replication requires a set of catalysts (enzymes).
The main enzyme is referred to as DNA-dependent DNA polymerase, since it uses a DNA
template to catalyse the polymerisation of deoxynucleotides.
 Furthermore, energetically replication is a very expensive process. Deoxyribonucleoside
triphosphates serve dual purposes. In addition to acting as substrates, they provide energy for
polymerisation reaction.
 For long DNA molecules, since the two strands of DNA cannot be separated in its entire
length (due to very high energy requirement), the replication occur within a small opening of
the DNA helix, referred to as replication fork.
 The DNA-dependent DNA polymerases catalyse polymerisation only in one direction, that is
5'  3'.
 Consequently, on one strand (the template with polarity 3'  5'), the replication is
continuous, while on the other (the template with polarity 5'  3'), it is discontinuous. The
discontinuously synthesised fragments are later joined by the enzyme DNA ligase.

Figure: replication Fork

 The DNA polymerases on their own cannot initiate the process of replication.
 There is a definite region in E. coli DNA where the replication originates. Such regions are
termed as origin of replication.
 In eukaryotes, the replication of DNA takes place at S-phase of the cell- cycle.
 The replication of DNA and cell division cycle should be highly coordinated.
A failure in cell division after DNA replication results into polyploidy.

102 E
24. Meselson & Stahl carried out centrifugation in CsCl2 density gradient to separate.
[CBSE 2020]
(1) DNA from RNA (2) DNA from protein
(3) The normal DNA from 15N-DNA (4) DNA from tRNA
Ans. (C) The normal DNA from 15N-DNA
25. Give below is one the strands of a DNA segment: [CBSE 2020]
TACGTACGTACGTACG 5'
3' 
(a) Write its complementary strand
(b) Write a Possible RNA strands that can be transcribed from the above DNA molecule formed.
Ans. (a) 5' ATGCATGCATGCATGC 3'
(b) 5' AUGCAUGCAUGCAUGC 3'
Note : 3' – 5' is tamplate strand
5' – 3' is coding strand
26. Explain the role of regulatory gene in a lac operon. Why is regulation of lac operon called as
negative regulation? [CBSE 2019]
Ans. Regulatory gene / i gene codes for the repressor of the lac operon, the repressor protein
(synthesised by i gene, binds to the operator site of the operon, and prevents the RNA polymerase
from transcribing the operon
The repressor of lac operon is synthesised constitutively / all the time, and thus the operon is in
‘switched off’ position generally, it is switched on only when lactose is present in the culture
medium of the E.coli when the operon gets ‘switched on’
27. Compare the processes of DNA replication and transcription in prokaryotes. [CBSE 2019]
Ans. Similarities - Both the processes involve -
i. Unwinding of the helix and separating the two DNA strands
ii. Breaking the hydrogen bonds between the bases / pairs
iii. Follow complimentary base pair rule
iv. Polymerization occurs in 5‘ 3' direction
v. Linking / Polymerization of nucleotides.
Dissimilarities

DNA replication Transcription

1. DNA nucleotides added are 1. RNA nucleotides added are


dATP, dGTP, dCTP, dTTP ATP, GTP, CTP, UTP

2. Deoxyribose sugar is the part of 2. Ribose sugar is the part of


nucleotide nucleotide

3. Adenine pairs with Thymine 3. Adenine with Uracil

4. Both strands copied 4. Only one strand copied.

5. Resulting into two DNA molecules 5. Resulting in formation of an


RNA molecule

E 103
28. (a) Explain Griffith’s ‘transforming principle’ experiment.
(b) In the above experiment, ‘‘heat which killed one type of bacteria, did not destroy the
properties of genetic material.’’ Justify [CBSE 2019]
Ans. (a) S strain  Inject into mice  Mice die
R strain  Inject into mice  Mice live
S strain  Inject into mice  Mice live
(heat-killed)
S strain
(heat-killed)
+  Inject into mice  Mice die
R strain
(live)
(b) The two DNA strands complementary get separated by heating come together, when
appropriate conditions are provided heat did not destroy the genetic properties.
29. What is an operon ? Explain the functioning of lac operon when in an open state.
[CBSE 2018]
Ans. The arrangement where a (Polycistronic) structural gene is regulated by a common promoter and
regulatory genes
Lactose acts as inducer , binds with repressor protein, RNA polymerase freely moves over the
structural genes, transcribes lac mRNA, which in turn produce enzymes - transacetylase,
permease, β-galactosidase (by lac z), responsible for digestion of lactose
p i p o z y a
Transcription

Repressor mRNA lac mRNA


Translation

-galactosidase permease transacetylase


Inducer

(Inactive repressor)
30. (a) Hershey and Chase carried their experiment in three steps : infection, blending,
centrifugation. Explain each step.
(b) Write the conclusion and interpretation of the result they obtained. [CBSE 2017]
Ans. (a) Infection - Radioactive phosphorus / phosphorus labelled bacteriophages were allowed to
infect E.coli - growing in a culture medium, simultaneously radioactive sulphur / sulphur
labelled bacteriophage was allowed to infect E.coli growing in another culture medium
Blending - As infection proceeds- the viral coats were removed from the bacteria by agitating in
a blender.
Centrifugation - Virus particles were seperated from the bacteria by spinning them in a
centrifuge.
(b) Conclusion - DNA is the genetic material.
Interpretation - Sulphur labelled viral protein did not enter the bacteria during infection,
whereas phosphorus labelled viral DNA entered into the bacteria to cause infection.
104 E
(C) MULTIPLE CHOICE QUESTIONS
1. In eukaryotic cell circular and double stranded DNA occurs in -
(1) Golgibody (2) Mitochondria (3) Nucleus (4) Ribosome
Ans. (2) Mitochondria
2. Double helix model of DNA which was proposed by watson and crick was of-
(1) C-DNA (2) B-DNA (3) D-DNA (4) Z-DNA
Ans. (2) B-DNA
3. If there are 10,000 nitrogenous base pairs in a DNA then how many nucleotides are there -
(1) 500 (2) 10,000 (3) 20,000 (4) 40,000
Ans. (3) 20,000
4. Double helix model of DNA is proposed by-
(1) Watson and Crick (2) Schleiden schwann
(3) Singer and Nicholson (4) Kornberg and Khurana
Ans. (1) Watson and Crick
5. Substance common in DNA and RNA -
(1) Hexose Sugar (2) Histamine (3) Thymine (4) Phosphate groups
Ans. (4) Phosphate groups
6. Nucleotide is -
(1) N2 - base, pentose sugar and phosphoric acid
(2) Nitrogen, Hexose sugar and phosphoric acid
(3) Nitrogen base, pentose sugar
(4) Nitrogen base, trioses and phosphoric acid
Ans. (1) N2 - base, pentose sugar and phosphoric acid
7. Unit of nucleic acids are-
(1) Phosphoric acid (2) Nitrogenous bases (3) Pentose Sugar (4) Nucleotides
Ans. (4) Nucleotides
8. Nucleic acid (DNA) is not found in-
(1) Nucleus & nucleolus (2) Peroxysome & ribosome
(3) Mitochondria & plastid (4) Chloroplast & nucleosome
Ans. (2) Peroxysome & Ribosome
9. A nucleic acid contains thymine or methylated uracil then it should be -
(1) DNA (2) RNA
(3) Either DNA or RNA (4) RNA of bacteria
Ans. (1) DNA

105
10. A nucleoside differs from a nucleotide is not having -
(1) Phosphate (2) Sugar (3) Phosphate & sugar (4) Nitrogen base
Ans. (1) Phosphate
11. The unequivocal proof that DNA is the genetic material came from the experiments of
(1) Hershey and Chase (1952) (2) Frederic Griffith (1928)
(3) Watson and Crick (1953) (4) Meselson and Stahl (1958)
Ans. (1) Hershey and Chase (1952)
12. Genetic information are transferred from nucleus to cytoplasm of cell through :-
(1) DNA (2) RNA (3) Lysosomes (4) ACTH
Ans. (2) RNA
13. Short DNA segment has 80 thymine and 90 guanine bases. The total number of nucleotides
are-
(1) 160 (2) 40 (3) 80 (4) 340
Ans. (4) 340
14. Prokaryotic DNA is:-
(1) double stranded circular (2) single stranded circular
(3) double stranded linear (4) double stranded RNA as nucleic acid
Ans. (1) double stranded circular
15. DNA molecule has uniform diameter due to -
(1) Double stranded
(2) Presence of phosphate
(3) Specific base pairing between purine and pyrimidine
(4) Specific base pairing between purine and purine
Ans. (3) Specific base pairing between purine and pyrimidine
16. Which of the following sugar present in RNA ?
(1) Deoxyribose (2) Ribose (3) Sucrose (4) Maltose
Ans. (2) Ribose
17. In a nucleotide H3PO4 binds to which carbon atom of pentose sugar :-
(1) Only 1st carbon (2) Only 3rd carbon (3) Only 5th carbon (4) Both 3rd and 5th carbon
Ans. (3) Only 5th carbon
18. DNA is acidic due to :-
(1) Sugar (2) Phosphoric acid (3) Purine (4) Pyrimidine
Ans. (2) Phosphoric acid
19. If the sequence of bases in one strand of DNA is known then the sequence in other strand
can be predicted on the basis of:-
(1) Antiparallel (2) Complementary (3) Polarity (4) Coiling
Ans. (2) Complementary
(D) ASSERTION & REASON QUESTIONS
 Directions: In the following questions, a statement of assertion is followed by a statement of
reason. Mark the correct choice as:
(1) If both Assertion and Reason are true and Reason is the correct explanation of Assertion.

(2) If both Assertion and Reason are true but Reason is not the correct explanation of Assertion.

(3) If Assertion is true but Reason is false.

(4) If both Assertion and Reason are false.

1. Assertion: The sugar phosphate backbone of two chains in DNA double helix show anti-parallel
polarity.
Reason: The phosphodiester bonds in one strand go from a 3' carbon of one nucleotide to a 5'
carbon of adjacent nucleotide, whereas those in complementary strand go vice versa.
Ans. (1)
2. Assertion: DNA is considered to be better genetic material than RNA for most organisms.
Reason: 2'-OH group present in DNA makes it labile and less reactive.
Ans. (3)
3. Assertion : Histones are basic proteins of major importance in packaging of eukaryotic DNA.
Reason : Histones are of five major types H1,H2A H2B,H3 and H4 .
Ans. (2)
4. Assertion: Histones are basic in nature.
Reason: Histones are rich in the amino acids lysine and arginine.
Ans. (1)
5. Assertion: The unequivocal proof that DNA is the genetic material came from the experiments
of Hershey and Chase.
Reason: They worked with viruses that infect bacteria called bacteriophages.
Ans. (2)
6. Assertion: DNA acts as a genetic material in all organisms.
Reason: It is a single-stranded biomolecule.
Ans. (4)
7. Assertion: In Griffith's experiment, a mixture of heat-killed virulent bacteria R and live
non-virulent bacteria S, lead to the death of mice.
Reason: 'Transforming principle' got transferred from heat killed R strain to S strain and made
virulent.
Ans. (4)
8. Assertion: The genetic material should be stable chemically and structurally.
Reason: It should be able to generate its replica.
Ans. (2)
9. Assertion : Central dogma is the flow of information from DNA to mRNA and then decoding
the information present in mRNA in the form of protein.
Reason : In retroviruses, reverse of central dogma occurs.
Ans. (2)
10. Assertion: DNA act as the genetic material in most of the organisms.
Reason: DNA is chemically and structurally a stable molecule; it has the power of replication.
Ans. (1)
E 107
(E) VERY SHORT ANSWER QUESTIONS
1. What is a cistron?
Ans. Cistron is a segment of gene which codes for a certain polypeptide or protein.

2. When and at what end does the 'tailing' of hnRNA take place?

Ans. 'Tailing' of hnRNA takes place during conversion of hnRNA into functional mRNA after

transcription. It takes place at the 3-end.

3. At which ends do 'capping' and 'tailing' of hnRNA occur, respectively?

Ans. Capping occurs at 5'-end and tailing occurs at 3'-end.

4. Mention two functions of the codon AUG.

Ans. Two functions of the codon AUG are:

(i) It acts as a start codon during protein synthesis.

(ii) It codes for the amino acid methionine.

5. How does a degenerate code differ from an unambiguous one?

Ans. Degenerate code means that one amino acid can be coded by more than one codon. Unambigous

code means that one codon codes for one amino acid.

6. Mention the role of the codons AUG and UGA during protein synthesis.

Ans. The codon AUG initiates protein synthesis whereas the codon UGA stops protein synthesis.

7. Write the function of RNA polymerase II.

Ans. RNA polymerase II transcribes precursor of mRNA or hnRNA.

8. Give an example of a codon having dual function.

Ans. AUG acts as an initiator codon and also codes for methionine (met).

9. Mention how does DNA polymorphism arise in a population.

Ans. DNA polymorphism in a population arise due to presence of inheritable mutations at high

frequency.

10. Suggest a technique to a researcher who needs to separate fragments of DNA.

Ans. Gel electrophoresis is used to separate DNA fragments.


(F) SHORT ANSWER QUESTIONS
1. Name the negatively charged and positively charged components of a nucleosome.
Ans. Negatively charged component is DNA, positively charged component is histone octamer.
2. Write the two specific codons that a translational unit of m-RNA is flanked by one on either
sides.
Ans. Start codon – AUG
Stop codon – UAA/UGA/UAG.
3. Length of a DNA is 1.36mm. Calculate the number of base pairs in this DNA?
Ans. Length of DNA = Total bp × 3.4A°
1.36mm = Total bp × 3.4 A°
1.36 × 10-3 = Total bp × 3.4 × 10-10
1.36 10 3
= Total bp
3.4 10 10
Total bp = 4 × 106
4. Calculate the number of nucleosomes in a diploid cell of human?
Ans. No. of nucleosomes = total bp/200
= 6.6 × 109/200
= 3.3 × 107
5. Write the conclusion Griffith arrived at the end of his experiment with Streptococcus
pneumoniae.
Ans. He concluded that the R-strain bacteria had somehow been transformed the by the heat - killed
S-strain bacteria, this must be due to transfer of genetic material.
6. Identify A, B, C, D, E and F in the following table
Sr.
No.

i.
ii.
iii.

Ans. (i) A - Nitrogenous base / A - Pentose sugar


B - Pentose Sugar / B- Nitrogenous base
C - N glycosidic linkage.
(ii) D - phosphate group.
E - phospho ester linkage
(iii) F - (3 -'5') phosphodiester linkage.

109
(G) LONG ANSWER QUESTIONS
1. Why is DNA molecule considered as a better hereditary material than RNA molecule?
Ans. DNA molecule is a better hereditary material as
(i) It is more stable (due to presence of thymine)
(ii) Less reactive than RNA (as RNA has 2’ - OH making it more reactive)
(iii) Being less reactive, DNA is not easily degradable (RNA being more reactive is easily
degradable)
(iv) Rate of mutation is slow (Rate of mutation in RNA is faster)
2. Name the three RNA polymerases found in eukaryotic cells and mention their functions.
Ans. RNA polymerase - I, transcribes rRNAs (28S -18S and 5.8S)
RNA polymerase - II, transcribes precursor of mRNA / hnRNA / heterogeneous nuclear RNA
RNA polymerase - III , transcribes tRNA / 5s rRNA / snRNA
3. Explain the post transcriptional modifications the hn-RNA undergoes in eukaryotic cell.
Ans. Capping, 7 methyl guanosine triphosphate / 7 mGPPP is added to the 5’ end of hnRNA
• Tailing, Polyadenylate residues are added to 3’-end in a template independent manner
• Splicing, Introns are removed and exons are joined.
• Splicing occurs with the help of SnRNA so introns are removed.
4. (a) Why does DNA replication occur in small replication forks and not in its entire length?
(b) Why is DNA replication continuous and discontinuous in a replication fork?
(c) State the importance of origin of replication in a replication fork.
Ans. (a) DNA being very long, requires high energy for opening along its entire length.
(b) DNA-dependent DNA polymerases catalyse polymerisation only in one direction, i.e.
5’  3’, Two strands of DNA are anti parallel and have opposite polarity.
(c) Site where replication originates
5. Give the answer of following questions :
(a) Describe aminoacylation of tRNA.
(b) Explain the process that takes place in the ribosomes when mRNA makes its entry into
it in prokaryote.
(c) Due to transcription error, ATG codon of DNA is transcribed into UAG in mRNA
which translate non-functional polypeptide chain in the ribosome. Justify the statement.
Ans. (a) Amino acids are activated in the presence of ATP and linked to their cognate tRNA
(b) Small subunit of ribosome binds to mRNA at start codon (AUG) at 5’ end, in the two sites of
large subunits of ribosome, the charged tRNA with the aminoacid corresponding to the codon
on mRNA align, formation of peptide bond between the two closely placed aminoacids in the
two sites occur, with the help of ribozyme in the ribosome, peptide chain elongation
continues till the stop codon (UAG, UGA, UAA) on the mRNA reach the big unit of
ribosome.
(c) UAG being a stop codon termination occur before the completion of the functional
polypeptide.
110 E
6. Explain the steps of DNA fingerprinting that will help in processing of the two or more
blood samples picked up from the crime scene.
Ans. The technique of DNA Fingerprinting was initially developed by Alec Jeffreys. He used a
satellite DNA as probe that shows very high degree of polymorphism. It was called as Variable
Number of Tandem Repeats (VNTR). The technique, as used earlier, involved Southern blot
hybridisation using radiolabelled VNTR as a probe. It included
(i) Isolation of DNA,
(ii) Digestion of DNA by restriction endonucleases,
(iii) Separation of DNA fragments by electrophoresis,
(iv) Transferring (blotting) of separated DNA fragments to synthetic membranes, such as
nitrocellulose or nylon,
(v) Hybridisation using labelled VNTR probe, and
(vi) Detection of hybridised DNA fragments by autoradiography.
(H) CASE-STUDY BASED QUESTIONS
1. Study the following and answer the questions given below:-

Gene regulation is the mechanism of switching OFF and ON of the genes depending upon the
requirement of cells and the state of development. Gene regulation is two type : negative and
positive. The given gene diagram is Operon model of gene regulation study it and give the
answer of asked questions.

(i) Identify the name the regulatory give in this operon?


Ans. Regularity gene in this process is i gene.
(ii) Why is Lac operon is regulation referred to as negative regulation?
Ans. The repressor binds to the operator, the operon is switched off and transcription is stopped. So
it is a negative regulation.
(iii) Name the inducer Molecule and products of the genes 'A' and 'B' of the operon. write the
function of these gene products.

E 111
Ans. Product of 'A' gene is  - galactosidase that helps in the hydrolysis of disaccharide lactose into

its monomeric units, galactose and glucose.

Product of B gene  Code for permease. That increase permeability of the cell to

 - galactosides.

(iv) What are X and Y in subject - 1 and subject - 2 respectively?


Ans. X = Repressor, Y-Inducer

2. Study the following and answer the questions given below:-


The given case is an experiment related to the discovery of genetic material is DNA. The
experiment is show in the subject-1 and subject-2. Studies it and give the answer of followed
questions.
Subject-1 Subject-2

(i) Name the kind of virus they worked with and why?
Ans. They worked with bacteriophage eg = viruses that infect bacteria. These viruses were used
because during infection they transfer their genetic material into bacteria.
(ii) What are 'X' and 'Y' in subject - 1 and subject - 2 respectively?
Ans. X - 35s, Y - 32p
(iii) The viral coat were removed from the bacteria by agitating them in a blender, process is
called as -
Ans. Blending
(iv) State the conclusion drawn by them after the experiments.
Ans. The conclusion drawn by them after the experiment was that the DNA is the genetic material.

112 E
3. Read the following and answer the questions given below:-
Nucleic acids are long polymers of nucleotides. While DNA stores genetic information, RNA
mostly helps in transfer and expression of information. Though DNA and RNA both function as
genetic material, but DNA being chemically and structurally more stable is a better genetic
material. However, RNA is the first to evolve and DNA was derived from RNA. The hallmark of
the double stranded helical structure of DNA is the hydrogen bonding between the bases from
opposite strands. The rule is that Adenine pairs with Thymine through two H-bonds, and Guanine
with Cytosine through three H-bonds. This makes one strand complementary to the other. The
DNA replicates semiconservatively, the process is guided by the complementary H-bonding. A
segment of DNA that codes for RNA may in a simplistic term can be referred as gene.

(i) Write the name of components of a nucleotide.

Ans. Nitrogen base, pentose sugar, and phosphoric acid

(ii) Which is better genetic material out of RNA and DNA?

Ans. DNA being chemically and structurally more stable is a better genetic material as compare to

RNA.

(iii) What is the nature of the 2 strands of a DNA double helix ?

Ans. The are anti - parallel and complementary to each other.

(iv) Write the appropriate name of nitrogenous base that correctly fill in the blanks.

Pyrimidines present in DNA are...... (A)....... and.....( B...) while pyrimidines present in

RNA are.... (C)...... and ....(D).....

Ans. (A)- Cytosine, (B)- Thymine, (C)- Cytosine, (D)- Uracil

(v) If a double stranded DNA has 20 per cent of cytosine, calculate the per cent of adenine in

the DNA.

Ans: Cytosine 20% , therefore Guanine =20%

According to Chargaff's rule,

A+T =100-(G+C)

A+T =100-40. Since both adenine and thymine are in equal amounts

Thymine = Adenine = = 30 %

E 113
4. Read the following and answer the questions given below:-

DNA fingerprinting involves identifying differences in some specific regions in DNA sequence
called as repetitive DNA, because in these sequences, a small stretch of DNA is repeated many
times. These repetitive DNA are separated from bulk genomic DNA as different peaks during
density gradient centrifugation. The bulk DNA forms a major peak and the other small peaks are
referred to as satellite DNA. Depending on base composition (A : T rich or G:C rich), length of
segment, and number of repetitive units, the satellite DNA is classified into many categories,
such as micro-satellites, mini-satellites etc. These sequences normally do not code for any
proteins, but they form a large portion of human genome. These sequence show high degree of
polymorphism and form the basis of DNA fingerprinting. Since DNA from every tissue (such as
blood, hair-follicle, skin, bone, saliva, sperm etc.), from an individual show the same degree of
polymorphism, they become very useful identification tool in forensic applications.

(i) What is satellite DNA?

Ans. It is the repetitive DNA sequences which form a large portion of genome and have high degree of

polymorphism but do not code for any proteins.

(ii) What is the basis of genetic mapping of human genome as well as of DNA fingerprinting.

Ans. The polymorphism in DNA sequence.

(iii) Expand VNTR.

Ans. Variable Number of Tandem Repeats (VNTR).

(iv) Which of the DNA was used by Alec Jeffreys as probe that shows very high degree of

polymorphism?

Ans. Satellite DNA

(v) What is the basis of classified satellite DNA into many categories, such as micro-satellites,

mini-satellites etc ?

Ans. Its depend on base composition (A : T rich or G:C rich), length of segment, and number of

repetitive units.

114 E
CHAPTER 6
(A) NCERT QUESTIONS & SOLUTIONS
1. Explain antibiotic resistance observed in bacteria in light of Darwinian selection theory.
Ans. Darwinian selection theory states that individuals with favourable variations are better adapted
than individuals with less favourable variation.
 It means that nature selects the individuals with useful variation as these individuals are better
evolved to survive in the existing environment. An example of such selection is antibiotic
resistance in bacteria. When bacterial population was grown on an agar plate containing
antibiotic penicillin, the colonies that were sensitive to penicillin died, whereas one or few
bacterial colonies that were resistant to penicillin survived.
 This is because these bacteria had undergone chance mutation, which resulted in the
evolution of a gene that made them resistant to penicillin drug. Hence, the resistant bacteria
multiplied quickly as compared to nonresistant (sensitive) bacteria, thereby increasing their
number. Hence, the advantage of an individual over other helps in the struggle for existence.
2. Find out from newspapers and popular science articles any new fossil discoveries or
controversies about evolution.
Ans. A recent study of fossil revealed a small terrestrial dinosaur with feathers covering the limb and
body. This finding established that feathers evolved earlier than wing and may be functioning as
thermoregulator to face adverse condition. These newly developed feather earlier helped in
gliding and then flying
3. Attempt giving a clear definition of the term species.
Ans. Species can be defined as a group of organisms which have the capability to interbreed in order
to produce fertile offspring.
4. Try to trace the various components of human evolution.
Ans.
S.No Human ancestor Time of Origin General Features
1 Dryopithecus 15 mya Ape-like, hairy, arms and legs of same
length, large brain, ate soft fruits and leaves
2 Ramapithecus 14-15mya More man-like, walked more erect, teeth
like modern man.
3 Australopithecus 3-4 mya Fossils found in Tanzania and Ethiopia,
manlike primates, 4 feet tall, walked upright,
ate fruit, hunted with stone weapons, brain
capacity was 400-600
4 Homo habilis 2 mya Fossils found in East Africa, first human like
being, brain capacity 650-800 cc, did not eat
meat
5 Homo erectus 1.5 mya Fossils found in Java, brain capacity 900cc,
(Java man)
6 Homo 100,000-40,000 Fossils found in east and central Asia, brain
neanderthalensis year ago size 1400 cc, used hides to protect body,
buried their dead.
7 Homo sapiens 75,000-10,000 Developed cave art, agriculture, started
(Modern man) year ago human civilization.

E 115
5. Find out through internet and popular science articles whether animals other than man has
self-consciousness.
Ans. There are many animals other than humans, which have self consciousness. An example of an
animal being self conscious is dolphins. They are highly intelligent. They have a sense of self and
they also recognize others among themselves and others. They communicate with each other by
whistles, tail-slapping, and other body movements. Not only dolphins there are certain other
animals such as crow, parrot, chimpanzee, gorilla, orangutan, etc., which exhibit self-
consciousness.
6. List 10 modern-day animals and using the internet resources link it to a corresponding
ancient fossil. Name both.
Ans.
S.No Modern Name of Animal Corresponding ancient fossil
1. Man Homo neanderthalensis
2. Chimpanzee Dryopithecus
3. Gorilla Dryopithecus
4. Orangutan Dryopithecus
5. Gibbon Propliopithecus
6. Nautilus Gypceros
7. Octopus Belemnite
8. Elephant Stegolophpdon
9. Camel Procamelus
10. Horse Pliohippus
7. Describe one example of adaptive radiation.
Ans. Darwin's finches in the galapogos island once had a common ancestor but with evolution they
modified into different types according to their food habitat.
8. Can we call human evolution as adaptive radiation?
Ans. No, human evolution cannot be called as adaptive radiation because parent species of Homo
sapiens have evolved by progressive evolution from Homo habilis to Homo sapiens lineage.
9. Using various resources such as your school library or the internet and discussions with
your teacher, trace the evolutionary stages of any one animal, say horse. [IMP.]
Ans. The evolution of horse is represented as following:-
(A) Eohippus:- This stage is characterized by a short head and neck .It had four functional toes
and a splint of 1 to 5 on each hind limb and a splint of 1 to 3 in each forelimb. The molars
were short crowned that were adapted for grinding the plant dies.
(B) Mesohippus:- It was slightly taller than Eohippus. It had three toes in each foot.
(C) Merychippus:- it had the size of approximately 100 cm. Although it still has three toes in
each foot, but it could run on one toe. The side toe did not touch the ground. The molars
were adapted for chewing the grass.
(D) Pliohippus:- It resembled the modern horse and was around 120 cm tall .It had a single
functional toe with splint of second and fourth in each limb,
(E) Equus :- Pliohippus gave rise to Equus or the modern horse with one toe in each foot. They
have incisors for cutting grass and molars for grinding food.
Epochs Bones of

(in cm)
Height
Appearance Horse No. of toes
Limbs
Pleistocene

1 – toed
Modern
160 (2 Splint
Horse Equus
bones)

1 – toed
Pliocene

120 Pony like Pliohippus (2 Splint


bones)

3-toed
Miocene

100 Donkey like Merychippus (No Splint


bones)
Oligocene

3 – toed
60 Sheep like Mesohippus (1 Splint
bones)

4 – toed
Eocene

40 Fox like (1 Splint


bones)

Evolutionary stages of horse

117
(B) PREVIOUS YEAR QUESTIONS
1. At which stage during evolution did human use hides to protect their bodies and buried their
dead? [CBSE-2023]
(1) Homo habilis (2) Neanderthal man (3) Java man (4) Home erectus
Ans. (2) Neanderthal man
2. Mention Darwin's observations made on finches during his visit to Galapagos Islands.
Write the explanation given by Darwin on his observations. [CBSE 2023]
Ans. During his journey Darwin went to Galapagos Islands.
 There he observed an amazing diversity of creatures of particular interest, small black birds
later called Darwin’s Finches amazed him.
 He realised that there were many varieties of finches in the same island.
 All the varieties, he conjectured, evolved on the island itself. From the original seed-eating
features, many other forms with altered beaks arose, enabling them to become
insectivorous and vegetarian finches .
 This process of evolution of different species in a given geographical area starting from a
point and literally radiating to other areas of geography (habitats) is called adaptive
radiation.
3. Industrial melanism in England after 1850 is an excellent example of Natural selection.
Explain how? [CBSE 2023]
Ans. Industrial melanism in England after 1850 is an excellent example of Natural selection. It is
because, before industrialization the air quality of England was good due to which lichen were
able to grow on tree barks. Lichens were white in color. Hence, moths living on tree bark evolved
with time to have a more white population due to less melanin production. This reduced the
number of black moths in the population. This was an adaptation to hide from predators.
With time as industrialization grew the air quality reduced. This eliminated the lichens from the
region. Now the dark brown to black color of tree bark was exposed. Due to this, moth
population evolved to have more melanin giving them black color. This reduced the number of
white moths in the population. This was an adaptation to hide from predators' attacks.
In short, evolving with the situation gave moths' population a benefit due to natural selection.
4. How would the gene flow or genetic drift affect the population in which either of them
happen to take place? [CBSE 2019]
Ans. Results in changed frequency of genes (or alleles) in both populations, causing variation, leading
to evolution / speciation / founder effect
5. According to Darwinian theory of natural selection the rate of appearance of new forms is
linked to the life-cycle or the life-span of an organism. Explain with the help of an example.
[CBSE 2019]
Ans. A colony of bacteria (say A) growing in a given medium has built in variation in terms of ability
to utilise a feed component, a change in the medium composition would bring out only that part

118 E
of the population(say B) that can survive under the new conditions , In due course of time this
variant population outgrows the others and appears as new species thus organisms with shorter
life-cycle or life-span will undergo evolution faster / for the same thing to happen in fish or fowl
would take millions of years as life spans of these animals are in years.
6. Describe S.L. Miller's experiment. Comment on the observations he made and his
contribution towards the origin of life on Earth. [CBSE 2019]
Ans. High temperature (800º C), high energy radiation, reducing atmosphere created, by electric
discharge in a closed flask, containing CH4 + H2 + NH3, and water vapours in the
experimental setup.

Electrodes

To vacuum
pump
CH4 Spark
NH3 discharge
H2O Gases
H2
Water out
Condenser
Water in
Water droplets

Water containing
Boiling water organic compounds
Liquid water in trap

Observation and Contribution


- Formation of amino acids
- The first form of life arose slowly through evolutionary forces from non- living molecules/
abiogenesis.
7. "Appearance of melanized moths post-industrialization in England is a classic example of

evolution by natural selection." Explain. [CBSE 2019]

Ans. Before industrialization more white winged moth than dark winged moth existed in England,

post industrialization tree trunks became dark as smoke and soot deposited, lichens could not

grow due to pollution, due to higher predation of white winged moth on a darker background,

dark winged moth survived, nature selected the fittest organism.

8. According to the Hardy-Weinberg principle, the allele frequency of a population remains


constant. How do you interpret the change of frequency of alleles in a population? [CBSE 2019]
Ans. (i) Resulting in evolution/Speciation/original drifted population becomes founders.

E 119
9. (a) How does the Hardy-Weinberg equation explain genetic equilibrium?
(b) Describe how this equilibrium is disturbed that may lead to founder effect. [CBSE 2019]
Ans. (a) Allelic frequencies in a population are stable and remains constant from generation to
generation / the sum total of all the allelic frequencies is one.
(b) gene migration / gene flow / genetic drift / mutation / gene recombination / natural selection
leads to disturbance in equilibrium , when changes in allelic frequencies occur many times in
a population , leads to the formation of a new species , original drifted population becomes
the founders and the effect is called founders effect.
10. Charles Darwin during his famous sea voyage around the world in a ship (HMS Beagle),
concluded that there has been gradual evolution of life. Answer the following questions:
(a) What is his theory known as ? Explain the silent features of his theory.
(b) Name a scientist who arrived at a similar conclusion as that of Charles Darwin.
[CBSE 2019]
Ans. (a) - Theory of Natural Selection
- Any population has built in variation , those characteristics which enables some to survive
better (in natural condition) will outbreed others that are less endowed to survive / better adapted
individuals will survive , those individuals would leave more progeny / reproductively fit
individuals , they will survive more and hence selected by nature
(b) Alfred Wallace (a Naturalist)
11. Write the names of the following : (a) A 15 mya primate that was ape-like (b) A 2 mya
primate that lived in East African grasslands [CBSE 2018]
Ans. (a) Dryopithecus
(b) Australopithecines/Australopithecus/Homo habilis
12. With the help of an algebraic equation, how did Hardy-Weinberg explain that in a given
population the frequency of occurrence of alleles of a gene is supposed to remain the same
through generations ? [CBSE 2018]
Ans. In a population of diploid organisms
If frequency of allele A = p
and frequency of allele a = q
Expected genotype frequency under random mating are
AA = p² (for the AA homozygotes)
aa = q² (for the aa homozygotes)
Aa = 2pq (for the Aa heterozygotes)
(In absence of selection , mutation , genetic drift or other forces allelic frequency p and q are
constant through generation) Therefore p2 + 2pq +q2 = 1
13. (a) Differentiate between analogous and homologous structures.
(b) Select and write analogous structures from the list given below :
(i) Wings of butterfly and birds
(ii) Vertebrate hearts
(iii) Tendrils of bougainvillea and cucurbita
(iv) Tubers of sweet potato and potato [CBSE 2018]

120 E
Ans. (a) Analogous - Anatomically not similar though perform similar functions/are a result of
convergent evolution,
Homologus - Anatomically similar (but perform different functions)/are a result of divergent
evolution.
(b) Option (i) Wings of butterfly and birds /(iv) Tubers of sweet potato and potato (Any one)
14. State two postulates of Oparin and Haldane with reference to origin of life. [CBSE 2017]
Ans. (i) First form of life could have come from pre-existing non-living organic molecules/RNA
& Protein.
(ii) Formation of life was preceded by chemical evolution/formation of diverse organic
molecules from inorganic constituents.
15. Write the characteristics of Ramapithecus, Dryopithecus and Neanderthal man. [CBSE 2017]
Ans. Ramapithecus : hairy/ walked like gorillas and chimpanzees , more man like.
Dryopithecus : hairy/ walked like gorillas and chimpanzees , more ape- like.
Neanderthal man : brain size is 1400cc, used hides to protect their body/buried their dead.

(C) MULTIPLE CHOICE QUESTIONS


1. Primitive atmosphere was made up of the mixture of :
(1) Oxygen, ammonia, methane, water
(2) Hydrogen, ammonia, methane, oxygen
(3) Hydrogen, steam, methane, ammonia
(4) Oxygen, methane, water, nickle
Ans. (3) Hydrogen, steam, methane, ammonia
2. Which compounds were formed in the direction of the origin of life :
(1) Urea, nucleic-acid (2) Urea, amino-acid
(3) Proteins, nucleic-acid (4) Protein, amino-acid
Ans. (3) Proteins, nucleic-acid
3. What is most important for origin of life :
(1) Carbon (2) Oxygen (3) Water (4) Nitrogen
Ans. (3) Water
4. Pasteur succeeded in disproving the theory of spontaneous generation because :
(1) The laboratory was clean
(2) He pulled out the neck of flask into a tube
(3) He was lucky
(4) Yeast used in flask were dead
Ans. (2) He pulled out the neck of flask into a tube
E 121
5. Now the basis of origin of life is :
(1) Spontaneous generation (2) God's desire
(3) Sunlight on mud (4) None of them
Ans. (4) None of them
6. Oxygen in atmosphere has been formed by :
(1) Evaporation of water (2) Photosynthesis of blue green algae
(3) Metabolism of microorganisms (4) Decaying organisms
Ans. (2) Photosynthesis of blue green algae
7. Primitive atmosphere was reducing because :
(1) Hydrogen atoms were few
(2) Hydrogen atoms were active and in greater number
(3) Nitrogen atoms were more
(4) Oxygen atoms were more
Ans. (2) Hydrogen atoms were active and in greater number
8. Who called larger colloidal particles of primitive sea as coacervates :
(1) Fox (2) Oparin (3) Empedocles (4) Haldane
Ans. (2) Oparin
9. Who called water of primitive sea as pre biotic soup :
(1) Haldane (2) Oparin (3) Fox (4) Huxley
Ans. (1) Haldane
10. Oparin's theory is based on :
(1) Artificial synthesis (2) Spontaneous generation
(3) God's will (4) All
Ans. (1) Artificial synthesis
11. Which biologist gave most logical biochemical theory of origin of life ?
(1) Urey (2) Oparin (3) Stanley Miller (4) Haeckel
Ans. (2) Oparin
12. During the course of origin of life what was the sequence of substances which appeared on earth :
(1) Water, oxygen, nucleic acids, enzymes
(2) Amino acids, ammonia, phosphates, nucleic acids
(3) Glucose, amino acids, nucleic acids, proteins
(4) Ammonia, Amino acids, proteins, nucleic acids
Ans. (4) Ammonia, Amino acids, proteins, nucleic acids
13. Which of the following sets do not have homologous organs :
(1) Wings of mosquito and butterfly (2) Wings of butterfly and bat
(3) Mouth parts of cockroach and butter fly (4) None of them
Ans. (2) Wings of butterfly and bat
14. Wings of locust, pigeon, and bat are example of :
(1) Vestigial organs (2) Analogous organs
(3) Homologous organs (4) Exoskeleton
Ans. (2) Analogous organs

122 E
15. Homology is exhibited by :
(1) Wings of butterfly, birds and bat
(2) Paddle of whale, forearm of horse and forelimbs of man
(3) Tail of monkey and bird
(4) Sting of scorpion and honey bee
Ans. (2) Paddle of whale, forearm of horse and forelimbs of man
16. Golden age of Dinosaurs was during :
(1) Cenozoic era (2) Palaeozoic era (3) Archeozoic era (4) Mesozoic era
Ans. (4) Mesozoic era
17. Evolution of birds and mammals occured in :
(1) Eocene and oligocene periods
(2) Silurian and devonian periods
(3) Carboniferous and Permian periods
(4) Cretaceous and triassic periods
Ans. (4) Cretaceous and triassic periods
18. The mesozoic era of earth is called the :
(1) Age of amphibians (2) Age of armoured fishes
(3) Age of primitive man (4) Age of ruling reptiles
Ans. (4) Age of ruling reptiles
19. An era "age of birds and mammals" is :
(1) Mesozoic (2) Palaeozoic (3) Cenozoic (4) Cretaceous
Ans. (3) Cenozoic
20. Origin of life took place in which of the following era :
(1) Mesozoic (2) Palaeozoic (3) Precambrian (4) Proterozoic
Ans. (3) Precambrian

E 123
(D) ASSERTION – REASON QUESTIONS
 Directions: In the following questions, a statement of assertion is followed by a statement of
reason. Mark the correct choice as:
(1) If both Assertion and Reason are true and Reason is the correct explanation of Assertion.
(2) If both Assertion and Reason are true but Reason is not the correct explanation of Assertion.
(3) If Assertion is true but Reason is false.
(4) If both Assertion and Reason are false.
1. Assertion : The primitive atmosphere was reducing once i.e., without oxygen.
Reason : In the primitive atmosphere, oxygen was involved in forming ozone.
Ans (1)
2. Assertion : Organic compounds first evolved in earth required for origin of life were protein and
nucleic acid.
Reason : All life forms were in water environment only.
Ans (2)
3. Assertion : Coacervates are believed to be the precursors of life.
Reason : Coacervates were self-duplicating aggregates of proteins surrounded by lipid
molecules.
Ans (4)
4. Assertion : The earliest organisms that appeared on the earth were non-green and presumably
anaerobes.
Reason : The first autotrophic organisms were the chemoautotrophs that never released oxygen.
Ans (2)
5. Assertion: We have lost all the direct evidence of origin of life.
Reason: The persons responsible for protecting evidences were not skilled.
Ans (3)
6. Assertion: Thorns and tendrils of Bougainvillea and cucurbita represent homology.
Reason: Homologous organs have similar functions but are different in their structural details
and origin.
Ans (3)
7. Assertion: Analogous structures are different in appearance with same function.
Reason: Divergent evolution leads to analogy.
Ans (3)
8. Assertion : Natural selection is the outcome of difference in survival and reproduction among
individuals that show variation in one or more traits.
Reason : Adaptive forms of a given trait tend to become more common; less adaptive ones
become less common or disappear.
Ans (1)
9. Assertion : Java Ape-man, Peking man and Heidelberg man are the fossils of Homo erectus.
Reason : Homo erectus evolved from Homo habilis.
Ans (2)
10. Assertion : Hardy Weinberg principle explains the variations occurring in population and species
over a number of generations.
Reason: This principle is applicable only when genetic drift occurs.
Ans (4)

124 E
(E) VERY SHORT ANSWER QUESTIONS
1. Write the name of Famous palaeontologist/Palaeobotanist of India.

Ans. B.Sahni

2. What is the basis of Darwin's Theory of Natural Selection?

Ans. Enormous rate of reproduction in organisms, struggle for existence and survival of the fittest

3. Who was give the theory of inheritance of acquired characters?

Ans. Lamarck

4. What is genetic drift ?

Ans. The chance of elimination of genes from a small population is know as genetic drift:

5. Write the name of most recent man found as fossil.

Ans. Cro-magnon man

6. What was the cranial capacity of java man.

Ans. 900 cc

7. Fulhrott made an important discovery in evolution and he discovered.

Ans. Neanderthal man

8. Characteristics of primitive monkey which was in the direction of evolution of man.

Ans. 32 teeth

9. What is the greatest advantage of bipedal movement ?

Ans. Fore arms becoming free for carrying out order of brain

10. Which is the most primitive ancestor of man ?

Ans. Ramapithecus

E 125
(F) SHORT ANSWER QUESTIONS
1. Explain Oparin-Haldane theory of chemical evolution of life.
Ans. The first life form could have come from the pre-existing, non-living organic molecules (like
RNA, Proteins, etc.) and that formation of life was preceded by chemical evolution, i.e.,
formation of diverse organic molecules from inorganic constituents.
2. Distinguish between convergent and divergent evolution giving one example of each.
Ans. Divergent Evolution - Development of different functional structures from a common ancestral
form is called divergent evolution.
Homologous organs show divergent evolution.
Examples: Darwin's Finches, Australian Marsupials, locomotion in mammals.
Convergent Evolution - Development of similar adaptive functional structures in unrelated
groups of organisms is called convergent evolution.
Analogous organs show convergent evolution.
Examples: Australian Marsupials and Placental mammals, various aquatic vertebrate and wings
of insect, bird and bat.
3. How did Louis Pasteur disprove spontaneous generation theory?
Ans. Louis Pasteur showed that in pre-sterilized swan neck flasks, life did not come from killed yeast
while in another flask open to air, new organisms arose from ‘killed yeast. Because germ laden
dust particles in the air were trapped by the curved neck which serves as filter.
4. Define homologous organs? Give one example of organ homologous to hand of man?
Ans. Homologous organs are those organs which are similar in basic structure & embryonic
developments but perform different functions. e.g. bones of forelimbs of whales, bat, birds and
human beings.
5. What is the role of variation in evolution?
Ans. Variations are useful for survival of species in changed environmental situations. If a population
of reproducing organisms are suited to a particular niche & if the niche is drastically altered the
population could be wiped out however if some variations were to be present in few individuals,
there would be some chances for them to survive.
6. Describe one evidence which decisively proves that birds have evolved from reptiles?
Ans. Missing link between birds & reptiles called. Archaeopteryx showed that “Birds have evolved
from reptiles”. These are organisms which show the characters of both birds (e.g. presence of
wings & feathers in the body) as well as of reptiles (e.g. gong tail & jaws with identical teeth).
126 E
7. What is the study of fossils called? Mention any three points how the fossils throw light on
past life?
Ans. Study of fossils is known as paleontology.
 Cross-section of the earth’s crust indicates the arrangement of sediments one over the other
during the long history of Earth.
 Different sediments contain different life forms which probably died during the formation of
particular sediment.
 Connecting or missing link – which contains characters of different groups.
8. Why has natural selection not eliminated sickle – cell anaemia?
Ans. Sickle cell anaemia is not eliminated during natural selection because in some cases, sickle cell
anaemia is beneficial as it provides natural defense against malarial parasite.
9. Life originated from the earth’s inorganic atmosphere in the post, but this no longer
happens today. Give two reasons?
Ans. Life cannot be originated in the present day atmosphere because:-
(i) The temperature of present day atmosphere is much less than that of primitive atmosphere.
(ii) The present day atmosphere is oxidizing & not reducing due to presence of oxygen.

E 127
(G) LONG ANSWER QUESTIONS
1. What is adaptive radiation? Explain with an example.

Ans. Adaptive radiation is an evolutionary process that produces new species from a single, rapidly
diversifying lineage. This process occurs due to natural selection. An example of adaptive
radiation is Darwin finches, found in Galapagos Island. A large variety of finches is present in
Galapagos Island that arose from a single species, which reached this land accidentally. As a
result, many new species have evolved, diverged, and adapted to occupy new habitats. These
finches have developed different eating habits and different types of beaks to suit their feeding
habits. The insectivorous, blood sucking, and other species of finches with varied dietary habits
have evolved from a single seed eating finch ancestor.
2. (i) State the Hardy-Weinberg principle.
(ii) When there is a disturbance in the Hardy-Weinberg equilibrium, what would it result in?
(iii) According to this principle, what is the sum total of all allelic frequencies?
Ans. (i) The allele frequency in a population are stable and constant from generation to generation.
(ii) Evolution.
(iii) One.
3. Classify the following as examples of homology and analogy
(i) Hearts of fish and crocodile
(ii) Wings of butterfly and birds
(iii) Eyes of Octopus and Mammals
(iv) Tubers of potato and Sweet potato
(v) Thorns of Bougainvillea and spines of Opuntia
(vi) Thorn of Bougainvillea and tendrils of cucurbits.
Ans. (i) Homology (ii) Analogy
(iii) Analogy (iv) Analogy
(v) Analogy (vi) Homology
4. Stanley Miller and Harold Urey performed an experiment by recreating in the laboratory
the probable conditions of the atmosphere of the primitive earth.
(i) What was the aim of the experiment?
(ii) In what forms was the energy supplied for chemical reactions to occur?
(iii) For how long was the experiment run continuously? Name two products formed.
Ans. (i) To prove Oparins theory of origin of life.
(ii) Electric discharge using electrodes.
(iii) One week : Amino acids and Sugar.
128 E
(H) CASE-STUDY BASED QUESTIONS
1. Read the following and answer the questions given below:-
About 15 mya, primates called Dryopithecus and Ramapithecus were existing. They were hairy
and walked like gorillas and chimpanzees. Ramapithecus was more man-like while Dryopithecus
was more ape-like. Few fossils of man-like bones have been discovered in Ethiopia and
Tanzania. These revealed hominid features leading to the belief that about 3-4 mya, man-like
primates walked in eastern Africa. They were probably not taller than 4 feet but walked up right.
Two mya, Australopithecines probably lived in East African grasslands. Evidence shows they
hunted with stone weapons but essentially ate fruit. Some of the bones among the bones
discovered were different. This creature was called the first human-like being the hominid and
was called Homo habilis. The brain capacities were between 650-800cc. They probably did not
eat meat. Fossils discovered in Java in 1891 revealed the next stage, i.e., Homo erectus about 1.5
mya. Homo erectus had a large brain around 900cc.

(i) Whom did Ramapithecus look like?


Ans. Ramapithelus was more man like.
(ii) Where have been discovered few fossils of man like bones?
Ans. Few fossils of man like bones have been discovered in Ethiopia and Tanzania.
(iii) What is the cranial capacity of Java man?
Ans. Cranial Capacity of Java man is 900 cc.
(iv) Where lived Australopithecines?
Ans. Australopithecines probably lived in East - African grasslands.
(v) Who was Homo - habilis?
Ans. The first human like being the hominid was called Homo - habilis.

2. Read the following and answer the questions from given below:-
In a given population one can find out the frequency of occurrence of alleles of a gene or a locus.
This frequency is supposed to remain fixed and even remain the same through generations.
Hardy-Weinberg principle stated it using algebraic equations. This principle says that allele
frequencies in a population are stable and is constant from generation to generation. The gene
pool (total genes and their alleles in a population) remains a constant. This is called genetic
equilibrium. Sum total of all the allelic frequencies is 1. Individual frequencies, for example, can
be named p, q, etc. In a diploid, p and q represent the frequency of allele A and allele a. The
frequency of AA individuals in a population is simply p2. This is simply stated in another ways,
i.e., the probability that an allele A with a frequency of p appear on both the chromosomes of a
diploid individual is simply the product of the probabilities, i.e., p2. Similarly of aa is q2, of Aa
2pq. Hence, p2+2pq+q2=1. This is a binomial expansion of (p+q)2.

(i) Explain the Genetic drift


Ans. The spread of genes from one breeding population to another by migration which may result in
change in gene frequency is called genetic drift.

E 129
(ii) Where genetic drift operates?
Ans. Genetic drift operates only in smaller populations.
(iii) State what do 'p' and 'q' denote in the equation p2 + 2pq + q2 = 1.
Ans. In this equation p and q are individual freqeuencies of different alleles.
(iv) What is Hardy - weinberg principle says.
Ans. Hardy - weinberg principle says that alleles frequencies in a population are stable and is constant
from generation to generation.
(v) If any fluctuation in genetic equilibrium. what will happen?
Ans. Any functuation in genetic equilibrium leads to evolution.

3. Read the following and answer the questions from given below:-

The origin of life is considered a unique event in the history of universe. The universe is vast.
Relatively speaking the earth itself is almost only a speck. The universe is very old – almost 20
billion years old. Huge clusters of galaxies comprise the universe. Galaxies contain stars and
clouds of gas and dust. Considering the size of universe, earth is indeed a speck. The Big Bang
theory attempts to explain to us the origin of universe. It talks of a singular huge explosion
unimaginable in physical terms. The universe expanded and hence, the temperature came down.
Hydrogen and Helium formed sometime later. The gases condensed under gravitation and
formed the galaxies of the present day universe. In the solar system of the milky way galaxy,
earth was supposed to have been formed about 4.5 billion years back. There was no atmosphere
on early earth. Water vapour, methane, carbon di oxide and ammonia released from molten mass
covered the surface. The UV rays from the sun broke up water into Hydrogen and Oxygen and
the lighter H2 escaped. Oxygen combined with ammonia and methane to form water, CO2 and
others. The ozone layer was formed. As it cooled, the water vapor fell as rain, to fill all the
depressions and form oceans. Life appeared 500 million years after the formation of earth, i.e.,
almost four billion years back.

(i) Name the scientist who disproved the spontaneous generation theory.
Ans. Louis Pasteur disproved the theory of spontaneous generation. He proved biogenesis
(ii) Name the different gases contained in the flask used as an experimental setup by S.L.
Miller.
Ans. CH4, NH3, H2O and H2.
(iii) What provided energy for abiotic synthesis on primitive earth?
Ans. Very high temperatures due to lightning or UV – rays provided energy for abiotic synthesis.
(iv) When did life appear on earth?
Ans. Life originated sometimes 3600 million years ago.
(v) Mention the type of nutrition in the cells that originated first during the origin of life.
Ans. Heterotrophic nutrition
4. Read the following and answer the questions given below:-
Comparative anatomy and morphology shows similarities and differences among organisms of
today and those that existed years ago. Such similarities can be interpreted to understand whether
common ancestors were shared or not. For example whales, bats, Cheetah and human (all
mammals) share similarities in the pattern of bones of forelimbs. Though these forelimbs perform
different functions in these animals, they have similar anatomical structure – all of them have
humerus, radius, ulna, carpals, metacarpals and phalanges in their forelimbs. Hence, in these
animals, the same structure developed along different directions due to adaptations to different
needs. This is divergent evolution and these structures are homologous. Homology indicates
common ancestry. Other examples are vertebrate hearts or brains. In plants also, the thorn and
tendrils of Bougainvillea and Cucurbita represent homology. Homology is based on divergent
evolution whereas analogy refers to a situation exactly opposite. Wings of butterfly and of birds
look alike. They are not anatomically similar structures though they perform similar functions.
Hence, analogous structures are a result of convergent evolution - different structures evolving
for the same function and hence having similarity.

(i) Why are analogous structures, a result of convergent evolution ?


Ans. Analogous structures are a result of convergent evolution because they are not anatomically
similar structures though they perform similar functions.
(ii) Identify the examples of convergent evolution from the following :
(a) Flippers of penguins and dolphins
(b) Eyes of octopus and mammals
(c) Vertebrate brains
Ans. Flippers of penguins and dolphins and Eyes of octopus and mammals.
(iii) “Sweet potato tubers and potato tubers are the result of convergent evolution.” Justify the
statement.
Ans. Sweet potato tuber is a root modification and potato tuber is a stem modification but they show
convergent evolution because both of them are unrelated and perform the functions similar like
storage of food and vegetative reproduction.
(iv) Name the type of evolution that has resulted in the development of structures like wings of
butterfly and bird. What are such structures called.
Ans. Convergent evolution. Such structures are called analogous structures.
(v) State the evolutionary relationship giving reasons between the thorn of Bougainvillea and
tendril of cucurbit.
Ans. Divergent evolution/Homologous organs. Similar in origin but perform different function.

E 131
IMPORTANT NOTE

______________________________________________________________________

_______________________________________________________________________
_______________________________________________________________________
_______________________________________________________________________
_______________________________________________________________________
_______________________________________________________________________
_______________________________________________________________________
_______________________________________________________________________
_______________________________________________________________________
_______________________________________________________________________
_______________________________________________________________________
_______________________________________________________________________
_______________________________________________________________________
_______________________________________________________________________
_______________________________________________________________________
_______________________________________________________________________
_______________________________________________________________________
_______________________________________________________________________
_______________________________________________________________________
_______________________________________________________________________
_______________________________________________________________________
_______________________________________________________________________
_______________________________________________________________________

_______________________________________________________________________

_______________________________________________________________________
_______________________________________________________________________
CHAPTER 7
(A) NCERT QUESTIONS & SOLUTIONS
1. What are the various public health measures, which you would suggest as safeguard against
infectious diseases?
Ans. Few of the public health measures as safeguard against infectious diseases are :
Proper disposal of waste and excreta
Periodic cleaning and disinfection of water reservoirs, pools, cesspools and tanks
Observing standard practices of hygiene in public catering
2. In which way has the study of biology helped us to control infectious diseases?
Ans. The advancements made in biological science have armed us to effectively deal with many
infectious diseases.
The use of vaccines and immunisation programmes have enabled us to completely eradicate a
deadly disease like smallpox and to control other infectious diseases like polio, diphtheria,
pneumonia and tetanus to a large extent.
Biotechnology is at the verge of making available newer and safer vaccines.
Discovery of antibiotics and various other drugs has also enabled us to effectively treat
infectious diseases.
3. How does the transmission of each of the following diseases take place? [IMP.]
(a) Amoebiasis (b) Malaria (c) Ascariasis (d) Pneumonia
Ans. (a) Amoebiasis:- Houseflies act as mechanical carriers and serve to transmit the pathogen
(Entamoeba histolytica) from faeces of infected person to food and food products, thereby
contaminating them. Drinking water and food contaminated by the faecal matter are the main
source of infection.
(b) Malaria :- Malaria is transmitted to a healthy person with the bite of an infected female
Anopheles mosquito. The female Anopheles mosquito is the vector (transmitting agent) for
the pathogen (Plasmodium).
(c) Ascariasis:- The eggs of the pathogen (Ascaris) are excreted along with the faeces of
infected persons which contaminate soil, water, plants, etc. A healthy person acquires this
infection through contaminated water, vegetables, fruits, etc.
(d) Pneumonia :- A healthy person acquires the infection by inhaling the droplets/aerosol
released by an infected person or even by sharing glasses and utensils with an infected
person.

E 133
4. What measure would you take to prevent water-borne diseases?
Ans. Measures particularly essential to prevent water-borne diseases are:
(1) Maintenance of personal hygiene by :
● Keeping the body clean
● Consumption of clean drinking water, food, vegetables, fruits, etc
(2) Maintenance of public hygiene by :
● Proper disposal of waste and excreta
● Periodic cleaning and disinfection of water reservoirs, pools, cesspools and tanks
● Observing standard practices of hygiene in public catering
5. Discuss with your teacher what does ‘a suitable gene’ means, in the context of DNA
vaccines.
Ans. ‘A suitable gene’ means that a disease-resistant gene present on the genome of lower organisms.
By the technique of genetic engineering, this ‘suitable gene’ is transferred in a vaccine to inject in
human beings to induce the development of immunity.
6. Name the primary and secondary lymphoid organs. [IMP.]
Ans. Primary lymphoid organs - Bone Marrow, Thymus gland.
Secondary lymphoid organs - Spleen, Lymph nodes, Tonsils, Peyer’s patches of small intestine,
Appendix.
7. The following are some well-known abbreviations, which have been used in this chapter.
Expand each one to its full form: [IMP.]
(a) MALT (b) CMI (c) AIDS (d) NACO (e) HIV
Ans. (a) MALT = Mucosa Associated Lymphoid Tissue
(b) CMI = Cell Mediated Immunity
(c) AIDS = Acquired Immuno Deficiency Syndrome
(d) NACO = National AIDS Control Organisation
(e) HIV = Human Immunodeficiency Virus
8. Differentiate the following and give examples of each:
(a) Innate and acquired immunity (b) Active and passive immunity
Ans. (a) Difference between innate and acquired immunity :
S.N. Innate Immunity Acquired Immunity

1. Pathogen non specific type of defence Pathogen specific type of defence

2. Not characterised by memory Characterised by memory

Examples : Different barriers(Skin etc.) Examples : B and T lymphocytes

134 E
(b) Difference between active and passive immunity :
S.N. Active Immunity Passive Immunity
1. Active participation of immune No participation of immune system of
system of host host
2. Antibodies are produced actively Preformed antibodies are received
by immune system of host passively by/administered to host
3. Slow and takes time to give its full Quick response
effective response
Examples : Vaccines etc. Examples : Antiserums, Antivenoms etc.
9. Draw a well-labelled diagram of an antibody molecule. [IMP.]
Ans.
Antigen binding site Antigen binding site
N

Light
chain
Disulfide Bond
Heavy chain

C C
Structure of Antibody molecule
10. What are the various routes by which transmission of human immuno - deficiency virus
takes place?
Ans. Transmission of HIV-infection generally occurs :
By sexual contact with infected person
By transfusion of contaminated blood and blood products
By sharing infected needles as in the case of intravenous drug abusers
From infected mother to her child through placenta
11. What is the mechanism by which the AIDS virus causes deficiency of immune system of the
infected person?
Ans. HIV enters into helper T-lymphocytes and produce progeny viruses. The progeny viruses
released in the blood attack other helper T-lymphocytes. This is repeated leading to a progressive
decrease in the number of helper T-lymphocytes in the body of the infected person.
Due to progressive decrease in the number of helper T-lymphocytes, the person becomes
immuno-deficient.
12. How is a cancerous cell different from a normal cell?
Ans. Unlike a normal cell,
Cancerous cell undergoes uncontrolled mitotic divisions.
Cancerous cell leads to the formation of tumour.
Cancerous cell does not show contact inhibition.
E 135
13. Explain what is meant by metastasis.
Ans. Cells sloughed from such tumors reach distant sites through blood and wherever they get lodged
in the body & they start a new tumor there. This property called metastasis is the most feared
property of malignant tumors.
14. List the harmful effects caused by alcohol/drug abuse.
Ans. Few of the harmful effects caused by alcohol/drug abuse are :
Immediate adverse effects of drugs and alcohol abuse are reckless behaviour, vandalism,
violence etc.
Excessive doses of drugs may lead to coma, death due to respiratory failure, heart failure or
cerebral haemorrhage.
A combination of drugs or their intake along with alcohol generally results in overdosing and
even deaths.
The chronic use of drugs and alcohol damages nervous system and liver (cirrhosis).
The use of drugs and alcohol during pregnancy is also known to adversely affect the foetus.
15. Do you think that friends can influence one to take alcohol/drugs? If yes, how may one
protect himself/herself from such an influence?
Ans. One of the reasons for alcohol drinking or drug abuse is peer pressure, so one can be easily
influenced by one`s friends to take alcohol /drugs. One must avoid undue peer pressure with
one`s own will-power or may seek the help of one`s parents and teachers.
16. Why is that once a person starts taking alcohol or drugs, it is difficult to get rid of this
habit? Discuss it with your teacher.
Ans. Use of alcohol or drugs even once can be a fore-runner to addiction. Thus, the addictive potential
of drugs and alcohol, pull the user into a vicious circle leading to their regular use (abuse) from
which the person may not be able to get out. In the absence of any guidance or counselling, the
person gets addicted and becomes dependent on their use. Dependence is the tendency of the
body to manifest a characteristic and unpleasant withdrawal syndrome if regular dose of
drugs/alcohol is abruptly discontinued. This is characterised by anxiety, shakiness, nausea and
sweating, which may be relieved when use is resumed again.
In some cases, withdrawal symptoms can be severe and even life threatening and the person may
need medical supervision.
17. In your view what motivates youngsters to take to alcohol or drugs and how can this be
avoided? [IMP.]
Ans. Causes for motivation among youngsters towards alcohol or drugs use include :
Natural curiosity
Need for adventure and excitement
Experimentation
Perception that it is ‘cool’ or progressive to smoke use drugs or alcohol
To escape facing problems (like stress, from pressures to excel in academics or examinations)
Unstable or unsupportive family structures and peer pressure
And measures particularly useful to avoid alcohol or drugs abuse among youngsters
include:
Avoid undue peer pressure
Education and counselling
Seeking help from parents and peers
Looking for danger signs
Seeking professional and medical help

136 E
(B) PREVIOUS YEAR QUESTIONS
1. Tetanus antitoxin (Tetanus toxoid) when injected into the human body it immediately
provides: [CBSE 2023]
(a) Innate immunity (b) Passive immunity
(c) Auto immunity (d) Active immunity
Ans. (b) Passive immunity
2. Select the pathogen mismatched with the symptoms of disease caused by it from the list given
below : [CBSE 2023]
(a) Entamoeba histolytica : Constipation, abdominal pain.
(b) Epidermophyton : Dry scaly lesions on nail.
(c) Wuchereria bancrofti : Chronic inflammation of lymphatic vessels of lower limb.
(d) Haemophilus influenzae : Blockage of the intestinal passage.
Ans. (d) Haemophilus influenzae : Blockage of the intestinal passage.
3. Interferons are proteins. In humans they are secrted by : [CBSE 2023]
(a) Thymus gland (b) B-lymphocytes
(c) Viral infected cells (d) Tonsils
Ans. (c) Viral infected cells
4. The decrease in the T - lymphocytes count in human blood will result in : [CBSE 2023]
(a) Decrease in antigens (b) Decrease in antibodies
(c) Increase in antibodies (d) Increase in antigens
Ans. (b) Decrease in antibodies
5. Immunotherapy these days is one of the most efficient way of treatment of cancer. The
therapy involved activates the immune system and destroys the tumour.
(i) Write an example of one such biological response modifier used in immunotherapy.
(ii) Why do patients need such substances if immune system is already working in body?
(iii) State what is 'Contact inhibition'. [CBSE 2023]
Ans. (i)  - interferon
(ii) Because tumor cell have been shown to avoid to detection and destruction by immune
system.
(iii) Contact inhibition:-normal cell show a property called contact inhibition by virtue of which
contact with other cell inhibit their uncontrolled growth.
5. (a) "Plasmodium protozoan needs both a mosquito and human host for its continuity.
"Explain.
OR
(b) We all must work towards maintaining good health because 'health is wealth' Enlist
any six ways of achieving good health. [CBSE 2023]
137
Ans. (a) Plasmodium enters the human body as sporozoites (infectious form) through the bite of
infected female Anopheles mosquito.
 The parasite initially multiply within the liver cells and then attack the red blood cells
(RBCs) resulting in their rupture. The rupture of RBCs is associated with release of a toxic
substance, haemozoin, which is responsible for the chill and high fever recurring every three
to four days.
  When a female Anopheles mosquito bites an infected person, these parasites enter the
mosquito's body and undergo further development.
 When these mosquitoes bite a human, the sporozoites are introduced into his /her body,
thereby initiating the events mentioned above.
 It is interesting to note that the malarial parasite requires two hosts - human and mosquitoes -
to complete its life cycle. the female Anopheles mosquito is the vector (transmitting agent)
too.
OR
(b) Balanced diet, personal hygiene and regular exercise are very important to maintain good
health.
 Yoga has been practised since time immemorial to achieve physical and mental health.
 Awareness about diseases and their effect on different bodily functions, vaccination
(immunisation) against infectious diseases, proper disposal of wastes, control of vectors and
maintenance of hygiene in food and water resources are necessary for achieving good health.
6. When a microorganism invades a host, a definite sequence of events usually occur leading
to infection and disease, causing suffering to the host. This process is called pathogenesis.
Once a microorganism overcomes the defense system of the host, development of the
disease follows a certain sequence of events as shown in the graph. Study the graph given
below for the sequence of events leading to appearance of a disease and answer the
questions that follow :
Inclubation
period Death if immune
Number of microorganisms

Period of responses and /or


illness medical treatments fail
Period of decline

Mild signs or symptoms


No signs or symptoms
Time

(a) In which period, according to the graph there are maximum chances of a person
transmitting a disease/infection and why?

(b) Study the graph and write what is an incubation period. Name a sexually transmitted
disease that can be easily transmitted during this period. Name the specific type of
lymphocytes that are attacked by the pathogen of this disease. [CBSE 2023]

138 E
OR
(b) Draw a schematic labelle diagram of an antibody.
(c) In which period, the number of immune cells forming antibodies will be the highest in a
person suffering from pneumonia? Name the immune cells that produce antibodies.
[CBSE 2023]
Ans. (a) Incubation period. Because we can’t identity the disease at early stage.
(b) The incubation period means the time between catching the pathogens and beginning to have
symptoms of the disease. AIDS , T cells.
OR

(b)

Structure of an antibody molecule

(c) At the pick of diseases, B cells.


8. Why a malignant tumour considered to be more damaging than a benign tumour? Explain.
[CBSE 2023]
Ans. Tumours are the celluar mass that develop inside the body due to accumulation of cells or their
over growth. These are divided into categories of benign and malignant tumors. Benign tumours
are less harmfull as compared to malignant tumours. It is because, malignant tumours have
maignant property i.e., the cells of malignant tumours can detach from their main site and spread
in other parts of the body. Thus, malignant tumours spread easily in the body. This leads to
chances of cancer spread in the patient with malignant tumours.
9. A boy developed some allergic reactions when he straight entered into his air conditioned room
after a game of football outside his house. Write any two symptoms that could be noticed in
such condition. How does our body combat such conditions? [CBSE Term – II 2022]
Ans. Symptoms of allergic reactions include sneezing, watery eyes, running nose and difficulty in
breathing. Immune system overreacts by producing antibodies called Immunoglobulin E (IgE).
These antibodies travel to cells that release chemicals, causing an allergic reaction.
10.(a) (i) Write the Scientific name of the plant from where natural cannabinoids are obtained.
(ii) Mention the parts of the plant that are used for extracting the drug. [3]
(iii) How does the drug affect human body ?
OR
(b) Epithelial lining of our intestine is considered as secondary lymphoid organ. Justify the
statement. [CBSE Term – II 2022]

E 139
Ans. (a) (i) Cannabis sativa
(ii) Cannabinoids are obtained from the inflorescence of the plant .
(iii) These drugs affect cardiovascular system of the body. They affect brain areas that
influence pleasure, memory, thinking, concentration, movement and coordination.
OR
(b) There is lymphoid tissue also located within the lining of the major tracts (respiratory,
digestive and urogenital tracts) called mucosal- associated lymphoid tissue (MALT). It
constitutes about 50 percent of the lymphoid tissue in human body.
11. (a) Write the complete name of the diagnostic test for AIDS. Explain the principle it works on.
(b) Name the type of genetic material present in AIDS causing pathogen.
[CBSE Term – II 2022]
Ans. (a) ELISA – Enzyme Linked Immuno Sorbent Assay. It is based on the principle of antigen-
antibody interaction where a pathogen can be detected by the presence of antibodies
(proteins, glycoproteins, etc.) on it.
(b) HIV is a retrovirus, which means it carries single-stranded RNA as its genetic material rather
than the double-stranded DNA human cells carry.
12. A patient complains of suffering from constipation, stomach ache, stool with blood clots
and excess mucous. The physician diagnosed it as amoebiasis, after stool test.
(a) Write the scientific name of the microbe identified in the stool sample.
(b) How do you think, the patient must have contracted it? [CBSE Term – II 2022]
(c) Write your suggestions to the patient to avoid infection in future.
Ans. (a) Entamoeba histolytica
(b) Houseflies act as mechanical carriers and serve to transmit the parasite from faeces of
infected person to food and food products, thereby contaminating them. Drinking water and food
contaminated by the faecal matter are the main source of infection.
(c) Perform hand hygiene frequently, especially before handling food or eating, and after using
the toilet or handling faecal matter. Wash hands with liquid soap and water, and rub for at least
20 seconds.
13. Identify and name the disease in which the patient’s cells lose the property of contact
inhibition. State its possible causes and explain any three methods to accurately detect the
pathological and physiological changes that take place due to the disease in living tissues.
[CBSE Imp. Question]
OR
A patient had tested positive to ELISA Test. Identify the disease and the pathogen
responsible, give reasons for the reduced/ weak immunity of the patient and trace the path,
spread and effects of this pathogen in the human body. [CBSE Imp. Question]
Ans. Disease: Cancer
Probable Causes:
Physical/ Environmental- Exposure to X – rays/ gamma rays/ UV rays; Chemicals/Nicotine in
tobacco/ other carcinogens
Biological- Viral oncogenes/ Mutations
Detection and diagnosis:
1. Cancer detection is based on biopsy and histopathological studies of the tissue; blood and
bone marrow tests for increased cell counts in the case of leukemias. In biopsy, a piece of the
suspected tissue cut into thin sections is stained and examined under microscope
(histopathological studies) by a pathologist.
2. Techniques like radiography (use of X-rays), CT (computed tomography) and MRI
(magnetic resonance imaging) are very useful to detect cancers of the internal organs.
Computed tomography uses X-rays to generate a three dimensional image of the internals of
an object. MRI uses strong magnetic fields and non-ionising radiations to accurately detect
pathological and physiological changes in the living tissue.
3. Antibodies against cancer-specific antigens are also used for detection of certain cancers.
4. Techniques of molecular biology can be applied to detect genes in individuals with inherited
susceptibility to certain cancers. (any three methods)
OR
Disease: AIDS (Acquired Immuno Deficiency Syndrome)
Pathogen: Human Immuno deficiency virus (HIV).
Reason:
Due to decrease in the number of helper T lymphocytes, the person starts suffering from
infections that could have been otherwise overcome such as those due to bacteria especially
Mycobacterium, viruses, fungi and even parasites like
Toxoplasma.
The path of this pathogen and its spread and effect on the human body:
 After getting into the body of the person, the virus enters into macrophages where RNA
genome of the virus replicates to form viral DNA with the help of the enzyme reverse
transcriptase.
 This viral DNA gets incorporated into host cell’s DNA and directs the infected cells to
produce virus particles.
 The macrophages continue to produce virus and in this way acts like a HIV factory.
 Simultaneously, HIV enters into helper T-lymphocytes (TH), replicates and produce progeny
viruses.
 The progeny viruses released in the blood attack other helper T-lymphocytes. This is
repeated leading to a progressive decrease in the number of helper T lymphocytes in the
body of the infected person.
  During this period, the person suffers from bouts of fever, diarrhoea and weight loss.
14. The main barrier that prevents the entry of micro organism into body is [CBSE 2020]
(A) Antibodies (B) Macrophages (C) Monocytes (D) Skin
Ans. (D) Skin
E 141
15. It is often observed that the chances of a person suffering from measles in his or her
lifetime are low, if he or she has suffered from the disease in their early childhood. Justify
the statement. [CBSE 2020]
Ans. When our body encounters a pathogen for the first time produces a response called primary
response which is of low intensity. Subsequent encounter with the same pathogen elicits a highly
intensified secondary or anamnestic response. This is ascribed to the fact that our body appears to
have memory of the first encounter.
Hence, the chances of a person suffering from measles in his or her lifetime are low, if he or she
has suffered from the disease in their early childhood.
16. A student on a field trip suddenly felt breathlessness and started to sneeze very badly.
Nature this response and explain what it is due to. [CBSE 2019]
Ans. It could be because of allergy to pollen, mites, etc., which are different in different places.
The exaggerated response of the immune system to certain antigens (or allergens) present in the
environment is called allergy. Allergy is due to the release of chemicals like histamine and
serotonin from the mast cells.
17. Mention one application for each of the following: [CBSE 2018]
(a) Passive immunization
(b) Antihistamine
(c) Colostrum
(d) Cytokinin-barrier
Ans. (a) Provides preformed antibodies for quick response in case of infection by deadly
microbes(e.g. tetanus) or snake bite etc.
(b) Reduces symptoms of allergy
(c) Provides passive immunity (IgA antibodies) to new born
(d) Protects non-infected cells from further viral infection
18. Name a human disease, its causal organism, symptoms (any three) and vector, spread by
intake of water and food contaminated by human faecal matter. [CBSE 2018]
Ans. ● Disease spread by intake of water and food contaminated by human faecal matter :
Amoebiasis (Amoebic Dysentery)
● Causal organism : Entamoeba histolytica (a protozoan)
● Vector : Housefly
● Symptoms : Constipation, abdominal pain and cramps, stools with excess mucous and blood
clots, etc
19. Explain the relationship between B-lymphocytes and T-lymphocytes in developing an
immune respons. [CBSE 2017]
Ans. B-lymphocytes- produce antibodies to fight pathogen.
T - lymphocytes - do not produce antibodies but help B cells to produce them
can also destroy pathgen directly.
142 E
(C) MULTIPLE CHOICE QUESTIONS
1. Widal test is done for detecting.
(1) Pneumonia (2) Malaria (3) Typhoid (4) Cholera
Ans. (3) Typhoid
2. The name of 'Mary Mallon' is associated with the disease
(1) typhoid (2) leprosy (3) tuberculosis (4) smallpox
Ans. (1) typhoid
3. The elephantiasis is caused due to infection of
(1) Entamoeba (2) Ascaris
(3) Wuchereia (4) Perionyx
Ans. (3) Wuchereia
4. Dengue fever is transmitted by :-
(1) Aedes aegypti (Tiger mosquito) (2) Culex fatigaus
(3) Anopheles (4) Aedes donovoni
Ans. (1) Aedes aegypti (Tiger mosquito)
5. Plasmodium enters in the human body as :-
(1) Female Anopheles mosquito (2) Sporozoite
(3) Trophozoite (4) Haemozoin
Ans. (2) Sporozoite
6. Haemophilus influenzae causes :-
(1) Typhoid (2) Plague (3) Pneumonia (4)Influenza
Ans. (3) Pneumonia
7. House flies are mechanical carriers of :-
(1) Amoebiasis (2) Malaria (3) Common cold (4) Plague
Ans. (1) Amoebiasis
8. Ringworm in humans is caused by :
(I) Viruses (2) Bacteria (3) Fungi (4) Nematodes
Ans. (3) Fungi
9. Infection of Ascaris usually occurs by :
(1) mosquito bite
(2) drinking water containing eggs of Ascaris
(3) eating imperfectly cooked pork.
(4) Tse-tse fly
Ans. (2) drinking water containing eggs of Ascaris
10. Sporozoites of Plasmodium is seen in :
(1) Human RBCs (2) Human liver
(3) Gut of male Anopheles (4) Salivary gland of Aedes
Ans. (2) Human liver
E 143
11. Antigen is :-
(1) Substances which stimulates the production of venom
(2) Vaccine
(3) Antibody production stimulating agent
(4) Part of the body defence system
Ans. (3) Antibody production stimulating agent
12. Which cell is not phagocytic in nature :-
(1) Monocyte (2) Macrophage (3) Neutrophil (4) N.K. cell
Ans. (4) N.K. cell
13. Colostrum, the first milk secretion of mammary gland is rich in immunoglobulin
(1) IgE (2) IgM (3) IgA (4) IgG
Ans. (3) IgA
14. Surgical removal of thymus of a new born shall result in failure to mature -
(1) Monocytes (2) B – lymphocytes (3) T – lymphocytes (4) Basophils
Ans. (3) T - lymphocytes
15. Vaccination is categorized under
(1) Pathophysiology (2) Pathogenesis (3) Diagnosis (4) Prophylaxis
Ans. (4) Prophylaxis
16. Vaccine is :-
(1) Type of antibody (2) Inactivated antigen
(3) Inactivated pathogen (4) Activated pathogen
Ans. (3) Inactivated pathogen
17. AIDS is caused due to :-
(1) Reduction in number of helper T-cells (2) Lack of interferon
(3) Reduction in number of killer T-cells (4) Autoimmunity
Ans. (1) Reduction in number of helper T-cells
18. The antibody produce during allergy are
(1) IgG type (2) IgM type (3) IgA type (4) IgE type
Ans. (4) IgE type
19. Which one of the following techniques is safest for the detection of cancers?
(1) Magnetic resonance imaging [MRI] (2) Radiography [X-Ray]
(3) Computed tomography [CT] (4) Histopathological studies
Ans. (1) Magnetic resonance imaging [MRI]
20. What are the reasons of spread of cancer in humans :
(1) Cell growth (2) Cell death
(3) Uncontrolled differentiation of cell (4) Both (1) and (3)
Ans. (4) Both (1) and (3)
(D) ASSERTION & REASON QUESTIONS
 Directions: In the following questions, a statement of assertion is followed by a statement of
reason. Mark the correct choice as:
(1) If both Assertion and Reason are true and Reason is the correct explanation of Assertion.
(2) If both Assertion and Reason are true but Reason is not the correct explanation of Assertion.
(3) If Assertion is true but Reason is false.
(4) If both Assertion and Reason are false.
1. Assertion: There is no chance of malaria to be spread in a man after the bite of male Anopheles
mosquito.
Reason: It carries a non-virulent strain of Plasmodium.
Ans. (2)
2. Assertion : Plasmodium vivax is responsible for malaria.
Reason : Malaria is caused by polluted water.
Ans. (3)
3. Assertion : Tapeworm, roundworm and pinworm are endoparasites of human intestine.
Reason : Improperly cooked food is the source of all intestinal infections.
Ans. (1)
4. Assertion : Escherichia coli, Shigella sp. and Salmonella sp. are all responsible for diarrhoeal
diseases.
Reason : Dehydration is common to all types of diarrhoeal diseases and adequate supply of
fluids and electrolytes should be ensured.
Ans. (2)
5. Assertion : Rabies is an infection of mammals, it involves central nervous system which may
result in paralysis and finally death.
Reason : This is caused by neurotropic bacteria in saliva of rabies animal.
Ans. (3)
6. Assertion : -cells work chiefly by secreting substances called antibodies into the body fluids.
Reason : Antibodies ambush foreign antigen circulating in the blood stream.
Ans. (1)
7. Assertion : Interferons help in the elimination of viral infections.
Reason : Interferons released by infected cells, reach nearby unaffected cells and make them
resistant to viral infection.
Ans. (1)
8. Assertion : An antibody is represented by H2L2.
Reason : Each antibody is made of four peptide chains.
Ans. (1)

9. Assertion : Innate immunity is non-specific defence.


Reason : It consists of four types of barriers.
Ans. (1)
10. Assertion : T-lymphocytes mediate CMI response.
Reason : The above response makes it easy to transplant organs.
Ans. (3)
E 145
(E) VERY SHORT ANSWER QUESTIONS
1. Malaria, typhoid, pneumonia and amoebiasis are some of the human infectious diseases.
Which ones of these are transmitted through mechanical carriers?
Ans. Malaria and amoebiasis are transmitted through mechanical carriers.

2. Name the two intermediate hosts which the human liver fluke depends on to complete its
life cycle so as to facilitate parasitization of its primary host.

Ans. Snail and Fish

3. How does haemozoin affect the human body when released in blood during malarial
infection?

Ans. Haemozoin is responsible for the chill and high fever recurring every three to four days during
malarial infection.

4. State two different roles of spleen in the human body.

Ans. Spleen is the secondary lymphoid organ that stores lymphocytes, it filters microbes and acts as a
reservoir to store erythrocytes.

5. A body of ten years had chicken pox. He is not expected to have the same disease for the
rest of his life. Mention how it is possible.

Ans. The boy when encounters a pathogen for the first time, his body produces antibodies that results
in the memory of the first encounter, to protect the body in future.

6. Why do pollen grains of some flowers trigger 'sneezing' in some people?

Ans. Pollen grains trigger sneezing by causing allergic reaction.

7. Why is secondary immune response more intense than the primary immune response in
humans?

Ans. This is because of presence of antiboides developed during primary.

8. How do cytokine barriers provide innate immunity in humans?

Ans. Cytokine barriers provide innate immunity by releasing interferons. These interferons are
secreted by virus infected cells and protect the non-infected cells from further viral infection.

9. Name any two physiological barriers that provide innate immunity?

Ans. Acid in Stomach/Saliva in mouth/tears in eyes.

146 E
(F) SHORT ANSWER QUESTIONS
1. Name two diseases whose spread can be controlled by the eradication of Aedes mosquitoes.
Ans. Dengue and Chikungunya

2. (a) Name the source plant of heroin drug. How is it obtained from the plant?

(b) Write the effects of heroin on the human body.

Ans. (a) Heroin is obtained by acetylation of morphine, which is extracted from the latex of poppy
plant Papaver somniferum.

(b) Heroin is a depressant and slows down body functions.

3. A student on a field trip suddenly felt breathlessness and started to sneeze very badly.
Nature this response and explain what it is due to.

Ans. It could be because of allergy to pollen, mites, etc., which are different in different places.

The exaggerated response of the immune system to certain antigens (or allergens) present in the
environment is called allergy. Allergy is due to the release of chemicals like histamine and
serotonin from the mast cells.

4. What is morphine? Give its use and abuse.

Ans. Morphine is obtained from opium. It acts as useful analgesic and abuse causes addition.

5. Name any two physiological processes which assist in increasing immunity of the body?

Ans. 1. Presence of HCl in stomach

2. Presence of bile in duodenum

6. Why is secondary immune response more intense than the primary immune response in
humans?

Ans. Body will have memory B cells of the first encounter/presence of antibodies developed during
primary immune response.

7. Why is Gambusia introduced into drains and ponds?

Ans. Gambusia is a king of fish that prays upon the larvae of mosquito. It is therefore introduced into
the drains and ponds. To feed on Mosquito larvae / to eliminate the vectors responsible for
causing malaria. It thus helps in controlling malaria.

8. How do interferons protect us?

Ans. Interferons are antiviral agents produced by virus infected cells and can fight tumors. The virus-
infected cells stimulate neighboring cells to release antiviral proteins by releasing interferons.
Thus, interferons protect non-infected cells from further viral infections by creating cytokine
barriers.

147
(G) LONG ANSWER QUESTIONS
1. Mention one application for each of the following:
(a) Passive immunization (b) Antihistamine (c) Colostrum (d) Cytokinin-barrier

Ans. (a) Provides preformed antibodies for quick response in case of infection by deadly microbes
(e.g. tetanus) or snake bite etc.

(b) Reduces symptoms of allergy

(c) Provides passive immunity (IgA antibodies) to new born

(d) Protects non-infected cells from further viral infection

2. Name a human disease, its causal organism, symptoms (any three) and vector, spread by
intake of water and food contaminated by human faecal matter.

Ans. ● Disease spread by intake of water and food contaminated by human faecal matter:

Amoebiasis (Amoebic Dysentery)

● Causal organism: Entamoeba histolytica (a protozoan)

● Vector: Housefly

● Symptoms: Constipation, abdominal pain and cramps, stools with excess mucous and blood
clots, etc.

3. Principle of vaccination is based on the property of “memory” of the immune system.


Taking one suitable example, justify the statement.

Ans. In vaccination, a preparation of antigenic proteins of pathogen or inactivated/weakened pathogen


(vaccine) are introduced into the body. The antibodies produced in the body against these
antigens would neutralise the pathogenic agents during actual infection. The vaccines also
generate memory – B and T lymphocytes that recognise the pathogen quickly on subsequent
exposure and overwhelm the invaders with a massive production of antibodies.

4. Name a human disease, its causal organism, symptoms (any three) and vector, spread by
intake of water and food contaminated by human faecal matter.

Ans. ● Disease spread by intake of water and food contaminated by human faecal matter:-

Amoebiasis (Amoebic Dysentery)

● Causal organism:- Entamoeba histolytica (a protozoan)

● Vector:- Housefly

● Symptoms:- Constipation, abdominal pain and cramps, stools with excess mucous and blood
clots, etc.
148 E
5. (a) Why is there a fear amongst the guardians that their adolescent wards may get trapped
in drug/alcohol abuse?
(b) Explain ‘addiction’ and ‘dependence’ in respect of drug / alcohol abuse in youth.
Ans. (a) Curiosity, need for adventure & excitement, experimentation and the perception among
adolescents that it is ‘cool’ or progressive to use drugs or alcohol constitute common causes,
which motivate adolescents to get trapped in drug / alcohol abuse.
(b) ‘Addiction’ is a psychological attachment to certain effects – such as euphoria and a
temporary feeling of well-being – associated with drugs / alcohol.
‘Dependence’ is the tendency of the body to manifest a characteristic and unpleasant withdrawal
syndrome if regular dose of drugs / alcohol is abruptly discontinued.
6. (a) It is generally observed that the children who had suffered from chicken - pox in their
childhood may not contract the same disease in their adulthood. Explain giving reasons
the basis of such an immunity in an individual. Name this kind of immunity.
(b) What are interferons? Mention their role.
Ans. (a) When our body encounters a pathogen for the first time produces a response called primary
response which is of low intensity. Subsequent encounter with the same pathogen elicits a
highly intensified secondary or anamnestic response.
This is ascribed to the fact that our body appears to have memory of the first encounter. Hence,
the children who had suffered from chicken pox in their childhood may not contract the same
disease in their adulthood.
This kind of immunity is called as acquired immunity.
(b) Interferons are cytokines secreted by virus-infected cells, and protect non-infected cells from
further viral infection.
7. How to detect cancer? What are a few approaches to treat cancer?
Ans. Cancer can be detected at early stages and early detection is essential. Following are a few can
diagnosis and detection areas:
Tests for increases cell counts (blood cancer)

Histo-pathological and biopsy of blood/tissues/bone marrow

 Radiography, CT, MRI to detect cancer of internal organs

 Identification of cancer-specific antigens

 Application of molecular biology techniques to detect genes with inherited susceptibility to a

few cancers
Treatment of cancer: Listed below are a few approaches that can be used to treat cancer.
Surgery to remove the tumour
Immunotherapy to boost the killing of cancer cells

Radiotherapy to kill cancerous cells


Chemotherapy

 Administration of biological response mediators such as ∝-interferons that activate the immune
system thus helping in destroying the tumour

E 149
(H) CASE-STUDY BASED QUESTIONS
1. Read the following and answer the questions given below:
The word AIDS stands for Acquired Immuno Deficiency Syndrome. This means deficiency of
immune system, acquired during the lifetime of an individual indicating that it is not a congenital
disease. ‘Syndrome’ means a group of symptoms. AIDS is caused by the Human Immuno
deficiency Virus (HIV), a member of a group of viruses called retrovirus, which have an
envelope enclosing the RNA genome Transmission of HIV-infection generally occurs by
(a) sexual contact with infected person,
(b) by transfusion of contaminated blood and blood products,
(c) by sharing infected needles as in the case of intravenous drug abusers and
(d) from infected mother to her child through placenta.
So, people who are at high risk of getting this infection includes - individuals who have multiple
sexual partners, drug addicts who take drugs intravenously, individuals who require repeated
blood transfusions and children born to an HIV infected mother.

(i) Expand AIDS?


Ans. AIDS  Acquired Immuno deficiency syndrome.
(ii) What is syndrome?
Ans. Group of symptoms are called as syndrome.
(iii) What is retrovirus?
Ans. AIDS is caused by the Immuno deficiency virus (HIV), member of group of viruses called
Retrovirus.
(iv) Mention the site of transmission of HIV infection?
Ans. (a) Sexual contanct with inferted person
(b) From Infected mother to her child through placenta.

2. Read the following and answer the questions given below:


The exaggerated response of the immune system to certain antigens present in the environment is
called allergy. The substances to which such an immune response is produced are called
allergens. The antibodies produced to these are of IgE type. Common examples of allergens are
mites in dust, pollens, animal dander, etc. Symptoms of allergic reactions include sneezing,
watery eyes, running nose and difficulty in breathing. Allergy is due to the release of chemicals
like histamine and serotonin from the mast cells. For determining the cause of allergy, the patient
is exposed to or injected with very small doses of possible allergens, and the reactions studied.
The use of drugs like anti-histamine, adrenalin and steroids quickly reduce the symptoms of
allergy. Somehow, modern-day life style has resulted in lowering of immunity and more
sensitivity to allergens – more and more children in metro cities of India suffer from allergies and
asthma due to sensitivity to the environment. This could be because of the protected environment
provided early in life.

150 E
(i) What is Allergy?
Ans. The exaggerated response of the immune system to certain antigens present in the environment is
called allergy.
(ii) Which antibodies primarily Participate in allergic response?
Ans. IgE
(iii) State the function of mast cells in allergy response?
(iii) The function of mast cell in allergy response is that it release histamine which cause
inflammatory reactions in the body.
(iv) Why do pollen grains of some flower triggers sneezing in some people?
Ans. Pollen grains are allergens that caused Allergy in some people due to release of chemicals like
histamine & serotonin from mast cells. These triggers the inflammatory response in body. e.g.,
Sneezing, running nose etc.

3. Read the following and answer the questions given below:


The principle of immunisation or vaccination is based on the property of ‘memory’ of the
immune system. In vaccination, a preparation of antigenic proteins of pathogen or
inactivated/weakened pathogen (vaccine) are introduced into the body. The antibodies produced
in the body against these antigens would neutralise the pathogenic agents during actual infection.
The vaccines also generate memory – B and T-cells that recognise the pathogen quickly on
subsequent exposure and overwhelm the invaders with a massive production of antibodies. If a
person is infected with some deadly microbes to which quick immune response is required as in
tetanus, we need to directly inject the preformed antibodies, or antitoxin (a preparation
containing antibodies to the toxin). Even in cases of snakebites, the injection which is given to
the patients, contain preformed antibodies against the snake venom. This type of immunisation is
called passive immunisation.

(i) What is a vaccine?


Ans. Vaccine is a preparation of antigenic proteins of pathogen or inactivated / weakened pathogen
introduced into the body.
(ii) What are Allergens?
Ans. Allergens are substances that cause allergy.
(iii) What is active immunity?
Ans. The immunity produced by an individual after undergoing primary immunity response.
(iv) Mention the site in the body where the B cells and T cells are formed.
Ans. Both the B cells and T cells are formed in the bone marrow.
(v) What is passive immunity?
Ans. The antibodies developed in other vertebrates in response to deliberate inoculation of antigen in
the body.
151
4. Read the following and answer the questions given below:

Health is not just the absence of disease. It is a state of complete physical, mental, social and

psychological well-being. Diseases like typhoid, cholera, pneumonia, fungal infections of skin,

malaria and many others are a major cause of distress to human beings. Vector-borne diseases

like malaria especially one caused by Plasmodium falciparum, if not treated, may prove fatal.

Besides personal cleanliness and hygiene, public health measures like proper disposal of waste,

decontamination of drinking water, control of vectors like mosquitoes and immunization are very

helpful in preventing these diseases. Our immune system plays the major role in preventing these

diseases when we are exposed to disease-causing agents.

(i) Which pathogenic bacterium causes typhoid fever in human beings?


Ans. Salmonella typhi is pathogenic bacterium that causes Typhoid fever.
(ii) Where is antigen binding site is found in an antibody?
Ans. Antigen binding site is found between one heavy and one light chain.
(iii) Why is tobacco smoking associated with rise in blood pressure?
Ans. Nicotine in tobacco stimulates adrenal glands to release adrenaline and nor-adrenaline in the
blood circulation raising blood pressure.
(iv) What is a disease?
Ans. Malfunctioning of one or more organs characterised by signs & symptoms is called disease.
(v) How does haemozoin affect the human body when released in blood during malarial
infection.
Ans. Haemozoin accounts for recurrence of high fever and chills in every 3-4 days.

152 E
CHAPTER 8
MICROBES IN HUMAN WELFARE

(A) NCERT QUESTIONS & SOLUTIONS


1. Bacteria cannot be seen with the naked eyes, but these can be seen with the help of a microscope.
If you have to carry a sample from your home to your biology laboratory to demonstrate the
presence of microbes with the help of a microscope, which sample would you Carry and why?
Ans. The most common household product that we would like to carry is curd which common in
Lactic acid bacteria (Lactobacillus sp.).
2. Give examples to prove that microbes release gases during metabolism.
Ans. Puffed-up appearance of dough which is used for making dosa ,idli and bread due to gas
production.
 Methanogens (bacteria) in the biogas plant produce methane and carbondioxide.
 Also large holes in the 'Swiss cheese' are due to production of a large amount of CO2 during
its production.
3. In which food would you find lactic acid bacteria? Mention some of their useful applications.
Ans. The lactic acid bacteria (LAB) are found in curd.
 LAB convert the lactose sugar of milk into lactic acid.
 Lactic acid coagulates the milk protein called casein.
 It also increases the nutritional quality of curd as the curd contains vitamin B 12 along with
other vitamins. They also check the growth of other harmful microbes.
4. Name some traditional Indian foods made of wheat, rice and Bengal gram (or their
products) which involve use of microbes.
Ans. 'Dosa' and 'idli' (from rice), bread (from wheat) and 'dhokla' (from Bengal gram) are the
traditional Indian foods which involve use of microbes.
5. In which way have microbes played a major role in controlling diseases caused by harmful
bacteria? [IMP.]
Ans. The major role of microbes in controlling the diseases is the 'antibiotic production, Antibiotics
have been used against pathogenic bacteria, e.g. penicillin from Penicillium notatum,
streptomycin from Streptomyces griseus, etc.
6. Name any two species of fungus, which are used in the production of the antibiotics.
Ans. (i) Penicillium notatum (for penicillin production).
(ii) Aspergilus fumigatus (for fumagillin production).

153
7. What is sewage? In which way can sewage be harmful to us?
Ans. Sewage is the municipal waste water containing large quantities of human excreta and other
organic wastes.
Sewage could be harmful to us as it contains many pathogenic microbes and produces foul smell.
It is the cause of many water-borne diseases.
It is also the cause of eutrophication of water bodies there by killing many aquatic organisms.
8. What is the key difference between primary and secondary sewage treatment? [IMP.]
Ans. The key difference between primary and secondary treatment of sewage is that primary treatment
is the physical process of removing grit and floating debris while secondary treatment is a
biological process that involves digestion of organic matter by microbes.
9. Do you think microbes can also be used as source of energy? If yes, how?
Ans. Yes, microbes can be used to produce energy indirectly.
- Methanogens (bacteria) like Methanobacterium are involved in the production of biogas which
is used as source of energy.
10. Microbes can be used to decrease the use of chemical fertilizers and pesticides. Explain how
this can be accomplished. [IMP.]
Ans. Microbes can be used both as fertilizers and pesticides called biofertilizers and biopesticides
respectively.
Microbes are used as biofertilisers to enrich the soil nutrients, eg Rhizobiun, Azotobacter,
Azospirillum, etc. which can fix atmospheric nitrogen in the soil.
Bacillus thuringiensis bacteria act as biopesticide to control the growth of insect pests.
Trichoderma , fungal species, is an effective biocontrol agent of several plant pathogens.
Baculoviruses used as control agents in genus Nucleopolyhedrovirus are excellent for
species-specific, narrow spectrum insecticidal applications.
11. Three water samples namely river water, untreated sewage water and secondary effluent
discharged from a sewage treatment plant were subjected to BOD test. The samples were
labelled A, B and C; but the laboratory attendant did not note which was which. The BOD
values of the three samples A, B and C were recorded as 20 mg/L, 8 mg/L and 400 mg/L,
respectively. Which sample of the water is most polluted? Can you assign the correct label
to each assuming the river water is relatively clean?
Ans. Sample C is most polluted (Highest BOD).
Sample A-River water
Sample B- Secondary effluent (Least BOD)
Sample C- Untreated sewage (Highest BOD)
12. Find out the name of the microbes from which cyclosporin A (an immuno suppressive
drug) and statins (blood cholesterol lowering agents) are obtained.
Ans. (a) Cyclosporin A is obtained from Trichoderma polysporum.
(b) Statins are obtained from the yeast Monascus purpureus.
13. Find out the role of microbes in the following and discuss it with your teacher.
(a) Single cell protein (SCP)
(b) Soil
Ans.(a) Single cell protein (SCP): It is a protein-rich microbial biomass which can be used as food.
Microbes are being grown on an industrial scale as source of good protein.
e.g., (i) Blue-green algae like Spirulina
(ii) Methylophilus methylotrophus bacteria
(iii) Mushrooms
(b) Soil: Soil is the habitat of numerous microbes. Microbes in the soil increase the fertility of soil by
decomposing organic matter. Some microbes convert nitrates into free nitrogen that escapes into
atmosphere for replenishment.
14. Arrange the following in the decreasing order (most important first) of their importance,
for the welfare of human society. Give reasons for your answer. [IMP.]
Ans. Penicillin, Biogas, Curd and Citric acid.
(1) Penicillin:- It is an antibiotic used in curing numerous bacterial diseases.
(2) Biogas :- It is a source of energy in rural areas, produced by anaerobic degradation of
organic matter
(3) Curd:- It is vitamin-rich milk preparation which is easily digested.
(4) Citric acid:- It is an organic acid used as preservative in juices, jams and jellies, etc .
15. How do biofertilizers enrich the fertility of the soil? [IMP.]
Ans. Biofertilizers are microorganisms which bring about nutrient enrichment of soil by enhancing the
availability of nutrients to crops. They are of following types:
(a) Nitrogen fixing bacteria and cyanobacteria- They form symbiotic association with plants.
They get food and shelter from plants and on the other hand, plants get nitrogen fixed by
these bacteria. For example Rhizobium and Anabena.
(b) Mycorrhiza: It is an association between a fungus and roots of higher plants. It takes part in
the solubilisation and absorption of nutrients from organic matter. Many members of the
genus Glomus form mycorrhiza.
(c) Manures: They are semi-decayed organic remains of various types-manure, green manure
compost and vermicompost.
E 155
(B) PREVIOUS YEAR QUESTIONS
1. Given below are the list of the commercially important products and their source organisms.
Select the option that gives the correct matches. [CBSE 2023]
List-A List-B
S. Bioactive S. Microbes
No. Products No. (Source Organism)
(A) Cyclosporin A (i) Streptococcus
(B) Statins (ii) Tricoderma polysporum
(C) Streptokinase (iii) Penicillium notatum
(D) Penicillin (iv) Monascus Purpureus
Options :
(a) (A)-(i) (B)-(ii), (C)-(iii), (D)-(iv) (b) (A)-(iii) (B)-(iv), (C)-(ii), (D)-(i)
(c) (A)-(iv) (B)-(iii), (C)-(ii), (D)-(i) (d) (A)-(ii) (B)-(iv), (C)-(i), (D)-(iii)
Ans. (d) (A)-(ii) (B)-(iv), (C)-(i), (D)-(iii)
2. Certain specific bacterial spores are mixed in water and sprayed over Brassica crop to
control butterfly catterpillars. [CBSE 2023]
Name this bacterium and its mode of action on the butterfly catterpillars.
Ans. Bacillus thuringiensis (Bt). These are available in sachets as dried spores which are mixed with
water and sprayed onto vulnerable plants such as Brassica and fruit trees, where these are eaten
by the insect larvae. In the gut of the larvae, the toxin is released and the larvae get killed. The
bacterial disease will kill the caterpillars, but leave other insects unharmed.
3. (a) (i) Give an example of a genus of fungi that forms mycorrhizial association with plants.
(ii) How does the plant derive benefits from this association? [CBSE 2023]
Ans. (a) (i) Nucleopolyhedrovirus
(ii) Baculoviruses are the viruses belong the genus. These viruses attack the arthropods/ insects.
These viruses are excellent species specific effective narrow spectrum insecticidal biocontrol
agent. This is especially important when treating an environmentally sensitive area or
conserving beneficial insects to support an overall integrated pest management (IPM)
programme.
4. On spraying Bacillus thuringiensis on an infected cotton crop field the pests are killed by
the toxin, however the toxin although produced by the bacteria does not affect it. Explain
giving reason? [CBSE 2023]
Ans. Because the Bt toxin protein exist as inactive protoxins but once an insect ingest the inactive
toxin, it is converted into an active form of toxin due to the alkaline pH of the gut which
solubilise the crystals.
5. Farmers are often suggested to use the following organisms in their crop land so as to
improve the soil fertility. Explain. [CBSE Term – II 2022]
(i) Rhizobium
(ii) Anabaena
Ans. (i) Rhizobium is a bacterium found in soil that helps in fixing nitrogen in leguminous plants. It
attaches to the roots of the leguminous plant and produces nodules. These nodules fix
atmospheric nitrogen and convert it into ammonia that can be used by the plant for its growth
and development.
(ii) Anabaena plays a significant role in farming where it is used as a biofertilizer and soil
stabilizer.
6. Organic farmer use Trichoderma and Baculovirus as biological control agents. Explain.
[CBSE Term – II 2022]
Ans. Trichoderma species are free-living fungi that are very common in the root ecosystems. They are
effective biocontrol agents of several plant pathogens.
Baculoviruses are pathogens that attack insects and other arthropods. The majority of
baculoviruses used as biological control agents are in the genus Nucleopolyhedrovirus. These
viruses are excellent candidates for species-specific, narrow spectrum insecticidal applications.
They have been shown to have no negative impacts on plants, mammals, birds, fish or even on
non-target insects.
7. Explain four advantages of mycorrhizal association to plants. [CBSE IMP Question]
Ans. The fungal symbiontin in mycorrhizal associations with plants:
i. absorbs phosphorus from soil and passes it to the plant.
ii. provides resistance to root-borne pathogens,
iii. enhances tolerance to salinity and drought,
iv. induces an overall increase in plant growth and development.
8. A farmer noticed that nematode infection in tobacco plants has resulted in the reduction in
the yield. Suggest a strategy which provides cellular defense for providing resistance to this
pest. Explain the technique. [CBSE IMP Question]
Ans.  Strategy based on the process of RNA interference (RNAi) - as a method of cellular defence
can be used.
 This method involves silencing of a specific mRNA due to a complementary dsRNA
molecule that binds to and prevents translation of the mRNA (silencing).
 The source of this complementary RNA can be from an infection by viruses having RNA
genomes or mobile genetic elements (transposons) that replicate via an RNA intermediate.
 Using Agrobacterium vectors, nematode-specific genes are introduced into the host plant.
The introduction of DNA produces both sense and anti-sense RNA in the host cells.
 Two RNA’s being complementary to each other form a double stranded (dsRNA) that
initiate RNAi and thus, silence the specific mRNA of the nematode.
 As a consequence, the parasite cannot survive in a transgenic host expressing specific
interfering RNA. The transgenic plant therefore gets protected from the parasite.
9. Name the genus of baculovirus that acts as a biological control agent in spite of being a
pathogen. Justify by giving three reasons that make it an excellent candidate for the Job.
[CBSE 2020]
Ans. The majority of baculoviruses used as biological control agents are in the genus
Nucleopolyhedrovirus.
 These viruses are excellent candidates for species-specific, narrow spectrum insecticidal
application.
 They have been shown to have no negative impacts on plants, mammals, birds, fish or even
on non-target insects

E 157
10. What are ‘flocs’, formed during secondary treatment of sewage? [CBSE 2019]
Ans. Masses of bacteria associated with fungal filament (to form mesh like structure).
11. Write any two places where methanogens can be found. [CBSE 2019]
Ans. They may found in anaerobic sludge (digester), rumen of cattle (ruminants), stomach of cattle
(gut of cattle), marshy area, flooded rice fields, biogas plant.
12. Briefly describe the process of secondary treatment given to municipal waste water (sewage)
before it can be released into fresh water bodies. Mention another benefit provided by this
process. [CBSE 2019]
Ans. Process of secondary treatment :
1) Passing of primary effluent into large aeration tank which is constantly agitated
mechanically & air is pumped into it that allows vigorous growth of useful aerobic microbes
into flocs.
2) Microbes consume major part of organic matter in effluent which significantly reduces BOD
13. Baculoviruses are good example of biocontrol agents. Justify giving three reasons.
[CBSE 2018]
Ans. Baculoviruses are pathogens that attack insects and other arthropods.
 Most of these bio control agents belongs to the genus Nucleopolyhedrovirus.
 These are species-specific, narrow spectrum insecticides.
 They do not harm plants, mammals, birds, fish and other non-target insects.
 Baculoviruses are helpful in integrated pest management(IPM) programme, in which
beneficial insects are conserved and there is no negative impact on plant mammals, birds,
fish or non target insects.
14. (a) Organic farmers prefer biological control of diseases and pests to the use of chemicals
for the same purpose. Justify.
(b) Give an example of a bacterium, a fungus and an insect that are used as biocontrol
agents. [CBSE 2018]
Ans. (a)
 Reduces dependence on toxic chemicals.
 Protects our ecosystem or environment.
 Protects and conserves non-target organisms / they are species – specific.
 These chemicals being non-biodegradable may pollute the environment permanently.
 These chemicals being non-biodegradable may cause biomagnifications
(b) Bacteria – Bacillus thuringiensis
Fungus – Trichoderma
Insect – Ladybird / Dragonfly / Moth or any other correct example.
15. Secondary treatment of the sewage is also called Biological treatment. Justify this statement
and explain the process. [CBSE 2017]
Ans. Involves biological organism such as aerobic and anaerobic microbes / bacteria and fungi to
digest organic waste.
158 E
Primary effluent is passed into aeration tank where vigorous growth of aerobic microbes
(flocs) take place,
BOD reduced (microbes consume major part of organic matter),
Effluent is passed to settling tank where flocs sediment to produce activated sludge,
Sludge is pumped to anaerobic sludge digester to digest bacteria and fungi.
16. How does the application of the fungal genus, Glomus, to the agricultural farm increase the
farm output? [CBSE 2017]
Ans. Glomus forms mycorrhizal association, absorbs phosphorus, provide resistance to root borne
pathogens, enhanced to tolerate salinity or drought
17. Describe how do ‘flocs’ and ‘activated sludge’ help in Sewage Treatment. [CBSE 2017]
Ans. Flocs - Aerobic microbes consume the major part of the organic matter in the effluent,
significantly reduces BOD
Activated sludge - Small part of activated sludge is used as inoculum and pumped back to
aeration tank and pumped into anaerobic sludge digesters where microbes or bacteria grow
anaerobically to produce CH4 or H2S or CO2 or biogas.
18. List the events that reduce the Biological Oxygen Demand (BOD) of a primary effluent
during sewage treatment. [CBSE 2017]
Ans. (i) Effluent from the primary settling tank passed into aeration tank.
(ii) It is agitated mechanically and air is pumped into it.
(iii) There is vigorous growth of aerobic microbes into flocs.
(iv) The microbes consume major part of the organic matter in effluent.

159
(C) MULTIPLE CHOICE QUESTIONS
1. Which gas is responsible for the puffed-up appearance of dough?
(1) CO2 (2) O2 (3) SO2 (4) NO2
Ans. (1) CO2
2. Which bacterium helps in the production of ‘Swiss cheese’?
(1) Propionibacterium sharmanii (2) Trichoderma polysporum
(3) Saccharomyces cerevisiae (4) Aspergillus niger
Ans. (1) Propionibacterium sharmanii
3. Microbes are present in
I. soil II. air III. water IV. thermal springs
(1) I, III, IV (2) I, II, III, IV (3) I, II (4) III, IV
Ans. (2) I, II, III, IV
4. Saccharomyces cerevisiae is employed in production of
(1) idli (2) beer (3) bread (4) All of the above
Ans. (4) All of the above
5. Lactic acid bacteria convert milk into curd and improve its nutritional quality by enhancing
(1) vitamin A (2) vitamin B (3) vitamin C (4) vitamin D
Ans. (2) vitamin B
6. Cheese and yogurt are products of
(1) pasteurisation (2) fermentation (3) dehydration (4) distillation
Ans. (2) fermentation
7. Microbes are diverse group which include
I. bacteria II. mosses III. protozoans IV. fungi
(1) I, III, IV (2) I, IV (3) I, II (4) III, IV
Ans. (1) I, III, IV
8. Lactobacillus mediated change of milk to curd occurs due to
(1) coagulation and partial digestion of milk fats.
(2) coagulation and partial digestion of milk proteins.
(3) coagulation of milk proteins and complete digestion of milk fats.
(4) coagulation of milk fats and complete digestion of proteins.
Ans. (4) coagulation of milk fats and complete digestion of proteins.
9. Baker’s yeast is
(1) Saccharomyces cerevisiae (2) S.ludiwigii
(3) S.octosporus (4) Schizosaccharomyces
Ans. (1) Saccharomyces cerevisiae
10. Match the following list of microbes and their importance:
(1) Saccharomyces cerevisiae (i) Production of immunosuppressive agents
(2) Monascuspurpureus (ii) Ripening of Swiss cheese
(3) Trichoderma polysporum (iii) Commercial production of ethanol
(4) Propionibacterium sharmanii (iv) Production of blood cholesterol lowering Agents
(1) (2) (3) (4)
(1) (iv) (iii) (ii) (i)
(2) (iv) (ii) (i) (iii)
(3) (iii) (i) (iv) (ii)
(4) (iii) (iv) (i) (ii)
Ans (4) (iii) (iv) (i) (ii)
11. Streptokinase which is used as a ‘clot buster’ obtained from
(1) Streptococcus (2) Staphylococcus (3) Lactobacillus (4) Saccharomyces
Ans. (1) Streptococcus
12. Statins, a bioactive molecule, inhibiting the enzyme responsible for synthesis of
(1) carbohydrate (2) protein (3) vitamins (4) cholesterol
Ans. (4) cholesterol
13. Antibiotics are used to treat diseases like
(1) plague (2) whooping cough, diptheria
(3) leprosy (4) All of these
Ans. (4) All of these
14. Which one of the following is not true about antibiotics?
(1) First antibiotic was discovered by Alexander Flemming.
(2) The term ‘antibiotic’ was coined by S. Waksman in1942.
(3) Some persons can be allergic to a particular antibiotic.
(4) Each antibiotic is effective only against one particular kind of germ.
Ans. (3) Some persons can be allergic to a particular antibiotic.
15. Which one of the following is used in the manufacture of alcohol?
(1) Bacteria (2) Water molds (3) Yeasts (4) Slime molds
Ans. (3) Yeasts
16. Which industrial products are synthesized from microbes?
I. Antibiotics II. Fermented beverages
III. Enzymes and chemicals IV. Bioactive molecules
(1) I, III, IV (2) I, II, III, IV (3) I, III (4) I, II, III
Ans. (2) I, II, III, IV
17. Which one of the following sets includes bacterial diseases?
(1) Tetanus, tuberculosis, measles (2) Diphtheria, leprosy, plague
(3) Cholera, typhoid, mumps (4) Malaria, mumps, poliomyelitis
Ans. (2) Diphtheria, leprosy, plague
18. The large vessels for growing microbes on an industrial scale are called
(1) petridish (2) stores (3) biogas vessel (4) fermenters
Ans. (4) fermenters
19. Antibiotics are drugs commonly used to cure diseases of
(1) fungi (2) viruses (3) protozoans (4) bacteria
Ans. (4) bacteria
20. The first antibiotic to be isolated was
(1) terramycin (2) streptomycin (3) neomycin (4) penicillin
Ans. (4) penicillin

E 161
(D) ASSERTION & REASON QUESTIONS

 Directions: In the following questions, a statement of assertion is followed by a statement of


reason. Mark the correct choice as:
(1) If both Assertion and Reason are true and Reason is the correct explanation of Assertion.
(2) If both Assertion and Reason are true but Reason is not the correct explanation of Assertion.
(3) If Assertion is true but Reason is false.
(4) If both Assertion and Reason are false.

1. Assertion: Streptococcus thermophilus increases nutritional value of milk.


Reason: Milk has lesser vitamin content than curd and yoghurt.
Ans. (1)
2. Assertion: After 24 hours, toddy becomes unpalatable.
Reason: The fermentation of toddy is continued by naturally occurring yeast.
Ans. (1)
3. Assertion: Beer & wine are called soft liquors white gin, rum etc. are hard liquors.
Reason: Beer & wine are made without distillation.
Ans. (2)
4. Assertion: Yeasts such as Saccharomyces cerevisiae are used in baking industry.
Reason: Carbon dioxide produced during fermentation causes bread dough to rise by thermal
expansion.
Ans. (1)
5. Assertion: An organ transplant patient if not provided with cyclosporin A may reject the
transplanted organ.
Reason: Cyclosporin A inhibits activation of T-cells and interferes with destruction of non-self
cells.
Ans. (1)
6. Assertion: The kneaded flour shows leavening, when yeast is added to it.
Reason: Enzymes secreted by yeast cause leavening.
Ans. (1)
7. Assertion : Saccharomyces species are utilized for making alcoholic beverages.
Reason :Yeast has an enzyme, zymase responsible for fermentations.
Ans. (1)
8. Assertion: Secondary treatment of sewage is also called biological treatment while primary
treatment is called physical treatment.
Reason: Primary sewage treatment depends only upon sedimentation properties of materials
present in sewage and filteration.
Ans. (1)
9. Assertion: Energy value of biogas is lower than that of organic matter.
Reason: Biogas minimises the chances of spread of fecal pathogens
Ans. (2)
10. Assertion: Dragonflies can be used to decrease occurrence of diseases like malaria, dengue, etc.
Reason: Baculoviruses are effective in controlling many insects and other arthropods.
Ans. (2)

162 E
(E) VERY SHORT ANSWER TYPE QUESTIONS
1. How is lactic acid bacteria beneficial to us other than helping in curdling the milk ?
Ans. Lactic acid bacteria improves the nutritional quality by increasing Vitamin B12.

2. Give the scientific name of the source organism from which the first antibiotic was

produced.

Ans. Penicillium notatum.

3. Name the gas released and the process responsible for puffing up of the bread dough when

Saccharomyces cerevisiae is added to it.

Ans. Gas – Carbon dioxide

Process – Fermentation

4. Name the metabolic pathway associated with the rising of dough in making bread. What

makes the dough rise ?

Ans. Metabolic pathway : Alcoholic fermentation by yeast. CO2 produced in this process is

responsible for rising of dough.

5. Write the scientific name of the microbe used for fermenting malted cereals and fruit

juices.

Ans. Saccharomyces cerevisiae.

6. Which of the following in the baker’s yeast is used in fermentation ?

Saccharam barberi, Saccharomyces cerevisiae.

Ans. Saccharomyces cerevisiae.

7. Name the group of organisms and the substrate that act to produce biogas.

Ans. Name of the group of organisms-Methanogens

Substrate-Cellulosic material, cow dung and agricultural waste.

8. Bottled fruit juices are clearer as compared to those made at home. Explain.

Ans. Bottled fruit juices are clearer as compared to those prepared at home because they are treated

with enzyme pectinase and protease. This enzymes acts on juices and make them clearer.

9. Name the first antibiotic discovered and by whom ?

Ans. Penicillin, Alexander Fleming.

163
10. (i) A patient who had an organ transplant was given cyclosporin –

A Mention the microbial source and state the reason for administration of this
bioactive molecule.

(ii) Bottled fruit juices bought from the market are clearer as compared to those made at
home. Give reason.

Ans. (i) Source – Trichoderma polysporum.

Reason – Immuno suppressive agent.

(ii) They are clarified by pectinases and proteases.

11. Name a bioactive molecule, its source organism and the purpose for which it is given to
organ transplant patients.

Ans. Cyclosporin A.

Source – Trichoderma polysporum.

Purpose – Immuno suppressive agent.

12. Make a list of three household products along with the names of the micro-organism
producing them.

Ans. Lactic acid bacteria - curd

Saccharomyces cerevisiae - bread

Propionibacterium shermanii - swiss cheese

13. Name the types of association that the genus Glomus exhibits with higher plants.

Ans. Symbiosis/Mycorrhiza/Mutualism.

14. Mention two advantages of adding blue-green algae to paddy fields.

Ans. In paddy fields, cyanobacteria (blue-green algae) add organic matter to the soil and increases its
fertility.

15. Mention the information that the health workers derive by measuring BOD of a water body?

Ans. By measuring BOD of a water bodies, health workers find the amount of dissolved O2. The lesser
amount of dissolved O2, the more polluted the water body will be.

164 E
(F) SHORT ANSWER TYPE QUESTIONS
1. Why is Rhizobium categorized as a ‘symbiotic bacterium’ ? How does it act as a biofertiliser?
Ans. Rhizobium is a symbiotic bacterium living in the root nodule of leguminous plants, it fixes
atmospheric nitrogen into organic forms to be used by plants as nutrient and in turn bacteria get
carbohydrate food and shelter from the plant. It is a biofertiliser as it is a living organism that
enriches nutrient content of the plant / soil.
2. How do mycorrhizae help the plants to grow better?
Ans. Mycorrhiza refers to symbiotic association between the fungus and the root of higher plants.
These fungi in these associations absorb water, phosphorus, nitrogen, potassium, calcium from
soil and pass it to the plant. The fungus brings about the solubilization of organic matter of soil
humus, release of inorganic nutrients absorption and their transfer to roots. The genus Glomus
forms a mycorrhizal association with plants.
3. Your advice is sought to improve the nitrogen content of the soil to be used for cultivation
of a non-leguminous terrestrial crop.
(a) Recommend two microbes that can enrich the soil with nitrogen.
(b) Why do leguminous crops not require such enrichment of the soil?
Ans. (a) Azospirillum/ Azotobacter/ Anabaena/ Nostoc / Oscillatoria /Frankia (Any two correct
names of microbes).
(b) They can fix atmospheric nitrogen, due to presence of Rhizobium/N2 fixing bacteria in their
root nodules.
4. (i) How do organic farmers control pests ? Give two examples.
(ii) State the difference in their approach from that of conventional pest control methods.
Ans. (i) Natural predation / biological control.
Ex. Lady bird used to kill aphids / dragon flies used to kill mosquitoes / Bacillus thuringiensis used
to kill cotton bollworm / caterpillar / butterfly caterpillar.
S. Conventional Pest Organic farming based
No. Control pest control
(a) Use of chemical insecticides No chemical used.
and pesticides.
(b) Harmful to non target Not harmful to non target
organisms. organisms.
(c) Cause environment No adverse impact on
environment.
5. Choose any three microbes, from the following which are suited for organic farming which
is in great demand these days for various reasons.
Mention one application of each one chosen.
Mycorrhiza, Monascus, Anabaena, Rhizobium, Methanobacterium, Trichoderma.
Ans. Mycorrhiza : Fungal symbiont of the association, absorb phosphorus from soil.
Anabaena : Fix atmospheric nitrogen / Adds organic matter to the soil.
E 165
Rhizobium : It is a symbiotic root nodule bacterium which fixes atmospheric nitrogen (in
leguminous plants) in organic form which is used by plant as nutrient.
Methanobacterium : They digest cellulosic material and the product / spent slurry can be used as
fertilizer.
Trichoderma : Biocontrol agent for several plant pathogens.
6. What are biopesticides ? Give the scientific name and use of the first commercially used
biopesticide in the world.
Ans. The biological agent which are used to control weeds, insects and pathogen are called
biopesticides. The micro-organisms that are used as biopesticides are some viruses, bacteria,
fungi and their products. The bacterium Bacillus thuringiensis was the first biopesticide to be
used on commercial scale.
7. Organic farmers prefer biological control of diseases and pests to the use of chemicals for
the same purpose Justify.
Ans. (i) Reduces dependence on toxic chemicals.
(ii) Protects our ecosystem or environment.
(iii) Protects and conserves non-target organisms / they are species – specific.
(iv) These chemicals being non-biodegradable may pollute the environment permanently.
(v) These chemicals being non-biodegradable may cause bio magnification
8. Give an example of a bacterium, a fungus and an insect that are used as biocontrol agents.
Ans. Bacteria: Bacillus thuringiensis
Fungus: Trichoderma
Insect: Ladybird / Dragonfly / Moth or any other correct example.
9. What are biopesticides ? Give the scientific name and use of the first commercially used
biopesticide in the world.
Ans. The biological agent which are used to control weeds, insects and pathogen are called biopesticides.
The micro-organisms that are used as biopesticides are some viruses, bacteria, fungi and their
products. The bacterium Bacillus thuringiensis was the first biopesticide to be used on commercial
scale.
10. Name any one symbiont which serves as a biofertilizer. Mention its specific role.
Ans. Rhizobium, a root nodule bacterium of legumes is a symbiont. It fixes nitrogen symbiotically and
thus help the plants in obtaining their nitrogen nutrition and therefore serve as a biofertilizer.

166 E
11. Which of the following is a cyanobacterium that can fix atmospheric nitrogen. Azospirillum,
Oscillatoria, Spirulina.
Ans. Oscillatoria
12. In the given diagram the gas X can be:

Gas

Gas-holder
Dung Water (CH4 + CO2 + .…) Sludge

Digester

Ans 1. Hydrogen sulfide 2. Carbon monoxide 3. Ammonia 4. Oxygen


13. Explain the different steps involved during primary treatment phase of sewage.
Ans. Physical removal of particles (large and small), by filtration and sedimentation, forming primary
sludge / sedimented solids, forming effluent (supernatant) for secondary treatment.
14. Name the bacterium responsible for the large holes seen in ‘Swiss cheese ‘. What are these
holes due to ?
Ans. The large holes in ‘Swiss cheese’ are due to the production of a large amount of CO2 by a
bacterium named Propionibacterium shermanii.
15. Name the microbes that help the production of the following products commercially :
(i) Statin (ii) Citric acid (iii) Penicillin (iv) Butyric acid
Ans. (i) Monascus purpureus (ii) Aspergillus niger
(iii) Penicillium notatum (iv) Clostridium butyricum

E 167
(G) LONG ANSWER TYPE QUESTIONS
1. Describe the process of waste-water treatment under the following heads:
(i) Primary treatment. (ii) Secondary treatment.
Explain the process of sewage water treatment before it can be discharged into natural
water bodies. Why is this treatment essential?
Ans. (i) Primary treatment
(a) Physical removal of particles like debris, soil, sand or silt through filtration, sedimentation in
stages.
(b) Solids settle to form primary sludge, the supernatants form the primary effluent.
(ii) Secondary Treatment: It is the biological treatment.
(a) Effluent passed into aeration tanks.
(b) Vigorous growth of useful aerobic microbes into flocs.
(c) Significant reduction of BOD due to use of organic matter by microorganisms.
(d) After fall in the level of BOD, the effluent is passed on to settling tanks where bacterial flocs
settle to form activated sludge.
(e) Activated sludge is passed on to anaerobic sludge digester, where bacteria and fungi are
anaerobically digested.
2. (i) How does Bacillus thuringiensis act as a biocontrol agent for protecting Brassica and
fruit trees? Explain.
(ii) (a) List the components of biogas.
(b) What makes methanogens a suitable source for biogas production?
Ans. (i) Bacterium Bacillus thuringiensis (Bt) are available in sachets as dried spores, mixed with
water and sprayed onto vulnerable plants, these are eaten up by the insect larvae, the toxins are
released in the gut and larva gets killed.
(ii) (a) Methane, H2S, CO2, H2.
(b) Methanogens grow anaerobically on cellulosic material, produce large amount of methane,
along with CO2 & H2.
3. The following flow diagram is the process of formation of curd from milk. Observe it and give
the answer of question that follow.
Milk is incubated with curd

Bacteria(A) grow in milk

Production of (B)

Coagulation and digestion of milk protein

Improved nutritional quality (C)


168 E
(i) Write the name of bacteria A which grow in milk.
(ii) What is the product (B)?
(iii) Write the name which nutritional quality become improved.
Ans. (i) The bacteria is LAB-Lactic acid bacteria
(ii) The product B is lactic acid
(iii) it is vitamin B12
4. Why should biological control of pests and pathogens be preferred to the conventional use
of chemical pesticides? Explain how the following microbes act as biocontrol agents:
(a) Bacillus thuringiensis
(b) Nucleopolyhedrovirus
Ans. Biological control of pests and pathogens is preferred because:
(i) The chemicals cause pollution of water bodies as well as ground water, besides getting stored
in the plants.
(ii) The chemicals are toxic thus extremely harmful to human beings and other animals.
(a) Bacillus thuringiensis : Bacillus thuringiensis is available in sachets as dried spores,
which are mixed with water and sprayed onto vulnerable plants. When they are eaten by
the insect larvae, the toxin is released in the gut where it become active and kills the
laevae.
The Bt toxins gene when introduced unto plant develop resistance to attack by insect
pests. Specific Bt toxin genes obtained from Bacillus thuringiensis are used in several
crop plants which make them resistant to insect pest.
(b) Nucleopolyhedrovirus: These viruses are excellent candidates for species specific narrow
spectrum insecticidal application. This is especially desirable when beneficial insects are
being conserved to aid in on overall integrated pest management programme.

E 169
(H) CASE BASED TYPE QUESTIONS
1. Read the following and answer the questions given below.
Villager in a place near Chambur started planning to make power supply for agricultural
purposes from cow dung.They have started a biogas plant for the purpose. Study the flow chart
for biogas production given below and answer the following questions.
Biogas
A
Fermentative Microbes
B
Suitable compounds
C

Protein Fat Cellulose Hemicellulose

(i) Analia developed the technology of producing biogas from coul dung largely due to the
efforts of -
Ans. (i) Methanogenic bacteria.
(ii) What is represent by B in the flow chart?
Ans. Organic acid
(iii) Which one produce gas by decomposing the gobar in gober gas.
Ans. Methanogenic bacteria
(iv) Where are methanogenic bacteria not found.
Ans. Activated sludge
2. Read the following and answer the questions given below.
Microorganisms include bacteria, viruses, fungi and protozoa. In our mind, we presume, most of
the time, that microbes are always harmful. Microbes are, of course, the causal agents of many
infections diseases of plants and animals including humans but they also have lots of beneficial
role. Lactic acid bacteria (LAB) are one of this kind of useful group. These are Gram positive,
non-sporulating, either rod-shaped or spherical bacteria. They produce lactic acid in milk
products as major metabolic end product of carbohydrate fermentation. LAB are considered as
natural fermentors. Lactobacillus is a common LAB which converts lactose sugar of milk into
lactic acid, that causes coagulation and partial digestion of milk protein casein. Milk is then
changed into curd, yoghurt and cheese. Lactobacillus is also used in probiotics which have
potentially beneficial effect on gut ecosystem of humans. Some other probiotic strain used belong
to the Genus Bifidobacterium.

170 E
(i) How is lactic acid bacteria beneficial to us other than helping in curdling the milk.

Ans. Lactic acid bacteria improves the nutritional quality by increasing vitamin B12.

(ii) What are probiotics ?

Ans. Probiotics are gut friendly live bacteria.

(iii) Name the microbe used for making bread.

Ans. Saccharomyces cerevisiae (Baker's yeast) is used in making bread.

(iv) How is LAB (Lactic Acid Bacillus) helpful for our stomach.

Ans. LAB play very beneficial role in checking disease-causing microbes in our stomach.

(v) How LAB helpful for conversion of milk sugar into Lactic acid.

Ans. They converts lactose sugar of milk into Lactic acid.

3. Read the following and answer the questions given below.

Microbes are also used for commercial and industrial production of certain chemicals like
organic acids, alcohols and enzymes. Microbes are also used for the production of enzymes.
Lipases are used in detergent formulations and are helpful in removing oily stains from the
laundry. Bottled fruit juices bought from the market are clearer as compared to those made at
home. Streptokinase produced by the bacterium Streptococcus and modified by genetic
engineering, another bioactive molecule, cyclosporin A. Statins produced by the yeast has been
commercialised used.

(i) Name the microbe used commercially in the production of blood cholesterol lowering
statins.

Ans. Monascus purpureus

(ii) Name the microbe commonly used as a insecticide

Ans. Bacillus thuringiensis

(iii) Name the microbe used commercially in production of citric acid.

Ans. Aspergillus Niger

(iv) Name the immune suppressive agent used in organ-transplant patients.

Ans. Cyclosporin A

(v) Name the organism which produced streptokinase.

Ans. Streptococcus

E 171
IMPORTANT NOTE

______________________________________________________________________

_______________________________________________________________________
_______________________________________________________________________
_______________________________________________________________________
_______________________________________________________________________
_______________________________________________________________________
_______________________________________________________________________
_______________________________________________________________________
_______________________________________________________________________
_______________________________________________________________________
_______________________________________________________________________
_______________________________________________________________________
_______________________________________________________________________
_______________________________________________________________________
_______________________________________________________________________
_______________________________________________________________________
_______________________________________________________________________
_______________________________________________________________________
_______________________________________________________________________
_______________________________________________________________________
_______________________________________________________________________
_______________________________________________________________________
_______________________________________________________________________

_______________________________________________________________________

_______________________________________________________________________
_______________________________________________________________________
172 E
CHAPTER 9
(A) NCERT QUESTIONS & SOLUTIONS
1. Can you list 10 recombinant proteins which are used in medical practice? Find out where
they are used as therapeutics (use the internet).
Ans.
S.No. Recombinant Proteins Therapeutic uses
1. Human Insulin (Humulin) Treatment of diabetes type-1
2. Human growth hormone Replacement of missing hormone in
short stature people.
3. Platelet Growth factor Stimulation of wound healing
4. Calcitonin Treatment of rickets
5. Blood clotting factor Replacement of clotting factor
VIII/IX missing in patients with
Haemophilla A/B.
6. Hirudin Used as an anticoagulant
7. Interferon Treatment of viral infection and
cancer
8. Chorionic Gonadotropin Treatment of infertility
9. Interleukins Enhancing activity of immune
system
10. Tissue Plasminogen Treatment for acute myocardial
Activator infarction, dissolves blood clot after
heat attack and stroke.
2. Make a chart (with diagrammatic representation) showing a restriction enzyme, the
substrate the DNA on which it acts, the site at which it cuts DNA and the product it
produces. [IMP.]
Ans.
EcoRI cuts the DNA between bases
The enzyme cuts both DNA strands G and A only when the sequence
at the same site GAATTC is present in the DNA

Vector DNA Foreign DNA

EcoRI
Sticky end

Sticky end
DNA fragments join at sticky ends

Recombinant DNA

Step in formation of recombinant DNA by action of


restriction endonuclease enzyme - EcoRI

E 173
3. From what you have learnt, can you tell whether enzymes are bigger or DNA is bigger in
molecular size? How did you know? [IMP.]
Ans. DNA is bigger in molecular size. DNA is made up of sugar, phosphate and nitrogenous bases. An
enzyme is made up of only one or few polypeptides.
Enzyme is synthesized from a portion of DNA.
4. What would be the molar concentration of human DNA in a human cell? Consult your
teacher.
Ans. The molar concentration of human DNA in a human diploid cell as follow:
 Total number of chromosomes × 6.023 × 1023
 46 × 6.023 × 1023

 277.06 × 1023 Moles.


Hence, the molar concentration of DNA in each diploid cell in human is 277.06 × 1023 moles.
5. Do eukaryotic cells have restriction endonucleases? Justify your answer. [IMP.]
Ans. Eukaryotic cells have no restriction enzymes as the DNA molecules of eukaryotes are heavily
methylated.
It is present in prokaryotic cell (like bacteria) where these act as defense mechanism to restrict
the growth of bacteriophages.
6. Besides better aeration and mixing properties, what other advantages do stirred-tank
bioreactors have over shake flasks?
Ans. Shake flask is used for a small–scale production but the stirred-tank bioreactors are used for large
scale production of biotechnological products.
Advantages of stirred - tank bioreactors over shake flasks are that these facilitate –
 Temperature control system,
 pH control system,
 Foam control system and
 Sampling ports from where small, volume of the cultures can be obtained and tested time to time.
7. Collect 5 examples of palindromic DNA sequences by consulting your teacher. better try to
create a palindromic sequence by following base - pair rules.
Ans. (i) 5' GAATTC 3' (ii) 5' GGATCC 3'
3' CTTAAG 5' 3' CCTAGG 5‘
(iii) 5' ACTAGT 3' (iv) 5' AAGCTT 3'
3' TGATCA 5' 3' TTCGAA 5'
(v) 5' AGGCCT 3'
3' TCCGGA 5'
8. Can you recall meiosis and indicate at what stage a recombinant DNA is made?
Ans. Meiosis is the cell division process of gamete formation. It occurs in two steps –
Meiosis-I and Meiosis-II
During pachytene stage of prophase–I of meiosis-I, crossing over takes place between non-sister
chromatids of homologous chromosomes and a recombinant DNA is made.
9. Can you think and answer how a reporter enzyme can be used to monitor transformation
of host cells by foreign DNA in addition to a selectable marker? [IMP.]
Ans. A reporter gene is used to monitor the transformation of host cells by foreign DNA. They act as a
selectable marker to determine whether the host cell has taken up the foreign DNA or the foreign
gene gets expressed in the cell.
Here, the reporter gene is used as a selectable marker to find out the successful uptake of gene of
interest.
An example of reporter gene includes lac Z gene which encodes β- galactosidase enzyme.
10. Describe briefly the following:
(a) Origin of replication (b) Bioreactors (c) Downstream processing
Ans. (a) Origin of replication:- It is a DNA sequence that initiates any piece of linked DNA to
replicate and is also called ori site. It controls the copy numbers of the linked DNA.
(b) Bioreactors:-Bioreactors are vessels of large volumes (100-1000 liters) in which raw
materials are biologically converted into specific products.
- It provides the optimal conditions for achieving the desired product by providing optimal
growth conditions like temperature, pH, substrates, salts vitamins and oxygen.
Stirred tank bioreactors are commonly used bioreactors.
A bioreactor has the following components.
(i) An agitator system
(ii) An oxygen delivery system
(iii) Foam control system
(iv) Temperature control system
(v) pH control system
(vi) Sampling ports to withdraw cultures periodically
(c) Downstream processing - All the process to which a product has to be subjected through a
series of processes before it is ready for marketing as a finished product called downstream
processing.
 It includes separation of the product from the reactor.
 Purification of the product.
 Formulation of the product with suitable preservatives.
 Quality control testing and trials in case of drugs.

E 175
11. Explain briefly.
(a) PCR (b) Restriction enzymes and DNA (c) Chitinase
Ans. (a) PCR - PCR stands for polymerase chain reaction, which is a method for amplification of
small segments of DNA.
(b) Restriction enzymes and DNA- Restriction enzymes are called ‘molecular scissors’
because they cut the helix of DNA at a specific site. DNA is the genetics material, which
carries and pass the genetic characters or information from one generation to other.
(c) Chitinase - Chitinase is an enzyme which is used to degrade the cell wall of fungi to release
its cellular components.
12. Discuss with your teacher and find out how to distinguish between. [IMP.]
(A) Plasmid DNA and chromosomal DNA (B) DNA and RNA
(C) Exonuclease and endonuclease
Ans. (A) Plasmid DNA and chromosomal DNA

S.No. Plasmid DNA Chromosomal DNA


1. This is present in prokaryotic cells. This is present in both prokaryotic and
eg -bacteria eukaryotic cells.
2. This is the circular extra-chromosomal It is linear and associated with histones
DNA not associated with histone proteins. proteins in eukaryotes, but it is double
stranded and circular in prokaryotes.
3. It gives extra characters to prokaryotes like It contains genes for characters
antibiotic resistance. essential for life of organism.
(B) DNA and RNA
S.No. DNA RNA
1. It has deoxyribose sugar . It has ribose sugar.
2. It has A, G, C, T nitrogen base in its It has A, G, C, U nitrogen base in its
nucleotide. nucleotide.
3. It is double stranded. It is mostly single stranded.
4. It acts as a genetic material in almost all It acts as genetic material in only some
organism. viruses.
5. DNA can’t be catalytic. It can be catalytic.(Ribozyme)
(C) Exonuclease and Endonuclease

S.No. Exonuclease Endonuclease


1. Remove nucleotides from ends of the DNA Cut at specific positions within the
DNA
2. They are end specific. They are site specific.
3. e.g : Exonuclease - I e.g. EcoRI
(B) PREVIOUS YEAR QUESTIONS
1. Assertion (A) : Synthetic oligonueleotide polymers are used during Annealing in a PCR.
[CBSE 2023]
Reason (R) : The primere bind to the double stranded DNA at their complementary regions.
Ans. (c) Assertion (A) is true but Reason (R) is false
2. (a) Write the scientific name of the source organism of the thermostable DNA polymerase
used in PCR. [CBSE 2023]
(b) State the advantage of using Thermostable DNA polymerase.
Ans. (a) Thermus aquaticus
(b) Thermostable polymerase (Taq polymerase) is used in the PCR due to their advantage to
remain active at very high temperatures. Unlike other polymerases. Taq polymerase do not
get denature in the process of PCR thus, effectively perform the task of DNA strand
polymeriozation even at 94 - 95ºC temperature.
3. (a) State the principle involved in separation of DNA fragments using gel electrophoresis.
(b) How are DNA fragments visuaised once they are separated by gel electrophoresis?
[CBSE 2023]
Ans. (a) DNA fragments are negatively charged molecules they can be separated by forcing them to
move towards the anode under an electric field through a medium/matrix.
(b) The separated DNA fragments can be visualised only after staining the DNA with a
compound known as ethidium bromide followed by exposure to UV radiation .You can see
bright orange coloured bands of DNA in a ethidium bromide stained gel exposed to UV
light.
4. (a) Given below is the stepwise schematic representation of the process of
electrophoresis.
P Q R

4
end 3 end
U S
2
1

T
Identify the 'alphabets' representing
(i) Anode end
(ii) Smallest/lightest DNA strand in the matrix
(iii) Agarose gel
(b) What is elution? State the importance of elution in this process.
[CBSE Term-II 2022]
177
Ans. (a) (i) Anode- S end (ii) R (iii) T
(b) The separated bands of DNA are cut out from the agarose gel and extracted from the gel
piece. This step is known as elution.
Importance:- The DNA fragments purified in this way are used in constructing recombinant
DNA by joining them with cloning vectors.
5. (a) Read the paragraph given below and answer the questions that follow:
Enzyme Taq polymerase, is extracted from a eubacterial microorganism Thermus
aquaticus from Yellowstone National Park in Montana, USA and isolated by Chien at
al. (1976). Taq polymerase successfully replaced the DNA polymerase from E.coli that
was being used in PCR earlier and this shift revolutionised the PCR technique.
(i) Taq polymerase after its discovery replaced E.coli DNA polymerase in PCR
technique. Explain giving reasons why was the need felt for the change?
(ii) What is a primer and its importance in PCR?
(ii) What the importance of PCR as a diagnostic tool. [CBSE Term-II 2022]
Ans. (a) (i) Thermostable DNA polymerase (isolated from a bacterium, Thermus aquaticus), which
remain active during the high temperature induced denaturation of double stranded DNA.
(ii) primer is a small segment of DNA that binds to a complementary strand of DNA. Primers
are necessary to start the functioning of DNA polymerase enzyme and therefore are
necessary in polymerase chain reaction.
(iii) PCR is important because it can generate several copies of a DNA sequence in a very
short time period. It is also important in forensic science as a tool for genetic engineering.
It helps in analyzing the gene expression
6. Name the commonly used vector for cloning genes into higher organisms.
[CBSE IMP Questions]
Ans. Retrovirus/ Adenoviruses/Papilloma virus/Cauliflower mosaic virus/Tobacco mosaic virus
7. Assertion: E. coli having pBR322 with DNA insert at BamHI site cannot grow in medium
containing tetracycline. [CBSE IMP Questions]
R
Reason: Recognition site for Bam HI is present in tet region of pBR322.
(A) Both assertion and reason are true, and the reason is the correct explanation of the assertion.
(B) Both assertion and reason are true, but the reason is not the correct explanation of the
assertion.
(C) Assertion is true but reason is false.
(D) Both assertion and reason are false
Ans. (A) Both assertion and reason are true, and the reason is the correct explanation of the
assertion.
8. What are sticky ends? State their significance in recombination DNA technology.
[CBSE IMP Questions]
Ans.  Restriction enzymes cut the strand of DNA a little away from the centre of the palindrome
sites, but between the same two bases on the opposite strands. This leaves single stranded
portions at the ends. These overhanging stretches on each strand are called sticky ends.
 They form hydrogen bonds with their complementary counterparts and facilitate the action
of DNA ligase enzyme.
178 E
9. (a) Identify step A and B in a cycle of polymerase chain reaction given below?

5' 3'
3' 5'
Primers
(A)
5' 3'
3' 5'

DNA polymerase
(Taq polymerase)
+ deoxynucleotides
5' 3'
3' 5'
(B)
5' 3'
3' 5'

(b) State the specific characteristic feature of the enzyme in carrying step B. [CBSE 2020]

Ans. (a) Step A- Annealing, B- Extension

(b) Thermostable DNA polymerase (isolated from a bacterium, Thermus aquaticus), which
remain active during the high temperature induced denaturation of ds DNA. It is known as
Taq DNA Polymerase.

10. How is a continuous culture system maintained in bioreactors and why? [CBSE 2019]

Ans. Used medium is drained out from one side of the bioreactor and fresh medium is added from the
other side.

- This type of culturing method produces a larger biomass leading to higher yields of desired
protein.

11. How are DNA fragments visualized during gel electrophoresis? What is elution?

[CBSE 2019]

Ans. Separated DNA fragments by agarose gel electrophoresis are stained with ethidium bromide,
followed by exposure to UV radiations and bright orange coloured DNA bands are visualized.
- The removal of DNA band by cutting a piece of gel with knife from agarose gel, this step is
called as elution.

12. Diagrammatically represent the process of amplification of “gene of interest” using PCR
technique. [CBSE 2018]
Ans. The process of amplification of “gene of interest” using PCR technique explain through
following structure.
179
5' 3'
ds DNA
3' 5'

Heat Denaturation(94°C)

5' 3'
3' 5'
Primers Annealing (54°C)
5' 3'
3' 5'

DNA polymerase
(Taq polymerase)
+ deoxynucleotides
5' 3'
3' 5'
Extension(72°C)
5' 3'
3' 5'

13. Briefly explain the roles of the following with the help of an example each in recombinant
DNA technology.
(a) Restriction Enzymes
(b) Plasmids [CBSE 2017]
Ans. (a) Restriction Enzymes - It recognizes a specific sequence of base pairs / palindromes, and
cuts the DNA strand at a specific site. eg. EcoRI
(b) Plasmids - Plasmids are extra-chromosomal and autonomously replicating circular ds
DNA in bacteria. They act as vector to transfer desired gene into the host cell.
eg. -pBR322, Ti plasmid of Agrobacterium.
14. Briefly explain the role (s) of the following in Biotechnology :
1. Restriction endonuclease
2. Gel – electrophoresis
3. Selectable markers in pBR322 [CBSE 2017]
Ans. (1) Restriction endonuclease - The enzyme which cuts at specific position within the DNA
known as restriction enzyme.
(2) Gel – electrophoresis-Separation of DNA fragments under the influence of electric field.
Agarose gel electrophoresis is employed to check the progression of a restriction
enzyme digestion.
(3) Selectable markers in pBR322- Helps in identifying and eliminating non-transformants
from transformants or selection of transformants .
Eg- (i) Ampicillin resistance gene
(ii) Tetracycline resistance gene
(C) MULTIPLE CHOICE QUESTIONS
1. The term “Biotechnology” was given by –
(1) Karl Ereky (2) Stanley Cohen (3) Herbert Boyer (4) Paul Berg
Ans. (1) Karl Ereky
2. Genetic engineering is:-
(1) Study of extra nuclear gene (2) Manipulation of genes by artificial method
(3) Manipulation of RNA (4) Manipulation of enzymes
Ans. (2) Manipulation of genes by artificial method
3. Who transferred gene of SV-40 virus into E.coli by –bacteriophage?
(1) Karl Ereky (2) Stanley Cohen (3) Herbert Boyer (4) Paul Berg
Ans. (4) Paul Berg
4. First Recombinant DNA produced by linking an antibiotic resistance gene with native plasmid of
(1) Salmonella typhimurium (2) Agrobacterium tumefaciens
(3) Escherichia coli (4) Haemophilus influenzae
Ans. (1) Salmonella typhimurium
5. Which of the following enzyme are known as molecular scissors?
(1) DNA Ligase (2) DNA Polymerase
(3) Reverse Transcriptase (4) Restriction endonuclease
Ans. (4) Restriction endonuclease
6. Restriction endonucleases are used in genetic engineering because :-
(1) They can degrade harmful proteins (2) They can join DNA fragments
(3) They can cut DNA at variable site (4) They can cut DNA at specific base sequences
Ans. (4) They can cut DNA at specific base sequences
7. According to EFB, "The integration of natural science and organisms, cells, parts thereof and
molecular analogues for products and services," is known as–
(1) Biochemistry (2) Bioinformatics (3) Biotechnology (4) Biology
Ans. (3) Biotechnology
8. Which of the following lytic enzyme used to isolation of DNA from fungal cell?
(1) Lysozyme (2) Cellulose (3) Pectinase (4) Chitinase
Ans. (4) Chitinase
9. The enzyme which remove nucleotides from ends of the DNA are known as
(1) Exonuclease (2) Endonuclease (3) Polymerase (4) Ligase
Ans. (1) Exonuclease
10. Which of the bond of DNA molecules cut by Restriction endonuclease?
(1) Phosphodiester bond (2) Hydrogen bond
(3) Glycosidic bond (4) Phosphoanhydride bond
Ans. (1) Phosphodiester bond
E 181
11. In enzyme EcoRI, the letter R indicate –
(1) Indicates species of bacteria
(2) Indicates genus of bacteria
(3) Indicates strain of bacteria
(4) Order in which the enzymes were isolated from bacteria
Ans. (3) Indicates strain of bacteria
12. The first isolated restriction endonuclease is-
(1) EcoR I (2) BamH I (3) Hind II (4) Hind III
Ans. (3) Hind II
13. The technique to check the progression of a restriction enzyme digestion is-
(1) Gel electrophoresis (2) PCR-technique
(3) Centrifugation (4) Southern blotting
Ans. (1) Gel electrophoresis
14. In agarose gel electrophoresis, DNA molecules are separated on the basis of their
(1) charge only (2) size only (3) charge to size ratio (4) all of the above
Ans. (4) all of the above
15. The separated bands of desired DNA are cut out from the agarose gel and extracted from the gel
piece is known as
(1) Southern blotting (2) Centrifugation
(3) Elution (4) Gel electrophoresis
Ans. (3) Elution
16. Which of the given statements is correct in the context of observing DNA separated by agarose
gel electrophoresis?
(1) DNA can be seen in visible light
(2) DNA can be seen without staining in visible light
(3) Ethidium bromide stained DNA can be seen in visible light
(4) Ethidium bromide stained DNA can be seen under exposure to UV light
Ans. (4) Ethidium bromide stained DNA can be seen under exposure to UV light
17. Which of the following is the example of molecular scissors.
(1) EcoRI (2) Hind - III (3) Bam – I (4) All the above
Ans. (4) All the above
18. Which of the following is the example of direct gene transfer?
(1) Microinjection (2) Electroporation (3) Particle gun (4) All the above
Ans. (4) All the above
19. Agrobacterium tumefaciens contains a large plasmid, which induces tumour in the plants it is
termed as -
(1) Ti plasmid (2) Ri plasmid
(3) Recombinant plasmid (4) pBR322
Ans. (1) Ti plasmid
20. The DNA molecules that can carry a foreign DNA segment into the host cell is known as
(1) Cloning vector (2) Selectable marker
(3) Recombinant DNA
Ans. (1) Cloning vector
(D) ASSERTION & REASON QUESTIONS
 Directions: In the following questions, a statement of assertion is followed by a statement of
reason. Mark the correct choice as:
(1) If both Assertion and Reason are true and Reason is the correct explanation of Assertion.
(2) If both Assertion and Reason are true but Reason is not the correct explanation of Assertion.
(3) If Assertion is true but Reason is false.
(4) If both Assertion and Reason are false.
1. Assertion: Bacterial cells are made competent by treating them with specific concentration of a
divalent cation.
Reason: Treatment of bacterial cell with a divalent cation increases the efficiency with which
DNA enters the bacterium through pores in its cell wall.
Ans. (1)
2. Assertion: Both the passenger and vehicle DNAs are treated separately with separate restriction
endonuclease.
Reason: Ligation is done by the use of alkaline phosphatase and DNA ligase.
Ans. (4)
3. Assertion : Vector DNA and foreign DNA are cut by same restriction endonuclease.
Reason: Digestion of vector DNA and foreign DNA with same enzyme produces complementary
sticky ends.
Ans. (1)
4. Assertion: Selectable marker is meant for distinguishing a recombinant from non-recombinant.
Reason: Every recombinant can flourish in medium having both ampicillin and tetracycline,
while the non- recombinants cannot.
Ans. (3)
5. Assertion: Restriction endonuclease recognises palindromic sequence in DNA and cuts them.
Reason: Palindromic sequence has two unique recognition sites Pst I and Pvu I recognised by
restriction endonuclease.
Ans. (3)
6. Assertion: Bacteriophage vectors are more advantageous than plasmid vectors.
Reason: Bacteriophage vectors can be easily detected at the time of cloning experiments.
Ans. (1)
7. Assertion : The separated DNA fragments can be visualised only after staining the DNA with
ethidium bromide in gel electrophoresis.
Reason: We can see pure DNA fragments in the visible light without staining.
Ans. (3)
8. Assertion: The ori site of vector responsible for controlling the copy number of the linked DNA.
Reason: Ori is a sequence from where replication starts.
Ans. (2)
9. Assertion: Retroviruses in animals have the ability to transform normal cells into cancerous cells.
Reason: The retrovirus should have been disarmed whenever it used to deliver desirable genes
into animal cells.
Ans. (2)
10. Assertion: Amplification of a gene of interest can be done by polymerase chain reaction.
Reason: It is possible to amplify DNA segment approximately 1 billion times within a span of 30 cycles.
Ans. (2)

183
(E) VERY SHORT ANSWER QUESTIONS
1. Mention the type of host cells suitable for the gene guns to introduce an alien DNA.

Ans. The host cells suitable for the gene guns to introduce an alien DNA are plant cells.

2. Name the host cells in which microinjection technique is used to introduce an alien DNA.

Ans. The microinjection technique to introduce alien DNA is usually carried out in animal cell, i.e.

directly into the nucleus.

3. Why is it essential to have a selectable marker in a cloning vector?

Ans. Selectable marker in cloning vector helps in identifying and selecting the recombinants and

eliminating the non-recombinants.

4. Why do DNA fragments move towards the anode during gel electrophoresis?

Ans. DNA fragments are negatively charged molecules and hence, moves toward the anode during gel

electrophoresis.

5. How is the action of exonuclease different from that of endonuclease.

Ans. Exonuclease removes nucleotides from the ends of DNA, while endonuclease cuts the DNA at

specific positions.

6. Mention the role of molecular scissors in recombinant DNA technology.

Ans. Molecular scissors or restriction enzymes cut DNA at specific site, thus allowing to extract

desired gene and like it with DNA of host.

7. Name the technique used for separating DNA fragments in the laboratory.

Ans. Gel electrophoresis is used for separating DNA fragments in the laboratory.

8. What is the role of ethidium bromide during agarose gel electrophoresis?

Ans. The separated DNA fragments during agarose gel electrophoresis are visualised after staining the

DNA with ethidium bromide, in UV light. This staining imparts DNA a bright orange colour.

9. Why it is not possible for an alien DNA to become part of a chromosome anywhere along its

length and replicate normally ?

Ans. Alien DNA must be linked to ori / origin of replication / site to start replication.

10. Which main technique and instrument is used to isolate DNA from a plant cell ?

Ans. Centrifugation and centrifuge.


(F) SHORT ANSWER QUESTIONS
1. Describe a palindrome with the help of an example.
Ans. A DNA sequence that reads the same, on the two strands from 5’– 3’ direction or 3' – 5' direction.
5’–GAATTC–3’
3’–CTTAAG–5’
2. List the key tools used in recombinant DNA technology.
Ans. Restriction enzymes / Polymerase enzymes / Ligase enzymes / Vectors / Host organisims/ E.coli/
Agrobacterium.
3. Name two commonly used vectors in genetic engineering.
Ans. Plasmid and Bacteriophage.
4. Name the bacterium that yields thermostable DNA polymerase.
Ans. Thermus aquaticus.
5. Write any two biochemical / molecular diagnostic procedures for early detection of viral
infection. Explain the principle of any one of them.
Ans. ELISA
ELISA – antigen antibody interaction / PCR – amplification of nucleic acid for its identification
6. Retroviruses have no DNA. However, the DNA of the infected host cell does possess viral
DNA. How is it possible?
Ans. Retrovirus have RNA as genetic material whenever it enter into host cell formed cDNA through
process of reverse transcription so that host cell possess viral DNA.
7. Why is 'plasmid' an important tool in biotechnology experiments?
Ans. Plasmids are commonly used to multiply or express particular genes and act as vectors to transfer
piece of foreign DNA attached to them.
8. Why is it essential to have 'selectable marker' in cloning vector ?
Ans. Selected marker helps in the identification and elimination of non-transformants and permitting
the growth of the transformants. Therefore, they are considered. Therefore, they are considered
essential in cloning vacter.
9. Why are molecular scissors so called? Write their use in biotechnology.
Ans. The restriction enzymes are known as molecular scissors as they cut the DNA at specific sites or
locations.
They help (in genetic engineering) to form recombinant molecules of DNA, which are composed
of DNA from different genomes.
10. What if EcoRI? How does EcoRI differ from an exonuclease?
Ans. EcoRI is restriction endonuclease enzyme.
Exonuclease removes nucleotides from the ends of DNA.
EcoRI makes cuts at specific position within the DNA.
185
(G) LONG ANSWER QUESTIONS
1. Briefly explain the roles of the following with the help of an example each in recombinant
DNA technology.
(a) Restriction Enzymes
(b) Plasmids
Ans. (a) Restriction Enzymes - It recognizes a specific sequence of base pairs / palindromes, and
cuts the DNA strand at a specific site. eg. EcoRI
(b) Plasmids - Plasmids are extra-chromosomal and autonomously replicating circular ds
DNA in bacteria. They act as vector to transfer desired gene into the host cell.
e.g., -pBR322, Ti plasmid of Agrobacterium.
2. Briefly explain the role (s) of the following in Biotechnology:
1. Restriction endonuclease
2. Gel – electrophoresis
3. Selectable markers in pBR322
Ans. (a) Restriction endonuclease The enzyme which cuts at specific position within the DNA
known as restriction enzyme.
(b) Gel-electrophoresis-Separation of DNA fragments under the influence of electric field.
Agarose gel electrophoresis is employed to check the progression of a restriction
enzyme digestion.
(c) Selectable markers in pBR322- Helps in identifying and eliminating non-transformants
from transformants or selection of transformants .
Eg- (i) Ampicillin resistance gene
(ii) Tetracycline resistance gene
3. (a) Why must a cell be made 'competent’ in biotechnology experiments? How does calcium
ion help in doing so?
(b) State the role of ‘biolistic gun’ in biotechnology experiments.
Ans. (a) To take up the (hydrophilic) DNA from the external medium.
• Divalent calcium ions increase the efficiency of DNA entering the cell through pores in the
cell wall.
(b) Biolistic gun helps to introduce alien DNA into the plant cell by bombarding high velocity
microprojectile (gold or tungsten) coated with DNA.
4. (a) How has the development of bioreactor helped in biotechnology ?
(b) Name the most commonly used bioreactor and describe its working.
Ans. (a) Larger biomass / large volume of culture can be processed leading to higher yields of
desired specific products (protein / enzymes),under controlled condition.
(b) Stirring type
• Mixing of reactor contents evenly (with agitator system or a stirrer)
• Facilitates oxygen availability
• Temperature / pH / foam control/under optimum conditions
5. Study the figure of vector pBR322 given below. Identify A, B and C and explain their roles
in cloning a vector.
EcorR I Cla I Hind III

Pvu I BamH I
A D
pBR322
B
C

Pvu II
Ans. A - antibiotic resistance genes, the ligation of alien DNA is carried out at a restriction site on this
gene / acts as selectable marker present in this antibiotic resistance gene.
B - ori, the sequence where replication starts
C - rop, codes for proteins involved in the replication of the plasmids
6. With the help of diagrammatic representation only, show the steps of recombinant DNA
technology.
Ans. Recombinant DNA technology involve the following steps:
(a) Isolation of DNA.
(b) Digestion of DNA by restriction endonuclease.
(c) Isolation of a desired DNA fragment.
(d) Amplification of the gene of interest.
(e) Ligation of the DNA fragment into a vector.
(f) Insertion of recombinant DNA into the host.
(g) Culturing the host cells on a suitable medium at a large scale.
(h) Extraction of the desired gene product.
(i) Downstream processing of the products as finished product, ready for marketing.
Foreign DNA Vector
DNA
(Plasmid)
Same restriction enzyme cutting both foreign
DNA and vector DNA at specific point

Ligases join foreign


DNA to plasmid

Recombinant
DNA Molecule

Transformation

E. coli
Cloning Host Cells divide

Diagrammatic representation of recombinant DNA technology

187
7. A typical agarose gel electrophoresis showing migration of DNA fragments. Observerd it

and give the answer of question that follow.

(a) How are the separated DNA fragments visualised?

(b) What are the criteria for separation of DNA fragments in gel electrophoresis.

(c) What is elution?

Ans. (a) The separated DNA fragments can be visualised only after staining the DNA with ethidium

bromide followed by exposure of UV radiation.

(b) The DNA fragments separate (resolve) according to size and length along with charge

through sieving effect provided by the agarose gel.

(c) The separated bands of DNA are cut out from the agarose gel and extracted from the gel

piece called as elution.

8. (a) Why should a bacterium be made 'competent' ?

(b) Explain the role of `microinjection' and 'gene gun' in biotechnology.

Ans. (a) The bacterial cells must first be made competent in order to receive the hydrophilic rDNA/

plasmid which cannot otherwise pass through the cell membrane.

(b) Microinjection - rDNA is directly injected into the animal cell nucleus

Biolistics (gene gun) - Plant cells are bombarded with high velocity micro-particles of gold

/tungsten coated with DNA.


(H) CASE-STUDY BASED QUESTIONS
1. Read the following and answer the questions given below:
The first restriction endonuclease–Hind II, whose functioning depended on a specific DNA
nucleotide sequence was isolated and characterised five years later. It was found that Hind II
always cut DNA molecules at a particular point by recognising a specific sequence of six base
pairs. This specific base sequence is known as the recognition sequence for Hind II. Besides
Hind II, today we know more than 900 restriction enzymes that have been isolated from over 230
strains of bacteria each of which recognise different recognition sequences.
(i) What distinguish exonuclease activity from Endonuclease?
Ans. Exonuclease remove nucleotides from the ends of DNA whereas, endonucleases make cuts at
specific positions within the DNA.
(ii) When a restriction enzyme discovers its recognition sequence, what happens?
Ans. Once restriction enzyme finds its specific recognition sequence, it will bind to the DNA and cut
each of the two strands of the two strands of the double helix at specific points in their sugar -
phosphate backbone.
(iii) Why are molecular scissors so called? write their use in biotechnology.
Ans. The restriction enzymes are as molecular scissors as, They cut the DNA at specific sites or
locations. They Help (in genetic engineering) to from recombinant molecules of DNA, which are
composed of DNA from different genomes.
(iv) In accordance with restriction enzyme naming conventions, expand Hind II.
Ans. In Hind II enzyme, H stands for Haemophilus, in stands for influenzae, d stands for strain, II
stands for type II restriction ezymes.
2. Read the following and answer the questions given below:
Rajat is a student of biotechnology. His professor tells him that for transformation with
recombinant DNA the bacterial cells must be made capable of taking up DNA as DNA do not
pass through membrane. While doing experiment in the lab, Rajat noticed that bacterial cells
were not taking up the foreign DNA even after treating it with sodium ion. He asked his
professor, the Reason behind this. His professor explained that he should check the valency and
charge of the ion that he is using for the treatment.

(i) Mention the type of host cells suitable for the gene guns to introduce an alien/foreign DNA.
Ans. Plant host cells are suitable for the gene guns to introduce an alien/foreign DNA.
(ii) Why DNA cannot pass through the cell membrane? Explain.
Ans. Since, DNA molecules are hydrophilic, they cannot pass through cell membranes. For
recombinant DNA to be Integrated into vector or host genome, it is necessary for the DNA to be
inserted in the cell.

E 189
(iii) How is a bacterial cell made competent to take up recombinant DNA from the medium?

Ans. The two ways by which cells can be made competent to take up DNA are

(1) Chemical action :- The host cell is treated with a specific concentration of Divalent cation,
i.e., calcium Increases the pore size in the cell membrane.

(2) Heat-shock treatment :- Incubating the cells with recombinant DNA on ice, followed by
brief Treatment of heat at 42ºC and again putting them back on ice.

(iv) Write any four ways used to introduce a desired DNA segment into a bacterial cell in
recombinant technology experiments.

Ans. (i) Microinjection

(ii) Disarmed Pathogen vectors

(iii) Biolistic or gene gun

(iv) Treatment of host cell by bivalent cation such as calcium.

3. Read the following and answer the questions given below:


The cutting of DNA by restriction endonucleases results in the fragments of DNA. These
fragments can be separated by a technique known as gel electrophoresis. Since DNA fragments
are negatively charged molecules they can be separated by forcing them to move towards the
anode under an electric field through a medium/matrix. The separated DNA fragments can be
visualised only after staining the DNA with a compound known as ethidium bromide followed by
exposure to UV radiation (you cannot see pure DNA fragments in the visible light and without
staining). You can see bright orange coloured bands of DNA in a ethidium bromide stained gel
exposed to UV light.

(i) In Gel-electrophoresis the separation of DNA fragments take place upon the basis of.
Ans. Charge and length of fragments.
(ii) The Agarose gel which used as a medium is electrophoresis obtained from.
Ans. See weeds.
(iii) What is the role of ethidium bromide during agarose gel-electrophoresis?
Ans. The separated DNA fragments during agarose gel-electrophoresis are visualised after staining the
DNA with ethidium bromide, in UV light, this staining imparts DNA a bright orange colour.
(iv) The separation of DNA fragments into electrophoresis take plane by which effect?
Ans. Sieving effect.
(v) Why do DNA fragments move towards the anode during gel electrophoresis?
Ans. Because DNA is negatively charged.
4. Read the following and answer the questions given below :
Small volume cultures cannot yield appreciable quantities of products. To produce in large
quantities, the development of bioreactors, where large volumes (100-1000 litres) of culture can
be processed, was required. Thus, bioreactors can be thought of as vessels in which raw materials
are biologically converted into specific products, individual enzymes, etc., using microbial plant,
animal or human cells. A bioreactor provides the optimal conditions for achieving the desired
product by providing optimum growth conditions (temperature, pH, substrate, salts, vitamins,
oxygen). The most commonly used bioreactors are of stirring type,which include simple stirred-
tank bioreactor and sparged stirred - tank bioreactor.
(i) Which of the apparatus used in large scale production of recombinant product?
Ans. Bioreactor
(ii) Which of the factors affect the quality of obtained product in a bioreactor?
Ans. These are (i) Temperature (ii) pH (iii) Oxygen supply
(iii) Write the name of most commonly used bioreactor.
Ans. Stirred type
(iv) The sparged stirred-tank bioreactor is well suited for large scale production why?
Ans. In sparged stirred tank bioreactor surface area for oxygen transfer is increased.
(v) Define bioreactor
Ans. A large vessel with stirring arrangement in which organic raw material are biologically
converted into specific product under optimum condition.

191
IMPORTANT NOTES
__________________________________________________________________________________

__________________________________________________________________________________

__________________________________________________________________________________

__________________________________________________________________________________

__________________________________________________________________________________

__________________________________________________________________________________

__________________________________________________________________________________

__________________________________________________________________________________

__________________________________________________________________________________

__________________________________________________________________________________

__________________________________________________________________________________

__________________________________________________________________________________

__________________________________________________________________________________

__________________________________________________________________________________

__________________________________________________________________________________

__________________________________________________________________________________

__________________________________________________________________________________

__________________________________________________________________________________

__________________________________________________________________________________

__________________________________________________________________________________

__________________________________________________________________________________

__________________________________________________________________________________

__________________________________________________________________________________
CHAPTER 10
(A) NCERT QUESTIONS & SOLUTIONS
1. Crystals of Bt toxin produced by some bacteria do not kill the bacteria themselves because-
(a) bacteria are resistant to the toxin
(b) toxin is immature;
(c) toxin is inactive;
(d) bacteria encloses toxin in a special sac.
Ans. (c) toxin is inactive.
2. What are transgenic bacteria? Illustrate using any one example.
Ans. The bacteria whose DNA is manipulated, carry and express a foreign DNA is called transgenic
bacteria.
For example :- Two DNA sequence (A and B chains of human insulin) were introduced into the
plasmid of bacteria E.coil. The transgenic bacteria start producing insulin chain.
3. Compare and contrast the advantages and disadvantages of production of genetically
modified crops. [IMP.]
Ans. Advantages of genetically modified crops:
(1) Reduces the use of chemical fertilizers and pesticides which cause pollution
(air, water and soil).
(2) Production of new resistant varieties against pathogen, droughts, salinity, etc.
(3) Provides raw materials to industries like pharmaceuticals.
(4) Genetically modified crops have enhanced nutritional quality and yield.
(5) These crops grow fast and produce high yield through modifications.
Disadvantages of genetically modified crops:
(1) Proteins produced by GM organisms might cause allergy and other reactions.
(2) Resistance characters might develop in intestinal bacteria against antibiotics.
(3) Resistant genes transferred by pollen to the weeds may also become resistant to pests.
4. What are Cry proteins? Name an organism that produce it. How has man exploited this
protein to his benefit? [IMP.]
Ans. Cry protein (crystal protein) is a toxin coded by a gene cry and is poisonous to some insects, thus
giving resistant characters to the plants.
Bacillus thuringiensis produces Cry protein.
Cry protein producing gene is transferred to the plants to provide resistance against insect larvae.
Man has developed several transgenic crops by introducing these genes from bacteria to crop
plants such as Bt cotton, Bt corn etc.
E 193
5. What is gene therapy ? Illustrate using the example of adenosine deaminase (ADA) deficiency.

Ans. Gene therapy is a method which corrects or replaces the defective genes. [IMP.]

In 1990, first clinical gene therapy was given to a 4-year old girl with adenosine deaminase

(ADA) deficiency.

This enzyme plays an important role in functioning of immune system. The disorder is caused

due to the deletion of the gene for adenosine deaminase.

In gene therapy, lymphocytes from the blood of the patient are grown in a culture outside the

body.

A functional ADA cDNA (using a retroviral vector) is then introduced into these lymphocytes,

which are returned to the patients. However, as these cells are not immortal, hence the patient

requires periodic infusion of such genetically engineered lymphocytes.

6. Diagrammatically represent the experimental steps in cloning and expressing an human

gene (Say the gene for growth hormone) into a bacterium like E.coli.

Ans. It is possible to produce HGH (human growth hormone) by recombinant DNA technology.

This is represented diagrammatically as follows :

Plasmid
Bacterial
chromosome
Human
Escherichia cell
coli
Restriction
Enzyme HGH
Gene
Cut Plasmid
Sticky ends HGH Gene
Joining foreign
DNA by ligase

Plasmid free
E.coli Recombinant DNA

Multiplication of
Recombinant DNA
Reintroduced

Clones of engieered
bacteria in a
fermenter
containing medium

Recombinant HGH

Fig. Formation of Growth Hormone by rDNA Technology.


194 E
7. Can you suggest a method to remove oil (hydrocarbon) from seeds based on your
understanding of rDNA technology and chemistry of oil?
Ans. To remove oil from seeds, the genes responsible for formation of glycerol or fatty acids need to
be identified and removed which is responsible for this synthesis. So, by rDNA technique one
can obtain oil-less seeds by preventing the synthesis of fats.
8. Find out from internet what is golden rice.
Ans. Golden rice is a genetically modified rice that contains β-carotene (provitamin A).
This rice is modified in order to enhance the quantity of vitamin - A in it. It is called golden due
to the gold like colour it gets from - carotene.
9. Does our blood have proteases and nucleases?
Ans. Blood does not contain proteases and nucleases because their function is to breakdown proteins
and nucleic acids.
10. Consult internet and find out how to make orally active protein pharmaceutical. What is
the major problem to be encountered?
Ans. Orally active protein pharmaceutical can be made by lining it with a substance that will dissolve
after it has passed through the stomach. The major problem encountered is that the stomach
enzymes and acids may denature the therapeutic protein and render it ineffective.

E 195
(B) PREVIOUS YEAR QUESTIONS
1. (a) Name (i) a GM cereal crop having enhanced nutritional value, (ii) the nutrient it is
rich in.
(b) State any two benefits of Genetically modified crops. [CBSE 2023]
Ans. (a) (i) Golden rice. (ii) Vitamin ‘A’ enriched
(b) GM plants have been useful in many ways. Genetic modification has:
(i) Increased tolerance against abiotic stresses (cold, drought, salt, heat).
(ii)Reduced reliance on chemical pesticides (pest-resistant crops).
2. "RNA interference has been used to produce transgenic tobacco plants to protect them
from the infestation by specific nematodes." Explain the novel strategy exploited by the
biotechnologists. [CBSE 2023]
Ans. A novel strategy was adopted to prevent this infestation which was based on the process of RNA
interference (RNAi). RNAi takes place in all eukaryotic organisms as a method of cellular
defense.
 This method involves silencing of a specific mRNA due to a complementary dsRNA
molecule that binds to and prevents translation of the mRNA (silencing).
 The source of this complementary RNA could be from an infection by viruses having RNA
genomes or mobile genetic elements (transposons) that replicate via an RNA intermediate.
 Using Agrobacterium vectors, nematode-specific genes were introduced into the host plant.
 The introduction of DNA was such that it produced both sense and anti-sense RNA in the
host cells.
These two RNA’s being complementary to each other formed a double stranded (dsRNA) that
initiated RNAi and thus, silenced the specific mRNA of the nematode. The consequence was that
the parasite could not survive in a transgenic host expressing specific interfering RNA. The
transgenic plant therefore got itself protected from the parasite.
3. Eli Lilly's contribution for diabetic patients through r-DNA technology has been
overwhelmingly accepted. Explain how? [CBSE 2023]
Ans. Insulin used for diabetes was earlier extracted from pancreas of slaughtered cattle and pigs.
Insulin from an animal source, though caused some patients to develop allergy or other types of
reactions to the foreign protein.
In 1983, Eli Lilly an American company prepared two DNA sequences corresponding to A and
B, chains of human insulin and introduced them in plasmids of E. coli to produce insulin chains.
Chains A and B were produced separately, extracted and combined by creating disulfide bonds to
form human insulin.
196 E
4. What are cry–proteins? With the help of a suitable example, explain how it acts as a
biological pesticide. [CBSE Term-II 2022]
Ans. Cry Protein: The insecticidal protein which is produced by soil bacterium named Bacillus
thuringiensis is called cry protein.
For example - The proteins encoded by the genes cryIAc and cryllAb control the cotton
bollworms, that of cryIAb controls corn borer.
5. Read the following paragraph and answer the questions that follow :
Biotechnology revolves around the "gene of interest", with an objective to open various
avenues for human welfare in health, medicine pharma, agriculture etc, using different
techniques, tools and process. One of the breakthroughs of biotechnology in medicine is the
gene therapy.
(i) Name the human disease for which of gene therapy was used for the first time.
(ii) Explain the steps of gene therapy carried to cure the disease using the lymphocytes of the
patient. Why is this therapy not a permanent cure of the disease?
(iii) Write the possible permanent cure of the disease by the gene therapy that is in progress.
[CBSE Term-II 2022]
Ans. (i) ADA deficiency [Adenosine deaminase deficiency]
(ii) As a first step towards gene therapy, lymphocytes from the blood of the patient are grown in
a culture outside the body. A functional ADA cDNA (using a retroviral vector) is then
introduced into these lymphocytes, which are subsequently returned to the patient. However,
as these cells are not immortal, the patient requires periodic infusion of such genetically
engineered lymphocytes.
(iii) If the gene isolate from marrow cells producing ADA is introduced into cells at early
embryonic stages, it could be a permanent cure.

6. Differentiate between pro-insulin and mature insulin. [CBSE IMP Question]


Ans. Pro-insulin contains an extra stretch called the C peptide which is not present in the mature
insulin.
7. Explain the procedure by which PCR aids in early detection of cancer.
[CBSE IMP Question]
Ans.  A single stranded DNA or RNA is tagged with a radioactive molecule(probe)
 It is allowed to hybridize to its complementary DNA in a clone of cells followed by
detection using autoradiography.
 The clone having the mutated gene will hence not appear on the photographic film,
 because the probe will not have complementarity with the mutated gene.
Hence, cancer induced mutation can be detected.

E 197
8. A farmer noticed that nematode infection in tobacco plants has resulted in the reduction in
the yield. Suggest a strategy which provides cellular defense for providing resistance to this
pest. Explain the technique. [CBSE IMP Question]
Ans.  Strategy based on the process of RNA interference (RNAi) - as a method of cellular defense
can be used.
 This method involves silencing of a specific mRNA due to a complementary dsRNA
molecule that binds to and prevents translation of the mRNA (silencing).
 The source of this complementary RNA could be from an infection by viruses having RNA
genomes or mobile genetic elements (transposons) that replicate via an RNA intermediate.
 Using Agrobacterium vectors, nematode-specific genes were introduced into the host plant.
The introduction of DNA was such that it produced both sense and anti-sense RNA in the
host cells.
 Two RNA’s being complementary to each other formed a double stranded (dsRNA) that
initiated RNAi and thus, silenced the specific mRNA of the nematode.
 As a consequence, the parasite could not survive in a transgenic host expressing specific
interfering RNA. The transgenic plant therefore got itself protected from the parasite.
9. When Bacillus thuringiensis enters a certain insect’s body .The insect gets killed, but itself
remains unaffected .Explain how it is possible. [CBSE 2020]
Ans. Bacillus thuringiensis forms protein crystals during a particular phase of their growth. These
crystals contain a toxic insecticidal protein, which killed insects but this Bt toxin protein exist as
inactive protoxins but once an insect ingest the inactive toxin, it is converted into an active form
of toxin due to the alkaline pH of the gut which solubilise the crystals. The activated toxin binds
to the surface of midgut epithelial cells and create pores that cause cell swelling and lysis and
eventually cause death of the insect.
10. A child is born with ADA-deficiency-
(a) Suggest and explain a procedure for possible life-long (permanent) cure.
(b) Name any other possible treatment for this disease. [CBSE 2019]
Ans. (a) Gene therapy - In gene therapy lymphocytes from the blood of the patient are grown in a
culture outside the body A functional ADA cDNA is then introduced into these
lymphocytes, these cells are returned to the patient’s body at early embryonic stage.
(b) Bone marrow transplantation and enzyme replacement therapy
11. A corn farmer has perennial problem of corn-borer infestation in his crop. Being
environmentally conscious he does not want to spray insecticides. Suggest solution based on your
knowledge of biotechnology. Write the steps to be carried out to achieve it. [CBSE 2019]
Ans. Isolation of Bt toxin genes from Bacillus thuringiensis, incorporated into corn, toxin coded by
gene cry I Ab in corn, kills the pests/ pest dies.
12. Why has the Indian Government set up the organization named GEAC? Give any two
reasons. [CBSE 2019]
Ans. (i) To check the Validity of GM research and safety of introducing of GM organisms for public
services.
(ii) To approve large - scale field trials and the release of transgenic crops in the environment.
13. (a) What are transgenic animals?
(b) Name the transgenic animal having the largest number amongst all the existing transgenic
animals.
(c) Mention any three purposes for which these animals are produced. [CBSE 2018]
Ans. (a) Animals that have had their DNA manipulated to possess and express an extra (foreign) gene
are known as transgenic animals.
(b) Over 95 % of all existing transgenic animals are mice.
(c) Three purposes for which transgenic animals produced-
(1) Normal physiology and development: Transgenic animals can be specifically designed to
allow the study of how genes are regulated, and how they affect the normal functions of the
body and its development.
(2) Study of disease: Many transgenic animals are designed to increase our understanding of
how genes contribute to the development of disease.
(3) Vaccine safety: Transgenic mice are being developed for use in testing the safety of
vaccines before they are used on humans.
14. Mention the chemical change that pro-insulin undergoes, to be able to act as mature
insulin. [CBSE 2018]
Ans. Removal of C - peptide (from pro-insulin).
15. What are Cry genes? In which organism are they present? [CBSE 2017]
Ans. The genes which code for Bt toxin / Cry proteins / toxic proteins, they are present in bacteria
Bacillus thuringiensis.
16. Why do lepidopterans die when they feed on Bt cotton plant? Explain how does it happen.
[CBSE 2017]
Ans. Bt cotton contains inactive toxin protein (Insecticidal protein), once an insect ingest it the
inactive protoxins are converted into an active form of toxin due to the alkaline pH of the gut
which solubilise the crystals, activated toxins binds to surface of midgut (epithelial cells), create
pores causes cell swelling and lysis and eventually cause the death of the insect.
E 199
(C) MULTIPLE CHOICE QUESTIONS
1. First transgenic plant was -
(1) Cotton (2) Maize (3) Rice (4) Tobacco
Ans. (4) Tobacco
2. The proteins encoded by the genes cryI Ac and cry II Ab control the
(1) Cotton bollworms (2) Roundworm (3) Moth (4) Fruit fly
Ans. (1) Cotton bollworms
3. A transgenic rice (Golden rice) has been developed for increased content of :-
(1) Vitamin A (2) Vitamin B1 (3) Vitamin C (4) Vitamin D
Ans. (1) Vitamin A
4. Golden rice is
(1) a variety of rice grown along the yellow river in China.
(2) long stored rice having yellow colour tint.
(3) a transgenic rice having gene for -carotene.
(4) wild variety of rice with yellow coloured grains
Ans. (3) a transgenic rice having gene for -carotene.
5. Some strains of Bacillus thuringiensis produce proteins that kill certain insects such as-
(1) Lepidopterans (2) Coleopterans (3) Dipterans (4) All the above
Ans. (4) All the above
6. The inactive protoxins of Bacillus thuringiensis are activated inside body of insect due to the-
(1) alkaline pH of the gut (2) alkaline pH of the mouth
(3) acidic pH of the gut (4) acidic pH of the mouth
Ans. (1) alkaline pH of the gut
7. The protein products of the Bt toxin genes cryIAc and cryIIAb are responsible for controlling:-
(1) Bollworm (2) Roundworm (3) Moth (4) Fruit fly
Ans. (1) Bolloworm
8. Transgenic Brassica napus has been used for the synthesis of :-
(1) Hirudin (2) Heparin (3) Polgalacturonase (4) Cry protein
Ans. (1) Hirudin
9. Which of the nematode infects the roots of tobacco plants(Root knot disease) and causes a great
reduction in yield.
(1) Ascaris lumbricoides (2) Taenia solium
(3) Meloidegyne incognitia (4) Faciola hepatica
Ans. (3) Meloidegyne incognitia
10. Transgenic tobacco plant was developed by the process of RNA interference, was resistant
against the infection of -
(1) Algae – Scenedesmus (2) Fungi - Fusarium
(3) Bacteria - Bacillus thuringiensis (4) Nematode :Meloidegyne incognitia
Ans. (4) Meloidegyne incognitia
11. The first genetically engineered insulin has been developed by Eli-Lilly company is .It is a
(1) British Company (2) Indian Company
(3) American Company (4) Chinese Company
Ans. (3) American Company
12. Genetically engineered bacteria have been used in commercial production of –
(1) Thyroxin (2) Testosterone (3) Human insulin (4) Melatonium
Ans. (3) Human insulin
13. The name of drug used in cancer treatment produced by biotechnology is –
(1) Interferon (2) Human growth hormone
(3) TSH (4) Insulin
Ans. (1) Interferon
14. During the processing of the prohormone "proinsulin" into the mature "insulin"
(1) C-peptide is added to proinsulin (2) C-peptide is removed from proinsulin
(3) B-peptide is added to proinsulin (4) B-peptide is removed from proinsulin
Ans. (2) C-peptide is removed from proinsulin
15. Which step proved to be the main challenging obstacle in the production of human insulin by
genetic engineering?
(1) Splitting A and B polypeptide chains.
(2) Addition of C-peptide to pro-insulin.
(3) Getting insulin assembled into mature form.
(4) Removal of C-peptide from active insulin.
Ans. (3) Getting insulin assembled into mature form.
16. What is the disadvantage of using porcine insulin (from pig) in diabetic patients?
(1) It leads hypercalcemia (2) It is expensive
(3) It may cause allergic reactions (4) It can lead to mutation in human genome
Ans. (3) It may cause allergic reactions
17. Why insulin not administered orally to diabetic patient?
(1) Insulin is bitter in taste
(2) Insulin is sour in taste
(3) Insulin will lead to sudden increase in blood sugar if given orally
(4) Insulin leads to peptic ulcer if given orally
Ans. (3) Insulin will lead to sudden increase in blood sugar if given orally
18. The first clinical gene therapy was given in 1990 to a 4 years old girl with enzyme deficiency of
(1) Adenosine deaminase (ADA) (2) Adenosine oxidase
(3) Adenosine diaminase (4) Adenosine dehydrogenase
Ans. (1) Adenosine deaminase (ADA)
19. Why is repeated transfusions of genetically engineered cells required in SCID patients?
(1) Transfused cells have a limited life span (2) The introduced gene is mutated
(3) Transfused cells are immortal (4) Both (1) and (2)
Ans. (1) Transfused cells have a limited life span
20. The milk for Rosie cow is nutritionally a more balanced product for human babies that natural
cow-milk .It contained gene of
(1) human β -lactalbumin (2) human α-lactalglobulin
(3) human β- lactalglobulin (4) human α-lactalbumin
Ans. (4) human α - lactalbumin
201
(D) ASSERTION & REASON QUESTIONS
 Directions: In the following questions, a statement of assertion is followed by a statement of
reason. Mark the correct choice as:
(1) If both Assertion and Reason are true and Reason is the correct explanation of Assertion.
(2) If both Assertion and Reason are true but Reason is not the correct explanation of Assertion.
(3) If Assertion is true but Reason is false.
(4) If both Assertion and Reason are false.
1. Assertion :- Bt-plant are resistant for insect because gene of Bacillus thuringiensis is inserted in
crops.
Reason :- It is toxic to insect and cause pores in mid gut of insect.
Ans. (1)
2. Assertion : Transgenic food may cause toxicity or produce allergy.
Reason : Transgenic plants have high nutrient content.
Ans. (2)
3. Assertion : Genetically modified microbes help in crop protection.
Reason : Transgenic bacteria control insects by producing endotoxins.
Ans. (2)
4. Assertion : Organs of pig such as heart, pancreas, etc., for human use can be grown through
transgenic animals.
Reason : Transgenic pigs show improved growth and meat production.
Ans. (2)
5. Assertion : A gene from Bacillus thuringiensis is incorporated in plant genome to increase their
yield.
Reason : Bacillus thuringiensis has Bt toxin producing gene, which kills the larva of insects.
Ans. (1)
6. Assertion : Bio-patent are awarded for biological entities and all products derived from them.
Reason : Patent on use of turmeric in wound healing was cancelled in 2008.
Ans. (3)
7. Assertion : In gene therapy, new gene is introduced only into the somatic cells.
Reason : Genetic modification is done to replace faulty genes by normal functional gene.
Ans. (2)
8. Assertion : Flavr-Savr tomato was the first transgenic commercial crop that entered the market.
Reason : RNA interference involves silencing of a speecific mRNA due to a complementary
RNA molecule that binds to and prevent translation of the mRNA.
Ans. (2)
9. Assertion : GM crops can affect human health by causing allergic reactions.
Reason : Transgenes in commercial crops can endanger native species, the Bt toxin gene
expressed in pollen might endanger pollinators like honeybees.
Ans. (2)
10. Assertion : The recombinant therapeutics do not induce unwanted immunological respones.
Reason : About 30 recombinant therapeutics have been approved for human used world wide.
Ans. (2)

202 E
(E) VERY SHORT ANSWER QUESTIONS
1. Which is the first transgenic cow? Which gene was inserted into it?

Ans. The first transgenic cow was Rosie. The gene inserted was human -lactalbumin.
2. What is GEAC? What are its main objectives?
Ans. GEAC is Genetic Engineering Approval Committee. It is an Indian government organization. Its
main objectives are as follows:
• To examine the validity of genetic modification research.
• To inspect whether the use of genetically modified crops is safe for public use or not.
3. For which Indian rice variety was patent filed by a USA company?
Ans. Basmati rice.
4. How was insulin obtained before the advent of recombinant DNA technology?
Ans. Before the advent of recombinant DNA technology insulin was obtained from slaughtered cattle
and pigs.
5. What are the disadvantages of GMO?
Ans. The disadvantages of GMO are:
• These can harm the insects that are beneficial to our ecosystem.
• It is not a natural way to cultivate plants and hence can damage the environment.
• It causes unwanted residual effects.
• These create more weeds.
6. Why was the second amendment of the country’s patent bill cleared by the Indian Parliament?
Ans. The second amendment to the country’s patent bill was cleared to prevent biopiracy by other countries,
i.e., unauthorized exploitation of our bio-resources and traditional knowledge by other countries.
7. What is a patent?
Ans. Patent is a form of intellectual property right that gives its owner the right to exclude others from
using, and selling their invention for a limited period of time.
8. How are ‘cry’ and ‘Cry’ different from each other?
Ans. ‘cry’ is the gene that encodes for Bt toxin. ‘Cry’ is the protein coded by the ‘cry’ genes.
9. Suggest a molecular diagnostic procedure that detects HIV in suspected AIDS patients.
Ans. PCR and ELISA.
10. Mention the chemical change that proinsulin undergoes, to be able to act as mature insulin.
Ans. Removal of C-peptide (from pro-insulin).

E 203
(F) SHORT ANSWER QUESTIONS
1. How is a probe used in molecular diagnostics?
Ans. A probe is a DNA or RNA fragment of variable length which can be labelled by a radioactive
molecule. It hybridises to its complementary DNA and can be detected by autoradiography.
2. Is Bt-cotton resistant to all pests other than lepidopteron, dipterans, and coleopterans?
Ans. Bt cotton contains genes against lepidopteron, dipterans, and coleopterans. But, these genes are
not effective against all types of pests that attack Bt-cotton.
3. Name the transgenic animal having the largest number amongst all the existing transgenic
animals ?
Ans. Over 95% of all transgenic animals are mice.
4. What is gene therapy ? Name the first clinical case where it was used.
Ans. (i) Collection of methods that allows correction of a gene defect that has been diagnosed in a
child / embryo.
(ii) Adenosine deaminase (ADA) deficiency.
5. (i) Mention the cause and the body system affected by ADA deficiency in humans.
(ii) Name the vector used for transferring ADA-DNA into the recipient cells in humans.
Name the recipient cells.
Ans. (i) Defective gene not producing ADA, immune system is affected.
(ii) A retroviral vector is used. Recipient cells are lymphocytes.
6. Why is functional insulin produced considered better than the ones used earlier by diabetic
patient ?
Ans. The functional protein is produced by rDNA. It does not produce allergic reaction and
complication while earlier insulin was produced or extracted from pancreas of cattle and pig. It
caused allergy and many complication to the diabetic patients.
7. What is GMO? List any five possible advantages of a GMO to a farmer.
Ans. Plants, bacteria, fungi, animals whose genes have been altered by manipulation.
Tolerance to abiotic stresses, like cold, drought, salt, heat, reduced reliance on chemical
pesticides, pest resistant crops, reduce post harvest losses,
Increased efficiency of mineral usage by plants, enhanced nutritional value to create tailor made
plant.
8. Why is molecular diagnosis preferred over conventional methods? Name any two
techniques giving one use of each.
Ans. To allow early detection.
Example : rDNA technology / PCR / ELISA / Probe (Any two).
PCR-to detect low concentration of bacteria /virus (HIV).
9. What are transgenic animals. Give an example.
Ans. Animals whose DNA is manipulated to possess and express an extra (foreign) gene e.g. Rosie -
transgenic cow.
10. What was the speciality of milk produced by the transgenic cow Rosie.
Ans. Rosie was the first transgenic cow. The milk produced by it was protein rich. It contained human a
lactalbumin. This milk was nutritionally richer and balanced as compared to that of normal cow.
(G) LONG ANSWER QUESTIONS
1. How is the Bt cotton plant created as a GM plant ?
How is it protected against bollworm infestation ?
OR
(i) Why are certain cotton plants called Bt cotton plants?
(ii) Explain how Bt cotton is resistant to pests.
Ans. (i) Certain cotton plants are called Bt cotton because specific Bt toxin genes were isolated
from Bacillus thuringiensis and incorporated into these cotton plants. The proteins encoded
by the genes cryIAc and cryIIAb control the cotton bollworms, that of cryIAb controls corn
borer.
(ii) Specific Bt toxin genes were isolated from Bacillus thuringiensis and incorporated into the
several crop plants such as cotton. The choice of genes depends upon the crop and the
targeted pest, as most Bt toxins are insect-group specific. The toxin is coded by a gene
named cry. There are a number of them, for example, the proteins encoded by the genes
cryIAc and cryIIAb control the cotton bollworms, that of cryIAb controls corn borer.
2. (i) What is gene therapy?
(ii) What is meant by ADA deficiency ? How is gene therapy a solution to this problem ?
Why is it not a permanent cure ?
Ans. (i) It is a collection of methods that allows correction of a gene defect that has been diagnosed
in a child or embryo.
(ii) ADA is adenosine deaminase deficiency, this enzyme is crucial for the immune system to
function. The patient lacks functional T-lymphocytes and fails to fight the infecting
pathogens. Children with ADA deficiency are cured by bone marrow transplantation or
enzyme replacement therapy, where ADA is given by injection. By using gene therapy
techniques, lymphocytes are taken from the patient’s bone marrow and the normal gene for
ADA is introduced into the lymphocytes using retrovirus. These cells are re-introduced in
the patient’s immune system.
3. How has the use of Agrobacterium as vectors helped in controlling Meloidegyne incognitia
infestation in tobacco plants? Explain in correct sequence.
Ans. • Using Agrobacterium vector nematode specific genes introduced into host plant
• Sense and antisense strands of mRNA are produced
• ds RNA is formed
• ds RNA initiates RNAi
• Prevents translation of mRNA / silencing of mRNA of parasite / nematode
• Parasite will not survive
4. (a) What are transgenic animals?
(b) Name the transgenic animal having the largest number amongst all the existing transgenic
animals.
(c) Mention any three purposes for which these animals are produced.

E 205
Ans. (a) Animals that have had their DNA manipulated to possess and express an extra (foreign) gene
are known as transgenic animals.
(b) Over 95 % of all existing transgenic animals are mice.
(c) Three purposes for which transgenic animals produced-
(1) Normal physiology and development: Transgenic animals can be specifically designed to
allow the study of how genes are regulated, and how they affect the normal functions of the
body and its development.
(2) Study of disease: Many transgenic animals are designed to increase our understanding of
how genes contribute to the development of disease.
(3) Vaccine safety: Transgenic mice are being developed for use in testing the safety of
vaccines before they are used on humans.
5. How does RNA interference help in developing resistance in tobacco plant against
nematode infection?
Ans. With RNA interference technique transgenic tobacco plant is protected against nematode
Meloidegyne incognitia using Agrobacterium as the vectors.
• Nematode – specific genes were introduced into the host plant.
• It produced both sense and antisense RNA.
• There 2 RNAs form dsRNA molecules.
• It silence specific mRNA of nematode (No protein synthesis / no translation)
• Hence nematode cannot survive in tobacco plant.
6. The maturation of pro-insulin into insulin is simplified by following diagram ,observed it
and give the answer of questions that follow.

Proinsulin

A peptide
Insulin
B peptide

Free C peptide
(a) What is the main challenge for production of genetically engineered insulin?
(b) What is main disadvantage of insulin obtained from the pancreas of slaughtered cow
and pigs.
(c) Mention one difference between the human insulin and genetically engineered insulin.
Ans. (a) It is using rDNA technologies was getting insulin assembled into a mature form.
(b) There is potential of immune response in humans against the administered insulin which
is derived from animals.
(c) The insulin in human pancreas is synthesized as a pro insulin containing the C peptide,
which is removed to from mature insulin while genetically engineered insulin did not
contain C peptide was directly prepared in mature form by combining of A and B
polypeptide.
(H) CASE-STUDY BASED QUESTIONS
1. Read the following and answer the questions given below:
Insulin used for diabetes was earlier extracted from pancreas of slaughtered cattle and pigs.
Insulin from an animal source, though caused some patients to develop allergy or other types of
reactions to the foreign protein. Insulin consists of two short polypeptide chains: chain A and
chain B, that are linked together by a type of bridge. A company prepared two DNA sequences
corresponding to A and B, chains of human insulin and introduced them in plasmids of E. coli to
produce insulin chains. Chains A and B were produced separately, extracted and combined to
form human insulin.

(i) State the role of C-peptide in human insulin.


Ans. The C - peptide is an extra strecth of the peptides. That connect the A and B - polypeptide chains
of Insulin in pro - hormone. During processing to release mature and functional Insulin, this
C-peptide is removed.
(ii) Mention the chemical transformation that Proinsulin goes through to become mature
insulin.
Ans. The C-peptide present in proinsulin is removed during its maturation.
(iii) Why is proinsuling so called? How is proinsulin Different from functional insulin in
human?
Ans. Human insulin when initially synthesised in human body consists of three peptide chains - A, B
and C. The C - peptide is an extra stretch of amino acids Joining the A and B - chains. This is
called proinsulin or pro - hormone. It undergoes processing or splicing to release the functional
mature insulin that can carry out its normal functions.
(iv) Name a molecular diagnostic technique to detect the presence of a pathogen in its early
stage of infection.
Ans. (i) Polymerase chain
(ii) DNA recobinant technology
(iii) ELISA
2. Read the following and answer the questions given below:
Golden rice was engineered from normal rice by Potrykus and Beyer in the 1990s. The typical
golden colour is due to the production of β-carotene a precursor of vitamin-A. Golden rice differs
from its parental strain by the addition of three β-carotene genes. These included two genes from
daffodil plant and third from a bacterium. The incorporation of these genes allows the rice plant
to modify certain metabolic pathways in its cells to produce β-carotene.

E 207
(i) What are transgenic plants? Give some example.
Ans. Transgenic plants are plants that have been genetically engineered a breeding approach that uses
recombinant DNA Techniques to create plants with new charateristics. Example:-
Bt cotton, Golden rice, flavr Savr tomato
(ii) Genese are transferred to make golden rice by?
Ans. Genese are transferred to make golden rice by?
(iii) Some crop plants are modified genetically by manipulating their genes. How are they made
beneficial?
Ans. More tolerant to abiotic stresses; pest-resistant; reduction in post harvest losses; enhanced
nutritional value of food.
(iv) What are genetically modified organism? Name two Factors on which their behaviour
depends?
Ans. Those organisms whose gene have been altered by manipulating are called genetically modified
organism.
Two factors - (i) Proper insertion of gene of interest.
(ii) Proper harvesting of genetically modified organisms to produce desired product.
3. Read the following and answer the questions given below:
The first clinical gene therapy was given in 1990 to a 4-year old girl with adenosine deaminase
(ADA) deficiency. As a first step towards gene therapy, lymphocytes from the blood of the
patient are grown in a culture outside the body. A functional ADA cDNA (using a retroviral
vector) is then introduced into these lymphocytes, which are subsequently returned to the patient.
However, as these cells are not immortal, the patient requires periodic infusion of such
genetically engineered lymphocytes. However, if the gene isolate from marrow cells producing
ADA is introduced into cells at early embryonic stages, it could be a permanent cure.
(i) Write the name of disease which caused due to deficiency of enzyme adenosine deaminase.
Ans. ADA Deficiency.
(ii) Mention a possible permanent cure for a ADA deficiency patient.
Ans. A possible permanent cure would be gene therapy, if it is detected as early embryonic stage.
(iii) What is gene therapy? Name the first clinical case where it was used.
Ans. Gene therapy is a collection of methods that allows correction of a gene defect that has been
diagnosed in a child/embryo.
(iv) Why do children cured by enzyme replacement therapy for ADA deficiency need periodic
treatment.
Ans. As enzyme replacement therapy does not cure the disease completely, it requires periodic
treatment.
(v) What is the reason for ADA – Deficiency?
Ans. The disorder is caused due to the deletions of the gene for adnosine deaminase.
4. Read the following and answer the questions given below:
Several nematodes parasitise a wide variety of plants and animals including human beings.
A nematode Meloidegyne incognitia infects the roots of tobacco plants and causes a great
reduction in yield. A novel strategy was adopted to prevent this infestation which was based on
the process of RNA interference (RNAi). Using Agrobacterium vectors, nematode-specific genes
were introduced into the host plant. The introduction of DNA was such that it produced both
sense and anti-sense RNA in the host cells. These two RNA’s being complementary to each other
formed a double stranded (ds RNA) that initiated RNAi and thus, silenced the specific mRNA of
the nematode. The consequence was that the parasite could not survive in a transgenic host
expressing specific interfering RNA. The transgenic plant therefore got itself protected from the
parasite.

(i) State a method of cellular defence which works in all eukaryotic organisms.

Ans. RNA interference.

(ii) Which of nematode responsible for decrease in yielding of tobacco by infestation.

Ans. Meloidegyne incognitia

(iii) Explain the process of RNA interference.

Ans. RNA interference take place in all eukaryotic organisms as a method of cellular defence. It

involves silencing of a specific mRNA due to complementary ds RNA molecules that binds to

and prevent translation of the mRNA.

(iv) Given below is a single stranded DNA molecules. Frame and label its sense and antisense

RNA molecules. 5ATGGGTC3

Ans. 5ATGGGGTC3 sense 


DNA
3TACCCCAG5antisense 

5AUGGGGUC3 sense 
RNA
3UACCCCAG5 antisense 

(v) What could be the source of complementary RNA in RNAi?

Ans. It could be an infection by viruses having RNA genomes or mobile genetic elements

(transposons, that replicate via an RNA intermediate.

E 209
IMPORTANT NOTE

______________________________________________________________________

_______________________________________________________________________
_______________________________________________________________________
_______________________________________________________________________
_______________________________________________________________________
_______________________________________________________________________
_______________________________________________________________________
_______________________________________________________________________
_______________________________________________________________________
_______________________________________________________________________
_______________________________________________________________________
_______________________________________________________________________
_______________________________________________________________________
_______________________________________________________________________
_______________________________________________________________________
_______________________________________________________________________
_______________________________________________________________________
_______________________________________________________________________
_______________________________________________________________________
_______________________________________________________________________
_______________________________________________________________________
_______________________________________________________________________
_______________________________________________________________________

_______________________________________________________________________

_______________________________________________________________________
_______________________________________________________________________
210 E
CHAPTER 11
(A) NCERT QUESTIONS & SOLUTIONS
1. List the attributes that populations possess but not individuals.
Ans. The attributes that populations but not individuals possess are:
1. Population density 2. Mortality or death rate
3. Population growth 4. Natality or birth rate
5. Age distribution 6. Sex ratio
2. If a population growing exponentially doubles in size in 3 years, what is the intrinsic rate of
increase (r) of the population?
Ans. Nt = N0en
Since, Nt = 2i; N0en = 1,
e = 2.71828, t = 3
2 = (1 × 2.71829)3r
log 2 = 3rlog (2.71828)
0.3010 = 3r × 0.4343
r = 0.2310
Interinsic rate of increase of = 0.2310 × 100 = 23.10%
3. Name important defense mechanisms in plants against herbivory.
Ans. Morphological traits Chemical defenses
Thorn, Spines Poisonous cardiac glycosides
Sticky glandular hair Offensive smell
Hairy coating Tannins
Latex Bitter taste
Harboring ants Alkaloids (nicotine, caffeine,
quinine, opium)

4. An orchid plant is growing on the branch of mango tree. How do you describe this
interaction between the orchid and the mango tree?
Ans. The interaction between an orchid and the mango tree is commensalism, because orchid is
benefited by getting shelter from mango tree whereas the mango tree is neither harmed nor
benefited.
211
5. What is the ecological principle behind the biological control method of managing with pest
insects?
Ans. The ecological principle operating in the biological control method of managing with pest insect
is checking their population through predators and parasites.
6. Define population and community.
Ans. Population: Groups of individuals of same species, which can reproduce among themselves and
occupy a particular area in a given time.
Community: Groups of organisms of different species that live in common area, which are
interrelated and interdependent. It is a natural aggregation of plants and animals in the same
environment.
7. Define the following terms and give one example for each: [IMP.]
(A) Commensalism (B) Parasitism
(C) Camouflage (D) Mutualism
(E) Interspecific competition
Ans. (A) Commensalism: It is an interaction between two different species where one is benefited
and other remains unaffected.
Example : Clown fish and sea anemone. Here, the clown fish gets protection from predators
which stay away from stinging tentacles of anemone but sea anemone does not derive any
benefit from fish.
(B) Parasitism: It is an interaction between two organisms in which one is benefited and the
other is harmed, i.e. one organism lives at the cost of other organism.
Example : Cuscuta, a parasitic plant that is found growing on hedge plants, do not have
chlorophyll and thus derives its nutrition from the host.
(C) Camouflage: It is a phenomenon of blending of an organism with the surrounding due to
similar colour, marking and shape so as to avoid the predators.
Example : Common tree frog, Giant leaf insect.
(D) Mutualism: Positive inter specific interaction in which members of two different species
completely depend on each other for growth and survival.
Example : Lichen (association between algae and fungi). Here, fungi helps in absorption of
nutrients and water while the algal partner manufactures food.
(E) Interspecific competition: It is the competition among the members of different species for
limited natural resources.
Example : The Abingdon tortoise in Galapagos Islands became extinct within a decade
after goats were introduced on the Island, apparently due to the greater browsing efficiency
of the goats.
8. With the help of suitable diagram describe the logistic population growth curve. [IMP.]
Ans. Logistic growth :
The resources become limited at certain point of time, so no population can grow
exponentially.
This growth is more realistic.
Maximum number of individuals of a particular species in a habitat that is allowed by nature
is called carrying capacity (K).
When N is plotted in relation to time t, the logistic growth show sigmoid curve and also
called Verhulst-Pearl logistic growth.
It is given by the following equation:
dN K  N 
 rN  
dt  N 
Where
N = population density at time t
r = intrinsic rate of natural increase
K = carrying capacity
Population density (N)

dN = rN
dt
a
K
dN K–N
= rN
dt K
b

Time (t) 
Graph show lag phase, followed by phases of acceleration and deceleration and finally an
asymptote when population density reaches the carrying capacity.
9. Select the statement which explains best parasitism.
(a) One organism is benefited.
(b) Both the organisms are benefited.
(c) One organism is benefited, other is not affected.
(d) One organism is benefited, other is affected.
Ans. (d) One organism is benefited, other is affected.
10. List any three important characteristics of a population and explain.
Ans. The three important characteristics of a population are as follows:
(i) Population density: Population density of a species is the number of individuals of a species
per unit area or volume.
N
PD  Where – PD = Population density
S
N = Number of individuals in a region
S = Number of unit area in a region.
(ii) Birth rate: It is expressed as the number of births per 1,000 individuals of a pollution per
year.
(iii) Death rate: It is expressed as the number of deaths per 1,000 individuals of a pollution per
year.
213
(B) PREVIOUS YEAR QUESTIONS
1. Important attributes belonging to a population but not to an individual are : [CBSE 2023]
(i) Birth rate and death rate (ii) Male and female
(iii) Birth and death (iv) Sex-ratio
Select the correct option from the given options :
(a) (i) only (b) (ii) only
(c) (ii) and (iii) (d) (i) and (iv)
Ans. (d) (i) Birth rate and death rate and (iv) Sex-ratio
2. Many copepods live on the body surface of marine fish. This relationship is an example of:
[CBSE 2023]
(a) Commensalism (b) Parasitism
(c) Amensalism (d) Mutualism
Ans. (b) Parasitism
3. A Tight one - to - one relationship between many species of fig tree and certain wasps is an
example of - [CBSE 2023]
(a) Commensalism (b) Parasitism
(c) Amensalism (d) Mutualism
Ans. (d) Mutualism
4. Interaction between clown fish living among the stinging tentacles tentacles of sea anemone is an
example of - [CBSE 2023]
(a) Amensalism (b) Parasitism (c) Mutualism (d) Commensalism
Ans. (d) Commensalism
5. Assertion (A) : Decomposition process is slower if detritus is rich in lignin and cutin.
Reason (R) : Decomposition is largely an oxygen requiring process. [CBSE 2023]
(a) If both Assertion and Reason are true and Reason is the correct explanation of Assertion.
(b) If both Assertion and Reason are true but Reason is not the correct explanation of Assertion.
(c) If Assertion is true but Reason is false.
(d) If both Assertion and Reason are false.
Ans. (b) If both Assertion and Reason are true but Reason is not the correct explanation of
Assertion.
6. "Some species of insects and frogs have evolved with various specific features that help
them from being detected."
(a) Justify the statement giving reasons.
(b) Mention any two such features. [CBSE 2023]
Ans. (a) Camouflage
(b) (i) Detected easily by the predator.
(ii) Which they get blended with their surrounding.
214 E
7. (a) Observe the schematic representation given above and answer the following questions :
(i) Identify A and B.

Immigration
(I)

B Population
Density
A
(N)

Emigration
(E)
(ii) Calculate the growth rate of bacteria in a curd sample, where 1 million bacteria
increased to two million, within a period of one hour. [CBSE Term-II 2022]
OR
(b)

Post Reproductive

Reproductive

Pre Reproductive
Identify the type of pyramid given above. Write the identifying feature on the basis of
which of you identified it.
Ans. (a) (i) A- Mortality B-Natality
(ii) The per cent growth or birth per individual per hour
Final population – Initial population
= 100
Initial population
2 million  1 million
=  100
1 million
1
=  100
1
= 100 %
OR
(b) The given pyramid is expanding. The population of pre- reproductive is higher than the post
reproductive population , which make the expanding pyramid of population.
8. Define interference competition. Give one example that supports competitive exclusion
occurring in nature. [CBSE IMP. Question]
Ans. Interference competition is the feeding efficiency of one species which might be reduced due to
the interfering and inhibitory presence of the other species, even if resources (food and space) are
abundant. Examples that support competitive exclusion occurring in nature are:
E 215
 The Abingdon tortoise in galapagos Islands became extinct within a decade after goats were
introduced on the island, apparently due to the greater browsing efficiency of the goats.
 The larger and competitively superior barnacle Balanus dominates the intertidal area and
excludes the smaller barnacle Chathamalus from that zone.
9. Explain the difference between commensalism and mutualism types of interactions, with
the help of a suitable example of each. [CBSE 2020]
Ans. S. No. Commensalism Mutualism
(i) Interaction where one organism of a Interaction where organisms of
species is benefitted whereas the other two species benefit from each
is neither benefitted nor harmed. other.
(ii) For example clown fish and sea Lichens are an example
anemone. of mutualism.
(iii) Clown fish gets protection from Algae is a source of nutrition for
predators which stays away from the fungi & the fungi provides
stinging tentacles. shelter for the algae.
10. Study the age pyramids ‘A’, ‘B’ and ‘C’ of the human population given below and answer
the questions that follow: [CBSE 2020]

(a) Identify pyramids ‘B’ and ‘C’


(b) Write the basis on which the above pyramids are plotted.
Ans. (a) Pyramid B is declining and Pyramid C is stable.
(b) The above pyramids are plotted based on the number of organisms in different age groups.
 The human population are categorized into three age groups namely pre-reproductive,
reproductive and post-reproductive.
 Pre-reproductive phase includes young children, reproductive phase includes adults
capable of reproduction and post-reproductive includes people who are in their senescent or
old age and have lost their reproductive ability.
11. Mention the term used to describe a population interaction between an or chid growing on
a forest tree. [CBSE 2019]
Ans. Commensalism.
12. (a) What is “population” according to you as a biology student?
(b) “The size of a population for any species is not a static parameter.” Justify the
statement with specific reference to fluctuations in the population density of a region in
a given period of time. [CBSE 2019]
216 E
Ans. (a) Total number of organisms of a species in a particular area at a particular time.

(b) The size of a population for any species is not a static parameter because of the factors like:-

 Birth rate or Natality is the number of births during a given period.

 Death rate or Mortality is the number of deaths during a given period.

 Immigration is the number of individuals of the same species that have come into the
habitat from elsewhere during the time period under consideration.

 Emigration is the number of individuals of the population who left the habitat and gone
elsewhere during the time period under consideration.

13. Give reason as to why a weed such as Calotropis flourishes in abandoned fields.

[CBSE 2019]

Ans. It is not grazed by animals as it produces poisonous substances i.e. cardiac glycosides.

14. If in a population of size 'N' the birth rate is represented as 'b' and the death rate as 'd', the
increase or decrease in 'N' during a unit time period 't' will be : [CBSE 2019]

dN
= (b – d) × N
dt

The equation given above can also be represented as

dN
= (r × N; where r = (b – d)
dt

What does 'r' represent? Write any one significance of calculating 'r' for any population.

Ans. r = intrinsic rate of natural increase. It is an important parameter for assessing impacts of any
biotic or abiotic factor on population growth.

15. Compare, giving reasons, the J-shaped and S-shaped models of population growth of a
species. [CBSE 2018]

Ans. S. No. J shaped –growth curve S shaped –growth curve


(i) Resources are unlimited. Limited resources.
(ii) Growth is exponentially. It is logistic.
(iii) As resources are unlimited all Fittest individual will survive
individuals survive and reproduce. and reproduce.
(iv) Growth equation dN/dt = rN. Growth equation
dN/dt = rN(KN/K).

217
(C) MULTIPLE CHOICE QUESTIONS
1. Ecology is a subject which studies the interactions among organisms and __________.
(1) amongst the organism and its chemical environment.
(2) between the organism and its physical environment.
(3) amongst the organism and its habitat.
(4) between the organism and its biosphere.
Ans. (2) between the organism and its physical environment.
2. Which of the following is not an attribute of a population?
(1) species interaction (2) Sex ratio (3) Natality (4) Mortality
Ans. (1) Species interaction
3. Natality refers to -
(1) Death rate (2) Number of individuals leaving the habitat
(3) Birth rate (4) Number of individuals entering a habitat.
Ans. (3) Birth rate
4. In the exponential growth equation Nt = N0ert, e represents:-
(1) The base of number logarithms (2) The base of exponential logarithms
(3) The base of natural logarithms (4) The base of geometric logarithms
Ans. (3) The base of natural logarithms
5. The logistic population growth is expressed by equation?
Nk KN
(1) dN / dt  rN   (2) dt / dN  N r  
 N   K 
KN
(3) dN / dt  rN   (4) dN / dt  rN
 K 
 
Ans. (3)  
 
6. The formula for exponential population growth is :-
(1) dt/dN = rN (2) dN/rN = dt (3) rN/dN = dt (4) dN/dt = rN
Ans. (4) dN/dt = rN
7. Amensalism can be represented as :-
(1) Species A (–1); species B(O) (2) Species A (+); species B(+)
(3) Species A(–); species B(–) (4) Species A(+); species B(O)
Ans. (1) Species A (–1); species B(O)
8. Cuscuta is an example of :-
(1) Predation (2) Endoparasitism (3) Ecotoparasitism (4) Brood Parasitism
Ans. (3) Ecotoparasitism
9. Which one is example of ectoparasite?
(1) Ticks (2) Plasmodium (3) Ascaris (4) Tania
Ans. (1) Ticks
10. The factors which affect percolation and water holding capacity of soil are
(1) Soil composition (2) Grain size (3) Aggregation (4) All of these
Ans. (4) All of these

218 E
11. The process to maintain constancy of its internal environment is
(1) Equilibrium (2) Epimorphosis (3) Homeostasis (4) Apoptosis
Ans. (3) Homeostasis
12. Cuscuta is an example of :-
(1) Predation (2) Endoparasition
(3) Ecotoparasitism (4) Brood Parasitism
Ans. (3) Ecotoparasitism
13. If a population of 50 Paramecium present in pool increases to 150 after an hour, what would be
the growth rate of population?
(1) 50 per hour (2) 200 per hour
(3) 5 per hour (4) 10 per hour
Ans. (4) 10 per hour
14. Lichens are association of -
(1) bacteria and fungus (2) alga and bacterium
(3) fungus and alga (4) Fungas and Virus
Ans. (3) fungus and alga
15. According to population Scientists, one of the factors responsible for limiting population is the -
(1) Availability of food (2) Daily Variation of environmental temp.
(3) time required for ecological succession (4) Life span of members of the population
Ans. (1) Availability of food
16. Most successful parasite are those which do not -
(1) Grow free (2) Kill their host
(3) Reproduce sexually (4) Survive in soil
Ans. (2) Kill their host
17. A population has more young individual compared to the older individuals. what would be the
status of the population after some years :-
(1) It will decline (2) It will stabilise
(3) It will increase (4) It will first decline and then stabilise
Ans. (3) It will increase
18. Mycorrhizae are the example of :-
(1) Amensalism (2) Antibiosis
(3) Mutualism (4) Fungistasis
Ans. (3) Mutualism
19. Which type of age pyramid obtained when the population is growing.
(1) Bell - shaped age pyramid (2) Urn - shaped age pyramid
(3) Triangular age pyramid (4) Square - shaped pyramid
Ans. (3) Triangular age pyramid
20. Cyptically - coloured (camoflaged) is a technique through which prey can.
(1) Feed abundantly (2) lessen the impact of predator
(3) Increase their number (4) Increase their reproductive fitness.
Ans. (2) lessen the impact of predator
219
(D) ASSERTION & REASON QUESTIONS

 Directions: In the following questions, a statement of assertion is followed by a statement of


reason. Mark the correct choice as:
(1) If both Assertion and Reason are true and Reason is the correct explanation of Assertion.
(2) If both Assertion and Reason are true but Reason is not the correct explanation of Assertion.
(3) If Assertion is true but Reason is false.
(4) If both Assertion and Reason are false.
1. Assertion : Cattle or goat never graze on Calotropis.
Reason : Thorns are present as a morphological means of defence.
Ans. (3)
2. Assertion : Leaf butter and stick insect show mimicry to dodge their ememies.
Reason : Mimicry is a method to acquire body colour blending with the surroundings.
Ans. (1)
3. Assertion : A stable population is depicted by bell shaped age pyramid.
Reason : The proportion of individuals in reproductive age group in higher than those in per
reproductive age group.
Ans. (3)
4. Assertion : Plant animal interaction do not generally involve co - evolution of the mutulist
organisms.
Reason : Evolution of plants and animals go side by side.
Ans. (4)
5. Assertion: Desert lizards have physiological ability to main body temperature.
Reason: Desert lizards are warm blooded animals.
Ans. (4)
6. Assertion: Immigration contributes to decrease in population.
Reason: Emigration contributes to increase in population.
Ans (4)
7. Assertion : A triangular population pyramid depicts population size is stable.
Reason : Epiphytes growing on branches of the tree exhibit commensalism.
Ans. (1)
8. Assertion : Emigration increases the size of local population.
Reason : During emigration individuals from other population are added in the given population.
Ans. (4)
9. Assertion: Predation and parasitism are considered to be negative interactions.
Reason: Predators and parasitism limit the population of their host species.
Ans. (1)
10. Assertion : Carrying capacity limits the size of population.
Reason : Density independent population exceeds the carrying capacity.
Ans. (2)
220 E
(E) VERY SHORT ANSWER TYPE QUESTIONS
1. Name the interaction that exists between cuscuta and shoe-flower plant.
Ans. Parasitism.

2. Name the interaction between a whale and the barnacles growing on its back.

Ans. Commensalism

3. State the type of interaction that exists between ticks and dogs.

Ans. Ecto (Parasitism).

4. Name the interaction that exists between sucker fish and shark.

Ans. Commensalism.

5. Name the type of interaction seen between fig and wasps.

Ans. Mutualism.

6. Name two intermediate hosts which the human liver fluke depends on to complete its life

cycle so as to facilitate parasitisation of its primary host.

Ans. Snail and Fish.

7. How are closely related species of warblers able to co-exist in a competitive environment ?

Ans. They can co–exist due to behavioural differences in their foraging activities.

8. Koel is clever enough to lay eggs in a crow’s nest. Write the reason for this peculiar

behaviour. Name the type of interaction

Ans. So that the crow can incubate the Koel’s eggs.

Interaction – Brood parasitism.

9. What does J-shaped growth curve of a populations indicates?

Ans. The J-shaped growth curve indicates the minimum or absence of environmental resistance.

10. Define homeostasis.

Ans. The process to maintain the constancy of internal environment of the body, despite varying

external environmental conditions is called homeostasis.

E 221
(F) SHORT ANSWER TYPE QUESTIONS
1. Name two basic types of competition found amongst organisms. Which one of them is more
intense and why?
Ans. The two basic types of competitions are :
(i) Interspecific competition
(ii) Intraspecific competition
The intraspecific competition is more intense because the requirement of the individual of the
species are similar.
2. (a) Explain "birth rate" in a population by taking a suitable example.
(b) Write the other two characteristics which only a population shows but an individual
cannot.
Ans. (a) Birth rate is expressed as the number of births per 1,000 individuals of a population per year.
For example, in a pond there were 200 frogs and 40 more were born in a year. Then, the birth
rate of the population will be 40/200 = 0.5 frogs per year.
(b) Sex ratio, age distribution, population density, population growth. (Any two)
3. Explain Verhulst - Pearl Logistic Growth of a population.
Ans. According to Verhulst-Pearl Logistic growth, a population growing in a habitat with limited
resources initially shows a lag phase, followed by phases of acceleration and deceleration and
finally an asymptote when the population density reaches the carrying capacity. It is given by the
following equation :
dN K – N
 rN  
dt  K 

where, N = population density at time t, r = intrinsic rate of natural increase, K = carrying


capacity.
4. Co - evolution is a spectacular example of mutualism between an animal and a plant.
Describe co - evolution with the help of an example.
OR
Describe the mutual relationship between fig tree and wasp and comment on the
phenomenon that operates in their relationship.
Ans. Co - evolution can be observed in Fig (plant) and wasp (animal). The female wasp uses the fruit
for oviposition or egg laying. It also uses developing seeds within the fruit for nourshing its
larvae. The wasp in turn pollinates the fig inflorescence. The given Fig species can be pollinated
by its 'partner wasp species and no other species.
5. Why do algae and fungi shift to sexual mode of reproduction just before the onset of
adverse conditions?
Ans. For survival during unfavourable conditions / Fusion of gametes helps to pool their resources for
survival (hunger theory of sex) / Zygote develops a thick wall that is resistant to dessication and
damage, undergoes a period of rest before germination.
222 E
6. What is mutualism? Mention any two examples where the organisms involved are

commercially exploited in agriculture.


Ans. Interaction between two species in which both are benefited is called mutualism.
(i) Rhizobium in the roots (nodules) of legumes.

(ii) Mycorrhiza  Glomus living with the roots of higher plants.

7. Mention any two activities of animals, which get cues from diurnal and seasonal variations
in light intensity.

Ans. The two activities of animals which get cues from diurnal and seasonal variations in light
intensity are:
(i) Timing their foraging (ii) Migratory activities (iii) Reproduction (any two)

8. Egrets are often seen along the with grazing cattle. How do you refer to this interaction?
Give a reason for this association.
Ans. The interaction between them can be referred to as commensalism. Egrets always forage close to
where the cattle are grazing because the cattle, as they move stir up and flush out insects from the
vegetation which otherwise might be difficult for the egrets to find and catch.

9. Explain brood parasitism with the help of an example.

Ans. Koel is a parasitic bird (which has lost the instinct to make its own nest to lay eggs), has evolved
the tenchnique of laying eggs in the nest of a crow.
Its eggs bear resemblances to those of crow.

10. How does the Mediterranean orchid Ophrys ensure its pollination by bees?
Ans. The petalss of the Ophrys resembles the female of a bee species in size, colour and odour, etc
Male bee mistakes the Ophrys for female bee and tries to copulate. Few pollen grains adhered
with the body of the male bee fall over stigma of the flower thereby leading to pollination.

11. (a) How is Cuscuta adapted to be a parasitic plant?


(b) Why do cattle avoid browsing on Calotropis plants.? Explain.

Ans. (a) Cuscata has lost its chlorophyll and leaves during evolution and thus it derives its nutrition
from host plant, thus, it is a parasititc plant.
(b) Cattle avoid browsing on Calotropis plants beacuase it produces poisonous cardiac
glycosides.

E 223
(G) LONG ANSWER TYPE QUESTIONS
1. (i) What is an age-pyramid?
(ii) Explain with the help of figures the three different types of age pyramids represented
by a human population.
Ans. (i) If the age distribution (per cent individuals of a given age or age group) is plotted for the
population, the resulting structure is called the age pyramid.
(ii) Expanding: Pre reproductive population is greater than reproductive or post reproductive
population / growing with maximum no. of individuals in pre reproductive phase and least
no. in post reproductive phase.
Stable: Pre-reproductive and reproductive population are almost similar / ideal for population /
maintains balanced continuity / no. of individuals in reproductive phase is almost same, less no.
of individuals in post reproductive phase.
Declining: Pre-reproductive population is less than reproductive population / less no. of
individuals in pre- reproductive phase than reproductive phase.
2. Differentiate between an 'Expanding age pyramid' and a 'Stable age pyramid'. Substantiate
your answer with diagrams.
Ans. Expanding age pyramid- Population of pre-reproductive age is greater than population of
reproductive age.
Stable age pyramid- Population of pre-reproductive age equals to population of reproductive
age.

3. Study the graph given below and answer the questions that follow:
(i) The curve 'b' is described by the following equation:
dN K – N 
= rN 
dt  N 
What does 'K' stand for in this equation? Mention its significance.

224 E
(ii) Which one of the two curves is considered a more realistic one for most of the

animal populations?

(iii) Which curve would depict the population of a species of deer if there are no predators

in the habitat? Why is it so?

Ans. (i) K-Carrying capacity

Any habitat in nature having enough resources can support a maximum number of

organisms, beyond which no further growth is possible.

(ii) Curve ‘b’

(iii) Curve ‘a’, deer population will reach enormous numbers.

4. (i) Why predators in nature are 'prudent'?

(ii) What is camouflaged ? Give examples.

(iii) How Monarch butterfly protect itself from predation?

(iv) How Acacia, Cactus plants are protect itself from predation

(v) Why you never see any cattle or goats browsing on Calotropis plant.

Ans. (i) If a predator is too efficient and overexploits its prey, then the prey might become extinct

and following it, the predator will also become extinct for lack of food. This is the reason

behind ‘prudent’ nature of predator.

(ii) It is a phenomenon of blending of an organism with the surrounding due to similar colour,

marking and shape called as camouflaged. Some species of insects and frogs are

cryptically-coloured (camouflaged) to avoid being detected easily by the predator.

(iii) The Monarch butterfly is highly distasteful to its predator (bird) because of a special

chemical present in its body. Interestingly, the butterfly acquires this chemical during its

caterpillar stage by feeding on a poisonous weed.

(iv) Acacia, Cactus plants have a thorn a common morphological means of defense.

(v) The Calotropis plant produces highly poisonous cardiac glycosides chemicals that make

the herbivore sick when they are eaten, inhibit feeding or digestion, disrupt its reproduction

or even kill it, the weed Calotropis growing in abandoned fields. that is the reason we

never see any cattle or goats browsing on this plant.

E 225
(H) CASE BASED TYPE QUESTIONS
1. Read the following and answer the questions given below :
Can you think of any natural habitat on earth that is inhabited just by a single species? There is
no such habitat and such a situation is even inconceivable. For any species, the minimal
equirement is one more species on which it can feed. Even a plant species, which makes its own
food, cannot survive alone; it needs soil microbes to break down the organic matter in soil and
return the inorganic nutrients for absorption. And then, how will the plant manage pollination
without an animal agent? It is obvious that in nature, animals, plants and microbes do not and
cannot live in isolation but interact in various ways to form a biological community. Even in
minimal communities, many interactive linkages exist, although all may not be readily apparent.
Interspecific interactions arise from the interaction of populations of two different species. They
could be beneficial, detrimental or neutral (neither harm nor benefit) to one of the species or
both. Assigning a ‘+’ sign for beneficial interaction, ‘-’ sign for detrimental and 0 for neutral
interaction, let us look at all the possible outcomes of interspecific interactions.
(i) What is mutualism?
Ans. (i) Mutualism is an interaction that confers benefits to both the interacting species.
(ii) Differentiate between the following interspecific interaction in a population.
"Mutualism and competition"
Ans.
Mutualism Competition
It benifits both the Both the interacting
interacting species. species lose.
(iii) Name the interspecific interaction in which one is deterimental while other is neutral.
Ans. Amensalism
(iv) What is Interspecific interaction?
Ans. The interaction populations of two different species.
2. Study the three different age pyramids for human population given below and answer the
questions that follow:

(i) Write the names given to each of these age pyramids.


Ans. (i) A - Expanding pyramid B - Stable pyramid
C - Declining pyramid
(ii) Mention the one which is ideal for human population and why.
Ans. (ii) Stable pyramid is ideal for human population because it maintains the stability in all
population phases.
(iii) What would be the growth rate pattern when the resources are unlimited?
Ans. Exponential.
3. Read the following and answer the questions given below:

In many species of fig trees, there is a tight one-to-one relationship with the pollinator species of
wasp. It means that a given fig species can be pollinated only by its ‘partner’ wasp species and
no other species.

The female wasp uses the fruit not only as an oviposition (egg-laying) site but uses the
developing seeds within the fruit for nourishing its larvae. The wasp pollinates the fig
inflorescence while searching for suitable egg-laying sites. In return for the favour of pollination
the fig offers the wasp some of its developing seeds, as food for the developing wasp larvae.

(i) What is Mycorrhiza?


Ans. Mycorrhiza is a mutualistic association between the fungus and the roots of higher plants. The
fungi get shelter and food and lieu of that helps the plant in absorption of phosphorus from the
soil. It also provides resistance to the pant from root-borne pathogens.
(ii) Give a suitable example for commensalism.
Ans. Egrets ride on the back of cattle. When cattle forage for grass, insects fly away abruptly. This
gives the egrets a chance to feed on those insects. In this relationship, cattle get no benefit from
egrets.
(iii) What is competition?
Ans. Competition is relationship in which both species members are at loss.
(iv) Name the bind of interaction present between the following:-
(i) Nodulated roots & rhizobium
(ii) Orchids & Mango tree
Ans. (i) Nodulated roots & rhizobium :- Mutualism
(ii) Orchids & Mango tree :- Commensalism.
(v) Egrets are often seen along with grazing cattle. How do you refer to this interaction?
Ans. Commensalism
227
IMPORTANT NOTE

______________________________________________________________________

_______________________________________________________________________
_______________________________________________________________________
_______________________________________________________________________
_______________________________________________________________________
_______________________________________________________________________
_______________________________________________________________________
_______________________________________________________________________
_______________________________________________________________________
_______________________________________________________________________
_______________________________________________________________________
_______________________________________________________________________
_______________________________________________________________________
_______________________________________________________________________
_______________________________________________________________________
_______________________________________________________________________
_______________________________________________________________________
_______________________________________________________________________
_______________________________________________________________________
_______________________________________________________________________
_______________________________________________________________________
_______________________________________________________________________
_______________________________________________________________________

_______________________________________________________________________

_______________________________________________________________________
_______________________________________________________________________
228 E
CHAPTER 12
(A) NCERT QUESTIONS & SOLUTIONS
1. Fill in the blanks.
(a) Plants are called as ____________________ because they fix carbon dioxide.
(b) In an ecosystem dominated by trees the pyramid (of numbers) is ___________ type.
(c) In aquatic ecosystems, the limiting factor for the productivity is__________________.
(d) Common detritivores in our ecosystem are__________________.
(e) The major reservoir of carbon on earth is__________________.
Ans. (a) Producers (b) Inverted (c) Light
(d) Earthworm, ants and mites (e) Oceans (71% dissolved carbon)
2. Which one of the following has the largest population in a food chain?
(a) Producers (b) Primary consumers
(c) Secondary consumers (d) Decomposers
Ans. (a) Producers (decomposers can be maximum but they are excluded from the food chain)
3. The second trophic level in a lake is
(a) Phytoplankton (b) Zooplankton (c) Benthos (d) Fishes
Ans. (b) Zooplankton
4. Secondary producers are
(a) Herbivores (b) Producers (c) Carnivores (d) None of the above
Ans. (a) Herbivores
5. What is the percentage of photosynthetically active radiation (PAR) in the incident solar
radiation?
(a) 100% (b) 50% (c) 1-5% (d) 2-10%
Ans. (b) 50%
6. Distinguish between [IMP.]
(a) Grazing food chain and detritus food chain
(b) Production and decomposition
(c) Litter and detritus
(d) Upright and inverted pyramid
(e) Food chain and food web
(f) Primary and secondary productivity
E 229
Ans. (a) Grazing food chain and detritus food chain -
S.No. Grazing food chain Detritus food chain
(i) It start with green plants called It begins with dead organic matter and
producers as first trophic level. decomposers called saprophytes as first trophic
level.
(ii) A much less fraction of energy A much large fraction of energy flows through
flows through this type of food this type of food chain.
chain.
(iii) Energy for food chain comes from Energy for the food chain comes from organic
sun. remains or detrirus
(b) Production and decomposition
S.No. Production Decomposition
(i) It is the process of formation of It is the process of degradation of waste
fresh biomass from inorganic matter biomass into its constituents by decomposers.
by producers using sunlight.
(ii) It is anabolic process which traps Its catabolic process and release energy.
energy.
(c) Litter and detritus
S.No. Litter Detritus
(i) It is made of dried fallen plant The base bas comprises producers in smallest
matter. numbers.
(ii) It is found above the ground. It is found both above and below the ground.
(d) Upright and inverted pyramid
S.No. Upright pyramid Inverted pyramid
(i) The base bar comprises of The base bar comprises producers in smallest
producers in large numbers. numbers.
(ii) The numbers of consumers decrease The numbers of consumers increase and
and become least in top consumer become largest in top consumers level.
level. e.g. Pyramid of number and biomass may be
e.g. Pyramid of energy is always inverted.
upright.

230 E
(e) Food chain and food web
S.No. Food chain Food web
(i) The transfer of energy from The numbers of food chain are inter connected
producers to top consumers through with each other forming a web like pattern is
a series of organisms is called as called food web.
food chain.
(ii) One organism hold only one One organism can hold more than one position
position of trophic level. of trophic level.
(iii) The flow of energy can be easily The flow of energy is very difficult to calculate.
calculated.
(iv) It is always straight and proceed in Instead of straight line it is a series of branching
a progressive straight line. lines.
(f) Primary and secondary productivity
S.No. Primary productivity Secondary productivity

(i) It is the rate at which organic matter It is the rate at which organic matter is build up
is build up by produces. by consumers.
(ii) It is the result of synthesis of fresh It is the result of synthesis of organic matter
organic matter from inorganic from plant organic matter.
materials.
7. Describe the components of an ecosystem.
Ans. An ecosystem consists of two types of components, ie., biotic or living and abiotic or non-
living. There are three main types of biotic components on the basis of mode of obtaining their
food- producers, consumers and decomposers.
(a) Producers (autotrophs): They are photosynthetic or autotrophic plants that synthesise their
own organic food from inorganic raw materials with the help of solar radiations. Common
producers are algae, plants and photosynthetic bacteria. Phytoplanktons are the producers of
aquatic ecosystems.
(b) Consumers (heterotrophs): They are animals which feed on other organisms or producers for
obtaining their nourishment. Common consumers are deer, goat, etc.
(c) Decomposers: They are saprotrophs which obtain nourishment from organic remains. They
release digestive enzymes to digest the organic matter. Common decomposers are detritivores,
e.g. earthworm.
Abiotic components of ecosystem consists of non-living substances and factors which are as
follow:
(a) Temperature (b) Light (c) Wind (d) Humidity
(e) Precipitation (f) Water
E 231
8. Define ecological pyramids and describe with examples, pyramids of number and
biomass.
Ans. The relationship between producers and consumers at different trophic levels in an ecosystem
can be graphically represented in the form of a pyramid called ecological pyramid.
Structure: The base always represents the producers or the first trophic level and the apex
represents top level consumer or the last trophic level.
Ecological pyramids are of three types :
(i) Pyramid of number (ii) Pyramid of biomass (iii) Pyramid of Energy
(i) Pyramid of number: The relationship between producers and consumers in an ecosystem
can be represented in the form of a pyramid in terms of number of organisms at different
trophic levels called pyramid of number.
Note: It is inverted, when you count number of insects feeding on a big tree.

Trophic level Number of individuals

TC (Tertiary consumer) 3

SC (Secondary consumer) 3,54,000


PC (Primary consumer) 708,000
PP (Primary producer) 5,842,000
Pyramid of numbers in a grassland ecosystem
(ii) Pyramid of biomass: The relationship between producers and consumers in an ecosystem
can be represented in the form of a pyramid in terms of biomass called pyramid of biomass.
It can be
(a) Upright, e.g. in case of grassland ecosystem;
(b) Inverted, e.g. in case of pond ecosystem as biomass of fishes for exceeds that of
phytoplankton

Trophic level Dry weight (kg m–2)

TC 1.5

SC 11 PC 21
PC 37
PP 4
PP 809
(a) Pyramid of biomass shows a sharp decrease in (b) Inverted pyramids of biomass of
biomassatathigher
in biomass highertrophic
trophiclevel
level pondpond
ecosystem
ecosystem

232 E
9. What is primary productivity? Give brief description of factors that affect primary
productivity. [IMP.]

Ans. Primary productivity is the rate of synthesis of biomass by producers, per unit time, per unit
area through the process of photosynthesis.

The factors affecting primary productivity are following :-

  Plant species inhabiting a particular area.

  Environmental factors:

(i) Sunlight : The sunlight directly regulates the primary productivity because the plants
perform photosynthesis with the help of sunlight. As tropical region receives maximum
sunlight so it exhibits higher productivity.

(ii) Temperature: Temperature regulates the activity of enzyme. So, optimum temperature is
required for proper functioning of enzyme.

(iii) Moisture: Rain (humidity) is required for higher primary productivity. Deserts have the
lowest primary productivity as the soil is deficient in moisture.
10. Define decomposition and describe the processes and products of decomposition.

Ans. The process of breaking down complex organic matter into inorganic substances like CO, water
and nutrients is called decomposition. The raw materials for decomposition including dead
plant remains like leaves, bark, flowers, and animal remains and their faecal matter are called
detritus.
Steps of Decomposition

(1) Fragmentation: The process of breaking down of detritus into smaller particle is called
fragmentation, eg., as done by earthworm (farmer's friend).

(2) Leaching: The process by which water-soluble inorganic nutrients go down into the soil
horizon and get precipitated as unavailable salts is called leaching.

(3) Catabolism: The enzymatic process by which bacterial and fungal enzymes degrade detritus to
simpler inorganic substances is called catabolism.

(4) Humification: The process of accumulation of a dark coloured amorphous substance, called
humus, that is highly resistant to microbial action and undergoes decomposition at an extremely
slow rate is called humification. Humus being colloidal is reservoir of nutrients.

(5) Mineralisation: The process by which humus is further degraded by some microbes to release
inorganic nutrients is called mineralisation.

E 233
A tree grows in the soil
Some are eaten by insects
and other animals.Nutrients
and energy enter food web.

A green leaf falls to the


ground

Leaves partially consumed


by decomposers such as fungi
and bacteria. They begin to
lose form and become litter. Some nutrients leach into
soil by chemical action.

Further
decomposition by
earthworms,bacteria,
Soil, mites, fungi, etc.
Organic rich soil

Diagrammatic representation of decomposition cycle in a terrestrial ecosystem


11. Give an account of energy flow in an ecosystem.
Ans. The sun is the only source of energy for all ecosystems on earth. Of the total incident solar
radiation, only 50% of it is photosynthetic active radiation (PAR).
 Plant capture only 2-10 % of the PAR and this small amount of energy sustains the entire
living world. So, there is unidirectional flow of energy from the sun to producers and then
to consumers
 The energy is transferred in an ecosystem, in the form of food which is degraded and lose
major part of food energy as heat during metabolic activities and only a very small fraction
becomes stored as biomass
 This is correlated to second law of thermodynamics, i.e., ecosystems need a constant
supply of energy to synthesize molecules they require, to counteract universal tendency
towards increasing disorderliness.
 The green plants in the ecosystem which can trap solar energy to convert it into chemical
bond energy are called producers.
 All the animals that depend for food on plants are called consumers or heterotrophs.
 Consumers are divided into the following categories:
(i) Primary consumers: Animals which feed directly on plants, i.e., herbivores.
(ii) Secondary consumers: Consumers that feed on primary consumers, i.e.. carnivores.
(iii) Tertiary consumers: Consumers that feed on secondary consumers
 Lindeman's 10 per cent law: At each step of food chain, when food energy is transferred
from one trophic level to the next higher trophic level, only about 10 per cent of energy is
passed on to the next trophic level. This is known as Lindeman's 10 per cent law given by
Lindeman in 1942.
234 E
(B) PREVIOUS YEAR QUESTIONS
1. The primary productivity in an ecosystem is expressed as : [CBSE 2023]
(a) gm–2 yr–1 (b) gm–2 yr (c) K cal m–2 yr–1 (d) K cal m–2
Ans. (a) gm–2 yr–1 and (c) K cal m–2 yr–1
2. Assertion (A) : Decomposition process is slower if detritus is rich in lignin and cutin.
Reason (R) : Decomposition is làrgely an oxygen requiring process. [CBSE 2023]
Ans. (b) Both Assertion (A) and Reason (R) are true but Reason (R) is NOT the correct
explanation of Assertion (A).
3. Mention four significant services that a healthy forest ecosystem provide. [CBSE 2019]
Ans. Purify air, Production of O2, Purify water, Mitigate droughts and floods, Nutrients cycling,
generate fertile soils, Provide wildlife habitat, Maintain biodiversity, Pollinate crops, Provide
site for carbon storage, Provide aesthetic - cultural - spiritual values, economic benefits, from
nature food, industrial products, products of medicinal importance.
5. Give the answer of following questions.
(a) What is a trophic level in an ecosystem ? What is ‘standing crop’ with reference to it ?
(b) Explain the role of the ‘first trophic level’ in an ecosystem.
(c) How is the detritus food chain connected with the grazing food chain in a natural
ecosystem ? [CBSE 2018]
Ans. (a) Specific place of an organisms in a food chain, mass of living material (biomass) at each
trophic level at a particular time.
(b) First trophic level has producers / autotrophs, which trap solar energy to produce food
(photosynthesis).
(c) Organisms of the Detritus food chain (DFC) are the prey to the Grazing food chain (GFC)
organism, the dead remains of GFC are decomposed into simple inorganic materials which
are absorbed by DFC organisms.
6. Give the answer of following questions. [CBSE 2018, 2019]
(a) The pyramid of energy is always upright.’ Explain.
(b) Explain with the help of labelled diagrams, the difference between an upright
pyramid of biomass and an inverted pyramid of biomass.
Ans. (a) As the energy always flows from a particular trophic level to the next trophic level, some
energy is always lost as heat at each step.
235
(b) Trophic level Dry weight (kg m–2)

TC 1.5

SC 11
PC 21
PC 37
PP 4
PP 809

(i) (ii)
Fig. (i) Pyramid of biomass shows a sharp decrease in biomass at higher
trophic level in most of the ecosystem.
(ii) Inverted Pyramid of biomass in sea ecosystem

Upright pyramid of biomass Inverted pyramid of biomass


Biomass of producer is more than In a sea consumer i .e fish has more
the consumer of successive trophic biomass than the producers i.e
level in most of the ecosystem phytoplanktons.
7. Describe the inter-relationship between productivity, gross primary productivity and net
productivity. [CBSE 2017]
Ans. Productivity is the rate of biomass production per unit area over a period of time. Gross primary
productivity is the rate of production of organic matter during photosynthesis in an ecosystem.
Net productivity is the gross primary productivity minus respiration losses (R).
Net Productivity = GPP - R

236 E
(C) MULTIPLE CHOICE QUESTIONS
1. Decomposers like fungi and bacteria are
(a) Autotrophs (b) Heterotrophs (c) Saprotrophs (d) chemo-autotrophs
(1) (a) and (c) (2) (a) and (d) (3) (b) and (c) (4) (a) and (b)
Ans. (3) (b) and (c)
2. The process of mineralisation by microarganisms helps in the release of
(1) inorganic nutrients from humus
(2) both organic and inorganic nutrients from detritus
(3) organic mutrients from humus
(4) inorganic nutrients from detritus and formation of humus.
Ans. (1) inorganic nutrients from humus
3. Productivity is the rate of production of biomass expressed in terms of
(a) (kcal m–3) yr–1
(b) g–2yr–1
(c) g–1yr–1
(d) (kcal m–2) yr–1
(1) (b) (2) (c) (3) (b) and (d) (4) (a) and (c)
Ans. (3) (b) and (d)
4. An inverted pyramid of biomass can be found in which ecosystem?
(1) Forest (2) Marine (3) Grass land (4) Tundra
Ans. (2) Marine
5. What is the difference between a community or group of communities and an ecosystem?
(1) A community and the abiotic environment comprise an ecosystem.
(2) An ecosystem is a type to community.
(3) A biome includes only the plant community or commnunities present in an environment.
(4) An ecosystem includes only the abiotic aspects of a particular environment.
Ans. (1) A community and the abiotic environment comprise an ecosystem.
6. Which of the following ecosystems is most productive in terms of net primary production?
(1) Deserts (2) Tropical rain forests
(3) Oceans (4) Estuaries
Ans. (2) Tropical rain forests
7. Pyramid of numbers is
(1) always upright
(2) always inverted
(3) either upright or inverted
(4) neither upright nor inverted
Ans. (3) either upright or inverted

E 237
8. Of the total amount of energy that passes from one trophic level to another, about 10% is
(1) respired and becomes heat (2) passed out as faces or urine
(3) stored as body tissue (4) recycled to autotrophs
Ans. (3) stored as body tissue
9. During the process of ecological succession the changes that take place in communities are
(1) orderly and sequential (2) random
(3) very quick (4) not influenced by the physical environment.
Ans. (1) orderly and sequential
10. Climax community is in a state of
(1) non-equilibrium (2) equilibrium (3) disorder (4) constant change
Ans. (2) equilibrium
11. Trophic levels are formed by -
(1) Only plants (2) Only Carnivores
(3) Only animals (4) Organisms linked in food chain
Ans. (4) Organisms linked in food chain
12. Which one is not a functional aspect of ecosystem?
(1) Energy flow (2) Productivity (3) Decomposition (4) Stratification
Ans. (4) Stratification
13. A pond is a :-
(1) Biome (2) Natural ecosystem
(3) Artificial ecosystem (4) Community of plants and animals
Ans. (2) Natural ecosystem
14. If the carbon atoms fixed by producers already have passed through three species, the trophic
level of the last species would be:
(1) Scavenger (2) tertiary producer
(3) tertiary consumer (4) secondary consumer
Ans. (3) tertiary consumer
15. Humans benefit from ecosystems because ecosystems provide
(1) bullers from natural disasters such as floods.
(2) maintenance of a clean water supply
(3) climate moderation.
(4) All of the above
Ans. (3) climate moderation
16. Which of the following type of ecosystem is expected in an area where evaporation exceeds
precipitation, and mean annual rainfall is below 100 mm?

(1) Grassland (2) Shrubby forest (3) Desert (4) Mangrove

Ans. (3) Desert

17. The zone at the edge of a lake or ocean which is alternatively exposed to air and immersed in
water is called:

(1) pelagic zone

(2) benthic zone

(3) lentic one

(4) littoral zone

Ans. (4) littoral zone

18. The source of energy in an ecosystem.

(1) Sunlight

(2) DNA

(3) ATP

(4) RNA

Ans. (1) Sunlight

19. Which of the following is an ecosystem service provided by a natural ecosystem?

(1) Cycling of nutrients

(2) Prevention of soil erosion

(3) Pollutant absorption and reduction of the threat of global warming

(4) All of the above

Ans. (4) All of the above

20. Which of the following ecosystem is most productive in terms of net primary productiion?

(1) Deserts

(2) Tropical rain forests

(3) Oceans

(4) Estuaries

Ans. (2) Tropical rain forests

E 239

(D) ASSERTION – REASON QUESTIONS
 Directions: In the following questions, a statement of assertion is followed by a statement of
reason. Mark the correct choice as:
(1) If both Assertion and Reason are true and Reason is the correct explanation of Assertion.
(2) If both Assertion and Reason are true but Reason is not the correct explanation of Assertion.
(3) If Assertion is true but Reason is false.
(4) If both Assertion and Reason are false.

1. Assertion : Food web consists of several food chains.


Reason : Food web decreases the stability of an ecosystem.
Ans. (3)
2. Assertion : The conversion of productivity at next trophic level is 10%.
Reason : Energy is lost in the respiration process.
Ans . (1)
3. Assertion : Biomes are the major ecosystem of the world.
Reason : Tundra is an example of biome.
Ans. (2)
4. Assertion : Chaparral is also called "shrub forest".
Reason : Trees are totally absent in chaparral.
Ans. (3)
5. Assertion : A network of food chains existing together in an ecosystem is known as food web.
Reason : An animal like kite cannot be a part of a food web.
Ans. (3)
6. Assertion : Pyramid of energy may be upright or inverted.
Reason: Only 20% of energy goes to next trophic level.
Ans. (4)
7. Assertion: The pyramid of energy is always upright.
Reason: Maximum number of autotrophs is present in the pyramid of energy.
Ans. (3)
8. Assertion : The example of primary consumers includes herbivores.
Reason : The driving force of any ecosystem is its producers.
Ans. (2)
9. Assertion : A biotic community has higher position than population in ecological hierarchy.
Reason : Population of similar individuals remains isolated in the community.
Ans (3)
10. Assertion : Net primary productivity is gross primary productivity minus respiration.
Reason : Secondary productivity is produced by heterotrops.
Ans. (2)
(E) VERY SHORT ANSWER QUESTIONS
1. What is secondary productivity?

Ans. Secondary productivity is defined as the rate of formation of new organic matter by consumers.

2. State what does 'standing crop' of a trophic level represent.

Ans. Standing crop represents the mass of living material (biomass) at a particular time.

3. List any two ways of measuring the standing crop of a trophic level.

Ans. Standing crop is measured as the biomass or the number of plant in a unit area.

4. Differentiate between standing state and standing crop in an ecosystem.

Ans. In an ecosystem, standing crop is the mass of living material in each trophic level at a particular

time. Whereas standing state refers to the amount of nutrients in the soil at any given time.

5. Why is a food web formed in nature?

Ans. Many organisms occupy positions in different food chains and several food chains become

interconnected to form a food web.

6. Name an omnivore which occurs in both grazing food chain and the decomposer food

chain.

Ans. Sparrow/crow.

7. Justify the pitcher plant as a producer.

Ans. Pitcher plant is chlorophyllous and is thus capable of photosynthesis and act as producer.

8. Name any two organisms which occupy more than one trophic level in an ecosystem?

Ans. Man and sparrow

9. What is common to earthworm, mushroom, soil mites and dung beetle in a ecosystem?

Ans. They are all detritivores, i.e., decomposing organisms which feed on dead remains of plants and

animals.

10. Standing crop and biomass are related to each other, how?

Ans. The standing crop is measured as the mass of living organism or the number of plants in a unit

area. The biomass of a species is expressed in terms of fresh or dry wright.

E 241
(F) SHORT ANSWER QUESTIONS
1. “In a food chain, a trophic level represent a functional level, not a species.” Explain.
Ans. A given species may occupy more than one trophic level in the same ecosystem (in different
food chai) at the given time. If the function of the mode of nutrition of species changes, its
position shall changes in the trophic level .The same species can be at primary consumer level
in one food chain and at secondary consumer level in another food chain in the same ecosystem
at the given time.
2. Construct a pyramid of biomass starting with phy phytoplanktons. Lebel three the
pyramid upright or inverted. Why?
T3 Fishes
Ans.
T2 Zooplanktons
T1 Phytoplanktons

The pyramid is inverted because the biomass of fishes is much than that of the phytoplanktons.
3. Can you work out at how many trophic levels human beings functions in a food chain.
Or
“It is possible that a species may occupy more than one trophic level in the same
ecosystem at the same time” explain with the help of one example.
Ans. Human beings can be placed at three trophic level in a food chain, if human being is vegetarian
than included in primary consumer (herbivore) while in case of non-vegetarian this is secondary
consumer (carnivore) ,in case both it is omnivore and placed at top consumer.
4. Name the type of food chain responsible for the flow of large fraction of energy in an
aquatic and a terrestrial ecosystem respectively. Mention one difference between the two
food chains
Ans. Aquatic ecosystem- Grazing food chain / GFC. Terrestrial ecosystem - Detritus food chain /
DFC
Difference: GFC begins with phytoplankton / producers whereas DFC begins with dead
organism/detritus.
5. Which is more stable from food chain or food web. Justify your answer.
Ans. The food web is more stable than food chain. In food web more than one species are present at
different tropic level while in food chain only one species is present at each tropic level. In food
chain if a species is eliminated caused ecological imbalance but in food web no such type of
imbalance can possible because the space of eliminated species fulfill by another species.
6. Justify the statement, "Pyramid of energy is always upright, and can never be inverted."
Ans. Energy flow is always in one direction / Energy is always more at the producer level / Energy is
lost at each successive trophic level in the form of heat.
7. "Decomposition is an oxygen requiring process "comment.
Ans. Detritus is rich in nitrogen and sugars for oxidation of nitrogen and sugars oxygen is required
by a class of aerobic microbes.
242 E
(G) LONG ANSWER QUESTIONS
1. Write the relationship between productivity, gross primary productivity, net primary

productivity and secondary productivity.

Ans. The rate of production of biomass is productivity, whereas rate of formation of organic matter

during photosynthesis is Gross primary productivity.

Gross primary productivity minus respiratory losses (NPP = GPP-R) is net primary

productivity, formation of new organic matter by consumers is secondary productivity.

2. What are the limitations of ecological pyramide?

Ans. Limitation of ecological pyramids -

(i) It never takes into account the same species belonging to two or more trophic levels.

(ii) It assumes a simple food chain, which never exists in nature. It does not accummodate a

food web.

(iii) In spite of the vital role played by Saprophytes/decomposers, they are not given any

position in ecological pyramids.

3. (a) Name the type of deteritus that decomposes faster. list any two factor that enhance the

rate of decomposition.

(b) Write the different step taken in humification and mineralisation during the process

of decomposition.

Ans. (a) Detritus rich in nitrogen decomposes faster.

These are water - soluble substances like sugar - factors enhancing rate of decomposition

warm - temperature, moist environment, availability of O2.

(b) Humification :- Accumulation of dark coloured amorphous substance called humus which

is resistant to microbial action and undergoes decomposition at a very slow rate.

Mineralisation - Humus is further degraded by microbes releasing inorganic nutrients.

4. (a) What is primary production?

(b) Explain energy flow in ecosystem.

(c) Give a diagrammatic representation of energy flow through different trophic levels.

E 243
Ans. (a) Primary production: Primary production is defined as the amount of biomass or organic

matter produced per unit area over a time period by plants during photosynthesis. It is

expressed in terms of weight (gm–2) or energy (kcal m–2).

(b) Energy flow in ecosystem:

• We know that plants and photosynthetic bacteria (autotrophs), fix suns' radiant energy to

make food from simple inorganic materials. Plants capture only 2-10 per cent of the PAR

and this small amount of energy sustains the entire living world.

• All organisms are dependent for their food on producers, either directly or indirectly. So we

find unidirectional flow of energy from the sun to producers and then to consumers.

• The green plants in the ecosystem terminology are called producers.

(c) Diagrammatic representation of energy flow through different trophic levels:


(H) CASE-STUDY BASED QUESTIONS
1. Read the following and answer the questions given below:
Primary production is defined as the amount of biomass or organic matter produced per unit
area over a time period by plants during photosynthesis. It is expressed in terms of weight (gm–2) or
energy (kcal m–2). The rate of biomass production is called productivity. It is expressed in terms
of gm–2 yr–1 or (kcal m–2) yr–1 to compare the productivity of different ecosystems. It can be
divided into gross primary productivity (GPP) and net primary productivity (NPP). Gross
primary productivity of an ecosystem is the rate of production of organic matter during
photosynthesis. A considerable amount of GPP is utilised by plants in respiration. Gross
primary productivity minus respiration losses (R), is the net primary productivity (NPP).

(i) Write the full name of GPP.


Ans. Gross primary production.
(ii) Write the names of the factors on which the primary productivity depends.
Ans. (1) Environmental factor
(2) Photosynthetic capacity of plant
(3) Availability of nutrients
(iii) How are productivity, gross productivity, net primary productivity and secondary
productivity inter related?
Ans. GPP - R = NPP = 1
(iv) What do you mean NPP?
Ans. Available biomass for the consumption of heterotrophs.
(v) What is NPP equal to?
Ans. NPP = GPP – R
2. Read the following and answer the questions given below:
Each trophic level has a certain mass of living material at a particular time called as the
standing crop. The standing crop is measured as the mass of living organisms (biomass) or the
number in a unit area. The biomass of a species is expressed in terms of fresh or dry weight.
Measurement of biomass in terms of dry weight is more accurate. Why? The number of trophic
levels in the grazing food chain is restricted as the transfer of energy follows 10 per cent law –
only 10 per cent of the energy is transferred to each trophic level from the lower trophic level.
In nature, it is possible to have so many levels – producer, herbivore, primary carnivore,
secondary carnivore in the grazing food chain.

245
(i) State what does standing crop of a trophic level represent.
Ans. Standing crop of a trophic level represents the total mass of living material or energy content of
all the organisms of a trophic level at a particular time and location.
(ii) List any two ways of measuring the standing crop of a trophic level.
Ans. Biomass of living organisms in a unit area.
Number of living organism in a unit area.
(iii) Man can be a primary as well as secondary Consumer? Justify this statement.
Ans. Man can be a primary as weel as secondary consumer because man eats plants of well
secondary consumer because man eats plants as well as meat of other animals so man is
herbivore as well as carnivore.
(iv) Difference between two different types of pyramids of biomass with the help of an
example.
Ans. Pyramids refers to the relationship between producers and consumers in terms of biomass. It
can be upright, eg.- ingrasslands ecosystem or inverted eg - in pond ecosystem.
3. Read the following and answer the questions given below:
The base of a pyramid is broad and it narrows towards the apex. One gets a similar shape,
whether you express the food or energy relationship between organisms at different trophic
levels. This, relationship is expressed in terms of number, biomass or energy. The base of each
pyramid represents the producers or the first trophic level while the apex represents tertiary or
top level consumer. The three types of ecological pyramids that are usually studied are (a)
pyramid of number; (b) pyramid of biomass and (c) pyramid of energy.

(i) Differentiate between two different types of pyramids of biomass with the help of one
example of each.

Ans. Pyramids of biomass are of both types, upright and inverted. Upright pyramid of biomass can be
found in forest and grassland ecosystems, while inverted

(ii) Why are the pyramids referred to as ‘upright’ or ‘inverted’ ? Explain.

Ans. Pyramids can be upright or inverted. For e.g. Pyramid of energy is always upright as only 10%
energy is transferred from one trophic level to the next while pyramid of biomass is inverted in
pond ecosystem. Inverted pyramid shows less number / biomass of producers when compared
to primary consumers
(iii) What is an ecological pyramid ?

Ans. Graphical representation of the relationship among the organisms at different trophic level.

TG: @Chalnaayaaar
246 E

TG: @Chalnaayaaar
(iv) Write any two limitations of ecological pyramids.
Ans (a) It does not take into account the same species belonging to two or more trophic levels.
(b) It assumes a simple food chain, something that almost never exists in nature.
(c) It does not accommodate a food web.
(d) Moreover, saprophytes are not given any place in ecological pyramids even though they
play a vital role in the ecosystem.
(v) Explain the significance of ecological pyramids.
Ans Significance of ecological pyramids:
(a) Helps in comparing different ecosystems.
(b) Helps in studying seasonal variations and changes in ecosystem.
(c) Helps in studying amount of energy transfer, biomass production, number of organisms at
each trophic level in ecosystems.

4. Read the following and answer the questions given below:


You may have heard of the earthworm being referred to as the farmer’s ‘friend’. This is so
because they help in the breakdown of complex organic matter as well as in loosening of the
soil. Similarly, decomposers break down complex organic matter into inorganic substances like
carbon dioxide, water and nutrients and the process is called decomposition. Dead plant remains
such as leaves, bark, flowers and dead remains of animals, including fecal matter, constitute
detritus, which is the raw material for decomposition. The important steps in the process of
decomposition are fragmentation, leaching, catabolism, humification and mineralisation.

(i) What is decomposition?


Ans. It is the process of breaking down of complex organic matter into inorganic substance. Like
water carbon dioxide and nutrients by decomposers.
(ii) Write different stages involve in the process.
Ans. Fragmentation, leaching, catabolism, humification and mineralisation.
(iii) What is mineralisation?
Ans. It is process of degradation of humus by microbial action and release of inorganic nutrients.
(iv) Write the factors affecting the rate of this process.
Ans. Chemical composition of detritus - The rate of decomposition is slower, If detritus is rich in
lignin and chitin.
Climiate factors - Temperature and soil moisture are the most important climatic factors that
controls decomposition.

247
5. Read the following and answer the questions given below:
The consumers that feed on these herbivores are carnivores, or more correctly primary
carnivores (though secondary consumers). Those animals that depend on the primary carnivores
for food are labelled secondary carnivores. A simple grazing food chain (GFC) is depicted
below:
Grass Goat Man
(Producer) (Primary Consumer) (Secondary consumer)
The detritus food chain (DFC) begins with dead organic matter. It is made up of decomposers
which are heterotrophic organisms, mainly fungi and bacteria. They meet their energy and
nutrient requirements by degrading dead organic matter or detritus. These are also known as
saprotrophs (sapro: to decompose). Decomposers secrete digestive enzymes that breakdown
dead and waste materials into simple, inorganic materials, which are subsequently absorbed by
them.
(i) Name two types of food chains.

Ans. Grazing food chain and Detritus food chain.

(ii) What is the role of plants, animals and bacteria in an ecosystem?

Ans. Producers, consumers and decomposers.

(iii) What are starting points of a grazing food chain and detritus food chain respectively?

Ans. Grazing food chain  producers

deteritus food chain  Detritus (dead organic matter)

(iv) If 20 J of energy's is traped at producer level, then

how much energy will be available to man as food in the following chain?

Grass Goat Man

(Producer) (Primary Consumer) (Secondary consumer)

Ans 0.2 J

248 E
CHAPTER 13
(A) NCERT QUESTIONS & SOLUTIONS
1. Name the three important components of biodiversity.
Ans. The three important components of biodiversity are :
(i) Genetic Diversity
(ii) Species Diversity
(iii) Ecological diversity
2. How do ecologists estimate the total number of species present in the world?
Ans. There are two methods to estimate the number of species in the world :
(i) By estimating the rate of discovery of new species.
(ii) By statistical comparison of the temperate-tropical species richness of an exhaustively
studied group of insects and extrapolate this ratio to other groups of animals and plants to
come up with a gross estimate of the number of species on earth.
3. Give three hypotheses for explaining why tropics show greatest levels of species richness.
[IMP.]
Ans. The three hypothesis to explain species richness in tropics are:
(a) The constant environment in tropics promotes niche specialization and increased species
diversity.
(b) There is longer exposure to solar radiation in the tropical regions that contributes directly to
higher productivity and indirectly to greater species diversity.
(c) There occurred no glaciation in tropical region and it remained undisturbed. Thus organisms
living in tropics continued to flourish and evolved more species diversity.
4. What is the significance of the slope of regression in a species-area relationship?
Ans. Slope of regression in a species-area relationship indicates that species richness decreases with
the decrease in area. Regression coefficient (Z) is 0.1 - 0.2 regardless of the taxonomic group or
the region e.g. plants in Britain or birds in California. However, when very large areas like the
entire continent is analysed, it was found that slope of the line is much steeper with Z values in
the range of 0.6 to 1.2.
5. What are the major causes of species losses in a geographical region?
Ans. There are four major causes (The Evil Quartet) :
(a) Habitat loss and fragmentation. (b) Alien species invasions.
(c) Over-exploitation. (d) Co-extinctions.

249
6. How is biodiversity important for ecosystem functioning? [IMP.]
Ans. Importance of biodiversity for ecosystem functioning -
(a) Stability: Biodiversity is an important aspect for stability of an ecosystem. Ecologists
believe that communities with more species tend to be more stable than those with less
species.
(b) Productivity: Ecosystem with higher biodiversity show more productivity than ecosystems
with lower biodiversity. David Tilman's long-term ecosystem experiments using outdoor
plots provide confirmation.
(c) Ecosystem health: Rich biodiversity is not only essential for ecosystem health but
imperative for the survival of the human race on earth. Species are interlinked and so, killing
or disappearance of one would effect the others also.
(d) Resilience: Increased biodiversity provides resilience of the ecosystem against natural or
man-made disturbances.
7. What are sacred groves? What is their role in conservation?
Ans. Sacred groves are forest patches for worship in several parts of India. All the trees and wildlife in
them are venerated and given total protection. They are found in Khasi and Jaintia Hills in
Meghalaya, Western Ghat regions of Karnataka and Maharashtra etc. Tribe do not allow anyone
to cut even a single branch of tree in these sacred groves thus sacred groves have been free from
all types of exploitations.
8. Among the ecosystem services are control of floods and soil erosion. How is this achieved by
the biotic components of the ecosystem?
Ans. Control of soil erosion: Plant roots hold the soil particles tightly and do not allow the top soil
to be drifted away by winds or moving water. Plants increase the porosity and fertility of the soil.
Control of floods: It is carried out by retaining water and preventing run off of rain water. Litter
and humus of plants function as sponges thus retaining the water which percolates down and get
stored as underground water. Hence, the flood is controlled.
9. The species diversity of plants (22%) is much less than that of animals (72%). What could
be the explanations to how animals achieved greater diversification? [IMP.]
Ans. Animals have achieved greater diversification than plants due to following reasons:
(a) They are mobile and thus can move away from their predators or unfavorable environments.
On the other hand plants are fixed and have fewer adaptation to obtain optimum amount of
raw materials and sunlight therefore they show lesser diversity.
(b) Animals have well-developed nervous system to receive stimuli against external factors and
thus can respond to them. On the other hand plants do not exhibit any such mechanism thus
they show lesser diversity than animals.
10. Can you think of a situation where we deliberately want to make a species extinct? How
would you justify it?
Ans. Species which are harmful to human beings can be made extinct e.g. HIV, polio virus, etc. Such
micro-organisms are not part of any food chain and thus, their extinction would not affect the
ecosystem.

250 E
(B) PREVIOUS YEAR QUESTIONS
1. The sixth extinction in progress currently is different from all previous extinctions on earth
as it is : [CBSE 2023]
(a) 10-100 times faster (b) 100-1000 times faster
(c) 100-10000 times faster (d) 1000-10000 times faster
Ans. (b) 100-1000 times faster
2. The IUCN Red Data List (2004) in the last 500 years documents the extinction of nearly 784
species including : [CBSE 2023]
(a) 330 invertebrates (b) 338 invertebrates
(c) 359 invertebrates (d) 362 invertebrates
Ans. (c) 359 invertebrates
3. Which one of the following groups faces maximum threat of extinction? [CBSE 2023]
(a) Gymnosperms (b) Birds (c) Amphibian (d) Mammals
Ans. (c) Amphibian
4. "Cattle and goats do not browse the Calotropis plant." Justify the statement giving reasons.
[CBSE 2023]
Ans. Organisms to survive in the complex food web have evolved characters to support their living
and prevent them from predators. One of such example is Calotropis. Calotropis plant also
known as milkweed produce a chemical compound cardiac glycoside that affect the mammalian
heart. It is the defence mechanism in Calotropis plant to keep cattle stay away from it. This is the
reason cattle and goats do no feed/browse on this plant.
5. "Biodiversity plays a major role in many ecosystem services that nature provides."
(a) Describe any two broadly utilatarian arguments to justify the given statement.
(b) State one ethical reason of conserving biodiversity. [CBSE 2023]
Ans. (a) The broadly utilitarian argument says that biodiversity plays a major role in many ecosystem
services that nature provides.
  The fast- dwindling Amazon forest is estimated to produce, through photosynthesis, 20 per
cent of the total oxygen in the earth’s atmosphere. Pollination (without which plants cannot
give us fruits or seeds) is another service, ecosystems provide through pollinators layer –
bees, bumblebees, birds and bats.
(b) The ethical argument for conserving biodiversity relates to what we owe to millions of plant,
animal and microbe species with whom we share this planet.
 Philosophically or spiritually, we need to realise that every species has an intrinsic value,
even if it may not be of current or any economic value to us.
251
6. (a) Write the inference drawn by Alexander von Humboldt after his extensive exploration
of South Americal jungle.
(b) Study the graph given below :

S = CAz

Species richness
log S = log C + Z log A

Area
As per Alexander von Humboldt, what do the symbols S, A, Z and C in the graph stand for,
in respect of a species and area relationship ? [CBSE Term-II 2022]
Ans. (a) German naturalist and geographer Alexander von Humboldt observed that within a region
species richness increased with increasing explored area, but only up to a limit.
(b) On a logarithmic scale, the relationship is a straight line described by the equation
log S = log C + Z log A
where
S = Species richness
A = Area
Z = Slope of the line (regression coefficient)
C = Y- intercept
7. (a) Explain the concept of co–extinction by taking two examples.
OR
(b) "Forests provide intangible benefits to us." Explain by taking three different areas,
how. [CBSE Term-II 2022]
Ans. (a) When a species becomes extinct, the plant and animal species associated with it in an
obligatory way also become extinct.
Example:
When a host fish species becomes extinct, its unique assemblage of parasites also meets the same
fate. Another example is the case of a coevolved plant-pollinator mutualism where extinction of
one invariably leads to the extinction of the other.
OR
(b) Intangible benefits from forests derived from forests, in other words influences of forests on
environment are described below:
Improvement of climate : Forests ameliorate climate influencing temperature, rainfall,
humidity, wind etc. Forests regulate temperature range balance in the atmosphere and water
cycle.
252 E
8. Which of the three forests- Temperate, Mangroves and Tropical Evergreen is more
vulnerable to invasion by outside animals and plants? [CBSE IMP Questions]
Ans. Tropical Evergreen Forests
9. Assertion: A community with more species is more stable than that with less species.
Reason: More the number of species, lesser the variation in the total biomass production year
after year. [CBSE IMP Questions]
(A) Both assertion and reason are true, and the reason is the correct explanation of the assertion.
(B) Both assertion and reason are true, but the reason is not the correct explanation of the
assertion.
(C) Assertion is true but reason is false.
(D) Both assertion and reason are false
Ans. (A) Both assertion and reason are true, and the reason is the correct explanation of the
assertion.
10. Explain how advanced ex-situ conservation techniques assist in preserving threatened
species of plants and animals. [CBSE IMP Questions]
Ans.  Advanced techniques are being used now for ex situ conservation. Gametes of threatened
species can be preserved in viable and fertile condition for long periods using
cryopreservation techniques. Eggs can, thus, be fertilized invitro.
 In plants, the explants can be propagated using tissue culture methods and can be kept for
long periods in seed banks.
11. The Tropical regions are likely to have more biological diversity than the Temperate ones.
Give two reasons to justify the statement. [CBSE IMP Questions]
Ans. Some possible reasons are:
 Speciation is generally a function of time, unlike temperate regions subjected to frequent
glaciations in the past, tropical latitudes have remained relatively undisturbed for millions of
years and thus, had a long evolutionary time for species diversification.
 Tropical environments, unlike temperate ones, are less seasonal, relatively more constant and
predictable. Such constant environments promote niche specialisation and lead to a greater
species diversity.
  There is more solar energy available in the tropics, which contributes to higher productivity;
this in turn might contribute indirectly to greater diversity.
12. Alien species invasion has been a threat to biodiversity. Justify with the help of a suitable
example. List any other causes responsible for such a loss. [CBSE 2020]
Ans. When alien species are introduced unintentionally or deliberately, some of them turn invasive
and cause decline or extinction of indigenous species.
For example the Nile perch introduced into lake Victoria in East Africa led to the extinction of
more than 200 species of cichlid fish. Other alien species examples includes African cat fish,
Parthenium and water hyacinth. Introduction of these species have caused loss of biodiversity.
Apart from Alien species invasion; the other causes of loss of biodiversity are -
(i) Co-extinction
(ii) Habitat loss & Fragmentation
(iii) Over exploitation
E 253
13. List six advantages of “ex-situ” approach to conservation of biodiversity. [CBSE 2019]
Ans. 1. An endangered or threatened species can be conserved.
2. Genetic strains of commercially important plants can be preserved for a long time
(seed banks). Biodiversity loss is reduced.
3. Gametes of threatened species can be preserved in a viable and fertile condition for long
periods (using cryopreservation).
4. In zoological parks it enables us to learn about their food habits and behaviour.
5. Plants can be propagated using tissue culture for economically beneficial.
6. Conserve large number of species for aesthetic value.
14. State 'two' observations made by German naturalist, Alexander von Humboldt during his
extensive explorations in South American jungles. [CBSE 2019]
Ans. (i) Within a region species richness increases with increasing explored area but only up to a limit.
(ii) The relation for a wide variety of taxa (angiosperm, birds, plants, fresh water fishes) turns out
to be a rectangular hyperbola.
15. Analyze the effects of 'Alien species invasion' on the biodiversity of a given area. Provide
two examples. [CBSE 2019]
Ans. Introduction of alien species causes decline or extinction of indigenous species due to tough
competition for utilization of resources.
Examples :
(a) Introduction of Nile perch in lake Victoria led to extinction of more than 200 species of
Cichlid fish.
(b) Introduction of African catfish Clarias gariepinus for aquaculture purposes threat to
indigenous catfishes.
16. What is cryopreservation? Mention how it is used in conservation of biodiversity.
[CBSE 2019]
Ans. It is a technique to preserve gametes for long period in viable and fertile condition at very low
temperature / - 196°C in liquid nitrogen.
Preserving gametes of threatened species.
17. How did David Tillman show that “stability of a community depends on its species
richness”? Explain. [CBSE 2019]
Ans. David Tilman (long term ecosystem experiments, using outdoor plots) found that presence of
more species showed less year-to-year variation in total biomass, and increased diversity
contributed to higher productivity.
18. Give the answer of following questions. [CBSE 2019]
(a) List any two ways the biodiversity loss affects any region.
(b) Explain any four causes of biodiversity loss, with the help of suitable examples.
Ans. (a)  Decline in plant production.
 Lowered resistance to environmental perturbations such as drought.
 Increased variability in certain ecosystem processes such as plant productivity, water
use, and pest and disease cycles.
(b)
(i) Habitat loss and fragmentation: This is the most important cause driving animals and
plants to extinction.
Examples - Tropical rain forests. Once covering more than 14 % of the earth’s land
surface, these rain forests now cover no more than 6 %. The Amazon rain forest (it is so
huge that it is called the ‘lungs of the planet’) harboring probably millions of species is being
cut and cleared for cultivating soya beans or for conversion to grasslands for raising beef
cattle. The degradation of many habitats by pollution also threatens the survival of many
species.
(ii) Over-exploitation : Humans have always depended on nature for food and shelter, but when
‘need’ turns to ‘greed’, it leads to over-exploitation of natural resources.
Example - many marine fish populations around the world are over harvested, endangering
the continued existence of some commercially important species.
(iii) Alien species invasions : When alien species are introduced unintentionally or deliberately
for whatever purpose, some of them turn invasive, and cause decline or extinction of
indigenous species.
Example - The Nile perch introduced into Lake Victoria in east Africa led eventually to the
extinction of more than 200 species of Cichlid fish in the lake. Threat posed to native species
by invasive weed species like carrot grass (Parthenium).
The African catfish for aquaculture purposes is posing a threat to the indigenous catfishes in
our rivers.
(iv) Co-extinctions : When a species becomes extinct, the plant and animal species associated
with it in an obligatory way also become extinct.
Example-When a host fish species becomes extinct, its unique assemblage of parasites also
meets the same fate. Covelved plant-pollinator mutualism where extinction of one invariably
leads to the extinction of the other.

E 255
19. Identify the areas labeled i, ii, iii and iv in the pie chart given below representing the
biodiversity of plants showing their proportionate number of species of major taxa.
[CBSE 2018]

Ans. (i) Lichen, (ii) Algae, (iii) Fungi, (iv) Mosses


20. Suggest two practices giving one example of each, that help protect rare or threatened
species. [CBSE 2017]
Ans. (1) In situ conservation, biodiversity hotspot, biosphere reserve, national parks, wildlife
sanctuaries and sacred groves.
(2) Ex situ conservation, Zoological parks, botanical gardens, wildlife safari parks,
cryopreservation techniques, seed bank.
21. ‘In-situ’ conservation can help endangered/ threatened species. Justify the statement.
[CBSE 2017]
Ans. Threatened organisms are conserved in their natural habitat or ecosystem, and such regions are
legally protected.
As hotspots, biosphere reserves, national parks, sanctuaries or sacred groves sites.

256 E
(C) MULTIPLE CHOICE QUESTIONS
1. Biodiversity term was popularized by –
(1) Edward Wilson (2) Paul Ehrlich
(3) Alexander von Humboldt (4) David Tilman
Ans. (1) Edward Wilson
2. log S = log C + Z log A
In the given equation of species-area relationship, the value of regression coefficient for a whole
continent, would be
(1) 0.1-0.2 (2) 0.5-0.0 (3) 0.6-1.2 (4) 0.3-0.5
Ans. (3) 0.6-1.2
3. Which of the following organisation is responsible for maintaining the Red Data Book?
(1) IDRI (2) IUCN (3) UNESCO (4) USDA
Ans. (2) IUCN
4. Alexander Von Humboldt described for the first time-
(1) Ecological Biodiversity (2) Laws of limiting factor
(3) Species area relationships (4) Population Growth equation
Ans. (3) Species area relationships
5. India has more than _____ genetically different strains of rice.
(1) 1,000 (2) 20,000 (3) 50,000 (4) 85,000
Ans. (3) 50,000
6. IUCN stands for -
(1) International Union For Conservation of Nature and Natural Resources.
(2) Indian Union For Conservation of Nature and Natural Resources.
(3) Indian Union For Conservation of Nature
(4) International Union of Conservation of names
Ans. (1) International Union For Conservation of Nature and Natural Resources.
7. Given below is the representation of the extent of global diversity of vertebrates. What groups the
four portions (A-D) represent
E
D
A
C
B

A B C D E
(1) Mammals Reptiles Birds Fishes Amphibian
(2) Amphibian Fishes Birds Reptiles Mammals
(3) Fishes Mammals Birds Reptiles Amphibian
(4) Fishes Amphibian Reptiles Birds Mammals
Ans. (4) A-Fishes B-Amphibian C-Reptiles D-Birds E-Mammals
E 257
8. Robert May places the global species diversity at about ____
(1) 10 million (2) 15 million (3) 2 million (4) 7 million
Ans. (4) 7 million
9. The enormous number of varieties of mango in India represents
(1) Genetic diversity (2) Species diversity
(3) Ecological diversity (4) Hybridisation programmes
Ans. (1) Genetic diversity
10. How many species of ants on Earth?
(1) 20,000 (2) 10,000 (3) 15,000 (4) 25,000
Ans. (3) 15,000
11. In Rivet Popper hypothesis, what do you mean by rivets?
(1) Species (2) Ecosystem (3) Biodiversity (4) Biosphere
Ans. (1) Species
12. The current species extinction rates are estimated to be ________ times faster than in the pre-
human times.
(1) 10-100 (2) 100-1000 (3) 200-2000 (4) 500-1000
Ans. (2) 100-1000
13. Which of the following is called the lungs of the planet?
(1) Tropical rain forest (2) Thar desert
(3) Amazon rain forest (4) Temperate deciduous forest
Ans. (3) Amazon rain forest
14. From his long-term ecosystem experiments, David Tilman showed that
(1) decreased diversity contributed to higher productivity
(2) decreased diversity contributed to decreased productivity
(3) increased diversity contributed to increased productivity
(4) increased diversity contributed to decreased productivity
Ans. (3) increased diversity contributed to increased productivity
15. Extinction of species like steller’s sea cow and passenger pigeon were due to ______ by humans.
(1) Co-extinctions (2) Over-exploitation
(3) Habitat loss and fragmentation (4) Alien species invasions
Ans. (2) Over-exploitation
16. What do you mean by “The Evil Quartet”?
(1) Causes of biodiversity loss (2) Biodiversity conservation
(3) Species-Area relationship (4) Genetic diversity
Ans. (1) Causes of biodiversity loss
17. The most important cause of extinction of animals and plants, especially in tropical rain forests is
(1) Habitat loss (2) Afforestation (3) Pollution (4) Soil erosion
Ans. (1) Habitat loss
18. Which of the following is not a cause for loss of biodiversity?
(1) Destruction of habitat (2) Invasion by alien species
(3) Keeping animals in zoological parks (4) Over-exploitation of natural resources
Ans. (3) Keeping animals in zoological parks
19. More than ____ percent of the drugs currently sold in the market worldwide are derived from
plants.
(1) 50 (2) 25 (3) 75 (4) 60
Ans. (2) 25
20. Amazon forest is estimated to produce _____ percent of the total oxygen in the earth’s
atmosphere.
(1) 10 (2) 15 (3) 20 (4) 30
Ans. (3) 20

E 259

(D) ASSERTION & REASON QUESTIONS
 Directions: In the following questions, a statement of assertion is followed by a statement of
reason. Mark the correct choice as:
(1) If both Assertion and Reason are true and Reason is the correct explanation of Assertion.
(2) If both Assertion and Reason are true but Reason is not the correct explanation of Assertion.
(3) If Assertion is true but Reason is false.
(4) If both Assertion and Reason are false.
1. Assertion: Steller’s sea cow becomes extinct.
Reason: It is due to over exploitation by humans.
Ans. (1)
2. Assertion: Biodiversity hotspots are example of in situ conservation.
Reason: It is a conservation of biological wealth on site.
Ans. (1)
3. Assertion: Ex situ conservation is carried out in biosphere reserves, national parks and
sanctuaries.
Reason: NGOs are involved in the maintenance of these protected areas.
Ans. (4)
4. Assertion: Insects are enormously diversified.
Reason: It is because of the presence of exoskeleton made up of chitin.
Ans. (1)
5. Assertion: Habitat loss and fragmentaion is the most important cause driving animals and plants
to extinction.
Reason: Habitat loss and fragmentaion causes alien species invasion.
Ans. (3)
6. Assertion: India has more than 50,000 genetically different strains of rice.
Reason: India has approximately 1,000 varieties of mango.
Ans. (2)
7. Assertion: According to the IUCN (2004), the total number of plant and animal species
described so far is lightly more than 1.5 million.
Reason: For many taxonomic groups, species inventories are more complete in temperate than in
tropical areas.
Ans. (2)
8. Assertion: During the long Period since the origin and diversification of life on earth there were
five episodes of mass extinction of species.
Reason: The six Extinctions is on progress different from the previous episodes.
Ans. (2)
9. Assertion: The diversity of microbes may run into millions.
Reason: If we use biochemical or molecular criteria to estimate microbial species.
Ans. (1)
10. Assertion: India is one of the 12 mega biodiversity economies of the world.
Reason: India has only 2.4% of the world’s land area. It shares 8.1% if the global species
diversity.
Ans. (1)
260 E
(E) VERY SHORT ANSWER QUESTIONS
1. Name the type of biodiversity represented by the following:
(i) 50,000 different strains of rice in India.
(ii) Estuaries and alpine meadows in India.
Ans. (i) Genetic diversity. (ii) Ecological diversity
2. Name the type of biodiversity represented by the following:
(i) 1000 varieties of mangoes in India.
(ii) Variations in terms of potency and concentration of reserpine in Rauwolfia vomitoria
growing in different regions of Himalayas.
Ans. (i) Genetic diversity. (ii) Genetic diversity.
3. An exotic variety of prickly pear introduced in Australia turned out to be invasive. How
was it brought under control?
Ans. It was controlled by introducing a cactus – feeding predator (a moth) from its natural habitat
4. Give an example of a plant which came into India as a contaminant and is a cause of pollen
allergy.
Ans. Parthenium / Carrot grass.
5. Name the unlabelled areas ‘a’ & ‘b’ of the pie chart representing biodiversity of vertebrates
showing the proportionate number of species of major taxa.

Ans. (i) Fishes (ii) Amphibians


6. Habitat loss and fragmentation has caused severe damage to a particular type of ecosystem.
Name it.
Ans. Tropical Rain Forest.
7. What trend is observed in respect of species diversity when we move from equator to poles?
Ans. In general, species diversity decreases as we move away from the equator towards poles.
8. Which region is considered as the one with highest biodiversity on earth? What is the name
given to such region.forests?
Ans. Amazonian rain forests. They are also called the ‘Lungs of the planet’.
9. Write the importance of cryopreservation in conservation of biodiversity.
Ans. Gametes of threatened species can be preserved in viable and fertile conditions for long periods
by cryopreservation.
10. What is the total number of species discovered and described presently on earth? What is
the predicted number?
Ans. 1.7 million and 50 million, respectively.

E 261
(F) SHORT ANSWER QUESTIONS
1. List three levels of biodiversity.
Ans. Biodiversity can be studied at the following levels:
1. Genetic diversity 2. Species diversity 3. Ecological/Ecosystem diversity.
2. Biodiversity must be conserved as it plays an important role in many ecosystem services
that nature provides. Explain any two services of the ecosystem.
Ans. The two ecosystem services are:
(i) Forest ecosystem, mitigates droughts and floods.
(ii) The wildlife help in pollination of crops.
3. Write what was the percentage of forest cover of India at the beginning and at the end of
the twentieth century. How different is it from the one recommended by the National
Forest Policy of our country ?
Ans. Beginning of 20th century - 30%
End of 20th century - 19.4%
Recommendations were 33% for the plains and 67% for the hills (thus forest cover shrunk
substantially)
4. What is meant by ‘alien species’ invasion ? Name one plant and one animal alien species
that are a threat to our Indian native species.
Ans. ‘Alien species’ invasion means introducing those that are, firstly, outside their natural
distribution area, and, secondly, threatens biological diversity.
Plants : Lantana camara
Animals : African catfish.
5.

The above graph shows Species-Area relationship. Write the equation of the curve ‘a’ and
explain.
Ans. (i) S = CAZ
Within a region, species richness increases with increasing explored area but only up to a limit.
6. Alien species are highly invasive and are a threat to indigenous species. Substantiate this
statement with any three examples.
Ans. (i) Nile perch introduced into Lake Victoria in East Africa led to the extinction of Cichlids fish.
(ii) Parthenium, Lantana, Eichhornia are invasive plants and pose a threat to indigenous species.
(i) Introduction of African catfish (Clarias gariepinus) for aquaculture is a threat to indigenous
catfishes.
(G) LONG ANSWER QUESTIONS
1. Observe the global biodiversity distribution of major vertebrate taxa in the given pie
diagram along side and answer the questions that follow.
Mammals
Fishes
Birds

Reptiles

Amphibians

(i) Which group of vertebrates have maximum diversity.


(ii) Which group of vertebrates are have minimum diversity in the Amazon rain forest of
South America.
(iii) Which group of vertebrates are completely regulators? Why?
Ans. (i) Fishes
(ii) Reptiles
(iii) All birds and mammals are regulators because they maintain homeostasis by
thermoregulation and osmoregulation.
2. Describe the advantages for keeping the ecosystems healthy?
Ans. The advantages of keeping the ecosystem healthy are as follow.
  The products of ecosystem processes are named as ecosystem services, as they are of
aesthetic goods and organism living within an ecosystem.
  Healthy ecosystem is the base for a wide range of economic, environmental and aesthetic
goods and services.
  It also mitigates drought and floods and cycling nutrients.
  Healthy forests ecosystem purify air and water
  It also provide aesthetic, cultural and spiritual values.
  Maintenance of biodiversity is also an important aspect of healthy ecosystem.
  Healthy ecosystem generate fertile soil and provide wide life habitats.
3. Which one of the two “in-situ” or “ex-situ” biodiversity conservation measures help the
larger number of species to survive? Explain.
Ans. In situ is onsite conservation which implies that species are conserved in their natural habitat,
while ex situ conservation implies conservation of genetic resources and different organisms
outside their natural habitat.
• To conserve species in their natural habitat the entire ecosystem has to be conserved
including all other organisms, biotic and abiotic components of the ecosystem associated
with the target species.
• In situ conservation helps in the restoration of degraded ecosystem and habitats that are
means of conserving genetic resources species ecosystem and landscapes,without uprooting
the local people.

E 263
4. Give the answer of following questions.
(a) List any two ways the biodiversity loss affects any region.
(b) Explain any four causes of biodiversity loss, with the help of suitable examples.
Ans. (a)  Decline in plant production.
 Lowered resistance to environmental perturbations such as drought.
 Increased variability in certain ecosystem processes such as plant productivity, water
use, and pest and disease cycles.
(b) (i) Habitat loss and fragmentation: This is the most important cause driving animals and
plants to extinction.
Examples- Tropical rain forests: Once covering more than 14 % of the earth’s land
surface, these rain forests now cover no more than 6 %. The Amazon rain forest (it is so
huge that it is called the ‘lungs of the planet’) harboring probably millions of species is being
cut and cleared for cultivating Soya beans or for conversion to grasslands for raising beef
cattle. The degradation of many habitats by pollution also threatens the survival of many
species.
(ii) Over-exploitation: Humans have always depended on nature for food and shelter, but when
‘need’ turns to ‘greed’, it leads to over-exploitation of natural resources.
Example - many marine fish populations around the world are over harvested, endangering
the continued existence of some commercially important species.
(iii) Alien species invasions: When alien species are introduced unintentionally or deliberately
for whatever purpose, some of them turn invasive, and cause decline or extinction of
indigenous species.
Example - The Nile perch introduced into Lake Victoria in east Africa led eventually to the
extinction of more than 200 species of Cichlid fish in the lake. Threat posed to native species
by invasive weed species like carrot grass (Parthenium).
The African catfish for aquaculture purposes is posing a threat to the indigenous catfishes in
our rivers.
(iv) Co-extinctions: When a species becomes extinct, the plant and animal species associated
with it in an obligatory way also become extinct.
Example-When a host fish species becomes extinct, its unique assemblage of parasites also
meets the same fate. Coevolved plant-pollinator mutualism where extinction of one
invariably leads to the extinction of the other.
264 E
5. Give the answer of following questions.
(a) Why should we conserve biodiversity? How can we do it?
(b) Explain the importance of biodiversity hot-spots and sacred groves.
Ans. (a) (i) Narrowly Utilitarian:
It is concerned with direct economic benefits from nature food, firewood, fibre, construction
material, industrial products and products of medicinal importance.
More than 25 % of the drugs currently sold in the market world wide are derived from plants.
(ii) Broadly Utilitarian:
It is concentred with indirect benefits from nature, like, photosynthesis, pollination.
A major role in many ecosystem services that nature provides Amazon forest is estimated to
produce 20% of the total oxygen in the earth's atmosphere pollination is another Service,
ecosystem provide through insects.
It can conserved by:
 In situ conservation by biosphere reserves, national parks, sanctuaries and sacred groves.
 Ex situ conservation by zoological parks, botanical gardens, wild life safari parks,
cryopreservation, seed banks and tissue culture (eggs in vitro).
(b) Hot spots - Regions with high level of species richness, high degree of endemism.
 Sacred groves - In many cultures, tracts of forest were set aside, and all the trees and
wildlife within were venerated and given total protection. Sacred groves are the last refuges
for a large number of rare, and threatened plants.

(H) CASE-STUDY BASED QUESTIONS


(1) Read the following and answer the questions given below:
Non-native or alien species are often introduced in advertently for their economic and other uses.
They often become invasive and drive away the local species. Exotic species have proved
harmful to both aquatic and terrestrial ecosystems. For example, water hyacinth (Eichhornia
crassipes) was introduced in Indian waters to reduce pollution. It was clogged water bodies
including wetlands at many places resulting in death of several aquatic plants and animals.
(i) Name the alien fish species which is posing a threat to indigenous caltfishes in our rivers
Ans. Clariass gariepinus (African catfish)
(ii) What is meant by alien species?
Ans. Non native powerful species which invade a new area are known as alien species.
(iii)What is the second major cause for the species extinction.
Ans. Alien species invasion
(iv) Eichhornia crassipes is an alien hydrophyte introduced in india. Mention the problem posed
by this plant.
Ans. When an alien hydrophyte Eichhornia were introduced unintentionally they turned invasive
caused decline or extinction of indigenous species.
(v) Give an example of a plant which came into india as contaminant and is a cause of pollen
allergy.
Ans. Parthenium/Carrotgrass.
265
(2) Read the following and answer the questions given below:
IUCN maintains a Red Data Book or Red List which is a catalogue of taxa facing risk of
extinction. The IUCN Red List (2004) documents the extinction of 784 species in the last 500
years. Some examples of recent extinctions include the dodo, quagga, thylacine and Steller's sea
cow. The last twenty years alone have witnessed the disappearance of 27 species. Red List has
eight categories of species.
(i) Dodo, an extinct taxon, belongs to which country?
Ans. Mauritius
(ii) Why Steller's sea cow and passanger pigeon become extinct?
Ans. Over - exploitation
(iii) Bali, Javan and Caspian belong to which species?
Ans. Species of tiger.
(iv) Which organization publisher the Red list of species?
Ans. ICUN
(v) What is Red Data book?
Ans. The Red Data book is a compilation of data on species threatened with extinction and is
maintained by IUCN.
(3) Read the following and answer the questions given below:
Wetlands are called Ramsar sites because the first international convention on their conservation
was held in Ramsar in Iran in 1971. Wetlands or Ramsar sites are low lying marshy areas which
get filled up during rains due to runoff and overflow from other water bodies. They are often
considered to be waste lands which are used as dumping areas and filled up to recover land for
various constructions activities. As a result, a large number of wetlands have disappeared.

(i) In which wetlands Migratory bird flamingo breeds in India?


Ans. Rann of Kutch
(ii) Which wetland in ecosystem is highly acidic and has a accumulation of decomposed plants
known as peat?
Ans. Bog
(iii) Write down importance of wetlands?
Ans. (1) They are an important source of recharging groundwater
(2) They provide protection from floods.
(3) They are good source of siltation and purification of water.
(iv) What are Ramsar sites?
Ans. They are conserved wetlands which are of international importance.
(v) Which is the largest Ramser site in India?
Ans. Sundarban Wet land (WB) India.

266 E
(4) Read the following and answer the questions given below:

Excessive exploitation of species, whether a plant or animal reduces the size of its population so

it becomes vulnerable to extinction. Such as Dodo and passenger pigeon have become extinct

due to over exploitation by humans. Thus the world is facing accelerated rates of species

extinctions, largely due to human interference.

(i) State how the current occurrence of species extinction is different from the earlier mass

extinction.

Ans. Species extinction in earlier times occurred due to natural calamities such as volcanic eruptions,

landslides, flood etc. while in the present times, the cause of species extinction is human

activities.

(ii) State how does ex-situ conservation help in protecting biodiversity.

Ans. Cryopreservation, in vitro fertilisation, micropropagation / tissue culture, sperm bank / seed bank

/ gene bank, are help full for conservation of biodiversity.

(iii) What is endemic species?

Ans. Endemic species belong to a local area and are of limited distribution due to biotic and abiotic

regulations, e.g. Lion Tailed Macaque

(iv) What is exotic species?

Ans. Exotic or Alien species are new species that enter a geographical regions.

(v) What do you understand by co-extinction?

Ans. When a species become extinct, the plant and animal species associated with it in the obligatory

ay, also become extinct.

267
IMPORTANT NOTES
__________________________________________________________________________________

__________________________________________________________________________________

__________________________________________________________________________________

__________________________________________________________________________________

__________________________________________________________________________________

__________________________________________________________________________________

__________________________________________________________________________________

__________________________________________________________________________________

__________________________________________________________________________________

__________________________________________________________________________________

__________________________________________________________________________________

__________________________________________________________________________________

__________________________________________________________________________________

__________________________________________________________________________________

__________________________________________________________________________________

__________________________________________________________________________________

__________________________________________________________________________________

__________________________________________________________________________________

__________________________________________________________________________________

__________________________________________________________________________________

__________________________________________________________________________________

__________________________________________________________________________________

__________________________________________________________________________________

268 E
SAMPLE QUESTION PAPER
CLASS: XII
Session : 2022-23
Biology (Code - 044)

Time Allowed : 3 hours Maximum Marks : 70


General Instructions :
1. All questions are compulsory.
2. The question paper has five sections and 33 questions. All questions are compulsory.
3. Section–A has 16 questions of 1 mark each; Section–B has 5 questions of 2 marks each;
Section– C has 7 questions of 3 marks each; Section–D has 2 case-based questions of 4 marks
each; and Section–E has 3 questions of 5 marks each.
4. There is no overall choice. However, internal choices have been provided in some questions. A
student has to attempt only one of the alternatives in such questions.
5. Wherever necessary, neat and properly labelled diagrams should be drawn.

SECTION – A
1. Remnants of nucellus are persistent during seed development in:
(A) pea (B) groundnut (C) wheat (D) black pepper
2. The wall layer of microsporangium which nourishes the pollen grain is :
(A) epidermis (B) endothecium (C) middle layers (D) tapetum
3. A short piece of DNA, having 20 base pairs, was analyzed to find the number of nucleotide
bases in each of the polynucleotide strands. Some of the results are shown in the table.
Number of nucleotide bases
Adenine Cytosin Guanine Thymine
Strand - I 4 4
Strand - II 5
How many nucleotides containing Adenine were present in strand 2?
(A) 2 (B) 4 (C) 5 (D) 7
4. In a certain species of insects, some have 13 chromosomes, and the others have 14
chromosomes. The 13 and 14 chromosome bearing organisms are
(A) males and females, respectively (B) females and males, respectively
(C) all males (D) all females
E 269
5. At a particular locus, the frequency of allele A is 0.8 and that of allele a is 0.2. What would be
the frequency of heterozygotes in a random mating population at equilibrium?
(A) 0.32 (B) 0.16 (C) 0.24 (D) 0.48
6. Variations caused due to mutations are
(A) random and directionless (B) random and directional
(C) random and small (D) random, small and directional
7. What is the smallest part of a DNA molecule that can be changed by a point mutation?
(A) Oligonucleotide (B) Codon
(C) Gene (D) Nucleotide
8. What should be the genotype of the indicated member?

(A) AA (B) Aa (C) XY (D) aa


9. A patient was advised to have a kidney transplant. To suppress the immune reaction, the doctor
would administer him:
(A) statins produced from Monascus purpureus
(B) statins produced from Streptococcus thermophilus
(C) cyclosporin A produced from Trichoderma polysporum
(D) cyclosporin A produced from Clostridium butylicum
10. Identify the activity of endonuclease and exonuclease in the given image.
Endonuclease Exonuclease

(A)

(B)

(C)

(D)
11. The main objective of production of pest resistant GM crops is to -
(A) encourage eco-friendly pesticides
(B) reduce pesticide accumulation in food chain
(C) eliminate pests from the field without the use of manual labour
(D) retain maximum nutritional content in the crop that would be otherwise consumed by pest
12. Observe the contents 1,2,3 and 4 of soil samples A,B and C shown in the graph. If the
temperature and soil moisture of all soil samples are identical, which soil sample (s) will show
faster decomposition?
20
Content (%)

15

10

0
1 2 3 4 1 2 3 4 1 2 3 4
Soil A Soil B Soil C
1 indicates lignin content, 2 indicates chitin,
3 indicate, Nitrogen content and 4 indicates sugar
content
(A) Soil Sample A (B) Soil Sample B
(C) Soil Samples A and B both (D) Soil Sample C
Question No. 13 to 16 consist of two statements – Assertion (A) and Reason (R). Answer
these questions selecting the appropriate option given below:
(A) Both A and R are true and R is the correct explanation of A.
(B) Both A and R are true and R is not the correct explanation of A.
(C) A is true but R is false.
(D) A is false but R is true.
13. Assertion : Primary endosperm nucleus is diploid.
Reason : It is the product of double fertilisation.
14. Assertion : Ribosomal RNA is synthesized in the nucleus of the cell.
Reason: It is translated with the enzyme RNA polymerase III.
15. Assertion : Smoking can raise blood pressure and increase heart rate.
Reason : Nicotine stimulates adrenal glands to release adrenaline and nor-adrenaline into the
blood circulation, both of which raise blood pressure and increase heart rate.
16. Assertion : PCR is a powerful technique to identify genetic disorders.
Reason : PCR can detect mutations in low amounts of DNA.

E 271
SECTION – B
17. Explain the process of hormonal regulation of spermatogenesis.
18. The diagram below shows the sequence of amino acids in part of a haemoglobin molecule.

Val His Leu Thr Pro Glu Glu haemoglobin


chain
mRNA
*
CAT GTA AAT TGA GGA CTT CTC
DNA
Key: Val = valine Thr = threonine
His = histidine Pro = proline
Leu = leucine Glu = glutamic acid

(a) If the base T* was substituted with A, how would it affect the haemoglobin chain?
(b) Name the condition and the effects associated with the above substitution.
19. The graph given below indicates the administration of the first (L) and second dose (M) of a
vaccine. The corresponding response of the body is indicated by X and Y. Interpret the graph
and explain the reason for such a response shown by the body.
Concentration of antibody

Y
L M

Time

20. The image below shows the result of plating bacteria in chromogenic medium after
incorporating the gene of interest in plasmid. Some plates had blue colonies; some plates had
white colonies. A single bacterium extracted from Plate I,II,III is shown below :

bacteria plasmid gene of


interest

bacteria from bacteria from bacteria from


plate I plate II plate III
On the basis of your observations
(a) Identify the plate(s) which is/are white. Give a reason.
(b) Identify the plate(s) which is/are blue. Give a reason.
272 E
21. Biomass of a standing crop of phytoplankton is 4 kg/m2 which supports a large standing crop of
zooplankton having a biomass 11 kg/m2. This is consumed by small fishes having biomass 25
kg/m2 which are then consumed by large fishes with the biomass 37kg/m2.
Draw an ecological pyramid indicating the biomass at each stage and also name the trophic
levels. Mention whether it is an upright or inverted pyramid.
OR
Use the information provided in the table given below to answer the following questions :
Tropic Level Net Productivity (kJm–2y–1) Respiration(kJm–2y–1)
Top Carnivore 50 35
Carnivores 420 378
Herbivores 4490 4041
Producers 45000 40,367
(a) Calculate the gross primary productivity.
(b) Analyze the trend in the Net Production from Producers to Top Carnivore. Give a reason for your
observation.
SECTION – C
22. The figure given below shows 3 sperms A, B and C.
(a) Which one of the three sperms will gain entry into the ovum?
(b) Describe the associated changes induced by it on P and Q.

B
Sperm C
A

R
P

Perivitelline space
Figure : Ovume surrounded by few sperms

23. Explain the phases in embryonic development from the morula stage till the establishment of
pregnancy in a human female.
24. A pregnant human female was advised to undergo MTP. It was diagnosed that the fetus she was
carrying had developed from a zygote having 45 chromosomes with only one X chromosome.
(a) What is this condition called and how does it arise?
(b) Why was she advised to undergo MTP?
273
25. The graphs below show three types of natural selection. The shaded areas marked with arrows
show the individuals in the population which are not selected. The dotted vertical lines show the
statistical means.
Number of individuals Number of individuals Number of individuals

Character Character Character


Graph A Graph B Graph C
(a) What names are given to the types of selection shown in graphs A, B and C.
(b) After the selection has operated for several generations in the above populations indicated as
Graph A, B and C, graphically illustrate the probable results.
26. The aeration tank of a sewage treatment plant is not functioning properly. Explain in detail the
impact of this on the treatment of sewage and BOD of the effluent.
27. A farmer grew 2 varieties of corn crop in field A and B. He grew normal corn crops in field A
and GM corn crops in field B. He observed corn borers attacked only in field A. To control it,
spores of Bt were sprayed in field A.
(a) Name the gene in the spores responsible for the control of this pest.
(b) What effect will the spores of Bt have on the insect pest?
(c) How has field B developed resistance against this pest?
OR
Lipoprotein lipase deficiency (LPLD)is a genetic disorder in which a person has a defective gene
for lipase. This leads to high triglycerides, stomach pain, fat deposits under the skin. It may
eventually affect the liver, pancreas and may also cause diabetes. The disorder occurs if a child
acquires defective genes from both parents (autosomal recessive). ERT (enzyme replacement
treatment) is one of the treatments offered to patients with LPLD.
(a) (i) What procedure is followed in ERT?
(ii) What could be one possible drawback of ERT?
(b) How can LPLD be treated using Biotechnology? Elaborate.
28. Give three reasons as to why the prokaryotes are not given any figures for their diversity by the
ecologists.
SECTION – D
Q. No. 29 and 30 are case-based questions. Each question has 3 subparts with internal
choice in one subpart.
29. The structure below shows pUC18 which is similar to pBR322 in its function. However, they
differ in some of their restriction sites and number of ori. The ori number for pBR322 is
approximately 20.

pUC18

ori more than


or equal to 100

274 E
(a) How are pUC18 and pBR322 used in biotechnological studies?
OR
What will be the impact if origin the above structure gets damaged?
(b) The lac z gene has many recognition sites. Study the segment of DNA given below and
answer the questions
5’... ATC GTA AAG CTT CAT…3’
3’... TAG CAT TTC GAA GTA…5’
(i) Applying your knowledge of palindrome sequences identify and mark the possible region
where the restriction enzyme X will act.
(ii) Restriction enzyme Y was used to extract gene of interest from a plant. This gene needs to be
inserted in the given DNA segment which has been treated with restriction enzyme X. Will
there be a successful recombination? Explain with a reason.
(c) Which one of the two (pUC18 and pBR322) would you prefer for biotechnological studies?
Justify.
30. Observe the graph given below. The graph represents inter-specific interaction between two
species of Paramecia competing for the same resource in a culture medium. Paramecium
caudatum and Paramecium aurelia were grown in separate cultures as well as in mixed cultures.
It was found that each species grew in numbers according to the logistic equation.
P
100 Alone
Number of paramecia

P.aurelia Mixed culture


80
60
Alone
40 P.caudatum
20
Mixed culture
t
0 2 4 6 8 10 12 14 16 18
Time (days)
(a) Which species is competitively superior? Support it with the data provided in the graph.
(b) State the underlying principle for the above result and name the scientist associated with this
principle.
(c) Explain the mechanism in which two or more species competing with each other can co - exist.
OR
Graphs A and B shown below depict interaction of two species. Which graph indicates
Mutualism? Give reason.

Species Species
introduced introduced
A
Population size

B
Population size

B A

Time Time
A B

E 275
SECTION – E
31. Placed below are case studies of some couples who were not able to have kids. These couples are
not ready for adoption or taking gametes from donors. After thoroughly examining the cases,
which Assisted Reproductive Technology will you suggest to these couples as a medical expert?
Explain briefly with justification of each case.
Couple Test Report of Female Test reports of male partner
Partner
Couple 1 Normal reports Normal sperms in testes, Missing
connection in epididymis and Vas
deferens
Couple 2 Blockage in the fallopian tube Normal reports
Couple 3 Normal reports Poor semen parameters in terms
of count, motility and morphology
Couple 4 low ovarian reserve Normal reports
Couple 5 Sterilization in male Morphologically abnormal sperms
OR
Given below are certain situations. Analyse the situation and suggest the name of suitable
contraceptive device along with mode of action.
Requirement of Name of
Situation contraceptive for - contraceptive Mode of action
device
1. Blocking the entry of
sperms through cervix
2. Spacing between
children
3. Effective emergency
contraceptive
4. Terminal method to
prevent any more
pregnancy in female
5. Sterilization in male

276 E
32. Given below is a stretch of DNA showing the coding strand of a structural gene of a transcription
unit?
5’--ATG ACC GTA TTT TCT GTA GTG CCC GTA CTT CAG GCA TAA—3’
(a) Write the corresponding template strand and the mRNA strand that will be transcribed, along
with its polarity.
(b) If GUA of the transcribed mRNA is an intron, depict the sequence involved in the formation
of mRNA /the mature processed hnRNA strand.
i. In a bacterium
ii. In humans
(c) Upon translation, how many amino acids will the resulting polypeptide have? Justify.
OR
In shorthorn cattle, the coat colours red or white are controlled by a single pair of alleles. A calf
which receives the allele for red coat from its mother and the allele for white coat from its father
is called a 'roan'. It has an equal number of red and white hairs in its coat.
(a) Is this an example of codominance or of incomplete dominance?
(b) Give a reason for your answer.
(c) With the help of genetic cross explain what will be the consequent phenotype of the calf
when
(i) red is dominant over white
(ii) red is incompletely dominant.
33. Explain the role of Primary and Secondary Lymphoid organs with the help of suitable examples.
OR
With the help of a flow chart illustrate how an infected animal cell can survive while viruses are
being replicated or released.

E 277
IMPORTANT NOTE

______________________________________________________________________

_______________________________________________________________________
_______________________________________________________________________
_______________________________________________________________________
_______________________________________________________________________
_______________________________________________________________________
_______________________________________________________________________
_______________________________________________________________________
_______________________________________________________________________
_______________________________________________________________________
_______________________________________________________________________
_______________________________________________________________________
_______________________________________________________________________
_______________________________________________________________________
_______________________________________________________________________
_______________________________________________________________________
_______________________________________________________________________
_______________________________________________________________________
_______________________________________________________________________
_______________________________________________________________________
_______________________________________________________________________
_______________________________________________________________________
_______________________________________________________________________

_______________________________________________________________________

_______________________________________________________________________
_______________________________________________________________________
SAMPLE QUESTION PAPER
SOLUTION

SECTION – A
1. (D) black pepper [1]
2. (D) tapetum [1]
3. (D) 7 [1]
4. (A) males and females, respectively [1]
5. (A) 0.32 [1]
6. (A) random and directionless [1]
7. (D) Nucleotide [1]
8. (D) aa [1]
9. (C) Cyclosporin A produced from Trichoderma polysporum [1]
10. (D) [1]
11. (B) Reduce pesticide accumulation in food chain [1]
12. (D) Soil Sample C [1]
13. (D) A is false but R is true [1]
14. (C) A is true but R is false. [1]
15. (A) Both A and R are true and R is the correct explanation of A. [1]
16. (A) Both A and R are true and R is the correct explanation of A. [1]
SECTION – B
17. Spermatogenesis starts at the age of puberty due to significant increase in the secretion of
gonadotropin releasing hormone (GnRH). This is a hypothalamic hormone. [0.5]
The increased levels of GnRH then act at the anterior pituitary gland and stimulate secretion of
two gonadotropins – luteinising hormone (LH) and follicle stimulating hormone (FSH).[0.5]
LH acts at the Leydig cells and stimulates synthesis and secretion of androgens. Androgens, in
turn, stimulate the process of spermatogenesis. [0.5]
FSH acts on the Sertoli cells and stimulates secretion of some factors which help in the process
of spermiogenesis. [0.5]
18.(a) CTT would become CAT which codes for valine. Thus, valine would replace glutamic acid at
that point. [0.5]
(b) Sickle cell anaemia [0.5], the mutant haemoglobin molecule undergoes polymerization [0.5]
leading to the change in the shape of the RBC from biconcave disc to elongated sickle like
structure. [0.5]
E 279
19. On administration of the first dose of the vaccine (L), the body shows a response of low
intensity (X) as the immune system comes in contact with the antigenic protein of the
weakened/inactivated pathogen for the first time. This is called primary immune response. [1]
On subsequent encounter with the same antigenic protein in the second dose (M), the body
elicits a highly intensified secondary response (Y). Because of the memory of the first contact
with the antigen, the secondary immune response is faster and stronger, leading to more
effective pathogen elimination in comparison to the primary immune response. [1]
20. (a) Plate 1, -galactosidase enzyme is responsible for blue colour. Gene is inserted in the
-galactosidase site of the plasmid thereby causing insertional inactivation of the enzyme,
so no blue colour is made. [1]
(b) Plate II - Gene of interest not inserted in the plasmid [0.5]
Plate III - No plasmid [0.5]
21.
Tertiary consumer Large Fishes 37 kg/m2
Secondary consumer Small Fishes 25 kg/m2
Primary consumer Small Fishes 11 kg/m2
Primary producer Phytoplankton 4 kg/m2

Inverted Pyramid of Biomass

OR
(a) Gross Primary Productivity is 45000 + 40367 = 85367 KJm-2y-1 [1]
(b) Net production is gradually reducing as we move from producers to consumers due to heat
loss/respiration /10% law. [1]
SECTION – C
22. (a) Sperm A [0.5]
(b) In the figure given, Sperm ‘A’has come in contact with the zona pellucida layer (P) of the
ovum (Q), it will induce changes in the membrane that will block the entry of additional
sperms (B and C). Thus, it ensures that only one sperm can fertilise the ovum. [0.5]
 The secretions of the acrosome of sperm A will help it to enter into the cytoplasm of the ovum
(Q) through the zona pellucida (P) and the plasma membrane, this will induce the completion of
the meiotic division of the secondary oocyte (Q). [1]
 The second meiotic division in Q being unequal will result in the formation of a second polar
body and a haploid ovum. Then, the haploid nucleus of the sperm ‘A’ and that of the ovum (Q)
will fuse together to form a diploid zygote. [1]

280 E
23. The embryo with 8 to 16 blastomeres is called a morula.

 The morula continues to divide and transforms into blastocyst as it moves further into the

uterus.

 The blastomeres in the blastocyst are arranged into an outer layer called trophoblast and

 An inner group of cells attached to trophoblast called the inner cell mass.

 The trophoblast layer then gets attached to the endometrium and the inner cell mass gets

differentiated as the embryo.

 After attachment, the uterine cells divide rapidly and covers the blastocyst.

 As a result, the blastocyst becomes embedded in the endometrium of the uterus. This is called

implantation and it leads to pregnancy. [0.5X6=3]

(a) (b) (c) (d)


(e) Morula

(h)
Implantation
of blastocyst

(f) (g) Blastocyst

Fig : Fertilisation and passage of growing embryo in fallopian tube

24. (a) The embryo has Turner’s Syndrome [0.5] due to aneuploidy of the sex chromosome. Such a
disorder is caused due to the absence of one of the X chromosomes, i.e., 45 with XO. [0.5]

(b) She was advised MTP as the child will have the following problems :

 udimentary ovaries

 poorly developed breasts

 lack of other secondary sexual characters

 delayed or no onset of the menstrual cycle and infertile. [Any 2; 2 marks]


281
25. (a) A-stabilising; B - directional; C - disruptive; [1.5]
(b) Graph A – Stabilising Graph B – Directional Graph C – Disruptive Graph [1.5]

Number of individuals

Graph A Graph B Graph C


Stabilising Directional Disruptive

26.  It will adversely affect the secondary treatment or biological treatment of sewage.
  When the aeration tank is not functional, the air will not be pumped into it.
 This will not allow the vigorous growth of useful aerobic microbes into flocs (masses of
bacteria associated with fungal filaments to form mesh like structures).
 Thus, the major part of the organic matter in the effluent will not be consumed by these
bacteria.
 The BOD (biochemical oxygen demand) of the effluent will not be reduced. BOD refers to
the amount of the oxygen that would be consumed if all the organic matter in one liter of
water were oxidised by bacteria.
 The greater the BOD of waste water, more is its polluting potential. Thus, the effluent will
remain polluted with high amount of organic matter and high BOD. [0.5X6=3]
27.(a) Cry I Ab [0.5]
(b) The spores of Bt contain crystalline toxin which is inactive [0.5]; for this crystalline toxin
protein to become active it needs alkaline pH, which is present in insect gut [0.5] The gut
lining is broken down/mid gut epithelial cells become porous/swollen/cell lysis. [0.5]
(c) The Bt-toxin gene is cloned and inserted into the plant genome by recombinant DNA
technology. These genetically modified (GM) plants express the Bt-toxin genes and become
pest-resistant. [1]
OR
(a) (i) Functional enzyme lipase is given to the patient by injection. [0.5]
(ii) This procedure is not completely curative. [0.5]
(b) The disease can be treated by using Gene therapy. [0.5]
 Gene therapy is a collection of methods that allows correction of a gene defect that has been
diagnosed in a child/embryo. [0.5]
 Here genes are inserted into a person’s cells and tissues to treat a disease. Correction of a genetic
defect involves delivery of a normal gene into the individual or embryo to take over the function
of and compensate for the non-functional gene. [1]
28. 'Prokaryotic organisms' diversity is not given any figures by ecologist because of following

reasons. [3]

 Classification and identification of vast diversity of microbes is very difficult and cannot be

efficiently done with use of currently available methods.

 For many microorganisms, it is difficult to culture them under laboratory condition.

 According to current biochemical and molecular techniques, it is estimated that microbes

diversity can range in billions with microbes inhabiting diverse habitat on earth, with enormous

diversity present in air, water and soil. Hence, more advanced molecular and biochemical

techniques are needed to classify and identify this enormous diversity of microbes.

SECTION – D
29. (a) Plasmids which can be used to insert the geneof interest from a desired organism into a
host/ they act as vectors to transfer gene of interest into the host. [1]
OR
Ori - Origin of replication (ori) - No replication will take place resulting in no copies of linked
DNA.
(b) (i) 5’... ATC GTA/AAG CTT /CAT…3’
3’... TAG CAT/TTC GAA /GTA…5’ [1 mark for both strand]
OR
5’... AAG CTT …3’
3’... TTC GAA …5’ ’ [1 mark for both strand]
(ii) No, as the restriction enzymes need to be the same which cut the DNA of the plasmid and the
gene of interest from the plant. [0.5+0.5=1]
UC18
(c) p as it has a higher copyrate. [0.5+0.5=1]
30. (a) P.aurelia species is competitively superior P.aurelia grows in numbers more quickly than
P.caudatum and shows more individuals in the same volume of culture/100 Paramecia
aurelia in 6 days whereas 60 P.caudatum in 8 days. [2]
(b) 'Competitive Exclusion Principle' which states that two closely related species competing for
the same resources cannot co-exist indefinitely and the competitively inferior one will be
eliminated. G.F. Gause, [1]
(c) One such mechanism is ‘resource partitioning’. If two species compete for the same resource,
they could avoid competition by choosing different times for feeding or different foraging
patterns, to avoid competition and co-exist due to behavioural differences in their foraging
activities. [1]
OR

Graph A - As both species grow simultaneously.

E 283
SECTION – E
31. Couple 1 : Normal reports of female, Normal sperms in testes, Missing connection in epididymis

and Vas deferens in male.

Assisted Reproductive Technology : Semen will be devoid of sperms in this case. So, In-vitro

fertilization (IVF) by collecting the sperms from epididymis, followed by ZIFT or IUT

(Test Tube Baby) is suggested. ZIFT is transfer of zygote or early embryo up to 8 blastomeres in

fallopian tube and IUT refers to transfer of embryos with more than 8 blastomeres in uterus.

[1]

Couple 2 : Blockage in the fallopian tube in the female, Normal reports of male.

Assisted reproductive Technology : Blockage of Fallopian Tube will not allow transfer of

sperms to the site of fertilisation. In-vitro fertilization (IVF) followed by IUT (Test Tube Baby).

It would involve transfer of embryo with more than 8 blastomeres in uterus. [1]

Couple 3 : Normal reports of female, Poor semen parameters in terms of count, motility and

morphology in male partner.

Assisted Reproductive Technology : Intracytoplasmic sperm injection (ICSI) in which sperm is

directly injected into the ovum. Artificial insemination procedure is used mainly when sperms

have poor characteristic or low sperm count. [1]

Couple 4 : Low ovarian reserve in female, Normal reports in male.

Assisted Reproductive Technology : In-vitro-fertilization (IVF) by selection of normal

blastocysts from ovary followed by Zygote intra-fallopian transfer involving transfer of zygote or

early embryos up to 8 blastomeres (ZIFT) or transfer of embryo with more than 8 blastomeres in

the uterus (IUT). [1]

Couple 5 : Poor ovarian reserve in female, morphologically abnormal sperms in male partner.

Assisted Reproductive Technology : ICSI intracytoplasmic sperm injection in which selected

normal sperms will be injected into the selected blastocyst. Intracytoplasmic sperm injection

(ICSI) procedure is used mainly when sperms have poor characteristic or low sperm count. [1]

OR
284 E
Situation Requirement of Name of contraceptive
Mode of action
No. contraceptive for - device
1. blocking the entry of Diaphragms/cervical Cover the cervix during coitus
sperms through caps/ vaults
cervix
2. spacing between Cu or hormone Cu ions from Cu containing IUDs
children releasing IUDs such as increase phagocytosis of sperms
Cu T/Cu7/ Multiload within uterus, suppress sperm
375/ Progestasert/LNG motility and fertilizing capacity/
20 hormone releasing IUDs make
uterus unsuitable for implantation
3. effective emergency Pills containing Pills inhibit ovulation and
contraceptive Progestrogens or implantation as well as alter the
progestrogen-estrogen quality of cervical mucus to
combination or IUDs prevent the entry of sperms/IUDs
within 72 hours of - Cu ions increase phagocytosis of
coitus sperms within uterus, suppress
sperm motility and fertilizing
capacity/ hormone releasing IUDs
make uterus unsuitable for
implantation
4. terminal method to Tubectomy Block gamete transport and
prevent any more prevent conception.
pregnancy in female
5. Sterilization in male Vasectomy Blocks sperm transport.
[0.5×10=5]
32. 5’--ATG ACC GTA TTT TCT GTA GTG CCC GTA CTT CAG GCA TAA—3’= CODING
(a) 3’- TAC TGG CAT AAA AGA CAT CAC GGG CAT GAA GTC CGT ATT---5’= TEMPLATE
[1 mark]
5’---AUG ACC GUA UUU UCU GUA GUG CCC GUA CUU CAG GCA UAA---3’ [1 mark]
(b) i. In a bacterium
5’---AUG ACC GUA UUU UCU GUA GUG CCC GUA CUU CAG GCA UAA---3’ [1 mark]
ii. In humans 5’---mGpppAUG ACC UUU UCU GUG CCC CUU CAG GCA UAA- Poly A
tail--3’ [1 mark]
(c) 9 amino acids in the polypeptide because UAA is stop/terminator codon and does not code for
any amino acid. [1 mark]
OR
(a) Codominance [0.5]
(b) Codominance is a condition in which two different alleles for a genetic trait are expressed.
Individuals receive one version of a gene, called an allele, from each parent. [0.5]

E 285
(c) (i) If pure breeding red coated cattles are represented as ‘RR’ and pure breeding white coated as
‘rr’. If Red is dominant over White. A cross between ‘RR’ and ‘rr’ would produce red coated
cattles (RR) and white coated cattle (rr) in the ratio of 3 : 1.
Parents: RR (Red) X rr (White) Gametes : Rr
R r
R RR [Red Coat] Rr [Red Coat]

r Rr [Red Coat] rr [White Coat]

F1 generation - 3 : 1
(ii) If the red and white coated cattles produce pink colour on a cross then, they exhibit incomplete
dominance in the inheritance of coat colour due to which they produce pink coloured coat upon
hybridisation. If pure breeding red coated cattles are represented as ‘RR’ and pure breeding white
coated as ‘rr’, then the pink coated cattles are ‘Rr’. A cross between ‘RR’ and ‘rr’ would produce
pink coated cattles (Rr) and white coated cattle (rr) in the ratio of 1 : 2 : 1
R r
R RR [Red] Rr [Pink]

r Rr [Pink]
rr [White]
[2]
33.  Lymphoid organs : These are the organs where origin and/or maturation and proliferation of
lymphocytes occur.
 The primary lymphoid organs are bone marrow and thymus where immature lymphocytes
differentiate into antigen-sensitive lymphocytes.
 After maturation the lymphocytes migrate to secondary lymphoid organs like spleen, lymph
nodes, tonsils, Peyer’s patches of small intestine and appendix.
 The secondary lymphoid organs provide the sites for interaction of lymphocytes with the
antigen, which then proliferate to become effector cells.
 The bone marrow is the main lymphoid organ where all blood cells including lymphocytes
are produced.
 The thymus is a lobed organ located near the heart and beneath the breastbone.
 Both bone-marrow and thymus provide micro-environments for the development and
maturation of T-lymphocytes.
 The spleen is a large bean - shaped organ. It mainly contains lymphocytes and phagocytes.
 It acts as a filter of the blood by trapping blood-borne micro - organisms. Spleen also has a
large reservoir of erythrocytes.
 The lymph nodes are small solid structures located at different points along the lymphatic
system. Lymph nodes serve to trap the micro-organisms or other antigens, which happen to
get into the lymph and tissue fluid. Antigens trapped in the lymph nodes are responsible for
the activation of lymphocytes present there and cause the immune response.
There is lymphoid tissue also located within the lining of the major tracts (respiratory,
digestive and urogenital tracts) called mucosa - associated lymphoid tissue (MALT). It
constitutes about 50 per cent of the lymphoid tissue in human body. [0.5 X 10 =5]
OR

Retrovirus (1/2)
Viral RNA core (1/2)
Virus infects
Viral protein normal cell (1/2)
coat
Plasma
membrane
Animal cell
Viral RNA is introduced into cell (1/2)
Cytoplasm
Viral DNA is
produced by Viral DNA incorporates
reverse (1/2)
into host genome
transcriptase
(1/2)
New viral RNA is
(1/2) produced by the
infected cell

New viruses are


produced (1/2)
Nucleus DNA
(1/2)

New viruses
can infect other cells (1/2)

Note : Infected cell can survive while viruses are being replicated
and released.
Fig. Replication of retrovirus

E 287
IMPORTANT NOTE

______________________________________________________________________

_______________________________________________________________________
_______________________________________________________________________
_______________________________________________________________________
_______________________________________________________________________
_______________________________________________________________________
_______________________________________________________________________
_______________________________________________________________________
_______________________________________________________________________
_______________________________________________________________________
_______________________________________________________________________
_______________________________________________________________________
_______________________________________________________________________
_______________________________________________________________________
_______________________________________________________________________
_______________________________________________________________________
_______________________________________________________________________
_______________________________________________________________________
_______________________________________________________________________
_______________________________________________________________________
_______________________________________________________________________
_______________________________________________________________________
_______________________________________________________________________

_______________________________________________________________________

_______________________________________________________________________
_______________________________________________________________________
288 E

You might also like